Download as pdf or txt
Download as pdf or txt
You are on page 1of 133

www.vet4arab.co.

cc

.c om
ob
ott
alk
w.
ww

www.alkottob.com
www.vet4arab.co.cc

om
Mosby's

ReView

b.c
Questions &Answers
for Veterinary Boards

tto
Clinical Sciences

al ko
w.
ww

www.alkottob.com
"---""
www.vet4arab.co.cc

Dedicated to Publishing Excel'ence

om
~ A Times Mirror
.. Company

Publisher: John A. Schrefer


Executive Editor: Linda L. Duncan
Contributors
Senior Developmental Editor: Teri Merchant
Project Manager: Linda McKinley
Production Editor: Julie Zipfel
Editing and Production: Top Graphics

b.c
Design: Renee Duenow
Manufacturing Manager: Unda Ierardi
Cover desigrl: Iennifer Marmarinos S.T. Finn-Bodner, DVM, MS, Dipl ACVR
Introduction Associate Professor, Department of Radiology, Auburn Uni·
Paul W. Pratt, VMD versity, Auburn, Alabama
Executive Editor, Mosby-Year Book, St. Louis, Missouri
judith A. Hudson, DVM, PhD, Dipl ACVR
jeffrey L. Rothstein, DVM, MBA Associate Professor, Department of Radiology, Auburn Uni-
Hospital Director, Elm Animal Hospital, Roseville, Michi- versity, Auburn, Alabama
gan

tto
William R. Klemm, DVM, PhD
SECOND EDmON Professor, College of Veterinary Medicine, Texas A & M
Copyright © 1998 by Mosby-Year Book, Inc. Anesthesiology University, College Station, Texas
Previous edition copyrighted 1993 Colin I. Dunlop, BVSc, Dipl ACVA Martha L. Moon, DVM, MS, Dipl ACVR
Advanced Anesthesia Specialists, Hunters Hill, New South Associate Professor, Virginia· Maryland Regional College of
All rights reserved. No part of this publication may be reproduced, stored in a retrieval system, or
Wales, Australia Veterinary Medicine, Virginia Polytechnic Institute,
transmitted, in any form or by any means, electronic, mechanical, photocopying, recording. or
otherwise, without prior written permission from the publisher. Etta M . Wertz, DVM, MS, Dipl ACVA Blacksburg, Virginia

ko
Permission to photocopy or reproduce solely for internal or personal use is permitted for libraries Staff Anesthesiologist, Alameda East Veterinary Hospital, Paul W. Pratt, VMD
or other users registered with the Copyright Clearance Center. provided that the base fee of $4.00 Denver, Colorado Executive Editor, Mosby- Year Book, St. Louis, Missouri
per chapter plus $.10 per page is paid directly to the Copyright Clearance Center, 222 Rosewood
Dr., Danvers, MA 01923. This consent does not extend to other kinds of copying. such as copying Donald E. Thrall, DVM, PhD, Dipl ACVR
for general distribution, for advertising or promotional purposes, for creating new collected works, Clinical Pathology Professor of Radiology, College of Veterinary Medicine,
or for resalc. North Carolina State University, Raleigh, North Carolina
Carol B. Grindem, DVM, PhD, Dipl ACVP
Printed in the United States of America Associate Professor of Clinical Pathology, College ofVeteri- Larry Patrick Tilley, DVM, Dipl ACVIM

al
Composition by Ibp Graphics nary Medicine, North Carolina State University, Raleigh, President, Vetmedfax Consultation Services, Santa Fe, New
Printing/binding by R.R. DonneDey & Sons Company
North Carolina Mexico
Mosby-Year Book, Inc. Barry Thomas Mitzner, DVM Erik R. Wisner, DVM, Dipl ACVR
11630 Westline Industrial Drive President, Southeast Vetlab, Inc., Miami, Florida Assistant Professor, Department of Radiology, School of
St. Lottis, Missouri 63146

Ubrary of Congress Cataloging In Publication Data


w. Rose E. Raskin, DVM, PhD, Dipl ACVP
Associate Professor and Service Chief of Clinical Pathology
Laboratory, Department of Physiological Sciences, Uni-
Medicine, and Department of Surgicai and Radiological
Sciences, School of Veterinary Medicine, University of
California, Davis, California

Clinical sciences / edited by Paul W. Pratt.-2nd ed. versity of Florida, Gainesville, Florida
p.
boards)
cm.-(Mosby's review questions & answers for veterinary Hematology and Cytology
Includes bibliographical refcrences. Diagnostic Imaging Rick L. Cowell, DVM, MS, Dipl ACVP
ww
ISBN 0·61 51-7462·4 Professor, Department of Anatomy, Pathology, and Phar-
1. Veterinary medicine-United States-Examinations. questions,
and Recordings macology, College of Veterinary Medicine, Oklahoma
etc. l. Pratt, Paul W. 1I. Series. john M. Bowen, DVM, PhD State University, Stillwater. Oklahoma
SF759.C56 1997
Professor of Pharmacology and 1bxicology, Associate Dean
636.069'076-<1c21 97-37128 W. jean Dodds, DVM
CIP for Research and Graduate Affairs, College of Veterinary
Medicine, University of Georgia, Athens, Georgia President, HEMOPET, Irvine, California
97 98 99 00 01 / 9 6 7 6 5 4 3 2 1

www.alkottob.com
vi CONTRIBUTORS
www.vet4arab.co.cc
Brad L. Hines, DVM Thomas J. Burke, DVM, MS
Internist, Metroplex Veterinary Centre, Irving, Texas Professor of Medicine, College of Veterinary Merlicine, Uni-
versity of Illinois, Urbana, Illinois; Consultant, Capitol

om
Illini Veterinary Services, Ltd., and lincoln Park Zoo
Immunology
Leslie A. Dierhauf, VMD
Laurel J. Gershwin, DVM, PhD, Dipl ACVM
Wildlife and Conservation BiolOgist, United States Fish and
Professor ofImmunology, VMTH Service Chief, Clinical
Immunology and Virology, School of Veterinary Medi-
cine, University of California, Davis, California
Wildlife Service, Albuquerque, New Mexico
Maarten Drost, DVM
Preface
Professor of Reproduction, College of Veterinary Medicine,
University of Florida, Gainesville, Florida
Nutrition
Bruce E. Eilts, DVM, MS, Dipl ACT

b.c
Linda D. Baker, VMD, MS, Dipl ACVN
Professor of Theriogenology, Department of Veterinary
Veterinarian, Dairy Production Systems Consultation,
Clinical Sciences, School of Veterinary Medicine, This series of five review books was developed to help
Mertztown, Pennsylvania thusiasm and ingenuity in developing challenging
Louisiana State University, Baton Rouge, Louisiana
candidates prepare for scholastic, licensure, and certi- questions are evident throughout the five volumes. Al-
Edward A. Moser, VMD, Dipl ACVN fication examinations. Although the books are not de-
Fredric L. Frye, DVM, MSc, CBiol, FIBiol though I had considered myself fairly well read in our
Veterinary Nutritionist, Veterinary Nutrition Specialists.
Visiting Professor at the University of London, University finitive texts, they can help candidates organize their field, I was humbled by the depth and breadth of
Selinsgrove, Pennsylvania
of Bristol, University of Edinburgh; Former Clinical Pro- study preparations and detect areas in which more knowledge illustrated in their questions.
fessor of Medicine, University of California, Davis, Cali- study is required. We have gone to great effort to root out all errors
Pharmacology fornia The five-volume series contains over 8,100 ques- and ambiguous statements. Despite these precau-

tto
tions, each with an accompanying answer. A short ex- tions, however, a number of flaws undoubtedly have
Terrence P. Clark, DVM, PhD Tina Gemeinhardt, DVM planation or rationale is provided for every answer. escaped notice. We would be grateful if readers would
Assistant Professor, Director, Clinical Pharmacology Ser- Huff Animal Hospital, Delta, British Columbia New to this edition are additional sections on special- notify us of any errors, ambiguities, or questionable
vice, Department of Physiology and Pharmacology,
Stormy Hudelson, DVM, Dipl ABVP ties, thousands of new questions, and updating of all statements in these books. We also encourage readers
Auburn University, Auburn, Alabama
Tucson, Arizona questions according to current medical and surgical to send their comments or criticism on any aspect of
Lloyd E. Davis, DVM, PhD practices. these books. In this way we can improve the quality of
David G. Huff, DVM
Professor Emeritus of Clinical Pharmacology, College of I am indebted to our group of 150 eminently quali- future editions.
Consulting Veterinarian, Vancouver Aquarium, Delta,
fied contributors, who have taken the time from their

ko
Veterinary Medicine, University of Illinois, Urbana, illi-
British Columbia
nois busy professional and personal lives to carefully craft Paul W. Pratt, l-MD
James D. Letcher, DVM questions on their respective subject areas. Their en- Santa Barbara, California
Sue Hudson Duran, RPh, MS
Okeechobee, Florida
Assistant Professor and Director of Pharmacy, Department
of Large Animal Surgery and Medicine, Auburn Univer- Seyedmehdi Mobini, DVM, MS, Dipl ACT
sity, Auburn, Alabama Professor of Veterinary Science, Research/Extension Veteri-
narian, Fort Valley State University, Fort Valley, Georgia

al
Jim E. Riviere, DVM, PhD
Burroughs Wellcome Distinguished Professor of Veterinary Daniel H. Nielsen, DVM
Pharmacology, College of Veterinary Medicine, North New Berlin Veterinary Hospital, New Berlin, New York
Carolina State University, Raleigh, North Carolina
Michael B. Paster, DVM
Avalon Animal Hospital and Bird Clinic, Carson, California
Principles of Surgery
Thomas P. Colville, DVM, MSc
Dorcas O. Schaeffer, DVM, MS, Dipl ACLAM
w.
Assistant Professor, Department of Comparative Medicine,
Associate Professor, Director, Veterinary Technology Pro- Facility Director, Office of Laboratory Animal Care, Col-
gram, Department of Veterinary and Microbiological lege of Veterinary Medicine, University of Tennessee,
Science, North Dakota State University, Fargo, North Knoxville, Tennessee
Dakota
Roderick C. Tubbs, DVM, MS, MBA, Dipl ACT, ABVP
ww
Swine Veterinarian, Bowling Green, Kentucky
Theriogenology
William F. Braun, Jr., DVM, Dipl ACT Toxicology
Veterinarian, Southwest Veterinary Services, Bloomington,
Gary D. Osweiler, DVM, PhD, Dipl ABVT
Wisconsin; Fonnerly Associate Professor, College of Vet-
Professor of Veterinary Toxicology, Director, Veterinary Di-
erinaryMedicine, University of Missouri, Columbia, Mis-
agnostic Laboratory, Iowa State University, Ames, Iowa
souri

vii

www.alkottob.com
'-
www.vet4arab.co.cc

om
Contents

b.c
SECTION 1 SECTION 6

Aulesdrresiology, 1 Nutrition, 143


CI. Dunlop, E.M. Wertz L.D. Baker, E.A. Moser

SECTION 2 SECTION 7

tto
Clinical Pathology, 35 Pharmacology, 155
CB. Grindem, B.T. Mitzner, R.E. Raskin T.P. Clark, L.E. Davis, S.H. Duran, J.E. Riviere

SECTION 3 SECTION 8

Diagnostic Imaging Principles of Surgery, 185

ko
and Recordings, 63 T.P. Colville

J.M . Bowen, S.T. Finn-Bodner, JA Hudson,


w'R. Klemm, M.L. Moon, P.W. Pratt, D.E. Thrall, SECTION 9
L.P. Tilley, E.R. Wisner
Theriogenology, 199
W.F. Braun, Jr., T.J. Burke, L.A. Dierauf, M. Drost,

al
SECTION 4 B.E. Eilts, F.L. Frye, T. Gemeinhardt, S. Hudelson,
Hematology and D.G. Huff, J.D. Letcher, S. Mobini, D.H. Nielsen,
M .B. Paster, D.O. Schaeffer, R.C Tubbs
Cytology, 101
R.L. Cowell, W.J. Dodds, B.L. Hines
w. SECTION 5
SECTION 10

Toxicology, 245
G.D. Osweiler
Immunology, 131
L.J. Gershwin
ww
Practice Aulswer Sheets, 255

Ix

www.alkottob.com
- - -----------.....-:.............
~------
www.vet4arab.co.cc SECTION

om
Anesthesiology
c.1. Dunlop, E.M. Wertz

b.c
Recommended Reading
Hall LW; Clarke KW: Veterinary anaesthesia, ed 9, London, 1992, Bailliere Tindall.
MuirWW, Hubbell IA: Equine anesthesia, St Louis, 1991, Mosby.
Muir WW; Hubbell 1A: Handbook of veterinary anesthesia, ed 2, St Louis, 1996, Mosby.

tto
Short CEo Principles and practice of veterinary anesthesia, Baltimore, 1987, Williams & Wilkins.
Thurman IC et al: Lumb and Jones' veterinary anesthesia, ed 3, Baltimore, 1996, Williams &
Wilkins.

',' Practice answer sheets are on pages .255-256.

ko
Questions
c.1. Dunlop
1. Which of the following is 1Wt an objective of d. competitive antagonism of acetylcholine
preanesthetic medication? e. stimulating adrenergic activity

al
a. alleviate or minimize pain
b. facilitate handling 3. Bradycardia during general anesthesia of dogs
c. minimize undesirable postanesthetic recovery can be treated with intravenous atropine, usually
w. complications at 0.01 to 0.02 mglkg Iv. If the bradycardia
worsens after atropine administration, the most
d. increase reflex autonomic activity
appropriate course of action is to:
e. decrease the dose of parenteral anesthetic
induction drugs a. repeat the dose after 1 or 2 minutes
b. wait for atropine to cause its sympathetic
effect
2. Atropine acts on the parasympathetic nervous
sYstem by: c. cease administration of anesthetic and
ww
insufflate with oxygen
a. depressing formation of acetylcholine d. administer epinephrine intravenously
b. enhancing formation of cholinesterase e. insert a transvenous pacemaker because the
c. directly inhibiting neurochemical reactions at dog must have sick sinus syndrome
the myoneural junction

Correct answers are on pages 25-33. 1

www.alkottob.com
2 SECTION 1 www.vet4arab.co.cc Anesthesiology 3
4. One of the differences between atropine and 9. Guaifenesin, a muscle relaxant commonly used 14. Elimination of atracurium from the body: d. decreased uterine muscle tone in pregnant
glycopyrrolate is that: for large animal anesthesia,:
a. produces the metabolite laudanosine, which animals
a. glycopyrrolate causes a greater increase in the a. acts at the myoneural junction depresses central nervous system function e. increased ventilation

om
heart rate than atropine b. causes profound cardiac depression b. is enhanced by its inherent instability at body
b. glycopyrrolate causes a smaller increase in the c. does not provide analgesia temperature and pH 19. In dogs appropriate treatment for bradycardia
heart rate than atropine d. causes respiratory paralysis because of its c. is prolonged in patients with renal disease associated with xylazine administration is:
c. atropine is longer acting than glycopyrrolate muscle relaxant properties only
d. atropine can be antagonized by epinephrine e. can be very effective in small doses a. an intravenous bolus dose of lactated Ringer's
d. is prolonged in patients with both renal and solution
but glycopyrrolate cannot liver disease
e. glycopyrrolate has a different site of action b. intravenous neostigmine
10. A drug with a similar mechanism of action to e. is prolonged following intravenous infusion
than atropine c. intravenous atropine
guaifenesin is:
d. intravenous yohimbine

b.c
a. diazepam 15. Atracurium has advantages over succinylcholine e. intravenous doxapram
5. One milliliter of a 10% solution of a drug contains in that:
how many milligrams of that drug? b. glycopyrrolate
c. butorphanol a. it has a shorter duration of action
a . 10mg/ml 20. Morphine, oxymorphone, and fentanyl produce
d. atracurium b. it does not cause ganglionic blockade analgesia and somnolence primarily by their
b.20mg/ml c. it does not cause muscle rigidity
e. succinylcholine action on:
c. 1.0 mg/ml d. reversal of succinylcholine requires
d. 0.1 mg/ml a. a 2 adrenergic receptors
administration of neostigmine
11. In horses the lethal effects ofguaifenesin: b. K-opiate receptors
e. 100mg/ml e. it paralyzes all skeletal muscles to the same

tto
a. are primarily manifested by respiratory center degree c. fl.-opiate receptors
depression d. internuncial neurons in the spinal cord
6. Of the following, which is rwt an action of
phenothiazine tranquilizers on the nervous b. are not seen because such large volumes must e. a-opiate receptors
16. In an anesthetized patient with profound
system? be administered
bradycardia, injection of neostigmine:
c. are primarily manifested by skeletal muscle 21. Which of the following is the most potent opioid
a. antiemetic relaxation resulting in respiratory muscle a. exacerbates the bradycardia agent?
b. sympathomimetic paralysis b. rapidly resolves the bradycardia

ko
c. potentiate seizures d. are seen at 3 to 4 times the therapeutic dose c. does not change the heart rate a. meperidine (Demerol)
d. affect the reticular formation and e. do not include cardiac depression d. causes sudden arousal of the anesthetized b. morphine
hypothalamus animal c. oxymorphone (Numorphan)
e. sedation e. prolongs muscle relaxation d. fentanyl (Sublimaze)
12. If you administer 350 ml ofa 5% solution of
guaifenesin to a 250-kg pony, how many mg/kg e. butorphanol (Torbugesic)
7. Acepromazine is classified as: would be administered? 17. In the central nervous system, what is the action

al
a. a phenothiazine tranquilizer a. 35 ofxylazine or detomidine? 22. Which drug does rwt have a narcotic effect?
b. an anticholinergic b. 250 a. antagonism of internuncial neurons a. levallorphan
c. an a2-adrenergic agonist c. 70 b. agonism of fl. -opiate receptors b. naloxone
d. an opioid agonist d. 10
w. c. antagonism of fl.-opiate receptors c. butorphanol
e. a benzodiazepine e. 100 d. agonism of presynaptic a2 adrenergic d. oxymorphone
receptors e. meperidine
B. In horses acepromazine should be used with 13. Atracurium: e. antagonism of presynaptic a2 adrenergic
caution in: receptors
a. reduces the requirement for inhalation 23. Concerning opioids, such as oxymorphone, which
a. pregnant mares anesthesia because of its analgesic properties statement is least accurate?
lB. In 'horses and cattle xylazine premedication
ww
b. horses with preexisting second-degree b. is a central-acting muscle relaxant a. They are good analgesics.
causes:
atrioventricular block c. crosses the placental barrier b. They cause bradycardia.
c. working stallions d. causes bradycardia following intravenous a. increased cardiac output and heart rate c. They improve ventilation.
d. adult draft horses administration b. second -degree atrioventricular block and d. They depress laryngeal function.
e. arthroscopic surgical patients positioned in e. should not be used unless patients are bradycardia
e. They cause minimal myocardial depression.
dorsal recumbency receiving intermittent positive-pressure c. hypoglycemia
ventilation

© 1998 Mosby-Year Book, Inc. Photocopying is prohibited by law.


Correct answers are on pages 25-33.

www.alkottob.com
4 SECTION 1 www.vet4arab.co.cc Anesthesiology 5
24. Parenterally administered opiate analgesics, such c. does not depress respiration as much as pure 33. After thiobarbiturate anesthesia, prolonged
as morphine, relieve pain by: fL-opioid agonists (e.g., morphine) d. It is associated with excessive muscle tone, so
recovery: diazepam should also be administered.
a. inhibiting transmission of afferent impulses d. is subject to the limitations of the Harrison
Narcotic Act a. may occur in thin, emaciated patients e. It can cause increased laryngeal reflexes and

om
b. interfering with perception of pain b. should be treated with mechanical ventilation coughing, especially at light anesthetic levels.
e. provides better visceral analgesia in dogs than
c. interfering with the response to pain c. may be due to hyperproteinemia
does oxymorphone
d. inhibiting initiation or transmission of d. should be treated with neuroleptics 37. Propofol:
impulses and disrupting perception of such e. may be due to hyperthermia
impulses 29. Narcotic analgesics for horses include morphine, a. lowers the seizure threshold
e. eliminating the source of pain impulses pentazocine, and butorphanol. Concerning these b. increases muscle tone
three drugs, which statement is most accurate? 34. Thiobarbiturates cause: c. causes renal toxiCity
25. Oxymorphone: a. They are all semisynthetic agents with opioid a. increased intracranial pressure d. causes postinduction apnea
agonistic and antagonistic effects.

b.c
b. renal toxicity e. increases intraocular pressure
a. provides rapid induction of anesthesia and is b. They have no potential for abuse by people, so c. bradycardia
suitable for patients with upper airway their use is not restricted.
obstruction d. postinduction apnea 38. Anesthetic induction with propofol causes:
c. They may cause central nervous system
b. can be administered intravenously in e. increased intraocular pressure a. bradycardia
excitement if administered to an otherwise
combination with diazepam for chemical untranquilized horse. b. increased intracranial pressure
restraint suitable to achieve endotracheal 35. Following anesthetic induction with thiopental in
d. Their effects can be antagonized with c. hypotension in hypovolemic patients
intubation a dog, you intubate the animal and administer
yohimbine. d. hypertension in normovolemic patients
c. does not cause significant respiratory halothane and oxygen via a circle system. The dog
e. They increase gastrointestinal motility. e. decreased cardiac output from myocardial

tto
depression immediately becomes apneic. The most depreSSion
d. increases vagal tone and heart rate appropriate course of action is to:
e. is likely to cause laryngospasm during 30. Which drug can antagonize opiate-induced
respiratory depression? a. institute intermittent positive-pressure
endotracheal intubation 39. In dogs recovery from propofol anesthesia is
ventilation at 12 breaths/min, at a volume of characteristically:
a. yohimbine 100 rnl/kg
26. In dogs intravenous administration offentanyl b. naloxone b. turn off the flow of halothane because the dog a. smooth and rapid
causes: c. ephedrine is deeply anesthetized b. rapid and accompanied by excitement

ko
a. hypotension d. etorphine (M 99) c. stimulate upper airway reflexes by gently c. rapid but rough because redistribution sites
e. dopamine moving the endotracheal tube backward and become saturated
b. tachycardia forward d. rapid because propofol is highly soluble in
c. atrioventricular block
d. institute intermittent positive-pressure water and rapidly excreted in urine
d. bradycardia 31 . Thiobarbiturates:
ventilation at 2 or 3 breathS/min, at a volume e. rapid and accompanied by marked ataxia on
e. decreased myocardial contractility a. are characterized by slow induction of of 10 rnl/kg standing

al
anesthesia e. administer another one-quarter dose of
27. Following intravenous administration offentanyl b. should never be used in cats thiopental because the dog's apnea is caused 40. Propofol:
to a dog, you observe marked bradycardia. The c. may produce cardiac arrhythmias following by light anesthesia
bradycardia could be appropriately treated with intravenous administration a. is very irritating when administered
any of the following agents except: perivascularly
d. are not bound to plasma proteins 36. Concerning use of thiopental in cats, which
a.
b.
lidocaine
atropine naloxone
w.
e. produce effects that can be reversed with statement is most accurate?
a. Thiopental should not be used in cats because
b. should be administered intramuscularly
because it is a colloidal solution
c. is dissolved in a soybean oil/egg lecithin
c. nalorphine they are more sensitive to the drug and more emulsion
d. naloxone 32. Which property of thiobarbiturates accounts for prone to toxicity than are dogs.
d. should be refrigerated because it is unstable
e. butorphanol their short duration ofaction? b., It can be given intramuscularly or at room temperature
subcutaneously to cats.
ww
a. rapidly metabolized e. can be used in multidose vials because it
b. quickly deposited in muscle and fat depots c. It is an appropriate choice for uremic, contains a bacteriostatic agent
28. As an analgesic, butorphanol: dehydrated cats.
c. distributed in extracellular tissues
a. has a longer duration of action than morphine
d. rapid renal clearance
b. is completely antagonized by naloxone
e. Hoffman elimination

© 1998 Mosby-Year Book, Inc. Photocopying is prohibited by law.


Correct answers are on pages 25-33.

www.alkottob.com
6 SECTION 1
www.vet4arab.co.cc Anesthesiology 7

45. Major classes of drugs usually considered 50. To produce epidural anesthesia in a dog, a local 54. Hypotension occurring during spinal anesthesia
41. Concerning ketamine, which statement is /east
preanesthetic medications include all the anesthetic drug should be injected at the: may be due to:
accurate?
following except: a. sacrococcygeal junction to avoid recumbency a. preganglionic sympathetic blockade
a. It is preferable to thiamylal as an induction

om
agent in patients with impaired a. dissocia tives b. junction of the fifth and sixth lumbar b. blockade of nerves to the adrenal medulla
cardiovascular function. b. opioids vertebrae to ensure mixing with cerebrospinal c. muscular paralysis
fluid d. loss of spinal fluid
b. It can be combined with acepromazine, c. hypnotics
diazepam, or xylazine. c. lumbosacral space, between the periosteum e. venous constriction
d. sympathomimetics
and dura mater
c. It can cause hallucinations in human beings. e. neuroleptanalgesics
d. lumbosacral space, between the pia mater
d. It should never be administered 55. Epidural local anesthesia may cause any of the
and arachnoid
intramuscularly. 46. Concerning injectable anesthetics, which following except:
e. lumbosacral junction, between the pia mater
e. It may be associated with rough, statement is most accurate? and arachnoid a. central nervous system depression and

b.c
hyperreflexive recoveries.
a. TIletamine is an opioid. seizures
b. Thiamylal is a hypnotic. 51. If2% lidocaine is appropriately administered b. hypotension
42. Ketamine: c. Detomidine is a phenothiazine tranquilizer. epidurally to a dog at 1 mll3 to 4.5 kg, the dog is c. respiratory muscle paralysis
a. causes tachycardia and hypertension via d. Propofol is a dissociative. not likely to: d. local tissue inflammation
sympathetic stimulation e. Meperidine is a benzodiazepine. e. analgesia of 12 hours' duration
a. become recumbent and have hind limb
b. causes circulatory depression analgesia
c. decreases intraocular pressure b. remain ambulatory and have only partial hind 56. An advantage of bupivacaine over lidocaine for
47. Dogs are given neuroleptanalgesic combinations

tto
d. decreases cerebral blood flow and intracranial to achieve profound sedation and analgesia. An limb analgesia epidural anesthesia for cesarean section in dogs is
pressure example of this combination is: c. maintain function of the diaphragm and that bupivacaine:
e. decreases myocardial oxygen consumption intercostal muscles
a. atropine and acepromazine a. has a short duration of action
d. have analgesia of the ventral midline caudal b. has minimal placental transfer and fetal
b. droperidol and glycopyrrolate to the umbilicus
43. Ketamine is a good choice for anesthetic uptake
c. oxymorphone and thiopental
induction in a: e. remain alert c. does not cause vasodilation and hypotension
d. butorphanol and ketamine
a. cat with an elevated blood urea nitrogen level d. is more concentrated than lidocaine

ko
e. fentanyl and acepromazine
and a history of chronic renal failure 52. Which of the following provides surgical e. has fewer cardiac effects than lidocaine when
b. dog that has been hit by a car and has analgesia of more than 2 hours'duration when inadvertently given intravenously
evidence of head trauma 48. Parenteral administration of which drug causes administered epidurally in an appropriate dose?
c. cat with hyperthyroidism and a resting heart some visceral analgesia?
a. bupivacaine 0.5% (Marcaine) 57. When attempting to administer an epidural local
rate of 200 beats/min a. ketamine b. lidocaine 2% (Xylocaine) anesthetic to a dog at the lumbosacral junction,
d. brachycephalic dog with upper airway b. acepromazine c. morphine at 0.5 mg/rnl (Duramorph) you aspirate cerebrospinal fluid. The most

al
obstruction c. glycopyrrolate d. mepivacaine 2% (Carbocaine) appropriate course of action is to:
e. male cat with a history of urethral obstruction d. pentobarbital e. meperidine at 50 mg/rnl (Demerol) a. not administer any epidural anesthetic but
e. atracurium instead use general anesthesia for the
44. Concerning preanesthetic agents, which procedure
53. An epidural local anesthetic block in dogs should
statement is /east accurate?
a. Meperidine is used for premedication and
pain relief.
w.
49. All the following drugs are almost completely
metabolized by the liver before renal excretion
except:
be used with caution or is contraindicated in:
a. a pregnant bitch presented for cesarean
b. administer the local anesthetic as planned
c. substitute morphine for lidocaine as the local
anesthetic
section
b. Glycopyrrolate may be used for a. diazepam d. administer the local anesthetic but at one
b. a dog with anal fistulae third less than the calculated dose
premedication to reduce the incidence of b. thiopental
vagus-mediated bradycardia. c. an old dog requiring repair of a ruptured e. keep trying until you correctly place the
c. fentanyl cranial cruciate ligament
ww
c. Ketamine is an excellent example of a needle in the epidural space
d. glycopyrrolate d. a dog with sacral fractures following
hypnotic premedicant.
e. ketamine automobile trauma
d. Some premedicants may prolong recovery
from general anesthesia. e. a dog with an infected phalangeal joint
requiring toe amputation
e. An advantage of many premedicants is that they
reduce the amount of anesthetic agent required
to induce and maintain general anesthesia.

© 1998 Mosby-Year Book, Inc. Photocopying is prohibited by law. Correct answers are on pages 25-33.

www.alkottob.com
8 SECTION 1 www.vet4arab.co.cc Anesthesiology
9
58. To produce spinal anesthesia, a local anesthetic d. as the sole anesthetic agent provides sufficient 66. Which of the following is lWt a characteristic of
analgesia for surgery d. should only be used with halothane
should be injected: an ideal inhalation agent?
e. does not produce forelimb analgesia e. should be used in closed-circuit anesthesia
a. between the pia mater and arachnoid a. nonflammable and nonexplosive at clinical

om
b. between the arachnoid and dura mater concentrations
62. As compared with halothane or methoxyflurane, 71. Concerning nitrous oxide, which statement is
c. into the epidural space b. low solubility in blood and tissues so that least accurate?
d. between the pia mater and spinal cord isoflurane has: induction and recovery are rapid
e. in the intramedullary area a. lower solubility, resulting in rapid changes in c. low fat solubility a. It should not be used in patients with
anesthetic depth d. high solubility in blood and tissues so that conditions involving gas pockets, such as a
pneumothorax.
b. less effect on cardiac and pulmonary function saturation is more rapid and induction is
59. When attempting to administer epidural hastened b. It is the most soluble inhalation anesthetic
morphine to a dog at the lumbosacral junction, c. greater potential to produce catecholamine-
e. only minimally metabolized and excreted and rapidly moves into and out of the body.
you obtain cerebrospinal fluid. The most induced arrhythmias
c. It can cause diffusion of hypoxemia at the end

b.c
appropriate course ofaction is to: d. greater potency largely unchanged
of inhalation anesthesia.
a. not administer any morphine but rather use e. a more rapid rate of metabolism
d. It requires high fresh gas flows, increasing the
parenteral analgesia 67. Which inhalation anesthetic is subject to the cost of anesthesia.
greatest degree of metabolism?
b. administer the epidural dose of morphine as 63. The minimum alveolar concentration (MAC) ofan e. It is not scavenged by activated charcoal.
planned inhalation anesthetic preventing response to a a. nitrous oxide
c. use fentanyl instead of morphine standard noxious stimulus is a measure of potency. b. halothane 72. Methoxyflurane:
d. administer morphine but at one third less Of the following, which does lWt affect MAC? c. methoxyflurane
than the calculated dose a. age d. desflurane a. is very potent (MAC 0.23 vol%) and so should

tto
e. use lidocaine instead of morphine not be administered in simple in-circuit
b. hypothermia e. isoflurane vaporizers
c. concurrent administration of sedatives and
b. has a low vapor pressure and can easily
60. Morphine for epidural/spinal use (Duramorph) is tranquilizers 68. Which inhalation anesthetic agent is most likely produce dangerously high concentrations
sold in 1 O-ml glass vials at a concentration of 0.5 d. duration of anesthesia to induce cardiac arrhythmias in the presence of c. is flammable in clinical concentrations
mglml. Morphine for parenteral administration is e. pregnancy elevated blood levels of catecholamines? d. is effective in low concentrations
sold in multiple-dose rubber-topped vials at a
a. isoflurane e. produces poor muscle relaxation

ko
concentration of 15 mglml. Morphine for
epidural/spinal use is packaged in smaller vials 64. Which inhalation agent is most potent? b. nitrous oxide
and at a lower concentration because: a. halothane c. methoxyflurane 73. Renal tubular damage can occur in dogs
a. drugs at concentrations greater than 1 mg/rnl b. nitrous oxide d. halothane subjected to prolonged methoxyflurane
should not be administered epidurally c. isoflurane e. desflurane administration and is secondary to:
b. solutions in multiple-dose vials are less d. methoxyflurane a. myocardial depression
convenient to use than solutions in glass vials e. propofol 69. Increasing depth of inhalation anesthesia is b. hepatic necrosis

al
c. the volume of morphine used in epidural accompanied by: c. toxic effects of fluoride ions
anesthesia is small d. hypoxia that occurs at 1.5 times the maximum
65. Mask induction of anesthesia using inhalation a. a marked increase in cardiac output
d. the morphine in the two solutions is different agents is: alveolar concentration
b. an increase in total ventilation per minute
e. the morphine solution supplied in glass vials e. hypotension
a. the best way to induce anesthesia in a patient c. an increase in arterial carbon dioxide tension
is preservative free and intended for single-
patient use with an upper airway obstruction
w.
b. best accomplished using a volatile anesthetic
that is highly soluble in blood, such as
d.
e.
a decrease in dead space ventilation
an increase in arterial blood pH and a 74. The term second-gas effect refers to:
decrease in respiratory frequency a. increased ventilation caused by nitrous oxide
61. In dogs presented for hind limb surgery. epidural methoxyflurane
injection ofmorphine: b. displacement of oxygen from the alveoli by
c. best accomplished using a volatile anesthetic 70. Nitrous oxide: rapid escape of nitrous oxide from the blood
a. increases the requirement for other anesthetic that is relatively insoluble in blood, such as during recovery from anesthesia
ww
drugs halothane a. has no analgesic activity
c. the additive effect of inhalation anesthetics
b. decreases the requirement for other d. not influenced by the adequacy of ventilation b. should not be used in concentrations less
than 80% when given in combination with nitrous oxide
anesthetic drugs e. less expensive than routine parenteral
c. is not sufficiently potent to produce d. distention of the bowel during inhalation of
c. does not change the requirement for other anesthetic inductions nitrous oxide
anesthetic drugs anesthesia in dogs
e. low arterial oxygen levels caused by fresh gas
mixtures that are less than 95% oxygen

© 1998 Mosby-Year Book, Inc. Photocopying is prohibited by law.


Correct answers are on pages 25-33.

www.alkottob.com
10 SECTION 1
www.vet4arab.co.cc Anesthesiology 11

c. is the only inhalation agent that lowers the 83. Inhalation anesthesia can be induced more rapidly c. nitrous oxide
75. Uptake of inhalation anesthetics from the alveoli
threshold to catecholamine-induced in neonatal animals than adults because neonates: d. cyclopropane
into the blood depends on the:
ventricular arrhythmias a. are relatively smaller but have a proportionately e. propane
a. MAC (potency) of the agent

om
d. depresses cardiac function by decreasing larger surface area/body weight ratio
b. patient's packed cell volume (hematocrit) myocardial contractility, causing vasodilation b. contain less body water 88. Cylinders full of compressed gas:
c. the agent's alveolar concentration and blood and depressing the sinoatrial node
c. become hypothermic, so less anesthetic is
solubility and the patient's cardiac output e. is more expensive to use than methoxyflurane a. are safe even if they are knocked over and the
required
d. inspired oxygen concentration and MAC valve fractures
d. have relatively greater alveolar ventilation and
e. tissue solubility of the agent 80. You anesthetize a healthy dog with halothane. b. are marked in different colors to identify the
cardiac output
During anesthesia the dog develops a ventricular manufacturer
e. have a relatively larger larynx
arrhythmia. The most appropriate course of c. have connecting thread sizes and diameters
76. Other constants being equal, the speed of
action is to: that are unique for each gas
induction with an inhalation anesthetic varies

b.c
84. Which of the following is not an appropriate d. do not need to be fixed to a solid object once
inversely with that agent's: a. increase the anesthetic depth action to combat hypotension in a dog inside a building
a. rubber/gas solubility ratio b. attempt defibrillation anesthetized with halothane? e. are lighter than empty cylinders
b. molecular weight c. infuse lidocaine intravenously at 20 to 40 a. decrease the anesthetic concentration
c. blood/gas solubility coefficient flog/kg/min
b. increase the rate of intravenous fluid 89. The oxygen flush valve:
d. MAC d. increase the flow of oxygen administration
e. saturated vapor pressure e. institute intermittent positive-pressure a. allows oxygen to flow into the breathing
c. treat the underlying cause (e.g., hypovolemia)
ventilation system without going through the vaporizer
d. administer an intravenous bolus of epinephrine

tto
b. is part of the vaporizer
77. Which of the following accounts for a patient's e. infuse dopamine or dobutamine
81 . Delivery of inhalation anesthetics from out-of- c. is used to rapidly increase the concentration
rapid awakening after halothane or isoflurane intravenously
circuit precision vaporizers depends on the fresh of anesthetic within the breathing system
anesthesia, as compared with recovery from
methoxyflurane anesthesia? gas flow rate. A 30-kg dog requires 5 ml of d. should only be used in emergencies
halothane per minute to stay anesthetized. If the 85. For which agent does the reading on the pressure e. allows excess pressure to escape from the
a. Liver metabolism and renal excretion occurs fresh gas (O~ flow rate is either 500 or 120 ml! gauge give no indication of quantity ofgas breathing system
rapidly. min, the appropriate vaporizer setting is: remaining in the cylinder?

ko
b. They are relatively insoluble in blood and a. nitrogen
clear rapidly from the lungs. a. 1.5% for both flow rates 90. With a vaporizer located within the circle
b. 1% and 4%, respectively b. oxygen breathing system, which of the following will
c. They have no analgesic properties, so
c. 5% for both flow rates c. helium increase the anesthetic concentration within the
ventilation is stimulated by pain.
d. halothane breathing circuit?
d. They depress ventilation less than d. 0.5% and 5%, respectively
methoxyflurane, and more normal ventilation e. 1% and 2.5%, respectively e. nitrous oxide a. increased tidal volume
does not slow recovery. b. increased body temperature

al
e. They are more potent than methoxyflurane, 82. Why are precision halothane vaporizers installed 86. The pressure in a nitrous oxide cylinder decreases c. increased inspired oxygen concentration
so there is less anesthetic to eliminate. outside the circle system? when: d. decreased humidity in the patient circuit
a. Vaporizers require complicated pressure a. the cylinder is one half full e. decreased rubber to gas solubility of the
78. All the following influence the speed of recovery valves for installation within the circle system.
w. b. the cylinder is one fourth full anesthetic
from inhalation anesthesia except: b. They use less halothane than in-circuit c. all the liquid nitrous oxide has vaporized,
a. duration of anesthesia vaporizers. which occurs when the tank is nearly empty 91. Concerning vaporizers lacated within the breathing
b. potency of the agent c. They increase absorption of carbon dioxide. d. the cylinder is heated circuit, which statement is most accurate?
c pulmonary blood flow d. They are less affected by inspiration and e. the cylinder is initially opened, then the a. They can only be used for anesthetics with
d. alveolar ventilation expiration than in-circuit vaporizers. pressure gradually decreases until the tank is low vapor pressure (e.g., methoxyflurane) .
e. High concentrations of halothane can be empty b. They can only be used for anesthetics with
e. solubility of the agent in body tissues
ww
delivered. high vapor pressure (e.g., halothane) .
87. Which gas is not liquefied by pressure applied at c. They do not alter resistance to gas flow in the
79. Halothane: room temperature? circuit.
a. is eliminated completely unchanged via the d. They must be high-efficiency vaporizers.
a. oxygen
lungs e. As fresh gas flow to the circuit increases, the
b. carbon dioxide
b. is the most potent respiratory depressant of circuit anesthetic concentration decreases.
the commonly used inhalation anesthetics

© 1998 Mosby-Year Book, Inc. Photocopying is prohibited by law. Correct answers are on pages 25-33.

www.alkottob.com
12 SECTION 1 www.vet4arab.co.cc Anesthesiology 13

92. Which of the following ensures unidirectional gas 96. Concerning soda lime and the carbon dioxide 99. To check for gas leaks within the breathing 103. Nonrebreathing anesthetic circuits:
flow in a circle system? absorption canister, which statement is kast circuit ofa circle rebreathing system:
accurate? a. are generally used in animals weighing less
a. unidirectional flow of fresh gas into the circuit a. the circuit must be immersed in a container than 5 kg

om
b. pop-off valve that allows gas flow out of the a. Unused soda lime is white before the patient of water
is connected to the circuit. b. have more resistance than circle systems
circuit b. the gas flow into the circuit must be
b. Soda lime becomes blue or pink after measured c. require lower fresh gas flows than circle
c. resistance by soda lime systems
d. reduced pressure in the patient's lungs prolonged service. c. the rebreathing hoses and bag must be
c. The soda lime canister becomes warm when disconnected d. produce less waste anesthetic gas pollution
e. inspiratory and expiratory one-way valves than circle systems
in service. d. a functional in-circuit pressure gauge is
d. The soda lime granules are easily crushed necessary e. cannot be used for patients requiring
93. Which item is not a component of the breathing between one's fingers when the granules are positive-pressure ventilation
circuit in a circle anesthetic system? e. resistance to collapse of the reservoir bag can
exhausted.

b.c
be assessed manually
a. rebreathing bag e. The soda lime granules are easily crushed 104. A characteristic of the T piece or Norman elbow
b. exhaust (pop-off) valve between one's fingers when the granules are anesthetic delivery system is that it:
unused. 100. A rebreathing circle anesthetic system is
c. oxygen flush valve described as semiclosed when: a. allows economical use of anesthetic gases
d. Ypiece and vapors
97. If the soda lime in the breathing circuit becomes a. the oxygen flowmeter is partly closed
e. inspiratory and expiratory one-way valves b. does not require a soda lime canister
exhausted during an anesthetic procedure, you b. gas flows out through the pop-off valve
c. requires low fresh oxygen flows
would expect to observe: c. the vaporizer is at a low setting
94. If the inspiratory unidirectional valve in a circle d. results in minimal environmental pollution
d. the pop-off valve is closed

tto
system patient breathing circuit is assembled a. only an increased respiratory rate e. can be used for closed -circuit anesthesia
without the valve leaflet in place, what would b. only an increased depth of breathing e. the oxygen flowmeter is completely closed
happen if the machine were put into service? c. an increased heart rate and decreased blood 105. In a Bain's nonrebreathing circuit, the flow of
a. no untoward results because the other pressure 101. The efficiency of carbon dioxide absorption by a fresh oxygen should be:
unidirectional valve would function properly d. an increased heart rate and increased blood soda lime canister depends on the:
pressure a. only sufficient to meet the animal's oxygen
b. anesthetic overdose a. length of the canister requirement
c. increased concentration of carbon dioxide in e. no physiologic changes b. volume of soda lime used

ko
b. approximately 30 mllkg/min
the inspired gas c. amount of water vapor in the expired gases c. sufficient to prevent rebreathing of expired
d. increased resistance to breathing 98. A problem within an anesthetic waste gas d. resistance the soda lime offers to gas flow gases
e. no untoward results because the rebreathing scavenging system that could be fatal to the e. volume and distribution of gas flOwing d. able to fill the reservoir bag
hoses would still be in place patient is: through the canister e. low enough to prevent gas flow out through
a. exhaustion of the activated charcoal canister the pop-off valve
95. The machine dead space ofa circle system (the gas b. occlusion of the tubing from the pop-off valve ·102. In a functional anesthetic circuit:

al
volume that must be rebreathed at each breath to the scavenger
a. unidirectional in-circuit valves ensure 106. When using a nonrebreathing circuit and an
containing carbon dioxide from the previous c. overfilling of the reservoir container with
movement of oxygen from the flowmeter to out-of-circuit vaporizer, the inspired
expired breath) is comprised of the: spillage out of the positive-pressure relief
die preciSion vaporizer concentration of anesthetic:
a. area from the inspiratory valve to the valve
b. the reservoir bag stores fresh anesthetic gas a. changes slowly with vaporizer adjustments
d. disconnection of the tubing from the pop-off
endotracheal tube connection
b. area from the Ypiece connection to the
endotracheal tube
valve
w.
e. occlusion of the vacuum hose from the
and must accommodate the patient's tidal
volume
c. soda lime canisters help cool the warm gas
b. is about 20% to 30% lower than the vaporizer
setting
c. is about 20% to 30% higher than the
c. entire patient breathing circuit reservoir container
expired by the patient vaporizer setting
d. area from the soda lime canister to the d. precision out-of-circuit vaporizers receive d. is the same as the vaporizer setting
endotracheal tube connection gas flow from the reservoir bag
e. depends on the blood/gas solubility ratio of
ww
e. area from the expiratory valve to the e. oxygen tanks and flow meters are in the the anesthetic agent
endotracheal tube connection patient circuit portion of a circle system

© 1998 Mosby-Year Book, Inc. Photocopying is prohibited by law. Correct answers are on pages 25-33.

www.alkottob.com
14 SECTION 1 www.vet4arab.co.cc Anesthesiology 15
107. The minimum fresh oxygen flow required for a 112. Which of the following is nota reliable indication 115. In horses, which of the following indicates a deep c. xylazine
patient rebreathingfrom a circle anesthetic ofsuccessful intubation? plane of anesthesia?
system is: d. tiletamine-zolazepam
a. condensation inside the tube with expiration a. mean arterial pressure of 60 mm Hg, e. propofol

om
a. determined by the size of the reservoir bag b. tube placement confirmed with a respiratory rate of6/min, corneal reflex
b. dependent on the location of the vaporizer laryngoscope present
119. Tachycardia during anesthesia:
c. determined by the patient's respiratory rate c. no tube evident on palpation of the b. mean arterial pressure of 75 mm Hg,
and volume esophagus at the level of the larynx respiratory rate of 4/min, spontaneously a. may be due to hyperkalemia
d. dependent on the volume of the anesthetic d. movement of a piece of lint held at the tube closes eye b. may be due to hypercapnia
circuit end with expiration or inspiration c. mean arterial pressure of 60 mm Hg, c. should be treated with lidocaine
e. determined by the patient's metabolic rate e. chest wall movement on manual respiratory rate of 3/min, periodic d. should be treated with isoproterenol
compression of the reservoir bag nystagmus
e. is a sign of deep anesthesia
d. mean arterial pressure of 76 mm Hg,

b.c
108. A dog's metabolic requirement for oxygen is
respiratory rate of 6/min, brisk palpebral
calculated as: 113. Ifyou were to measure halothane concentration reflex 120. Palpation of the arterial pulse provides an
in a circle system during the first 5 minutes of indication of
a. twice the volume of carbon dioxide produced e. mean arterial pressure of70 mm Hg,
anesthesia, which of the following would be true?
b. 30 mllkg/min x body surface area respiratory rate of 4/min, spontaneously a. systolic arterial pressure
c. 200 mJ/kg/min x body surface area a. Concentrations of expired halothane would closes eye, lacrimation b. mean arterial pressure
be lower than concentrations of inspired
d. kg" 75 x 10 mJ/min c. systolic and diastolic pressures
halothane.
e. kg"·5 x 5 mil min 116. An esophageal stethoscope: d. the difference between systolic and diastolic
b. Concentrations of expired halothane would
pressures

tto
be higher than concentrations of inspired a. should be placed just cranial to the thoracic
109. Which breathing system could be used for halothane. inlet e. the difference between systolic pressure and
closed-system/low-flowanesthesia? c. Concentrations of inspired halothane would b. is useful for assessing cardiac function no blood pressure
a. circle rebreathing system be higher than concentrations of halothane c. should be positioned to auscultate the
in the vaporizer. loudest heart and lung sounds 121. Arterial blood pressure:
b. McGill breathing system
d. Concentrations of expired halothane would d. requires considerable technical training for
c. Ayres T piece a. reflects only the amount of vasoconstriction
be equal to concentrations of inspired competent use
d. Norman elbow present

ko
halothane. e. should be placed just cranial to the
e. Bain's breathing circuit b. may appear spuriously high if an overly wide
e. Concentrations of inspired halothane would esophageal sphincter cuff is used for indirect measurement
be lower than concentrations of expired
halothane. c. is more accurately measured using the
110. Which of the following is not an advantage ofa
.117. During ovariectomy in a dog anesthetized with Doppler indirect method than using an
closed anesthetic system?
halothane, the heart rate drops from 120 to 50 arterial catheter and pressure transducer
a. economical use of anesthetic agents 114. Which of the following values is least likely to beats/min. Which of the following is most d. depends on cardiac output, total peripheral

al
b. high humidity in the inspired gas change when the depth ofanesthesia is changed? appropriate for treatment ofbradycardia? resistance, and circulating blood volume
c. slow changes in anesthetic concentration a. blood pressure a. ketamine intravenously at 6 to 10 mg/kg e. indicates the adequacy of tissue perfusion
d. conservation of body heat b. expired concentration of carbon dioxide b. atropine intravenously at 0.01 to 0.02 mg/kg
e. reduced anesthetic pollution c. jaw muscle tone c. epinephrine intravenously at 0.5 to 1.0 mg/kg 122. Which of the following is not used to detect the
d. oxygen saturation of lingual tissues (pulse
111. Which of the following is a disadvantage of
tracheal intubation?
oximetry)
e. minute ventilation
w. d.
e.
dopamine intravenously at 5 to 10 fJ.g/kg/ min
isoproterenol intravenously at 0.5 to 1.0 mg/kg
arterial pulse for indirect determination of blood
pressure?
a. pulse oximetry
a. Connects the patient's airway with the 118. In cats a heart rate of30 beats/min is considered b. Doppler ultrasound
anesthetic circuit. dangerously low. When used at recommended c. oscillometry
b. Prevents dilution of inspired anesthetic gases dosages, the drug most likely to cause d. palpation
ww
with room air. bradycardia is: e. auscultation
c. May decrease anatomic dead space. a. thiopental
d. May increase mechanical dead space. b. ketamine
e. Facilitates positive-pressure ventilation.

© 1998 Mosby-Year Book, Inc. Photocopying is prohibited by law. Correct answers are on pages 25-33.

www.alkottob.com
SECTION 1
www.vet4arab.co.cc Anesthesiology 17
16

127. Mean arterial pressure is: 131. A vasoconstrictor drug that exerts little or no d. Vt 160 ml, respiratory rate of IOlmin
123. Indirect blood pressure measurement techniques inotropic effect is: e. Vt 250 mI, respiratory rate of 151min
use a pneumatic cuffand a method to detect a. equal to the pulse pressure
pressure changes, sound, or blood flow. b. equal to diastolic pressure plus one third of a. epinephrine

om
Compared to direct measurements of blood the difference between systolic and diastolic b. ephedrine 137. An average tidal volume for a dog at a light
pressure via an arterial catheter, indirect pressures c. isoproterenol anesthetic level is:
methods: c. the difference between the average systolic d. phenylephrine a. 200 mllkglbreath
a. are just as reliable and automated and diastolic pressures e. dopamine b. 12 mllkglbreath
b. accurately measure both systolic and d. not as important as systolic pressure c. 30 mllkglbreath
diastolic pressures e. a measure of perfusion 132. In small doses dopamine: d. kg"·75 x 10 mil breath
c. accurately measure mean arterial pressure e. 5 mllkglbreath
a. dilates renal arteries
d. are not influenced by the size of the cuff or 128. An anesthetized animal would be considered

b.c
b. stimulates a-receptors
the animal's limb hypotensive if the:
c. increases the heart rate 138. The best indicator of adequacy of ventilation in
e. are influenced by the animal's position anesthetized animals is:
a. systolic arterial pressure were below 120 mm d. increases the respiratory rate
Hg e. causes bradycardia a. respiratory rate
124. When indirectly measuring blood pressure: b. systolic arterial pressure were below 100 mm b. mucous membrane color
a. a loose cuff yields more accurate readings Hg
133. When administered for hemostasis in an animal c. Paco2
than a snugly applied cuff c. diastolic arterial pressure were below 80 mm
anesthetized with halothane, which drug is most d. pH of arterial blood
b. the cuff should be placed distal to the site of Hg
likely to cause premature ventricular contractions? e. tidal volume
d. diastolic arterial pressure were below 60 mm

tto
pulse palpation or auscultation
c. the cuff width should be 30% to 40% of limb Hg a. epinephrine
circumference e. mean arterial pressure were below 60 mm Hg b. phenylephrine 139. The effect of increased arterial carbon dioxide levels
c. dopamine in an otherwise normal anesthetized animal is:
d. a loose cuff yields spuriously low readings
e. an overly narrow cuff yields spuriously low 129. Appropriate treatment for hypotension during d. dobutamine a. a decrease in respiratory rate
readings inhalation anesthesia includes any of the e. ephedrine b. a decrease in respiratory depth
following except: c. a sudden decrease in blood pressure

ko
125. In a laterally recumbent anesthetized dog, a. administration of a vasopressor (e.g., 134. The objective ofadministering a positive inotrope d. a sudden decrease in heart rate
systolic arterial pressure is highest in the: dobutarnine) to a hypotensive anesthetized animal is to: e. injected mucous membranes
b. decreasing the concentration of inspired a. increase the blood pressure
a. aorta
anesthetic b. increase the heart rate 140. Lung compliance is a measure of:
b. carotid artery
c. intravenous administration of boluses of
c. renal artery c. increase tissue perfusion a. the resistance to lung inflation by manual
fluid at 10 to 20 mllkg
d. decrease hemoglobin saturation with oxygen compression of the reservoir bag

al
d. femoral artery d. administration of a stimulant (e.g.,
e. cranial tibial artery e. decrease production of carbon dioxide b. the amount of pressure required to inflate
doxapram)
e. administration of a j3-adrenergic agonist the lung
126. If a transducer used to measure the blood (e.g., dobutarnine) 135. Anatomic dead space: c. the difference in pressure between the
pressure (e.g., Doppler crystal, dir~ct pre~sure ventilator and the rebreathing circuit
a. cannot change during anesthesia
transducer, aneroid manometer) IS applied to an
animal and located lower than the level of the
heart:
130. Cardiac output:
w.
a. is the product of heart rate X arterial blood
b.
c.
usually is greater than the tidal volume
can increase during anesthesia
d. the change in lung volume for each unit of
transpulmonary pressure change
e. end-inspiratory pressure indicated on the
pressure d. is the same as mechanical dead space pressure gauge located in the breathing circuit
a. the systolic and diastolic pressure readings
b. is increased by most general anesthetics e. decreases as the duration of anesthesia
will be spuriously high _. increases
b. the systolic and diastolic pressure readings c. is increased by most inhalation anesthetics 141. Carbon dioxide acts on the heart by causing:
ww
will be spuriously low d. may be monitored using the Doppler unit
a. decreased cardiac output
c. the systolic and diastolic pressure readings placed on a front or hind limb 136. Which tidal volume (VJ and respiratory rate
produce the most ventilation? b. increased cardiac output
will be accurate e. may decrease with bradycardia
c. decreased ventricular filling in diastole
d. only the diastolic pressure reading will be a. Vt 160 mI, respiratory rate of 15/min
d. increased venous return
spuriously high b. Vt 250 mI, respiratory rate of 12/min
e. decreased afterload
e. only the diastolic pressure reading will be c. Vt 400 mI, respiratory rate of 8/ min
spuriously low

Correct answers are on pages 25-33.


© 1998 Mosby-Year Boo/;;, Inc. Photocopying is prohibited by law.

www.alkottob.com
18 SECTION 1 www.vet4arab.co.cc Anesthesiology 19
142. An appropriate treatment to combat 147. Which arterial oxygen tension (Paaz) would
hypercapnia in anesthetized small animals is: markedly alter the arterial oxygen content?
E.M. Wertz
151. In dogs anatomic landmarks used to identifY the 154. Concerning nitrous oxide (NOz). which statem ent
a. ephedrine to stimulate ventilation a.560mmHg epidural space are the:

om
is most accurate?
b. dobutamine to increase the heart rate b.300mmHg
a. dorsal spinous process of L7 and the dorsal a. N0 2 disassociates to produce O2 and N.
c. increased flow rate of oxygen c. 200mrnHg
aspect of the ilial wings eliminating the need for simultaneous O2
d. decreased anesthetic depth d.140mmHg
b. dorsal spinous processes of the sacrum and administration.
e. addition of nitrous oxide to the inspired gas e.70mmHg the iliac crests b. N0 2 is a potent anesthetic (MAC =
c. dorsal depression on the midline cranial to approximately 200%) in dogs.
143. In a nonanesthetized animal ventilating 148. Which of the following is least likely to increase the dorsal spinous process of L5 c. The high potency of N0 2 is related to its low
normally. you would expect the Pacoz to be: the tissue oxygen delivery in an anesthetized d. dorsal spinal processes of L5-6 and the solubility.
horse breathing 90% oxygen and with a Paaz of proximal aspect of both femurs
a. approximately 25 mrn Hg

b.c
d. The rapid onset of action of N02 is caused by
65 mm Hg and a Pacaz of 45 mm Hg? e. first obvious midline depression caudal to
b. approximately 40 mrn Hg its low blood:gas partition coefficient (0.47) .
a. a blood transfusion to raise the hemoglobin the sacrum
c. approximately 55 mm Hg e. N0 2 is easily scavenged using an activated
concentration by 20% charcoal absorbent scavenging system.
d. twice the value of Pao2
b. intravenous dobutamine infusion at 3 152. In an attempt to inject bupivacaine epidurally.
e. half the value of Pao 2
",g/kg/min the needle enters the subarachnoid space. What 155. Several scavenging methods are employed to
c. positive-pressure ventilation to reduce the is the most appropriate course ofaction? minimize occupational exposure to inhalant
144. An increase in arterial carbon dioxide levels Paco2 to 40 mrn Hg
a. Only half of the calculated dose should be anesthetic agents (halogenated hydrocarbons).
during anesthesia in an otherwise healthy animal: d: a reduction in the alveolar halothane level administered. Which statement most accurately describes an

tto
a. decreases the plasma bicarbonate level from 1.8 to 1.1 vol% acceptable method ofexhaust ofanesthetic vapors?
b. The entire calculated dose can be
below 20 mrnollL e. intravenous administration of a bolus of fluid administered without complication. a. The exhaust hose coming from the
b. increases the base excess above 3 mEq / L at 20 mllkg and ephedrine at 0.1 mg/kg
c. A volume of cerebrospinal fluid equal to the anesthetic machine is connected to the
c. has no effect on blood pH volume ofbupivacaine to be injected should be recirculating air flow system of the building.
d. increases the blood pH 149. An anesthetized dog breathing 33% oxygen has a withdrawn and then the bupivacaine injected. b. The exhaust hose coming from the
e. decreases the blood pH Pacoz of39 mm Hg. a Paaz of 43 mm Hg. and a d. Two times the calculated dose should be anesthetic machine is connected to a

ko
blood pH of 7.39. What is the most appropfiate administered. canister containing activated charcoal.
course of action? e. The needle should be withdrawn and no c. The exhaust hose coming from the anesthetic
145. Oxygenation of tissues depends on the:
a. increase blood carbon dioxide levels to further attempt made to administer machine is positioned so it ends just above
a. oxygen-carrying capacity of the blood create greater ventilatory drive bupivacaine epidurally. the level of the floor. dropping the vapors just
b. saturation of hemoglobin b. administer sodium bicarbonate at ground level.
c. oxygen content of arterial blood and blood intravenously 153. In an attempt to inject morphine epidurally. the d. The exhaust hose coming from the anesthetic
flow c. increase the flow rate of oxygen needle enters the subarachnoid space. What is machine is connected to a floor drain.

al
d. body temperature d. replace the soda lime in the canister the most appropriate course ofaction? e. The pop-off valve is kept closed during
e. arterial oxygen level anesthesia to prevent escape of anesthetic
e. continue with the procedure. since these a. Only half of the calculated dose should be vapors into the room.
values indicate no need for changes in the administered.
146. The hemoglobin-oxygen dissociation curve is anesthetic protocol w. b. The entire calculated dose can be
shifted to the left by: administered without complication. 156. Propofol is classified as:

a. decreased blood pH 150. Which of the following is notan effect of carbon c. A volume of cerebrospinal fluid equal to the a. a phenothiazine tranquilizer
b. decreased body temperature dioxide on the brain? volume of morphine to be injected should be b. an <X2-agonist
a. increases cerebral blood flow withdrawn and then the morphine injected. c. a sedative-hypnotic
c. increased blood 2.3-diphosphoglycerate levels
d. Two times the calculated dose should be d. an anticholinergic
d. increased arterial carbon dioxide levels b. decreases neuronal activity
administered.
e. increased hemoglobin levels c. increases cerebrospinal fluid pressure e. an opioid agonist
ww
e. The needle should be withdrawn and no
d. causes an inert gas-like narcotic effect further attempt made to administer
e. decreases cerebral blood pressure morphine epidurally.

© 1998 Mosby-Year Book. Inc. Photocopying is prohibited by lLlw. Correct answers are on pages 25-33.

www.alkottob.com
SECTION 1
www.vet4arab.co.cc Anesthesiology 21
20

158. The duration of analgesia produced by 165. Which instrument is used to measure a patient's 170. Which species is most susceptible to the effects of
157. In dogs hypotension and apnea may occur on
morphine administered epidurally is: tidal volume? xylazine?
induction of general anesthesia with propofol.
These effects can be minimized by: a. 20 to 40 minutes a. capnometer a. cats

om
a. slow intravenous injection (over 30 to 60 b. 45 to 60 minutes b. pulse oximeter b. rabbits
seconds). given to effect c. 4 to 6 hours c. Wright's respirometer c. horses
b. administration of half of the calculated dose, d. 16 to 24 hours d. Doppler d. swine
rapid, followed by slow infusion of the e. 48 to 72 hours e. ventilator e. cattle
remainder
c. no sedation or tranquilization before 166. During an anesthetic procedure, the pulse 171. In a dog breathing room air at sea level, the
159. After epidural administration ofmorphine, the onset
propofol administration ofanalgesia becomes evident in approximately: . oximeter reads 91%. Which partial pressure of expected partial pressure of oxygen would be:
d. administration of the calculated dose oxygen would not correlate with this reading?

b.c
intramuscularly and waiting 5 minutes for a. 15 to 30 seconds a.55t060mmHg
onset of the full effect b. 5 to 10 minutes a. 91 mmHg b.65t070mmHg
e. heavy premedication with morphine before c. 15 to 30 minutes b.150mmHg c. 75 to 80 mm Hg
induction d. 1 hour c.300mmHg d. 85t090mmHg
e. 2 to 3 hours d.52mmHg e. 95 to 100 mm Hg
e.200mmHg
172. In a dog breathing 100% oxygen at sea level, the
Questions 160 through 162 167. Which of the following is not an agonist: expected partial pressure of oxygen would be:

tto
Sevoflurane Desflurane Isoflurane Halothane Methoxyflurane antagonist pair?
a.60t065mmHg
664 240 243 23
Vapor pressure (mm Hg) at 20' C 160 a. morphine and naloxone b.80t085mmHg
None Yes Yes
Preservatives Yes None b. butorphanol and yohimbine c. >llOmmHg
Stable Decomposes Decomposes
Stability in soda Hme No(?) Yes c. detomidine and tolazoline
91 224 970 d. 100 to 105 mm Hg
Oil:gas partition coefficient 47 18.7
1.4 2.4 13.0 d. medetomidine and atipamezole e.90t095mmHg
Blood:gas partition coefficient 0.68 0.42
1.5 0.87 0.23 e. diazepam and flumazenil
MAC (dogs) 2.6 Approx.6.0

ko
160. Which anesthetic has the most rapid onset of 163. Most precision vaporizers are temperature and 173. A packed cell volume of36% most closely
action? flow compensated. With which vaporizer must 168. Which drug is administered to partially approximates a hemoglobin content of
you manually compensate for fresh gas flow and antagonize the effects of morphine while
a. 6 g/dl
a. halothane temperature? maintaining some analgesia?
b. isoflurane b. 9 g/dl
a. Ohio#8 a. flumazenil c. 12 g/dl
c. sevoflurane
b. Vapor b. yohimbine d. 18 g/dl

al
d. desflurane
c. "Tec" types c. butorphanol e. 36 g/dl
e. methoxyflurane
d. Copper kettle d. naloxone
e. "Matic" types e. acepromazine
161. Which anesthetic is the most potent? 174. Which packed cell volume indicates the lowest
acceptable oxygen-carrying capacity?
a.
b.
c.
desflurane
sevoflurane
isoflurane based on:
w.
164. When an anesthetized patient is ventilated by
wind, the volume of each breath delivered is
169. Which species is least susceptible to the effects of
xylazine?
a. 12%
b.21%
a. cats c. 18%
d. halothane a. the size ofthe rebreathing bag attached to b. rabbits d. 30%
e. methoxyflurane the anesthetic machine c. horses e.35%
b. the patient's minute ventilation d. swine
ww
] 62. Which anesthetic can be safely administered c. the patient's tidal volume e. cattle
using a nonprecision vaporizer? d. a desired peak airway pressure of 40 to 50 cm
of water
a. desflurane
e. expansion of the patient's chest wall on
b. isoflurane
observing the patient
c. methoxyflurane
d. halothane
e. sevoflurane
© 1998 Mosby-Year Book, Inc. Photocopying is prohibited by law. Correct answers are on pages 25-33.

www.alkottob.com
22 SECTION 1
www.vet4arab.co.cc Anesthesiology 23
175. Which of the following is the least appropriate d. determine appropriate preanesthetic 183. What is the best preanesthetic to manage this case? c. procaine and bupivacaine
treatment for a mean blood pressure of60 mm medications for the patient
Hg? e. choose an appropriately sized a. oxymorphone and atracurium d. mepivacaine and chloroprocaine

om
intramedullary pin for fracture fixation b. oxymorphone and acepromazine e. procaine and lidocaine
a. Decrease the percent of inhalation anesthetic
being administered. c. ketamine and diazepam
b. Administer fluids intravenously. 180. Concerning use of acepromazine as a d. atropine and acepromazine 189. Which local anesthetic has the longest duration
c. Administer dobutamine. preanesthetic agent, which statement is least e. No preanesthetic should be used. of effect?
d. Administer ephedrine. accurate? a. procaine
e. Begin intermittent positive-pressure a. Acepromazine decreases the amount of 184. Which anesthetic would be the best for b. lidocaine
ventilation. anesthetic required. maintenance ofanesthesia? c. mepivacaine
b. Acepromazine decreases the incidence of a. halothane in oxygen d. chloroprocaine

b.c
176. Which neuromuscular relaxant produces vomiting. b. methoxyflurane in oxygen e. bupivacaine
depolarization at the motor end-plate, causing c. Acepromazine smoothes the recovery period, c. intravenous bolus of pentobarbital
muscle fasciculations? minimizing paddling and vocalization.
d. isoflurane in oxygen 190. Which local anesthetic is ineffective when
d. Acepromazine increases the amount of agent
a. propofol e. constant intravenous infusion of thiopental applied topically?
necessary for anesthetic induction.
b. pancuronium a. procaine
e. Acepromazine decreases the incidence of
c. atracurium epinephrine-induced arrhythmias. 185. How would ventilation best be managed? b. hexylcaine
d. succinylcholine c. lidocaine
a. intermittent positive-pressure ventilation
e. midazolam

tto
181. Concerning propofol, which statement is least b. continuous positive airway pressure d. dibucaine
accurate? c. positive end-expiratory pressure e. tetracaine
177. Which neuromuscular relaxant cannot be d. spontaneous ventilation
a. It is administered intravenously or
effectively antagonized?
intramuscularly to induce anesthesia. e. assisted ventilation Questions 191 through 196
a. gallamine b. It induces apnea when administered
~~ a 40-kg German shepherd with osteosarcoma ofthe
b. pancuronium intravenously. 186. Which method of noninvasive monitoring is zllum, hemzpelvectomy is selected as the treatment.
c. atracurium c. It induces hypotension when rapidly

ko
necessary in this patient? Preoperative blood studies are normal. Morphine and
d. succinylcholine administered intravenously. atropine are given as premedicants. Anesthesia is
d. Because the vial contains no preservatives, a. indirect blood pressure measurement
e. vecuronium induced with intravenous thiopental to effect and main-
any unused portion should be discarded. b. pulse oximetry tained with isoflurane in oxygen. A morphine epidural is
e. It does not cause tissue sloughing following c. tidal volume measurement administered to help manage pain during and after
178. An E cylinder of oxygen contains approximately d. electrocardiography surgery. Patient monitoring includes electrocardio-
inadvertent perivascular administration.
700 L of oxygen when full. Approximately how
e. capnometry graphy. capnometry. pulse oximetry. and direct blood
many liters of oxygen is contained in an E

al
pressure measurement. Systolic blood pressure is initially
cylinder when the pressure is 1100 psi? Questions 182 through 186 100 mm Hg and diastolic pressure is 55 mm Hg. However,
a. 350L 187. Local anesthetics exert their effect by: as surgery progresses, the systolic pressure drops to 70 mm
A depressed, 15-year-old golden retriever has a head
b. 700L tilt to the right. The complete blood count and serum a. interruption of the Na-Kpump, inhibiting Hg and diastolic pressure to 30 mm Hg; blood pressure
c. 175 L chemistry results are within normal limits. To confirm any ion exchange continues to fall. A large pool ofbloody fluid on the floor
d. More information is necessary to determine
the volume of gas remaining.
w.
your suspicions of an intracranial mass, computerized
tomography (CT scan) is necessary. The procedure
b. occupation and blockade of muscarinic
cholinoreceptors
under the surgery table is rapidly increasing in size.

191. What is the mean arterial blood pressure initially?


requires general anesthesia. c. prevention of rapid influx of Na into nerve
e. 525 L
axons, preventing the action potential a. 70mmHg
182. Which of the following poses the greatest danger d. inactivating slow Ca channels b.78mmHg
179. The pin index safety system is used to: during anesthesia?
e. occupation and inhibition of nicotinic c.85mmHg
ww
a. prevent overfilling of the anesthetic vaporizer a. hepatic dysfunction cholinoreceptors d. it cannot be determined from the
b. choose an appropriately sized endotracheal b. kidney dysfunction information given
tube for the patient c. hypotension e. lOOmmHg
188. Which pair of drugs are "amide" types of local
c. prevent interchange of oxygen and nitrous d. hypertension anesthetics?
oxide E cylinders at the yoke hookup e. hypercapnia a. procaine and mepivacaine
b. lidocaine and bupivacaine

© 1998 Mosby-Year Book, Inc. Photocopying is prohibited by law.


Correct answers are on pages 25-33.
www.alkottob.com
SECTION 1
www.vet4arab.co.cc Anesthesiology 25
24

192. At what mean arterial blood pressure would you 197. In a patient with which of the following blood
values would it be appropriate to administer
Answers
begin to suspect inadequate perfusion of vital
organs? only a colloidal solution (e.g., plasma)? 1. d Premedication agents, such as atropine, are Atracurium is a non depolarizing blocker and

m
a. packed cell volume 30%, total plasma protein used to minimize undesirable reflex autonomic causes relaxation alone. Some muscle groups are
a. BOmmHg activity. more susceptible (e.g., extraocular and limb
concentration 6.S g/dl
b.70mmHg 2. d Atropine acts by competitive antagonism of muscles) than others (e.g., facial muscles,
b. packed cell volume 20%, total plasma protein
c.60mmHg acetylcholine at muscarinic receptors. This diaphragm), so different doses are required to
concentration 7.0 g/ dl
d. SOmmHg competition can be overcome by increasing the achieve equivalent relaxation.
c. packed cell volume 40%, total plasma protein

.co
e. The mean arterial pressure does not indicate concentration of acetylcholine at receptor sites 16. a Neostigmine inhibits acetylcholine esterase,
concentration 2.S g/dl
this problem. (as seen following administration of causing increased levels of acetylcholine at such
d. packed cell volume 3S%, total plasma protein anticholinesterases). receptors. This increases preganglionic
concentration 4.S g/dl muscarinic activity, slowing the heart rate.
193. What is the mean arterial blood pressure after 3. a Atropine stimulates the vagus center in the
e. packed cell volume 4S%, total plasma protein brain, which can slow the heart rate initially in Atropine (0.04 mg/kg IV) should be given
the decrease in blood pressure is observed? concentration 7.0 g/ dl response to IV administration of small doses. immediately before neostigmine.
a. SOmmHg 4. b Glycopyrrolate causes less of an increase in 17. d These drugs are classified as " z-adrenergic
b.66mmHg 198. A 45-kg dog is seen after acute loss ofblood, and heart rate than atropine. agonists.
you decide to administer intravenous fluids as .

ob
c.70mmHg S. e A 10% solution = 10 g/100 ml = 10,000 mgllOO lB. b "z.agonists cause second -degree
d.43mmHg treatment. The dog's packed cell volume is 35%, mI = 100 mg/ml. ~trioventricular block and bradycardia, reSUlting

e. It cannot be determined from the information and total plasma protein concentration is 4.2 g/dl. 6. b Phenothiazine tranquilizers are sympatholytic III decreased cardiac output. Hyperglycemia,
Which of the following is most appropriate as the (cause vasodilation). increased urine output, and increased
given.
initial fluid therapy to prevent vascular colfapse? intrauterine pressure also occur. Respiratory
7. a Acepromazine is one of the most commonly
a. 2BOO mI of 0.9% saline solution infused IV function is depressed because of sedation.
194. What is the estimated blood volume of this dog? used phenothiazines.
over 4 hours 19. c Intravenous atropine rapidly antagonizes the
B. c Acepromazine can cause persistent
xylazine-induced bradycardia. An " z-adrenergic

ott
a. 2400 mI b. 4S0 mI of lactated Ringer's solution injected paraphimosis or priapism in adult intact male
b. 3200ml antagonist, such as yohimbine, would only
SC at multiple sites horses. Working stallions are at higher risk for
partially antagonize the bradycardia and
c. 2BOO mI c. 1BO mI of 7% saline solution infused IV over these problems.
decrease the sedative-analgesic (desirable)
d. 1600ml 10 minutes 9. c Guaifenesin causes muscle relaxation by its effects of xylazine.
e. 2000 mI d. 2S0 mI of plasma infused IV over 30 minutes action on internuncial neurons in the spinal
20. c Pure opioid agonists exert their effects via
e. 4S0 mI of whole blood infused IV over 1 hour cord. It has no recognized analgesic properties
agonism of fJ.-opiate receptors.
and so should not be used as a sole agent for
195. Intraoperatively, blood should be administered 21. d Assuming the potency of morphine (mg/kg

alk
restraint for surgery.
when the packed cell volume falls below: 199. Midazolam is classified in the same category as: basis) is 1, the potency offentanyl is 100 x,
10. a Diazepam is a centrally acting muscle relaxant.
oxymorphone 10 x , butorphanol 2 x , and
a.4S% a. diazepam 11. d Overdosage of guaifenesin is associated with meperidine O.S X to O.2S x.
b. 40% b. atipamezole skeletal muscle rigidity and often cardiac arrest
22. b Naloxone (Narcan), a pure opioid antagonist,
c.3S% c. mepivacaine preceding respiratory arrest by several minutes.
has no narcotic/ agonistic effects of its own.
d.30% The therapeutic dosage is 100 mg/kg.
d. neostigmine 23. c Opioid fJ.-agonists generally cause respiratory
e.2S% 12. c 3S0 mI x SO mg/mI = 17,SOO mg/2S0 kg = 70
e. propofol depression, especially after intravenous or
mg/kg.
repeated intramuscular administration.
196. Approximately how many milliliters of red blood
cells have been lost when this critical packed cell
200. Medetomidine is classified as:
a. a hypnotic
w. 13. e Atracurium blocks postsynaptic acetylcholine
receptors at the neuromuscular junction and so
causes paralysis of the intercostal muscles and
24. b Opiates alter the perception of pain by the
central nervous system. Human patients may
volume is reached? still be aware of the pain without being in pain.
b. an amide type of local anesthetic diaphragm.
2S. b In dogs either oxymorphone or fentanyl is
a. 160 mI 14. b Atracurium's molecular stability depends on
c. a tranquilizer commonly given to allow intubation. The drug is
b.320ml an alkaline pH and low temperature. This
d. an "z agonist usually administered in combination with
ww
c. 4BO mI method of limiting the drug's action is known as
e. an opioid antagonist diazepam. The induction is slow (2 to S minutes)
Hoffman elimination.
d.640ml and causes respiratory depression. Therefore it
IS. c Succinylcholine is a depolarizing should be preceded by S minutes of
e. BOO mI
neuromuscular blocking agent that initially preoxygenation via face mask.
causes rigidity, tachycardia, and hypertension. It
26. d Apart from bradycardia, there is minimal
has a short duration of action because it is
cardiac depression from intravenous
metabolized by plasma cholinesterase.
oxymorphone and fentanyl.

© 1998 Mosby-Year Book, Inc. Photocopying is prohibited by law.

www.alkottob.com
SECTION 1
www.vet4arab.co.cc Anesthesiology 27
26

37. d Similar to thiobarbiturates, propofol depresses 50. c The epidural space is located immediately 63. d The anesthetic requirement is reduced by
27. a Nalorphine and naloxone are opioid outside the dura mater. increasing age, hypothermia, concurrent use of
antagonists and butorphanol is a partial seizure activity, decreases intraocular pressure,
and causes muscle relaxation. 51. b An appropriate dose of epidural local other central nervous system depressants and
antagonist; therefore, all would combat the

om
anesthetic produces excellent analgesia and hind pregnancy.
bradycardia. Atropine would also combat 38. c Hypotension can be observed following
administration of propofol in healthy patients. limb muscle paralysislrelaxation. 64. d MAC values for these anesthetics in dogs are
bradycardia without antagonizing the desirable
This effect is exacerbated by dehydration. 52. a Bupivacaine produces both sensory and motor 0.9 vol % for halothane, 215 vol % for nitrous
sedative or analgesic/ properties of fentanyl.
Propofol does not directly alter the heart rate or blockade of 2 to 3 times longer duration than oxide, and 1.4 vol % for isoflurane, versus 0.23 vol
28. c Butorphanol is a mixed (K) agonist and ("') % for methoxyflurane. Propofol is not an
myocardial contractility. lidocaine or mepivacaine.
antagonist and so is associated with less inhalation agent.
respiratory depression. 39. a Typically, propofol does not cause excitement 53. d Epidural anesthetics should not be
in recovery. Patients are quickly ambulatory and administered when there is trauma, bleeding, or 65. c Mask inductions are slow (undesirable if rapid
29. c Opioids can cause central nervous system airway control is desirable) , with speed of
not ataxic. In part, recovery is due to rapid infection at the injection site.
stimulation in horses, especially if administered induction dependent on drug solubility (insoluble
metaboHsm. 54. a Preganglionic sympathetic blockade causes

.c
intravenously and without other tranquilizers/ agents produce faster induction) and ventilation
sedatives. 40. c In the commercially available product, vasodilation in the anesthetized region.
(hyperventilation produces faster induction).
30. b Naloxone is a ",-receptor antagonist that can propofol is dissolved in a nonirritant milky oil 55. e Local anesthetic analgesia persists
Mask induction can be expensive, especially with
reverse respiratory depression. emulsion and can be given intramuscularly approximately as long as muscle
isoflurane (about $5 for induction in a dog) .
(requires large volume) as well as intravenously. paralysis/relaxation. The longest duration of
31. c In healthy dogs there is a high incidence (up to 66. d Highly soluble agents (e.g., methoxyflurane)
It is stable at room temperature but contains no effect is from bupivacaine, whlch lasts 4 to 6

ob
40% of cases) of transient ventricular arrhythmias have slow equilibration times and therefore slow
preservative agents. ' hours.
following anesthetic induction with intravenous induction and recovery times.
thiobarbiturates. Thls does not usually cause 41. d Ketamine can be administered intra- • 56. b Bupivacaine has the lowest placental transfer
muscularly, although this causes pain. of any of the commonly used local anesthetic 67. c About 50% of an inhaled dose of methoxyflurane '
clinical problems in healthy patients. and 25% of a dose of halothane are metabolized.
42. a Ketamine stimulates the cardiovascular agents. This is because ofits high protein
32. b Thiobarbiturates owe their short duration of Other agents have minimal metaboHsm.
system, increasing myocardial oxygen binding in maternal blood.
action to almost immediate distribution from the 68. d In decreasing order of the likelihood of
plasma to well-perfused tissue, and then to fat consumption, and also increases intraocular and 57. d If the same dose as required for epidural
arrhythmogenesis: halothane> methoxyflurane

ott
depots. Therefore there is rapid recovery of intracranial pressures. anesthesia is administered into the cerebrospinal
fluid, the anesthetic moves far cranially and > isoflurane, desflurane, and nitrous oxide.
consciousness after a single anesthetic dose. 43. d Rapid induction of anesthesia and
maintenance of laryngeal function are desirable could cause respiratory muscle paralysis. 69. c Dose-dependent ventilatory and cardiovascular
33. a Thiobarbiturates can produce prolonged depression occurs. Respiratory depression
recovery because of saturation of muscle and fat in brachycephalic dogs. In addition, rapid 58. a In spinal anesthesia the anesthetic agent is
recovery is desirable. Although not producing the administered into the cerebrospinal fluid, whlch increases arterial carbon dioxide tension.
storage depots and their subsequent release and
fastest induction and smoothest recovery circulates in the subarachnoid space. 70. c The MAC of nitrous oxide in dogs is
slow metabolism and elimination. Hypo-
(thiopental or propofol would be better), 59. b Giving the same dose as a spinal injection that approximately 215 vol %, so under normal
proteinemia, dehydration, acidosis and
ketamine would be acceptable. conditions (1 atmosphere of pressure), it cannot

alk
hypothermia all exacerbate this problem. was calculated for epidural administration would
44. c Ketamine is a dissociative agent. produce excellent analgesia and incur no more alone produce anesthesia.
34. d Thiobarbiturates decrease intracranial and
45. d Sympathomimetics (e.g., dopamine, risk of adverse effects (as compared with 71. b Nitrous oxide is the most insoluble of the
intraocular pressure and usually cause no
dobutamine) are usually administered to epidural administration). This is not the commonly used inhalation agents.
change or an increased heart rate.
stimulate cardiovascular function. situation for local anesthetics administered into 72. d 1.3 to 1.5 times the MAC (0.23 vol %) produces
35. d Apnea is commonly seen following thiopental
46. b Hypnotic agents include pentobarbital, the cerebrospinal fluid. a surgical plane of anesthesia.
induction of anesthesia, especially with a hlgh
inspired oxygen level. Giving 2 or 3 normal breaths thiopental, thiamylal, propofol, and chloral 60. e Epidural drugs are also occasionally 73. c Fluoride ions produced by metabolism of
per minute ensures that the patient remains hydrate. Tiletamine is the dissociative agent in administered into the cerebrospinal fluid and so methoxyflurane are nephrotoxic.
should be preservative free. Although the volume
oxygenated and receives some halothane; it also
allows the arterial carbon dioxide to rise to a level
sufficient to stimulate spontaneous ventilation. If
Telazol (tiletamine-zolazepam).
w.
47. e A neuroleptanalgesic combination usually
consists of a sedative/tranquilizer (e.g.,
of drug administered epidurally is important
(hence the different concentrations), this is not the
only reason that the solution is packaged in a vial.
74. c When nitrous oxide is given with halothane,
the peak blood concentration of halothane is
attained more rapidly than if halothane were
no halothane were administered, this dog would acepromazine) and an opioid analgesic (e.g., used alone. Concurrent use of nitrous oxide
be awake in 10 to 15 minutes. Propofol also causes fentanyl). 61. b Epidural morphine reduces the anesthetic causes halothane to be taken up at an
postinduction apnea. 48. a Ketamine produces mild visceral analgesia in requirement to produce analgesia in the hind accelerated rate, known as the second-gas effect.
36. e Thiobarbiturates cause tissue irritation and limbs, and also to a lesser extent in the forelimbs. Answer b describes diffusion hypoxia.
people.
ww
sloughing after perivascular injection and are 49. e The kidney is the major excretory organ for 62. a Isoflurane causes respiratory depression in
associated with prolonged recovery in almost all sedative and hypnotic drugs. Because dogs and horses, is relatively less arrhythmogenic,
dehydrated, emaciated, hypoproteinemic they have high lipid solubility and are poorly and is excreted virtually unchanged. It is one sixth
animals. They produce relatively good muscle ionized, they are metabolized by the liver before as potent as methoxyflurane and two thirds as
relaxation at anesthetic doses. excretion. Ketamine is excreted without potent as halothane.
metabolism via the kidney in dogs and cats.

© 1998 Mosby-Year Book, Inc. Photocopying is prohibited by law.


www.alkottob.com
28 SECTION 1 www.vet4arab.co.cc Anesthesiology 29

75. c The greater an agent's alveolar concentration, 82. d With in-circuit vaporizers, output depends on 93. c The oxygen flush valve is located between the 109. a For closed-circuit anesthesia the system must
the more rapid is its absorption. Uptake also ventilation (increases with increasing ventilation). ~ource of compressed oxygen and the fresh gas incorporate a soda lime carbon dioxide
depends on blood solubility. An anesthetic that is In-circuit vaporizers are of simple "flow over inlet of the circuit. absorber.

m
very soluble is rapidly taken up by the blood. liquid" design and generally use less anesthetic 94. c Without a patent inspiratory valve, exhaled 1l0. c The other answers are all advantages of
Cardiac output determines the blood flow than out -of- circuit (precision) vaporizers. The gas would move into the inspiratory side of the closed-circuitllow-flow systems. The closed
through the lungs and therefore the quantity vapor pressure of halothane at 20· C is 243 mrn rebreathing hose, so the patient would inspire system permits inhalation of warm gases,
absorbed. The correct ratio of inspired air to lung Hg. At sea level (760 mni Hg), 243/760 = 32%, gas containing carbon dioxide. maintenance of a constant anesthetic level,
perfusion (ventilation/perfusion ratio) ensures which means a 32% concentration of halothane 95. b If the one-way valves are patent, there is economical use of volatile agents, and
optimal anesthetic uptake. can be achieved. There is a similar situation for unidirectional gas flow from the Ypiece connector. maintenance of high humidity of inspired

.co
76. c The ideal agent, producing rapid induction isoflurane. Out -of-circuit precision vaporizers gases.
96. d Soda lime granules become harder when they
and recovery, would have low blood solubility only deliver a 5% maximum for these agents.
are exhausted. 111. d An endotracheal tube extending well beyond
and low lipid solubility, rapidly achieving 83. d Uptake of anesthetic is improved with the animal's nose increases dead space.
97. d Hypercapnia results in sympathetic nervous
equilibrium (partial pressure) among gas, blood increased ventilation. The "centralized" 112. e With this maneuver, stomach inflation/
system stimulation.
and tissue phases/compartments. Solubility is circulation of neonates ensures rapid delivery of
98. b Occlusion of the tubing from the pop-off deflation can be mistaken for chest excursion.
an index of the capacity of the blood or tissue to anesthetic to the brain.
valve to the scavenger would cause pressure in 113. a Early in anesthesia, the patient removes most
retain anesthetic. 84. d Halothane markedly lowers the
the breathing circuit to increase. of the anesthetic from the inspired gas and
77. b A small amount of vapor entering tissue in arrhythmogenic dose of epinephrine in dogs. exhales gas with a low anesthetic
99. d To check the circuit for leaks, it should be

ob
which the agent is not soluble causes a high 85. e Nitrous oxide is a liquid when stored' under concentration. This gas goes back around the
pressurized to 20 to 30 cm H20, and the
vapor tension. Therefore during anesthetic pressure at room temperature. Halotl'lane is a circuit and dilutes the concentration of gas
manometer should be observed for 1 minute for
recovery, arterial and brain concentrations drop vapor, not a gas, and therefore is not stored coming from the vaporizer.
pressure change. A pressure decrease indicates
to negligible levels within minutes and the under pressure. Gases that can be liquefied at
a leak. 114. d Tissue oxygen saturation (Sao2, measured
patient awakens rapidly. room temperature are purchased by weight.
100. b If the pop-off valve were closed, pressure with pulse oximetry) will not change
78. b The duration of anesthesia determines the Pressure is an accurate index of quantity for
would increase. It is possible to close the pop- perceptibly unless there is a marked change in
saturation level of body tissues. Anesthetics that nonliquefiable compressed gases. tissue perfusion or hemoglobin saturation with
off valve in a low-flow system where the oxygen

ott
are highly soluble in fatty tissues take longer to 86. c The pressure in the cylinder begins to drop oxygen. Neither will change enough within the
flow equals the patient's oxygen consumption
be expelled from the tissues. If pulmonary blood when all the liquid nitrous oxide has been range of clinical anesthetic levels to change
(a closed circuit).
flow or ventilation is inadequate, inhalation vaporized and the gas loses pressure. Cylinders Sa0 2, especially if the animal is breathing a high
agents cannot be cleared through the lungs and must be watched thereafter for rapid emptying. 10 1. e Theoretically, the volume of the soda lime
container should be at least twice the patient's inspired concentration of oxygen.
recovery is delayed. Hthe agent is highly soluble Another cause of decreased pressure is chilling of
tidal volume. 115. a A corneal reflex can persist even at deep
in the blood, it is not removed quickly. An the cylinder.
102. b The reservoir bag is the compliant levels of anesthesia. Loss of the corneal reflex is
example is methoxyflurane, which is highly 87. a All except oxygen are liquefied at room
(collapsible) part of the circuit and usually a sign of very deep anesthesia. Repeated testing
soluble in blood and has a long recovery period.

alk
temperature under pressure. As such, they are of the corneal reflex can damage the cornea.
79. d Up to 20% of the inhaled dose of halothane is safely handled as liquids in cylinders. should have a volume of about 5 times the
patient's tidal volume (approximately the 116. c The esophageal stethoscope should be
metabolized. Isoflurane causes more respiratory 88. c Gas cylinders for various agents are identified
functional residual capacity). positioned in the esophagus at the level where
depression in dogs and horses. Methoxyflurane by a unique color, thread size, and thread
103. a These circuits require high fresh gas flows heart and lung sounds are best heard. These
and isoflurane are arrhythmogenic, although diameter. E cylinders also have a pin index
and so are not economical for larger patients. sounds allow evaluation of heart and
significantly less so than halothane. Halothane is combination. respiratory rates, neither of which alone is a
by far the cheapest commonly used inhalation 89. a The emergency oxygen flush valve bypasses 104. b B?th are examples ofnonrebreathing circuits,
useful guide to anesthetic depth.
agent. the vaporizer to deliver only oxygen to the so hIgh fresh gas flow is necessary to prevent
rebreathing of expired carbon dioxide. 117. b The bradycardia is most likely the result of
80. a Painful stimulation of lightly anesthetized circuit. It should be used to fill the circuit as part
patients causes catecholamine release, which
can lead to ventricular arrhythmias in
halothane-anesthetized patients. Increasing the
anesthetic procedures.
w.
of the preanesthetic "pressure check" before all 105. c Fresh gas flow rates of approximately 200
ml/kg/min are necessary to prevent rebreathing
of carbon dioxide.
excessive vagal tone and could be treated with
intravenous atropine. It should also respond to
cessation of the noxious stimulus (i.e., release of
90. a Increased ventilation increases the anesthetic traction on the ovary).
anesthetic depth can eliminate these concentration within the circuit. 106. d With nonrebreathing circuits, there is no
arrhythmias. 118. c "'2-adrenergic agonists cause marked
91. e With in-circuit vaporizers, increasing fresh gas drop in anesthetic concentration from the
bradycardia. In dogs and cats, they should be
81. b Halothane, 5 ml/500 ml O2 (per minute) =1% flow "washes" anesthetic out through the pop-off vaporizer to the patient.
used after premedication with an
ww
Vaporizer setting; 5 ml/120 ml O2 (per minute) = valve, decreasing the concentration within the 107. e For semiclosed circuits, oxygen flow is usually anticholinergic drug, such as atropine.
4.2% Vaporizer setting. Each setting would result 30 ml/kg/min. However, for closed or low-flow
circuit. 119. b A high blood carbon dioxide level causes
in a similar anesthetic depth. This illustrates the circuits, the fresh oxygen flow can be as low as 4
importance of fresh gas flow in determining 92. e Both one-way valves must be patent to ensure cardiovascular stimulation, mediated via the
unidirectional gas flow. to 6 ml/kg/min for dogs.
anesthetic delivery to the circuit. sympathetic nervous system.
108. d This is approximately 4 to 6 ml/kg/min in
dogs.

© 1998 Mosby- Year Book, Inc. Photocopying is prohibited by law.

www.alkottob.com
30 SECTION 1 www.vet4arab.co.cc Anesthesiology 31
120. d Assessment of pulse quality does not enable by norepinephrine administration, including 145. c Oxygen delivery is related to cardiac output,
you to estimate systolic or diastolic pressure. increased stroke volume, presumably from of this, the anesthetic effect produced by NO,
hemoglobin concentration, and the percent has a relatively rapid onset of action.
121. d Arterial pressure =Cardiac output x increased contractility. saturation with oxygen.
155. b A canister containing activated charcoal will

m
Peripheral resistance. Cardiac output = Heart 132. a At a dosage of 1 to 3 f.l.g/kg/minute, dopamine 146. b Shifting of the oxyhemoglobin curve to the left
rate X Stroke volume. increases renal output by increasing glomerular absorb halogenated anesthetic agents.
increases the affinity of oxygen for hemoglobin, However, canister use must be monitored to
122. a The response time of pulse oximeters is filtration via dilation of the afferent arterioles. reducing tissue oxygen availability. As blood pH prevent overuse (exhaustion of the activated
currently not fast enough to enable them to be 133. a Phenylephrine may be given in reasonable decreases, the oxygen dissociation curve shifts to carbon), resulting in reduced scavenging and
used with a pneumatic cuff to detect blood doses without fear of ventricular irritability. the right. Carbon dioxide and lactic acid pollution. Recommended governmental levels

.co
pressure. However, epinephrine is likely to cause liberated during metabolism cause this of exposure to the halogenated hydrocarbons is
123. e Currently available indirect methods all have premature ventricular contractions. phenomenon (the Bohr effect). increasing body no greater than 2 ppm for volatile agents and 25
inaccuracies associated with cuff size, limb size, 134. c We assume that a hypotensive patient has temperature above normal, an increased organic ppm for nitrous oxide. Nitrous oxide is not
the amount of hair covering the site, and the poor tissue perfusion (not always a true phosphate level (2,3-diphosphoglycerate and absorbed by activated charcoal and must be
animal's position. Some of these effects can be assumption) . The objective of therapy is to adenosine triphosphate), and an acidic pH all vented to the outside.
standardized for more repeatable measurements. improve tissue perfusion with oxygenated cause a shift to the right.
156. c Propofol is a sedative-hypnotic and is
In general, indirect methods most reliably blood. 147. e When the Pao, is above 100 mm Hg, all the unrelated to other anesthetic agents. Because
determine systolic arterial pressure. 135. c Anatomic dead space comprises the area of hemoglobin (which carries over 97% of the propofol has minimal analgesic properties,
124. cAn overly wide cuff yields spuriously low the respiratory tract not involved in gas oxygen) is saturated.
analgesics should be administered concurrently

ob
readings. An overly narrow cuff and a loose cuff exchange, including the oronasal cavity and 148. c Reducing the Paco, by 5 mm Hg would only for painful procedures.
yield spuriously high readings. conducting airways. In healthy unanesthetized raise the Pao, by about 5 mm Hg. Answers b, d, 157. a Slow administration ofpropofol requires a
125. e As the pressure wave travels distally, animals, it is equal to about one third of the and e would raise cardiac output, which would lesser amount of drug necessary to induce
impedance causes the systolic pressure to tidal volume but increases during anesthesia, improve ventilation/perfusion in the lung. general anesthesia and reduces the degree of
increase, although mean arterial pressure tends particularly as a result of atelectasis. 149. c This dog is likely cyanotic and, if not hypotension and apnea that commonly occur
to decrease. 136. e The minute volume depends on both tidal anesthetized, may be unconscious because of on induction. Opiate premedication tends to
126. a Hydrostatic force (gravity) causes a false volume and respiratory rate (Vt x Respiratory hypoxemia. increase the likelihood of apnea on induction

ott
elevation in pressure, proportional to the height rate). 150. e Cerebral blood flow and pressure are with propofol, but is not of consequence as long
difference (1.36 mm Hg/cm difference). 137. b An average range is 10 to 15 ml/kg/breath. enhanced by increasing Paco, or decreased by as ventilation is possible.
127. b Mean arterial pressure is the mean perfusion 138. c Minute ventilation and arterial carbon decreasing Paco,. Brain volume increases as 158. d The duration of analgesia produced by
pressure (not blood flow) . It is the area under the dioxide levels are equivalent measures of the more blood flows into the closed cranial cavity, morphine administered epidurally is much
pressure waveform. About one third of the effectiveness of ventilation. which also increases cerebrospinal fluid longer than from other routes of
cardiac cycle is systole, two thirds is diastole. 139. e Increased arterial carbon dioxide levels pressure. Carbon dioxide is an anesthetic at administration.
Therefore, to calculate mean arterial pressure, increase the depth and usually the rate of higher concentrations (e.g., 30%), although it 159. d Analgesia produced by morphine becomes

alk
systolic and diastolic pressure must be weighted. ventilation; increase heart rate, cardiac output, causes marked cardiovascular responses. apparent approximately 1 hour after epidural
128. e Hypotension is usually defined as a systolic and blood pressure; and cause injection of the 151. a These are the anatomic landmarks for administration. The delay in onset of effect is
pressure below 80 to 90 mrn Hg, which would mucous membranes. identifying the epidural space in dogs. Answer e related to its relatively lipophobic nature. It is
generally result in a mean arterial pressure 140. d Compliance is a measure of static pressure- states the landmarks to identify the epidural slowly absorbed into the spinal cord.
below 60 to 65 mrn Hg. volume relationships (Llcm) , whereas space in horses.
160. d The blood/gas partition coefficient indicates
129. d Doxapram increases the metabolic rate of the resistance is a measure of dynamic pressure 152. a The dose oflocal anesthetic should be that desflurane is the least soluble in blood of
brain and may stimulate ventilation. It does not flows (Llcm H,O/sec). reduced by approximately half (1 ml reduced to those listed. This solubility reflects the time
directly improve cardiovascular function unless 141. b Carbon dioxide causes cardiac epinephrine 0.5 ml per 4.5 kg) to prevent a higher level of necessary for the drug to exert its effect. The
its administration has caused or follows
hypoxemia.
130. e Cardiac output = Heart rate X Stroke volume.
w.
to be released from the sympathetic nerve
endings, resulting in increased cardiac output.
anesthetic block than is desired. By decreasing
the dose, the incidence of "total spinal
anesthesia" is reduced, decreasing the possibility
less soluble the drug, the more rapid the onset
of effect. The order of onset of effect for this
group of drugs would be as follows: desflurane
142. d In general, anesthetic drugs cause dose-
131. d Phenylephrine and methoxamine are of respiratory and cardiovascular collapse. > sevoflurane > isoflurane > halothane>
dependent respiratory depression.
regarded as pure alpha -adrenergic stimulators. 153. b There is no possibility of "total spinal methoxyflurane.
143. b Paco, is used to assess ventilation, whereas
Isoproterenol has powerful positive inotropic anesthesia" when an opioid is administered 161. e The lower the MAC, the more potent the
Pao, is used to assess oxygenation.
into the subarachnoid space. There are
ww
and chronotropic myocardial effects, causing 144. e CO, + H,O ... H,C0 3 ... H+ + HC03 • Large drug. MAC reflects the potency of inhalant
increased cardiac output. Peripheral vascular currently no documented problems from anesthetic agents.
changes in blood bicarbonate levels produce administration of the entire dose into the
dilation and reduced resistance are other only small changes in blood pH. Changes in 162. c Methoxyflurane may be administered using a
prominent effects of this beta-adrenergic subarachnoid space in dogs. nonprecision vaporizer, such as the Ohio #8
hydrogen ion concentration have a direct effect
stimulant. Ephedrine causes central nervous 154. d NO, is a relatively insoluble agent, with a vaporizer.
on blood pH.
system-mediated release of norepinephrine; blood:gas partition coefficient of 0.47. Because
therefore, its effects are similar to those caused

© 1998 Mosby-Year Book, Inc. Photocopying is prohibited by law.

www.alkottob.com
32 SECTION 1 www.vet4arab.co.cc Anesthesiology 33

163. d The copper kettle vaporizer must be manually 170. e Cattle are most susceptible to the effects of 182. e Hypercapnia may increase intracranial 189. e Bupivacaine has an intermediate onset of
adjusted to compensate for changes in xylazine. Sheep and goats are also very sensitive pressure by causing vasodilation of cerebral action (20 to 30 minutes) and lasts
temperature and fresh gas flow to deliver the to xylazine. blood vessels, resulting in increased blood flow approximately 4 to 6 hours.

om
desired amount of anesthetic vapor. These units 171. e This is calculated using the equation [(% and leading to possible brain herniation if a mass 190. a Pr?caine produces no effect following topical
are still available for use in veterinary hospitals. oxygen inspired) x (Barometric pressure - is present. Inappropriately managed anesthesia appitcatlOn.
The Ohio #8 is a nonprecision vaporizer and has Water vapor pressure)] - Respiratory quotient. could result in this. Also see answer 185.
191. a Mean arterial pressure is calculated using the
no means to compensate for temperature and 20.9% x (760 - 47) - 40/0.8 =99 mm Hg. 183. d Because the dog is very depressed, it is
changes in fresh gas flow. The Vapor (North equation [(Systolic blood pressure - Diastolic
172. c This is calculated using the equation [(% probably best not to administer any of the blood pressure)/3] + Diastolic blood pressure.
American Drager) is a precision vaporizer that is oxygen inspired) x (Barometric pressure - premedicants listed. Oxyrnorphone is a potent
flow compensated, but manual adjustments must [(100 - 551 3) + 55] = 70 mm Hg.
Water vapor pressure)] - Respiratory quotient. oplOld and depresses respiration; atracurium is
be made to correct for temperature changes. The 192. c A mean arterial pressure less than 60 mm of
100% x (760 - 47) - 40/0.8 = 663 mm Hg. a neuromuscular relaxant and should not be
newer precision vaporizers are both temperature Hg of pressure indicates the potential for
173. c Hemoglobin content is approximately one given as a premedicant (it is administered to
and flow compensated. Examples of these are the inadequate perfusion of the vital organs, such

.c
third the packed cell volume. anesthetized patients); ketarnine increases
"Tec" and "Matic" types of vaporizers, which as the heart, liver, and kidneys.
174. b A packed cell volume of21% would indicate a cerebral blood flow and could further increase
automatically compensate for changes in intracranial pressure and may induce seizure 193. d Using the equation cited in answer 191, [(70
temperature and fresh gas flows within flow hemoglobin content of approximately 7 gl dl, - 30) 13] + 30 = 43 mm Hg.
which is the lowest acceptable hemoglobin activity; acepromazine can induce seizures.
ranges normally used in veterinary anesthesia. 194. b The blood volume of dogs is approximately
content necessary for adequate oxygen content. 184. d Isoflurane in oxygen is the most appropriate

ob
164. e The appropriate volume per breath can be choice in this case. Also see answer 185. 7.5:ro to 9% of body weight (in kilograms). A safe
determined by simple observation of rise of the 175. e Intermittent positive-pressure ventilation estimate for general use is 8%. To estimate
may decrease cardiac output. All the other 185. a Intermittent positive-pressure ventilation is
patient's chest wall when squeezing the blood volume, multiply the body weight (in
answers are potential treatments. Usually used to prevent accumulation of carbon dioxide
rebreathing bag to inflate the lungs. If a
which could increase intracranial pressure. ' kilograms) by 80 ml/kg. 40 x 80 =3200 mi.
pressure manometer is located in the airway treatment progresses as follows if the above
When controlled ventilation and isoflurane are 195. e A packed cell volume of 25% indicates a
circuit, a peak airway pressure approximating measures have not corrected the situation:
decrease the percent of inhalation anesthetic used in cases such as this, an increase in cerebral hemog!obin content of approximately 8 gl dl. At
15 to 20 cm of water pressure indicates thiS pomt, more hemoglobin (in the form ofred
administered, administer a fluid bolus, blood flow, resulting in increased intracranial
adequate lung expansion. blood cells in whole blood) is needed.

ott
administer ephedrine, and, last, administer a pressure, is minimized. If halothane were
165. c Wright's respirometer is used to measure tidal administered instead, even with intermittent 196. d The dog initially has 1440 mI of red blood
more potent inotrope, such as dobutamine.
and minute volumes in small animals. It is positive-pressure ventilation, cerebral blood flow cells (Blood volume x Packed cell volume =
commonly inserted into the breathing circuit of 176. d Succinylcholine interacts with the nicotinic
is less well controlled and there is potential for Red blood cells [mill. When the packed cell
the anesthetic machine for these receptor, just as acetylcholine does, and
increased intracranial pressure. volume falls to 25%, there is 800 mI of red blood
measurements. stimulates muscle depolarization. Because
186. e Capnometry allows measurement of end tidal cells. Therefore the dog has lost 640 mI of red
succinylcholine is not rapidly metabolized, the
166. d The pulse oximeter measures hemoglobin's carbon dioxide. This estimates the arterial blood cells (1440 - 800 = 640).
duration of receptor occupation is longer than
saturation with oxygen. With 90% saturation

alk
acetylcholine and muscle stimulation carbon dioxide partial pressure. End tidal 197. c The total plasma protein concentration is low
(P90), the partial pressure of oxygen is carbon dioxide pressure is approximately 5 mm and would result in hypovolemia because of
(fasciculation) is followed by flaccidity.
approximately 60 rnrn Hg in dogs. of Hg less than the arterial value. End tidal inadequate protein to maintain intravascular
177. d SUCcinylcholine is hydrolyzed by
167. b Butorphanol is an opioid agonist; yohimbine carbon dioxide pressure should be maintained osmotic pressure and retain fluids in the
cholinesterases. It must be metabolized to end
is an ",, -antagonist. Yohimbine does not in the range of 25 to 30 mm Hg. vascular compartment.
its effects.
antagonize the effects ofbutorphanol. 187. c Local anesthetics produce their effects 198. c HypertOnic (7%) saline can be rapidly infused
178. a A full E cylinder contains approximately 700
168. c Butorphanol has antagonistic activity at the through interaction with the sodium channel of N and should be followed by slower infusion of
L, with a service pressure of approximately 2000
fL-receptor and has agonistic activity at K- and cells. They are thought to prevent action isotonic fluids (e.g., 0.9% saline). Hypertonic
to 2200 psi. The pressure is proportional to the
0-- receptors. When butorphanol is given to an potential generation by obstructing the sodium salme IS generally infused over approximately
animal that has received morphine (a fL-
agonist), morphine's effects at this receptor are
antagonized. However, the agonistic action of
gas volume (L) in the cylinder.
w.
179. c The pin index safety system is used to prevent
inadvertent interchange of nitrous oxide and
c~annel from inside the membrane or by
dlsruptmg electrical activity by lodging within
10 minutes.
199. a Midazolam is a water-soluble benzo-
the lipid membrane of the axon. diazepine in the same class as diazepam.
butorphanol at the K-receptor produces oxygen on the hanger yoke of the anesthetic
machine. It consists of two pins and a nipple, 188. b Lidocaine, mepivacaine, and bupivacaine are 200. d M~detomidine is an ",,-agonist approved for
analgesia; therefore butorphanol can be all amide types of local anesthetics. Procaine vetennary use in 19?6. It is the most potent "', -
administered to partially antagonize morphine's with spacing specific for each anesthetic gas.
and chloroprocaine are ester types of local agent currently available.
ww
effects. 180. d Tranquilizers administered as preanesthetics anesthetics.
169. d Swine are the least susceptible of all species help to decrease the amount of anesthetic
to the effects of xylazine. It is usually used in induction agent necessary.
combination with other anesthetic agents to 181. a It is only administered intravenously to
produce the desired effect in pigs. For sedation, induce anesthesia.
pigs require 20 to 30 times the low dosage used
in ruminants (0.1 mglkg 1M).

© 1998 Mosby-Year Book, Inc. Photocopying is prohibited by law.

www.alkottob.com
34 SECTI ON 1 www.vet4arab.co.cc SECTION

NOTES
2

m
, ,
Clinical Pathology

.co
C.B. Grindem, B.T. Mitzner, R.E. Raskin

Recommended Reading

ob
Cowell RL, 'lYler RD: Cytology and hematology of the horse, St Louis, 1992, Mosby.
Cowell RL, 'lYler RD: Diagnostic cytology of the dog and cat, ed 2, St Louis, 1997, Mosby.
Duncan JR et al: Veterinary laboratory medicine, ed 3, Ames, Iowa, 1994, Iowa State University
Press.
Eades SC, Bounous Dr: Laboratory profiles of equine diseases, ed 2, St Louis, 1997, Mosby.
Jain NC: Essentials of veterinary hematology, Baltimore, 1993, Williams & Wilkins.

ott
Meyer DJ et al: Veterinary laboratory medicine: interpretation and diagnosis, ed 2, Philadelphia,
1997, WB Saunders.
SodikoffCH: Laboratory profiles of small animal diseases, ed 2, St Louis, 1996, Mosby.
Willard MD et al: Small animal clinical diagnosis by laboratory methods, ed 2, Philadelphia, 1994,
WB Saunders.

alk
Questions
C.B. Grindem
1. The anemia of chronic inflammatory disease is c. deficiency of erythropoietin
caused by: d. deficiency of transferrin
a. chronic blood loss e. ineffective use of stored iron
w. b. deficiency of iron
Questions 2 and 3
Equine patient's sample Reference range
Red blood cells (RBC) ( x 1()6/!1L) 2.70 6.0-10.43
Hemoglobin (Hb) (g/rll) 5.5 10.1-16.1
ww
Packed cell volume (PCV) (%) 13 27-43
Mean corpuscular volume (MCV) (tI) 37-49
Mean corpuscular hemoglobin
concentration (MCHC) (gl rIl) 35.3-39.3
Total protein (gl rIl) B.B 6.0-B.5
Heinz bodies seen on blood smear

© 1998 Mosby-Year Book, Inc. Photocopying is prohibited by law. Correct answers are on pages 55-62. 35

www.alkottob.com
36 SECTION 2 www.vet4arab.co.cc Clinical Pathology 37

c. in vivo hemolysis 12. A false-negative Coombs' test can occur with any d. transfusion oftype-DEA 1.1 blood into a dog
2. How is the anemia in this horse most accurately
of the following except: with type-DEA 7 (tr) blood
described? d. in vitro hemolysis
e. extreme overanticoagulation with EDTA a. glucocorticoid therapy e. transfusion of the dam's RBCs into a foal
a. normocytic, hyperchromic

m
b. improper antigenl antibody ratio
b. normocytic, normochromic
8. If the automated white blood cell (WBC) count in c. insufficient quantity of antibody on RBCs 15. What is the best test to confirm failure of passive
c. macrocytic, hyperchromic transfer of maternal immunoglobulins in a foal?
a cat is 30,OOO/II'.L and 200 nucleated RBCsllOO d. testing an unseparated, refrigerated serum
d. macrocytic, hypochromic
WBCs were observed on the blood smear, the sample a. serum IgG levels
e. microcytic, hypochromic
corrected WBC count would be: e. not performing the test at various temperatures b. serum IgM levels

.co
a. 1000/fJ.L c. serum gamma glutamyltransferase (GGT) levels
3. What is the most likely cause of the anemia?
b. 1500/fJ.L 13. What is the preferred test to confirm a diagnosis of d. serum total protein
a. equine infectious anemia c. 7500/fJ.L systemic lupus erythematosus? e. serum albumin levels
b. lead poisoning d. 10,OOO/fJ.L a. Coombs' test
c. red maple poisoning e. 15,0001 fJ.L b. rheumatoid factor test 16. Neonatal isoerythrolysis in foals is best diagnosed by:
d. bracken fern poisoning c. antinuclear antibody test a. cross matching the foal's RBCs and the mare's
e. blister beetle poisoning d. lupus erythematosus cell test
9. An 8-year-old dog has a fever, weight loss, and serum

ob
oculonasal discharge. Vaccinations are current e. direct immunofluorescence test b. cross matching the mare's RBCs and the foal's
4. Which of the following causes extravascular (given 6 months ago). The WBC count is 20,OOO/fJ.L, serum
hemolytic anemia? . with 50% lymphocytes that are morphologically
14. Which patient is most likely to develop an c. blood typing the mare for Aa and Qa RBC
normal. The platelet count is decreased, and
a. babesiosis immediate transfusion reaction? antigens
plasma globulin levels are increased. What is the
b. anaplasmosis most likely cause of these findings? d. crossmatching the stallion's RBCs and the
a. transfusion of type-A blood into a cat with
c. leptospirosis foal's serum
type-B blood
a. ehrlichiosis

ott
d. copper toxicity e. crossmatching the mare's colostrum against
b. trypanosomiasis b. transfusion of type-B blood into a cat with
the stallion's RBCs
e. hypophosphatemia type-A blood
c. physiologic lymphocytosis
c. transfusion oftype-DEA 1.2 blood into a dog
d. acute lymphoblastic leukemia with type-DEA 1.1 blood
5. Neutrophils remain in the peripheral circulation
e. postvaccinationallymphocytosis For Questions 17 through 21, match the most likely
for approximately:
cause with the following five canine hemograms.
a. 2 days 10. What is the best test to differentiate iron

alk
b. 14 days deficiency of chronic blood loss from relative iron Test a b c d e Reference range
c. 120 days deficiency of chronic inflammatory disease? WBCs (per ILL) 2000 25,000 20,000 12,000 20,000 5000-18,000
d. 10 hours PCV(%) 10 28 15 58 20 33-56
a. serum iron MCV(fI) 70 64 74 70 74 63-73
e. 30 minutes
b. red cell indexes MCHC (g/dl) 36 34 42 36 33 34-38
c. transferrin saturation Platelets (x 103 ILL) 50 600 150 300 450 200-500
6. Neutrophil functional defects have been reported d. bone marrow iron stores Reticulocytes (%) 0 0 15 0 5 <1.5
with all of the following except: Total plasma protein (gl dl) 7.0 8.5 8 8.5 5.0 6.0-7.5
e. total iron-binding capacity w.
a. estrogen toxicity
b. after vaccination 17. Acute blood loss anemia persisting for more than 19. Anemia of chronic inflammatory disease
11. A dog has a PCVof 78% and normal blood urea 3 days
c. diabetes mellitus nitrogen (BUN), total plasma protein, and
d. selenium deficiency erythropoietin levels. What is the most likely cause 20. Relative polycythemia
e. bovine neutrophil CDllb/CDI8 surface of these findings? 18. Hemolytic anemia
marker deficiency 21. Aplasticanemia
ww
a. hypoxia
b. dehydration
7. Causes of erroneous refractive index-derived total c. renal disease
plasma protein values include all the following d. splenic contraction
except: e. polycythemia rubra vera
a. lipemia
b. Heinz bodies

© 1998 Mosby-Year Book, Inc. Photocopying is prohibited by law. Correct answers are on pages 55-62.

www.alkottob.com
38 SECTION 2
www.vet4arab.co.cc Clinical Pathology 39

For Questions 22 through 26, match the most likely 29. Amyloidosis 36. In horses, which of the following is most helpful in
cause with the following five canine hemograms. distinguishing changes in peritoneal fluid caused
by pathologic peritonitis versus those related to
30. Myoglobinuria

m
recent abdominal surgery?
Test a b c d e Reference range
22,000 4500 5000-18,000 a. total plasma protein
WECs (per fl.L) 20,000 14,000 2000 31. Pyelonephritis
9800 600 17,160 1125 3000-6000 b. RBCcount
Segmented neutrophils (per fl.L) 17,200
Band neutrophils (per fl.L) 0 0 0 2200 1350 0-300 c. neutrophil morphology
1000 660 2025 1000-5000 32. Diabetes mellitus
Lymphocytes (per fl.L) 800 2800 d. percentage of neutrophils

.co
Monocytes (per fl.L) 2000 700 300 1980 0 150-1350
e. total nucleated cell count
Eosinophils (per I'll 0 700 100 0 0 100-750 33. Iatrogenic hematuria
Platelets ( X 10' I'L) 250 500 50 450 38 200-500
6.5 6.8 7.0 8.5 8.7 6.0-7.5 37. What is the best procedure to confirm a cytologic
Total plasma protein (gt dl) 34. In a dog with ethylene glycol toxicity, what is the diagnosis of chylous effusion?
least likely finding on urinalysis?
22. Physiologic response to acute stress (epinephrine c. only intravascular and extravascular a. ether extraction
release) hemolytic anemia a. bilirubin crystals
b. fluid/serum cholesterol ratio
d. only hematuria and hemoglobinuria b. pH5.5
c. fluid/ serum triglyceride ratio

ob
e. hematuria, myoglobinuria, and hemoglobinuria c. 5 WBCs/high-power field
23. Physiologic response to chronic stress d. Sudan IV staining of the cytologic smear
(glucocorticoid release) d. specific gravity 1.009
e. cytomorphology and differential cell count
28. If a urine sample is diluted 1:1 (because the e. negative ketones
reading is higher than the refractometer scale)
24. Bone marrow suppression 38. Which species normally has neutrophils in its
and the diluted sample has a specific gravity of 35. What type of effusion is most likely to be observed
1.030, what is the actual specific gravity of the abdominal fluid?
in a cat with feline infectious peritonitis (PIP)?
25. Inflammation, with a degenerative left shift sample? a. cats

ott
a. transudate
a. 1.015 b. dogs
b. septic exudate
26. Inflammation and stress b. 1.030 c. cattle
c. nonseptic exudate
c. 1.060 d. sheep
27. Which of the following causes a positive reaction d. modified transudate
d. 2.030 e. horses
for blood with a urine dipstick? e. hemorrhagic effusion
e. 2.060
a. only hematuria For Questions 39 through 43, match the most likely

alk
b. only intravascular hemolytic anemia cause with the findings from the follOwing five
For Questions 29 through 33, match the most likely peritoneal fluid samples.
cause with the following five urinalyses.
Assay a b c d e
Assay a b c d e
Color Red Yellow Milky white Green Colorless
Color Red-yellow Yellow Yellow Yellow Brown Turbidity Cloudy Cloudy Cloudy Cloudy Clear
Turbidity Clear Slightly turbid Turbid Turbid Turbid WECs (per I'L) 5600 89,000 8000 1000 1000
Specific gravity 1.037 1.033 1.017 1.020 1.022 Differential count
6.5
pH
Glucose
Ketones
6.5
Negative
Negative
7.0
Negative
Negative
7.0
Negative
Negative
5.5
4+
3+
w.Negative
Negative
Mononuclear cells
Polymorphonuclear cells
Blast cells
30%
70%
0
5%
95%
0
5%
5%
90%
35%
65%
o
98%
2%
o
Bilirubin Negative 1+ Negative Negative Negative Other cells Platelet clumps Extracellular bacteria,
Protein 1+ 1+ 4+ Trace 2+ ciliate protozoa, plant
Blood 3+ 1+ Negative Negative 4+ material
WECsthigh-power field 10 30-40 3-5 0-3 0-2 Protein (gt ell) 4.5 5.0 2.5 1.0 1.0
ww
RBCsthigh-power field Too numerous to count 5-10 1-2 1-2 0-2
Other observations Triple phosphate crystals Granular casts 39. Hypoalbuminemia 42. Contamination of the sample with blood

40. Lymphosarcoma 43. Enterocentesis

41. Peritonitis

© 1998 Mosby-Year Book, Inc. Photocopying is prohibited by law. Correct answers are on pages 55-62.

www.alkottob.com
40 SECTION 2
www.vet4arab.co.cc Clinical Pathology 41

44. Which two enzyme assays are best for diagnosing 47. A decreased serum Na/K ratio is likely to be Question 53 CI 93m£q/L
observed with any of the following disorders Total CO, 11 mEq/ L
muscle disease? Blood samples from an 8-month-old mixed-breed dog
except: Serum osmolality 411 mOsm / kg
with poor weight gain yield the following laboratory

m
a. alanine aminotransferase (ALT) and lactate BUN nmg/dl
dehydrogenase (LDH) a. uroabdomen results.
Glucose 170 mg/dl
b. LDH and aspartate aminotransferase (AST) b. chylothorax Patient's Reference Phosphorus 10.1 mg/dl
c. hypothyroidism samp'" range
c. creatine phosphokinase (CK) and AST
d. hypo aldosteronism BUN (mg/dl) 7 8-28
d. CK and LDH 56. What is the anion gap in this dog?
Creatinine (mgl dl) 0.8 0.5-1.7

.co
e. CKandALT e. whipworm infection
Total bilirubin (mgl dl) 0.2 0.0-0.3 a. 39.5
ALP (lUlL) 160 1-114 b. 40.4
45. Which test is most specific for diagnosing exocrine Question 48 ALT (lUlL) 54 10-109 c. 50.5
pancreatic insufficiency? Canine Bile acid (llIDollL) 269 0-8
d.56.0
patient's Reference Ammonia ()lg/dl) 190 19· 120
a. fecal fat range Albumin ()lg/dl) 1.1 2.3-3.1 e. 61.5
samp'"
b. corn oil absorption Bromsulphalein (BSP) 6 0-5
Albumin (gl dl) 2.8 2.3-3.1
c. D-xylose absorption 25 8-28 (% retention) 57. What is the most likely cause of these findings?
BUN (mg/dl)

ob
d. trypsinlike immunoreactivity Ca (mg/dl) 14.1 9.1-11.7 a. renal failure
e. bentiromide (BT-PABA) absorption P (mg/dl) 7.2 2.9-5.3 53. What is the most likely cause of these findings?
b. lactic acidosis
a. shock
c. hyperadrenocorticism
46. Thyroid hormone levels are likely to be decreased 48. What is the most likely cause of these findings? b. hepatitis
d. diabetic ketoacidosis
in animals in any of the following except: a. renal failure c. cholestasis
e. ethylene glycol intoxication
a. young animals b. hypoadrenocorticism d. hemolytic anemia
c. pseudohyperparathryoidism e. portosysternic shunt

ott
b. lymphocytic thyroiditis
d . primary hyperparathyroidism For Questions 58 through 62, select the correct
c. phenobarbital administration answer from the five choices below.
d. glucocorticoid administration e. toxicity from a rodenticide containing 54. Using an ion-selective electrode method for assay,
cholecalciferol which of the following causes a falsely elevated a. ammonia
e. animals with concurrent illness
For Questions 49 through 52, match the most likely chloride value? b. alkaline phosphatase (ALP)
cause with the findings from the following four a. lipemia c. alanine aminotransferase (ALT)
blood acid-base profiles. b. icterus d. gamma-glutamyltransferase (GGT)

alk
Reference range c. hemolysis e. aspartate aminotransferase (AST)
a b c d
7.5 7.5 7.0 7.31-7.42 d. bromide poisoning
pH 7.3
Pco, (mmHg) 27 18 53 53 29-42 e. metabolic acidosis 58. A test used to evaluate liver function
HCO, (mEq / L) 14 19 37 13 17-24
Total CO, (mIDollL) 15 19.5 38.6 14.6 14-26
55. Hypocalcemia has been associated with all of the 59. A cytosolic and mitochondrial leakage enzyme
following conditions except: used to evaluate liver damage in domestic animals
49. Metabolic alkalosis with respiratory 51. Metabolic acidosis and respiratory acidosis
a. acidosis
compensation
w.
52. Metabolic acidosis with compensatory respiratory
alkalosis
b.
c.
hypoproteinemia
acute pancreatitis
60. A membrane-associated enzyme used to detect
cholestatic liver disease in small animals
50. Respiratory alkalosis
d. blister beetle toxicity
e. ethylene glycol toxicity 61. A cytosolic leakage enzyme used to evaluate liver
damage in small animals
ww
Questions 56 and 57
62. Urinary enzyme used to evaluate renal tubular
A dog has the following blood chemistry profile: disease
Na 149 mEq/L
K 5.5mEq/L

© 1998 Mosby-Year Book, Inc. Photocopying is prohibited by law. Correct answers are on pages 55-62.

www.alkottob.com
42 SECTION 2 www.vet4arab.co.cc Clinica l Pathology 43

Question 63 65. Used to recover larvae ofStrongyloides, 73. Recomme,u:lo.tiolls for submission offecal samples 77. Which procedure is recommendedfordefi"itive
Aeluroslrongylus abstrusus. Metaslrongylus, and for culture incl/lde all ofthe following except Else of diag"osis ofdermatomycoses?
Canine
patient's Reference Dictyocaulus from feces a. swabs a. India ink preparations

om
sample range
b. fresh feces b. Wood's light examination
BUN (m g/dl) 39 8-24 66. Used to confirm ?xyuris equi infecrion c. 5 to 10 9 offeces c. culture in Mueller-Hinton medium
Creatinine (mg/dt) I.. <l.8
d. sterile container
Urine specific gravity 1.033 1.001·L065
d. culture in Sabouraud's dextrose agar
Total bilimbin (mg/dl) 0.5 <0.6
67. Used to idelllify trophozoites o/Balantidium, e. air-tight containers e. potassium hydroxide slide preparations
4.7 5.4-7.5
Giardia, trichomonads, and amoebae
Totail)lasma protein (g/ d l)
PCV(%) 30 33-56 74. Concerning microbial sensitivity to arrtimicrobiaf For Questions 78 through 82, select the cor rect
68. Used to estimnte 'lUmbers 0/ parasite eggs or cysts drugs, whiell statement is most correct? answer from the five choices below.
63. Wilat is the most likely cause of tllese findings? per gram o/feces

b.c
a. Sensitivity test results correlate well with the a. Demodex s pp
u. dehydrat ion clinical response to antimicrobial treatment. b. Psoroptes ovis
b. chronic renal fail ure 69. Used to distinguish Eimeriafrom Isospora b. Drugs that do not inhibit bacterial growth in c. Sarcoptes scabiei
c. intravascular hemolysis vitro will not likely be useful in vivo. d. Otodectes cy"otis
d. extravascular hemolysis 70. What is tile most effectiveflolation solution for c. The antimicrobial wilh the largest zone of e. Knemidokoptes mutans
e. gastrointestinal hemorrhage recovery of pa rasile eggs from feces? inhibition is the most suitable drug.
a. salt (sodium chloride) d. A Kirby-Bauer zone of inhibition of6 mm
(none beyond perimeter of the disk) indicates 78. Causes otitis extema ill camivores
Question 64 b. s ugar (sucrose)
complete inhibition of bacterial growth.

tto
Can ine
c. d istilled water
e. Tube dilution is the most commonly used 79. Causes ~scaly leg" and depillming scabies in birds
patient's Referenre d. zinc sulfate
sample range
method in veterinary medicine to determine
e. sodium nitrate minim um inhibitory concentratio n of
Alkaline phosphatase {lUlL) 1080 1-1 14 antimicrobial agents. 80. Tunnels deeply ifllo the epidermis and causes
Alkaline p hosphatase-- 94 intense irritation
71. Which criterion is most reliable/or differen tiating
levamisole resistance (%)
Alanine aminotransferase 394 10-109 the microfilariae ofDirofilaria/rom those of 75. If aerobic microbiologic swabs canllot be
Dipetalonema? 81. Remains 011 the skill surface or under scabs and

ko
(lUl L) immediately (within several hours) irweulated oruo
Glucose (mg/dl) 220 76- 119 culture medium, the recommended procedure is to: may cause ear mallge
a. s ize
ACTH stim ula tion
Pre-ACfH conisol (..,.g/dl)
Post-ACTI-I cortisol (~g/ dl)
0.5
0.8
<.
6-17
b. head s hape
c. body shape
a. freeze the swab specimen
b. refrigerate the swab specimen
82. Cigar-shaped mite that lives ill the hair follicles,
sebaceous glands, and epidermis
d. buttonhook tail c. store the swab s pecimen anaerobicaIJy at
e. relative numbers room temperature
64. W1wt is tile most likely ca/lse of thesefilldings? For Questions 83 through 87. select the correct

al
d. inoculate the swab specimen into transport
a. hepatitis answer from the five choices below.
72. What is the preferred test to diagnose heartworm medium and store at room temperature
b. diabetes mellitus a. EDTA
infection i1l dogs receiving macrolide heartworm e. inoculate the swab specimen into transport
c. ialTogenic Cushing's disease medium and store at refrigerator temperature b. citrate
preventive?
d. adrenal-dependent Cushing's disease
a. filter test
c. heparin
e. pituitary-dependent Cushing's disease
b.
c.
Knott's test
direct blood smear
w. 76. Concerning skill scrapings fo r diagnosis 0/
ectoparasitism, which stateme"t is least accurate?
d. oxalate
e. fluoride
For Questions 65 through 69, select the correct a. Select an area near the margin of a lesion.
answer from the five choices below. d. antigen test
e. antibody test b. Select an unmedicated, nonexcoriated area. 83. Anticoagularrt ofchoice for blood gas analysis
a. Baermann technique c.. Pinch lhe affected skin ben.veen the lhumb and
b. McMaster technique fo refinger and then scrape with an oily scalpel
ww
84. Anticoagulant ofchoice for a routine complete
c. cellophane tape preparation blade until a s mall amount of blood appears. blood count (CBC)
d. sporulation technique d. Transfer the scraping from the scalpel blade
e. staining of a wet mount direct smear with into a drop of saline on the microscope slide.
85. Anticoagulam of choice for routine coagulation
iodine solution e. Apply a coverslip to the scraping on the tests (prothrombin time {PTJ, actillated partial
microscope slide.
thromboplastin time {AP1TlJ

Ci 1998 Mosby-Year Book, Inc. PllOtocopying is prohibited by law. Correct answers are on pages 55·62.

www.alkottob.com
44 SECTION 2 www.vet4arab.co.cc Clinical Pathology 45

86. Anticoagulant that inhibits glycolysis for accurate 93. Vitamin K antagonism is most readily diagnosed 102. What is the most importalll periodic c. rubricytes
glucose testing WiTh which test? maimenance procedure required by impedance-
d. reticulocytes
type hematology analyzers?
a. prothrombin time e. erythrocytes

m
B7. Anticoagulant thatean be used/or serum b. activated partial thromboplastin time a. oiling the vacuum pump
electrolyte assay c. PrvKA test (pI;oteins induced by vitamin K b. changing the vacuum tubing
108. What arnfactual change in the complete blood
absence or antagonism) c. deproteinizing the counting aperture COUTit is most likely to occur if the blood sample
For Questions 88 through 92, select the COlTect d. antithrombin III assay d. cleaning the diluter probe is collected from an excited or agitated patient?
answer from the five choices below. e. factor VIII assay e. adjusting the vacuum pressure a. leukocytosis

.co
a. new methylene blue stain b. decreased hematocrit
h. Wright's stain 94. Which group ofclottingfactors is vitamin K 103. When using an impedance-type hematology c. platelet aggregation
c. Papanicolaou stain dependent? analyzer, the background count is performed on: d. left shift
d. Nat! and Herrick's solution a. I, V, IX, XII a. a normal blood sample e. leukopenia
e. periodic acid-Schiff stain b. II, V. VIII, X b. a dilmed control sample
c. IV; VI, Xl, XII c. deionized water 109. In which species are nucleated erytllrocytes a
88. Used for identifying fungal structures on cytologic d. II, VII, VITI , X, antithrombin III d. an aliquot of isotonic diluent Ilormalfinding?

ob
smears e. II, VII , IX, X, protein C e. an aliquot of cleaning solution a. cats
b. pigs
89. Used/or staining reticulocytes 95. Wllat is the best test to diagnose von Willebrand's 104. Hematology quality-control teslS are used to c. rabbits
disease? assess the:
d. chickens
90. Used for routine staining ofcytologic smears a. prothrombin time a. performance of the ope rator e. horses
b. activated partial thromboplastin time b. accuracy of the counting method

ott
91. Used for WBC examination in avian blood samples c. factor VIII assay c. accuracy of the sample dilutions 110. Whic/, cell type is kast likely to befound on a
d. dilute clot retraction d. integrity of the reagents peripheral blood smear from a dog with
92. Used/or routine stainingofblood smears e. von Willebrand's factor antigen (vWF) assay e. performance of the entire system autoimmune hemolytic anemia?

For Questions 96 through 100, select the correcl a. reticulocyte


answer from the foUowing five coagulation profiJes, 105. Most errors tllm occur with automated b. spherocyte
hematology analyzers can be traced back to:
a b , d , c. target cell

alk
a. improper specimen collection d. erythrocyte
Prothrombin time Normal Normal
b. inadequate premixing of the specimen e. metamyelocyte
Activated partial thromboplastin time Normal Normal t
Fibrin degradation products Normal Normal Normal Normal
c. improper dilution of the specimen
Platelet numbers 1 Normal Normal Normal d. lack of familiarity with the analyzer 111. Til a complete blood count, a left shift refers to:
e. improper transposition of results
96. Factor Vll deficiency 99. Disseminated intravascular coagulation a. movement of the microscope slide to the left
b. decreased numbers of platelets
106. Avian blood smears may colltain all tlte c. an abundance of immature WBC forms
97. Factor Vlll deficiency 100. Tmmlllle-mediated thrombocytopenia

98. Factor X deficiency


w. following cell types except:
a. erythrocytes
d.
e.
increased numbers of nucleated RBCs
a trend toward macrocytosis
b. thrombocytes
c. heterophils 112. An abundanceofeosinophils on a peripheral
B.T. Mitzner d. neutrophils blood smear is most commonly associated with:
101. Specimellsfor hematologic analysis should be c. red e. eosinophils
ww
collecred in vacuum tubes with a stopper of a. neoplastic disease
d . green
what color? b. infection
e. gray 107. The erythrocyte (red cell) series comprises all of c. trauma
a. lavender the following cell types except:
d. allergic conditions
b. blue
a. prorubricytes e. stress
b. metamyelocytes

© 1998 Mosby-Year Book, Inc. Phorocopyillg is prohibired by law. Correct answers are on pages 55-62.

www.alkottob.com
46 SECTION 2
www.vet4arab.co.cc Cllnicld Pathology 47

d. bleeding time 124. Decreased serum cholinesterase activity often 129. Which method is most reliable for confirmation
113. How long should microJiemmocrir rubes be
accompanies: ofactive Ehrlichia canis infeerion?
cenrrifuged? e. erythrocyte sedimentation rate
a. renaJ disease a. blood smear examination
a. 1 minute

om
119. Wllich abnormality is least likely to befound in b. trauma b. buffy coat examination
b. Sminutes
a nonseparated blqod sample collected for serum c. organophosphate insecticide toxicity c. indirect fluorescent antibody test on serum
c. to minutes
chemistry analysis? d. diabetes mellitus d. wet preparation of peripheral blood stained
d. 3 minutes with new methylene blue
a. increased potassium level e. pancreatitis
c. Calibrate centrifuge to determine time.
b. increascd aspartate aminotransferase activity e. serum titer
c. increased phosphorus level 125. Wh ich of the following is not used to calculate
J 14. lNhich stain is not appropriate for rOil cine
d. decreased glucose level anion gap? 130. Which method is not IIsedfordetection of
staining otblODd smears? heartworm microfilariae?
e. decreased cholesterol level a. serum sodium level

.c
a. Wright's a. direct blood s mear
b. serum caJcium level
h. Wright's-Gicmsa b. buffy coat examination
120. Wllich technique is most likely to prevent c. serum CO 2 level
c. Diff-Quik c. modified Knott's technique
hemolysis during blood collection? d. serum potassium level
d. trichrome d. filtration/concentration method
a. Use a large-gauge (19-gauge or larger) needle. c. serum chloride level

ob
e. Giemsa e. enzyme-linked immunosorbent assay
b. Use a small-gauge (21-gauge or smalle r)
needle. 126. The constituents in most serum chemistry
115. As a minimum standard. blood collection tubes
c. Soak the skin well with alcohol before control samples are stabl€ for apprOXimately 7 131. 11le test of choice for rourine silroeillance ofdogs
colltaining EDTA should befiUed: days after reconstitution. Wlliell constiwem is
venipuncture. treated monthly with heartworm preventives is:
a. to the top d. Shake the specimen vigorously after collection. stable for tile shortest time after reconstitution?
a. Knott's technique
b. one fourth of the way to the top c. Use a large (I0-ml or larger) collection tube. a. glucose b. filtration/concentration

ott
c. two thirds of the way to the top b. aJbumin c. enzyme-linked immunosorbent assay of high
d. three quarters of the way to the top 121 . Wllich clinicopatliologic abnormality is least c. creatinine sensitivity
c. to me level s pecified by the collection tube lilcely to befound ill a patient with advanced d. carbon dioxide d. enzyme-linked immunosorbent or
manufacturer renal disease? e. urea n itrogen immunochromatographic assay of high
a. increased BUN level specificity
116. TI,e most commoll bleeding disorders seen in b. increased serum creatinin e level e. filrration/concentration plus enzyme-linked
127. Which immersion oil is m ost appropriate/or
companion animals are reInted to: immunosorbent assay

alk
c. decreased serum calcium level normal light microscopy?
a. decreased clotting factor activity d. decreased hematocrit a. typeA
b. lhrombocytopenia e. increased serum phosphorus level 132. w:llich met/lod is least appropriate for urine
b. type B collection?
c. platelet dysfun ction c. mineral oil
d. hemophilia A and B 122. Wllich of tile following does rwt reflect liver a. midstream free catch
d. linseed oil
e. hypocalcemia function? b. manuaJ expression of the bladder
e. SAE 30 a utomotive oil
c. aspiration of urine from the cage floor or
a. serum albumin level w. Iitterbox
117. Bleeding time is the best test to detect: b. serum alanine aminotransferase activity 128. In an infected animal, life cycle stages of which d. bladder catheterization
a. platelet dysfunction c. serum alkaline phosphatase activity parasite are least likely to be found on a fecal
e. cystocentesis
b. anemia d. serum creatinine level flotation preparation?
c. platelet deficiency c. serum aspartate aminotransferase activity a. Giardia mti
133. In which type of patient should cystocentesis not
d. clotting factor deficiency b. Toxocara canis be performed?
e. reticulocytosis 123. Serum amylase activity generally is not c. 1hchuris vulpis
ww
increased in animals with:
a. 3-month-old puppy
d. Allcylostoma caninum
b. obese beagle
118. Which of the following is rwt a test afhemostasis? a. pancreatitis e. Isospora
c. female dog in heat
a. activated clotting time b. renal disease d. adult male cat with disease of the bladder
b. prothrombin time c. dehydration wall
c. partial thromboplastin time d. pancreatic abscess e. old dog with a fever
c. colitis

e 1998 Mosby-Year Book, Illc. Photocopying is prohibited by law. Correct answers are on pages 55-62.

www.alkottob.com
48 SECTION 2 www.vet4arab.co.cc Cli"ical Pathology 49

134. A urine specimen with afruity odor is most likely c. I minute at 3000 rpm 143. You test a eat's serum for feline leukemia virus 146. Concerning cats whosesafiva tests are positive
to cantail!: d. 400 RCF (relative centrifugal force) for 5 usillg a microlVelf-type enzyme-linked for feline leukemia virus, which statemellt is
minutes immullosorbelH assay kit. Color develops in the most accurate?
a. metabolized fruit juice

m
e. Time and speed are of no significance. negative con trol and pariellt sample wells, as
b. bacteria well as in the positive well. What is the most a. They will soon die.
c. hemolyzed blood likely cause of these results? b. They are only tranSiently infected.
d. acetone 139. Cystine crystals are most likely to befound in the c. They have a latent infection.
a. failu re to properly time the test
urine 0/ d. They can transmit the virus to other cats.
e. myoglobin b. inadequate washing of wells after addition of
a. female collies e. They are probably not contagious.

.co
the enzyme conjugate
135. WllUl is tile nomlUl pH of canine and feline b. fema le Doberman pinschers c. prolonged storage of the kit at room
urine? c. female cocker spaniels temperature 147. When imerpretillg the results ofan
d. male dachshunds d. nonspecific cross-reactive antibodies in the antimicrobial susceptibility test, a drug that is
a. 7 or greater appropriate for treatment is indicated bya disk:
c. male labrador retrievers eat's serum
b. 7 or less
e. failure to add the enzyme conjugate a. with a large zone of inhibition
c. 8to 10
140. CAlcium oxalate crystals in urine sediment are b. with a small zone of inhibition
d. 10 or greater
mosloftell associated with: 144. A 7-year-old outdoor male cat has a historyalld c. wi th a zone diameter designared as
e. highly variable p H, from 4 to 9

ob
clinical sigm strongly suggestive of heartworm inhibitory, according to standard charts for
a. ethylene glycol (antifreeze) loxiciry
infection. However,filter testsfor heartworm ant imicrobial susceptibility
136. Wilen present in urine, wlliell substance can b. gout microfilariae arid enzyme-finked d. with a zone of inhibition indicating poor
mask a positiueglucose reaction 011 urine c. bacterial cystitis immunosorberlt assay are both negative. MlDt is ability to diffi.lse through the agar
dipsticks? d. end-stage renal disease the most likeiy explanarion? e. that inhibits growth of the greatest numbe r
a. kelOnes e. organophosphate insecticide poisoning a. The cal probably does not have heartworm of organisms
b. blood infection.

ott
c. penicillin 141. All commerciallyavailable/efine leukemia virus b. The cat may have small numbers of adult 148. Preciseamimicrobial susceptibility testing
d. ascorbic acid (FeLV) lest kits designed for in-clinic use detect: heartworl11s. requires tlwt the agar plme containing the
e. creatinine a. FelVantibodies c. The test procedures were perform ed antimicrobial disks be inoculated:
b. FelV-infected red blood cells incorrectly. a. and incubated immediately after the
c. FelVantigens d. The tests are designed for use only in dogs. differential media plates have been inoculated
137. The urine nitrite test is based 011 tile ability of
hacteria co convert nitrate to nitrite and is a d. Feline oncornavirus-associated cell e. The cat is probably infected with feline b. with a standard suspension ofa single

alk
crude indicator of urinary tmct infection itl membrane antigen (FOCMA) antibodies leukemia virus. organism selected from the preincubated
people. Why is it a/limited use in dogs and cats? e. FelV-infected lymphoid cells blood agar plate
a. Dogs and cats are rarely affected by bacterial 145. You test a 2-year-old clinically normal catfrom a c. with a mixed c ulture of all the organisms
single-cat household for feline leukemia I!ims present on the preincubated blood agar plate
infections of the urinary tract. 142. A positiveenzyme-finked immullosorberIC assay
infection with an enzyme-linked d. after p rei ncubation of triple sugar-iron agar
b. The acidic p H of dog and cat urine prevents for feline leukemia virus (FeLV) may indicate
immunosorbent assay kit designed for ill-office slants and broth subcultures
this reaction. allY of tile following except:
use. The test is positive. What is the mosl
c. The bacteria tend to react with Olher
e. after preincubation ofSabouraud's dextrose
a. The cat may be transiently infected. appropriate advice for the catS owner? slants and Hektoen broth subcultures
proteins in the urine of dogs and cats.
d . The diets of most dogs and cats do not
contain sufficient nitrate 10 result in a
b.
c.
d.
w.
The cat may be chronically infected.
The cat will definitely die from FelV infection.
The cat may become a latent carrier.
a. Isolate the cat and repeat the test in I or 2
months.
b. Euthanize the cat before it develops full -
149. For IOrlg-term storage, antimicrobial sensitivity
positive reaction. disks should be held at:
e. Enzymes in the urine of dogs and cats break e. The cat may be contagious for other cats. blown infection.
a. room tempe rature
down nitrate. c. The result was probably inaccurate.
b.2"to25"C
d. Isolate the cat but do not bother to retest
ww
because the second test will probably be c. below 2" C
138. For best results, urine specimens for microscopic d. above 5" C
positjve.
analysis should be centrifuged for: e. any convenient temperature
e. Do not breed this cat.
a. 5 minutes at 10,000 rpm
b. 5 minutes at 6000 rpm

C 1998 Mosby-Year Book, Inc. Photocopying i5 prohibited by law. Correct answers are on pages 55-62.

www.alkottob.com
50 SECTI O N 2 www.vet4arab.co.cc Clin ical PalllOlogy 51

150. III cytologic specimens stained with c. exposure to formalin vapors 161. Associated with ethylene glycol poisoning 167. A dog has severe thrombocytopenia (20,{J(X)
Romanowsky-type sWins (e.g., Diff-Quik, d. smear that is too thick p lateletsliJ-L}. Bone marrow evaluation shows
DipStat, Wright's), Wl l iell aftlle/allowing is most e. stain or diluent that is too acidic 162. MIen presellf in urine, Bence-Jones protein is megakaryocytes with normal morphology

om
likely to result ill excessive pin k staining? detected by: present i" normal to increased numbers. The
a. inadequate washing of stained slide most appropriate course ofaction is to:
a. spectrophotometry
b. delayed fixation b. suJfosalicylic acid test a. submit blood fo r an antimegakaryocyte
R.E. Ras kin antibody test
c. urine dipstick assay for protein
b. dete rm ine the activated clotting time
151. A dog is polydipsic. and you suspect renal d. hyperadrenocorticism d. microscopic examination of urine sediment
c. carefully scan the blood smear for evide nce
disease. You perform a urinalysis alldfind the e. hypothyroid ism e. freezing the urine to detect cryoprecipitares
of e hrlichiosis
specijicgravity is 1.010. You conclude tllat this
animal's urinc specijicgraviry: d. submit blood fo r prothrombin time,
163. Whicll test is useful in detecting retial tubular

b.c
ISS. Bilirubill itl tlleuritleofdogs: activated partial throm boplasin time, and
a. is within normal limits for dogs dysfunction? fib rin degradation products assay
a. is abnormal, even whe n present in trace
b. indicates isosth en uria amounts a. BUN assay e. give corticosteroids to control immune-
c. has been affected by consumption of water b. is only detected aft er th e dog becomes icteric b. urine protein /creatinine ratio m ediated destruction of platelets
d. indicates the patie nt 's kidneys retained their c. creatinine clearance
c. consists primarily of th e unconjugated form
abiliry to dilute urine d. fractional albumin clearance 168. WI/icl/flo tatioll solution is best to usefor
of bilirubin
e. is insufficie nt to determine the concentrating e. fractio nal sodium excretion qualitative exam ination offeces?
d. is increased with mild anemia
ability of the kidneys without additional tests
e. is fou nd with severe cholestatic d isease a. distilled water (specific gravity 1.000)

tto
/ 64. Glucosuria has been associated with all of tile b. sod ium chloride solution (specific gravity
/52. /n a horse with suspected excreianaf myopathy. following except: 1.050)
tile red-brown u rinc is positive for occult blood 156. WIlen urine is collected from a normal atlimal
by cystocelltesis, any of tile following is likely to a. heartworm disease c. acetic acid solution (specific gravity 1.100)
and protein using a dipstick test. Wh ich
additio nal piece of inform arion supports the be found except: b. urinary hemorrhage d. sugar solution (specific gravity 1.300)
diagllosisofmyoglobinuria and 1Iot a. sperm e. copper sulfate solution (specific gravity
c. nephrotoxins causing renal tu bular disease
Ilemogiobinuria? 1.500)
b. le ukocytes (3/high -power fie ld) d. uri nary obstruction in cats

ko
a. A d ear supe rnatant occurs when ammonium c. bacteria e. intravenous infusio n of dextrose-containing
sulfa te is added to the urine. d. hyaline casts (2/ Iow-power field) fluids 169. How soon after infection with hearrworm
b. There is an increase in serum sorbitol m icrofilariae are serolOgic test results positive?
e. e pithelial cells (2/ 10w-power fi eld)
dehydrogenase activity. 165. Wldch anioll is specifically measured by the a. 2 weeks
c. There is an increase in serum creatine kinase urine reagent strip fo r ketones? b. I month
For Questions 157 through 161, select the correct
activity. answe r from the five choices below. c. 3 mo nths
a. o-Iactate

al
d. Horses do not exhi bit hemoglobinuria. d. 6 months
a. triple phosphate b. o-glucuronate
e. The plasma appears pink. e. 1 year
b. calcium carbonate c. acetone
c. calcium oxalate d . acetoacetate
153. Tile urine of horses is normally thick a nd cloudy.
d. ammon ium biurate e. l3- hydroxybutyrate
This is ca used by the presellce of
a.
b.
e pithelial cells and mucus
epithelial cells and crystals
e. bilirubin
w.
157. Foulld normally in herbiuores (e.g., horses, rabbits)
166. A practical a nd sensitiue way to evaluate a
patie1ll's platelet function in the clinic without
c. mucus and crystals senditlK blood samples to tlte laboratory is by:
d. leukocytes and normal bacte rial fl ora
158. Associated with portosystemicsh unts a. clot relraction
e. leukocytes and hyaline casts
b. bleeding time
ww
159. Forms golden·yellow needle-shaped crystals j tl
c. von Willebrand's factor assay
154. Proteinuria may befound;1/ animals with: d. platelet count
uritle
a. a fever e. activated clotting time
b. diabetes inSipidus
160. Found jtl alkaline urine ofdogs and cats
c. a portosyste mic shunt

10 1998 Mosby-Year Book, Inc. Photocopying is pro/libited by law. Correct answers are on pages 55-62.

www.alkottob.com
52 SECTI O N 2 www.vet4arab.co.cc Clinica l PatilOlogy 53

170. Omcemirlg the microjiiariaeofDirofilaria d. have no adverse effects on cellular detail, 178. !fa dog's PCVis 15% and the reticulocytecounr CI 87 mEq/L
immitis alld Dipetalonema reconditum, which e. give cell oUllines a crisper image is 12%, the corrected reticulocyte count is: Ca 8.7 mg/dl
statement is most accumte? a. 10% P 12.1 mg/dl
Total CO 2

om
14 mEq/L
a. Microfilariae of Dirofilaria immitis move 174. Particle smears made from bone marrow b.8% BUN 86 mg/dl
progressively. whereas those of aspirate samples provide all the following c.6% Glucose 73 mg/ dl
Dipelalonema recondilllm move but remain information except: d.4%
in place.
a. degree of marrow fi brosis e.2%
b. Microfilariae of Dirofilaria immitis are longer 182. W1wt is (he anion gap in this animal?
b. proportion of hematopoietic cells to adipose
and wider than those of Dipetalonema a. 20 mEq/ L
recOflditum.
tissue 179. CBC in a cat reveals the/aI/owing: PCV 1096;
c. megakaryocyte numbers b. 32 mEq/L
c. The tail of Dirofilaria immitis microfilariae is MCV 48 fl; MCHC 34 gldI; aggregate reticulocytes
d. hemosiderin content 0.1% or IO,OOO/iJL 50 nucleated RBCsIlOO WBCs; c. 45 mEq/L
curved or hooked, whereas th at of

b.c
Dipetalonema reconditum microfilariae is e. myeloid/erythroid ratio normal erythrocyte morphology; and 90{)() d. 57 mEq/L
straight. WHCs/Ili-. Concerning this hemogram, which e. 66 mEq/L
d. Microfilariae of Dipetalonema reconditum
statemer/l is most aCCllrate?
175. Equipmem needed to perform a complete blood
are more common in canine blood than COlt /It i"cludes all ofllle/ollowing except a: a. The n umber of aggregate reticulocytes Question 183
those of Dirofilaria imm itis. indicates regenerative anemia.
c. Microfilariae of Dipetalonema reconditum a. microscope A dog has IITefollolVing blood chemistry values.
b. The number of nucleated RBCs indicates
have a tapered head, whereas those of b. cell counter regenerative anemia. Ca 7.9 mg/dl
Dirofilaria immitis have a blunt head. c. high-speed centrifuge c. The MCV indicates the bone marrow is Total plasma protein 4.0 g/dl

tto
d. refractometer responding to anemia by RBC regeneration. Albumin 1.8 g/dl
171. Conceming the direct Coombs' test, which e. 37"-C waler bath d. The PCV suggests moderate anemia for a cat. Globulin 2.2 g/d]
statement is least accurate? e. The corrected WBC count is 6000/ J.lL
a. It is performed at 4°, 20", and 37" C. 176. Conceming hemograms seer! ill IIQrious 183. W1lat is the corrected calcium value?
conditiollS, which statement is least accurate? 180. Increased serum alanilleaminotTansferase a. 7.6 mg/ dl
b. It requires use of species-specific reagen lS.
c. It is affected by administration of a. Eosinophilia may be associated with allergic activity is common with all of the/allowing b. 8.3 mg/dl
l!XCept:

ko
corticosteroids. or hypersensitivity reactions. c. 8.6 mg/ dl
d. It is perfonned on serum from the patient. b. Eosinophilia may be associated with a. abdominal trauma d. 9.3 mg/dl
e. It is positive when hemagglutination is parasitic infections. b. immune-mediated thrombocytopenia e. 9.6 mg/dl
observed. c. Basophilia is associated with IgG-mediated c. aClite pancreatitis
disorders.
d. severe skeletal muscle injury 184. Hypercalcemia has been associated with all of
172. Cotlceming the antinuclear antibody test, which d. Monocytosis is associated with chronic
e. hemolysis of the blood sample the following conditions excep t:
inflammatory d iseases.

al
statement is most accurate?
e. Persistent lymphocytosis may be associated a. lymphoid malignancies
a. It may be less sensitive than the lupus with bovine leukemia virus infection in cattle. 181. Acidosis is most likely to produce which change? b. anal sac apocrine-gland carcinoma
erythematosus ceUtest. c. hypoadrenocorticism
a. shift of potassium from extracellular to
b. It is not affected by large doses of exogenous intracellular d. fungal infections
corticosteroids. 177. Concemillg loxic neutTophils, which statement
w.
is least accurate? b. shift of potassium from intracellular to e. milk fever
c. It does not require use of species-specific
extracellular
reagents. a. Their presence suggests inflammation.
b. Their presence is usually related to localized c. shift of sodium from extracellular to 185. You suspect that a neonatal/oal has a ruptured
d. It is not positive in healthy dogs or cats.
intracellular bladder; and you collect fluid by
e. It is not specific for systemic lupus or systemic bacterial infections.
d. decrease in serum ionized calcium level abdominocentesis. ASsay for which biochemical
erythematosus. c. Changes in the neutrophil cytoplasm arise
from disturbed maturation of neutrophil e. increase in total serum calcium level constituent of lhe fluid is most likely to aid
ww
precursors. diagnosis ofbladder rupture?
173. Formalinfumes in the vicinityoffreshly made
blood films: d. Dohle bodies indicate severe toxicity. Question 182 a. sodium
e. The prognosis is more favorable when A patient has the /ollowing blood chemistry profile:
b. phosphorus
a. have no adverse effects on staining neutrophil numbers and the severity of c. potassium
b. alter the staining features of erythrocytes toxicity dimin ish over time. N. 153 mEq/L
K 5,4 mEq/L
d. urea nitrogen
c. enhance the staining features of leukocyte e. creatinine
nuclei

C 1998 Mosby-Year Book, Inc. Pllotocopying is prohibited by law. Correct answers are on pages 55·62.

www.alkottob.com
54 SECTION 2 www.vet4arab.co.cc Clinical Pathology 55

186. A direct smear preparation is generally not c. a long-standing pathologic condition /95. A dog's transtracheal wash maycofllain farge 198. III tests determinillg the semitiuityofbacleria to
advisable for cytologic examination of: d. normal canine neutrophils numbers ofeosinophiis in any of tile following antimicrobial agents, a bsellceofa clear ring
e. a degenerative change conditions excep t: around tllealltimicrobial disks indicates:
a. pleural fluid

m
h. pericardial fluid a. heamvorm·d isease a. contaminalll bacteria
c. peritoneal fluid /91. Mononuclear pleocytosiS is mosllikely to be b. a1lecgic reaction b. a slow-growing organism
d. cerebrospinal fluid observed in an animal willi: c. lung fluke infection c. antimicrobial susceptibili£}'
e. synovial fluid a. canine distemper d. Toxocara migration d. intermediate antimicrobial susceptibi lity
e. Mke nnel cough~ e. antimicrobial res istance

.co
b. feline infectious peritonitis
187. Qmcerning trallSudates, which statemem is c. streptococcal meningitis
least accurate? d. cryptococcal meningitis 196. Concemillgcollection of samples for 199. Concerning examination ofskin scrapillgsfor
e. toxoplasmosis microbiologic CIIlture, which statemem is least mites, which stalemel/t is least accurate?
n. Their protein concenuation is <2.5 gtdl. accurate?
b. Cell counts are <500/ fJ.L in normal dogs. a. AHIO scalpel blade may be used for skin
c. In normal dogs most of the cells are 192. Concerning protein in cerebrospinal fluid, which a. Cystocelllesis is preferred to catheterization scrapings.
statemelH is least accurate? to obtain urine. b. Mineral oil may be placed on the lesion to
neutrophils.
d . They arc associated with hypoalbuminemia b. Transtracheal aspiration may be used to aid recovery of parasites.
a. Protein levels can be estimated using tuin ary

ob
<1.0 g/d!.
obtain lower airway specimens. c. Whcn looking for Demodex mites, the skin
reagent strips.
e. They are associated with portal hypertension c. A su rgical scrub of the skin is required before should be firmly pinched to help express the
b. An increased protein level is seen with collection of samples from a lesion of
secondary to hepatic insufficiency. inflammatory conditions. mites from hair follicles.
supecficial pyoderma. d. Scrapings for Sarcoptes must be made deep
c. The Pandy test is used to measure globulin
d. Exudate from deep cutaneous lesions may be into the dermis because th e parasites reside
188. uJrlceming an acute or recem hemorrhagic levels. aspirated with a sterile needle and syringe. within hair follicles.
effusion, which scatement is least accurate? d. Protein levels are routinely measured as total
c. Several blood samples should be obtained e. TIle slide is examined under low power (lO x J.
solids with a refractometer.

ott
a. It has a clear supernatant and red sediment. for aerobic and anaerobic cultures 10 rule out
e. Protein levels may be increased without an bacterial septicemia.
b. Intact eryth rocytes may be present.
increase in cell counts in degenerative 200. Collcernillgdiagllosis ofdermatomycosis, which
c. It contains macrophages with phagocytized condi tions. statement is least accllrate?
erythrocytes. 197. 171e O/IQ of wllich parasite have a flattened
d. It contains hemosiderin -laden macrophages. operculum at one end? a. When present, ultraviolet fluorescence is
193. WI,iel, cell type is likely to be prese1lt in highest best seen at the tip of intact hairs.
e. It contains platelets. a. Eimeria
numbers in a normal transtracheal wash? b. Hairs should be plucked from the periphery

alk
b. Paragonimus kellicotti of the lesion.
189. Concemillg tile viscosity ofjoiTll fluid in horses, a. goblet cells
c. Trichuris vulpis c. A few drops of 10% potassium hydroxide may
wllich statement is least accurate? b. ciliated columnar epithelial cells
d. Strongyloides stercoralis be used to clear debris from the specimen.
c. alveolar macrophages
a. Hyaluronic acid is most responsible for joint e. Giardia d. Arth rospores on the hair shaft are best seen
fl u id viscosity. d. neutroph ils wi th the 40 X lens.
b. Viscosity is decreased by certain bacterial e. lymphocytes e. The appearance of conidia, location of
enzymes. arth rospores, and a bility to fluoresce are
c. Viscosity is decreased by dilurion of synovial 194. A transtracheal wash from a dog wich a chronic lIsed to distinguish the different
fl uid with blood.
d. Synovial fl uid appears yellow because of likely to be characterized by:
w.
cough associated with a collapsed trachea is dermatophytes.

decreased viscosi£}'. il. epithelial cells and mucus


e. Viscosi£}, may be estimated by the string test
or mucin clot test
b. mast cells and epithelial cells Answers
c. neutrophils and mucus 1. e Bone marrow iron stores are normal to 4. b The other diseases listed are associated with
ww
d. neutrophils and eosinophils increased, but macrophages sequester the iron, intravascular hemolysis.
190. [n a dog hypersegmemed neurropllils i" an e. eosinophils and mast cells so iI is not available fo r erythropoiesis.
effusion indicate: 5. d Neutrophils remain in circulation about 10
2. a MCV=PCV/ RBC x 1O=48%,MCHC=Hb/PCV hours.
a. sepsis X 100 = 42 gfdJ. 6. a Estrogen toxicity is associated with early
b. immune-mediated disease 3. c Red maple causes hemolytic anemia in horses, neutrophilia, followed later by decreased
often associated with Heinz bodies andlor production (aplastic anemia) and neutropenia,
methemoglobin. anemia, and thrombocytopenia.

C 1998 Mosby-Year Book, Inc. Photocopyillg is prollibilCd by law.

www.alkottob.com
56 SECTION 2 www.vet4arab.co.cc Clinical Pathology 57

7. b Heinz bodies do not interfere with plasma 22. b Epine phrine causes mature, mild leukocytosis 37. c An increased triglyceride level. with a 53. e A portosystemic shunt causes increased bile
protein values, but they can erroneously elevate (neulrophilia in dogs and lymphocytosis in cats fluidlse rum triglyceride ratio a bove I, is the most acid and ammonia levels and low albumin and
hemoglobin values. and horses) and an increased PCV reliable test indication of chylous effusion. BUN levels. Other common findings include

om
8. d Corrected WBC count = 23. a Glucocorticoids cause mature neutrophilia. 38. e Horses normally have neutrophils in their ammonium biurate crystals in the urine, target
ObservedWBC x 100 30,000 x 100 30,000 lymphopenia, monocytosis, and eosinopenia. abdominal flu id. celis, microcytosis. hypoglycemia, and
24. c Decreased bone marrow production is 39. e Hypoalbuminemia causes a clear transudate. hypocholeste rolem ia.
Nucleated RBe + 100 200 + 100 300 associated with neutropenia without a left shift. 40. c Lymphosarcoma produces a modified 54. d Bromide (used to treat seizures) poisoning
=10,000 25. e The hallmarks of inflammation with a causes a falsely elevated chloride level.
9. a Ehrilichiosis can be associated with benign transudate characte rized by blast cells.
degenerat ive left shirt are decreased numbers of 41. b Peritonitis causes a suppurative exudate. 55. a Acidosis is associated with increased calcium
lymphocytosis and monoclonal or polyclonal
WBCs. with immature cells (bands and levels. especially ionized calcium.
gammopathy. 42. a Red fluid with platelet clumps but no
metamyelocytes, etc.) oU (llumbe ring mature 56. c Anion gap = (Na + K) - (Cl + total CO 2),
10. d Ferritin is the preferred lest, hut it is species erythrophagia typifies contamination with
neutrophils.

b.c
specific and often not available. Bone marrow blood. 57. e Ethylene glycol causes a greater anion and
26. d Inflammation and stress are typified by an osmolar gap than the other disorders listed. An
iron stores are increased in chronic 43. d Green fluid with plant material and bacteria but
inflamm atory leukogram (neutrophilia, left shift. estimate of osmolality is l.86(Na + K) + BUN/2.8 +
inflammatory disease. no inflammatory cells typifies bowel puncture.
toxic changes) and lymphopenia . Glucosell8 =323, yielding a gap of 411 - 323 = 88.
11. e The PCV is higher than anticipated for relative 44. c CK is the most sensitive and specific indicator
27. e A dipstick test for blood detects RBCs, 58. a Ammonia levels are a measure of liver function
polycythemia (answers b and d) . Erythropoietin of muscle damage. Activities of AST and LDH
hemoglobin, and myoglobin. (rema ining viable cells). Activities of some
levels should be elevated with secondary both rise more slowly and are nonspecific
polycythemia (answers a and c) , 28. c The last two digits of the specific gravity of the (m uscle and liver). Activity ofLDH is high in enzymes are a measure of liver dysfunction
diluted sample (30) are multiplied by the dilutio n RBCs. so minor hemolysis may give erroneous (damaged cells) .
12. d A blood sample anticoagulated with EDTA is
factor (2) to yield the specific gravity for the elevations, especially in dogs but less likely in 59. eAST is used to evaluate liver disease in large
required. C3 coating ofRBCs can occur if clotted

tto
und iluted sample. horses. and small animals.
blood is left in the refrigerator.
29. c Amyloidosis causes marked proteinuria, with 45. d Trypsinlike immunoreactivity reflects 60. b ALP activity is used to evaluate cholestatic
13. c The antinuclear antibody test is more specific
granular casts and retained a bili ty to concentrate circulating trypsinogen and tryps in levels. his disease in small animals but is not a valuable test
and sensi tive than (he other tests listed.
the urine (glomerulotubular imbalance). not affected by oral administration of pancreatic for cholestatic disease in large animals.
14. a Cats with rype-B blood have naturally
30. e Myoglobin imparts a brown color to the urine. extracts; there is no need to halt use o f such 6 1. c ALT is used to evaluate liver disease in small
occurring anti-A antibodies.
4+ blood without RBCs indicates myoglobinuria products before testing. animals but not in large animals.
15. a IgG is the prevalent immunoglobulin in mare or he maglobinuria. An ammonium sulfate 46. a Young animals have high levels of thyroid 62. d Elevated urinary GGT activity indicates

o
colostrum. preCipitation test is needed to differentiate these hormone. tubula r damage. Serum GGT activity is also
16. a In neonatal isoerythrolysis, the mare's serum two (precipitates the hemoglobin). Also, the
contains antibodies against the foal's RBCs. 47. c Potassium excretion is decreased with valuable in evaluating liver disease. especially in
blood serum should be colorless rather than pink large animals.
hypoadrenocorticism and renal failure / urethral
17. e Acute blood loss causes regenerative anemia with myoglobinuria. and the history should be

alk
obstruction. Potassium is redistributed from cells 63. e GI hemorrhage can increase the BUN level
(increased reticulocytes. often with macrocytic compatible with a muscle disorder.
hypochromic indices) and decreased total to serum in acidosis. crush injuries. or cytosis. without increasing the creatinine level. A low
31. b Pyelonephritis is inflammatory; increased Sodium loss can occur with hypoadrenocorticism , total plasma protein level supports a diagnOSis of
plasma protein levels. WEC numbers with relatively few RBCs indicate diarrhea, vomiting, polyuria, or bums. blood loss.
18. c He molytiCanemia is regenerative, often with inflammation.
48. e Cholecalciferol causes vitamin D tOxicity, with 64. c Low pre- and POSl-ACTH cortisol levels
macrocytic indices and decreased platelets. The 32. d 4+ glucose with ketones suggests diabetes hype rcalcemia. Primary hyperparathyroidism characterize iatrogenic Cushing's disease.
total plasma protein level and MCHC are mellitus. Glucose in the urine increases the and pseudohyperparathyroidism are associated 65. a Larvae recove red from fres h feces of large
artificially elevated in he molyzed plasma. specific gravity. with hypophosphate mia. animals are almost always those ofiungworms.
19. b Anemia of ch ronic inflammatory disease is 33. a This combination ofresuhs indicatesw. 49. c Metabolic alkalosis with respiratory The Baermann technique is not recommended
associated with le ukocytosis (ne utrophilia). hematuria. compensation shows an alkaline pH with fo r recovery of Filaroides from the fe ces of dogs.
increased total plasma protein level, mild 34. a Calcium oxalate crystals support a diagnos is of increased HCOl and Pc0 2. 66. c The cellophane tape method is used to detect
anemia (often normocytic normochromic. with a ethyle ne glycol toxiciry.
tendency toward microcytic hypochromic 50. b Respiratory alkalosis shows an alkaline pH eggs of pinworms. Horses are the only domestic
35. c FlP is associated with a moderately cellular with decreased Pc0 2 • species infected by pinworms.
indices), and often increased platelet numbers.
effusion , often of mixed cell type and with a high 5 1. d Metabolic acidosis and respiratory acidosis
20. d Relative polycythemia has an increased PCV 67. e Trophozoites lack rigid walls and may collapse
protein level. On smears the background has a
ww
(not marked) and total p lasma protein level show an acidic pH, decreased HeOl • and and be difficult to identify on fecal flotation . Wet
characteristic eosinophilic s tippled a ppearance. increased Pc0 2•
when associated with dehydration. mounts allow recognition of movement patterns
36. c Postsurgical abdominal fluid is normally 52. a Metabolic acidosis with respiratory to differentiate groups of protozoa. Addition of
2 1. a Aplastic anemia is associated with pancyto- associated with high cell counts (often above
penia (decreased WBC. RBC, and platelet compensation shows an acidic pH \vith iodine facil itates identification of protozoa.
100.000/ .... L. with more than 90% neutrophils). decreased HC0 3 and Pco 2 •
numbers). 68. b The McMaster technique is the most accurate
Degenerative neutrophils. intracellular bacteria,
and commonly used method to quantitate eggs
and fecal material indicate a pathologic process. in feces.

o 1998 Mosby- Year Book, Inc. Photocopying is prohibited by law.


www.alkottob.com
58 SECTION 2
www.vet4arab.co.cc Cli nical Pathology 59

69. d Fecal culture causes sporulation of coccidial 86. e Fluoride inhibits glycolysis and should be used 102. c Protein accumulation in the aperture can 11 7. a Bleeding time is also prolonged with platelet
oocysts and is used to distinguish Eimeria (four for glucose testing if serum sample cannot I?e result in e rro neous hematocrit readings and an deficiency, but a direct count of platelets is a
sporocysts) from Isospora (two sporocysts). immediately separated from the red cells or if a increased freq uency of obstructions. better method to assess thrombocytopenia.

om
70. e Sodium nitrate is the most efficient solution routine plasma sample cannot be immediately 103. d Con tamination of the diluent with bacte ria or 11 8. e Erythrocyte sedimentation rate is not a test of
for fecal flotations. analyzed for glucose. other particles results in countin g e rrors. As the hemostasis.
71. a Size is the best criterion to differentiate the 87. c A heparinized sanlple can be used for count inc reases, errors increase expone ntially. 119. e Serum values of potassium, aspartate
microfilariae of Dirofilaria from those of electrolyte analysiS, but serum is the preferred 104. e The enti re system includes the reagent, aminotransferase, and phosphorus increase as
Dipetalonema (microfiiariae of Dirofilaria are sample. instrument, and operator. a result of red blood cell leakage a nd hemolysis.
longer and wider). 88. e Periodic acid-5chiff stain is used to stain 105. b Although a ll of the other choices can also Glucose levels decrease up to 5% pe r hour as a
72. d Macrolides (milbemycin and ivermcctin) fungal structures. cause errors, inadequate mixing is the most result of anaerobic glycolysis by red blood cells.
interfere with production of m icrofilariae by adult 89. a New methylene blue is used to stain common error. An automatic specimen rotator 120. b Large-bore needles cause "spiraling~ of blood
heartwonns. The antigen test is highly sensitive reticulocytes. improves mixing. cells as they enter the needle. Alcohol and

b.c
and specific and does not produce as many false· 90. b Romanowsky-type stains (e.g., Diff-Quik, 106. d The avian heterophil is analogous to the rough handling can result in cell lysis. A 10-ml
positive and false-negative reactions as the DipStat , Wright's) are used for routine staining mammalian neutrophil. tube would likely create too much negative
antibody test when screening fo r infections. of cytologic specime ns. Papanicolaou staining 107. b The metamyelocyte is a granulocyte pressure with subseque nt hemolysis, when
73. a Insufficient sample is the m ost common error is more time consuming and technically precursor. The other cells a re etyth rocyte collecting blood from a small a nimal.
associated with submission of fecal samples. difficult. precu rsors. 121. c Some renal diseases are related to increased
Fastidious microaerophilic or anaerobic 9 1. d Natt and Herrick's solution or phloxine B 108. a Physiologic leukocytosis occurs whe n serum calcium levels.
organisms cannot usually be recovered from stain is used for avia n WBC counts. marginated granulocytes e nter the general 122. d The serum creatinine level is a bener
swabs. 92. b Romanowsky-type stains (e.g., Diff-Quik, circulatio n as a result of excitemen t or stress. assessment of renal function.

tto
74. b Antimicrobial testing distinguishes drugs that DipStat) are used for routine staining of blood 109. d All avian erythrocytes are nucleated. 123. e Because amylase is eliminated through the
maybe useful from those that a re not. smears. 110. e The metamyelocyte is a granulocyte kidneys, any disorder that reduces renal blood
75. e Samples for aerobic culture should be 93. c The PIVICA test (proteins induced by vitamin K precursor. The other cells are a Uof the flow or impairs kidney function can result in
refrigerated and stored in transport medium. antagonism) is a modified prothrombin test that erythrocyte series. elevated serum amylase activity.
Samples for anaerobic culture should be stored is very sensitive in diagnosing vitamin K toxicity. Ill. c Immature forms include band and stab cells. 124. c Organophosphate insecticides can be
in transport medium at room tempe rature. 94. e Factors II, VII, IX, a nd X a nd protein Care 112. d Eosinophilia occurs commonly in response cholinesterase inhibitors.
Tissues should be f.roze n. vitamin K dependent. to antigen-antibody reactions, as well as with 125. b Anion gap = (Na + K) - (el + Total CO 2),

o
76. d Scraped mate rial should be added to mineral 95. e Assay for vWF antigen is the best test to inflammation of certain organs, such as the expressed in mEq/L
oil. diagnose von Willebrand's d isease. lungs, which tend to be allergy targets. 126. d Ammonia (NH]) is also unstable. Such
77. d Culture is me ultimate diagnostic test for 96. c Factor VII deficiency causes increased 113. e Calibration should be performed every I to 3 enzymes as alanine aminotransferase and

alk
ring-...onn. Microsporum ctmis is the only common extrinsic (prothrombin time) test times. months to account fo r changes in the aspartate aminotransferase usually show some
dennatophyte of dogs and cats that fluoresces on 97. e Factor VITI deficiency causes increased centrifuge that occur as a result of wear a nd loss of activity after a few days unless the
exposure to a Wood's (ultraviolet) light. intrinsic (activated partial thromboplastin time) tear. reconstituted control is frozen.
78. d Otodectes causes otitis externa in dogs and cats. test times. 114. d Trichrome stain is typically used for 127. b Type B is a high-viscosity oil and is the best
79. e Knemidokoptes causes scaly leg a nd 98. d Factor X defici ency causes increased extrinsic visualization of parasites in feces. choice for light microscopy.
depluming scabies in birds. (prothrombin time) , intrinsic (activated partial 1l5. e Because blood collection tube manufacturers 128. a Giardia organisms a re more likely to be
80. c Sarcoptes mites deposit their eggs in the deep thromboplastin time), and common pathway are now producing fewer tube s izes than found in direct saline smears unless zinc sulfate
e pidermis and cause intense pruritus. test times. previously, it is more importan t than ever to p ay is used as the flotation medium.
81. b Psoroptes mites do not burrow.
82. a Demodex mites have a classic cigar shape and
are found in hair follicles, sebaceous glands, and
w.
99. a Disseminated intravascular coagulation is
characterized by increased prothrombin and
activated partial thromboplastin times, increased
fibrin degradation products, and decreased
attention to the recommended fill volume
printed on the package. For example, several
commonly used lavender-top (EDTA) tubes o n
the market have an internal volume of 5 m1 but
129. a In terms of reliability, finding the characteristic
Ehrlichia morulae on a blood smear examination
confimls an active infection. A negative blood
smear, however, is inconclusive. Elevated
the e pide rmis.
83. c Heparin is the anticoagula nt of choice for platelet numbers and fibrinogen levels. are used to collect only 2 to 2.5 ml of blood a nd Ehrfic/tia antibody titers can persist for many
blood gas analysis. 100. b Immune-mediated thrombocytopenia contain enough EDTA to anticoagulate only 2 to months after the animal has apparently
ww
causes decreased platelet numbers but does not 2.5 mJ of blood. Filling such a 5-mJ tube recovered. A1mough suggestive of infection,
84. a EDTA is the anticoagulant of choice for routine completely could result in a clotted specimen;
affect prothrombin time, activated pa rtial antibody titers are not as sensitive for detecting
CBCs. therefore 2.5 ml (o r half the tube volume) is the active infection.
thromboplastin time, or fibrin degradation
85. b Citrate is the anticoagula nt of choice for m aximum fi ll recomme nded by the
products. 130. e Enzyme-lin ked immunosorbent assays and
routine coagulation tests. manufacturer.
101. a Lavender-top tubes contain EDTA the newer immunochromatographic assays
anticoagulant. 116. b Th rombocytope nia refers to a reduct jon in detect adul t heartwonn antigen, not
platele t numbers. micro filaria e. These tests are not affec ted by the
presence nor a bsence of microfilariae.

10 1998 Mosby-Year Boole, Inc. Photocopying is prohibited by law.


www.alkottob.com
60 SECTION 2 www.vet4arab.co.cc Clinical Pathology 61

131. d In a low-incidence population, a test with a 148. b Direct sensitivity tests (those performed with 165. d Some reagent strips can detect acetone as 180. b Alanine aminotransferase is not liver specific;
high positive pred ictive value is the best choice. mixed cultures) may only yield equivocal well as acetoacetate, but the reactions are most serum ALT activity may be increased with
Tests of high specificity have a high positive results. specific fo r acetoacetate.l3-hydroxybutyrate is severe skeletal muscle disorders.

om
predictive value. 149. c Unopened disk cartridges are best stored in not detected in tllese reactions. 181. b Acidosis increases ionized calcium levels but
132. c Urine specime ns collected from the floo r or the freezer. Once cartridges are opened, 16G. b ~si ng the buccal mucosa and a cutting does not change total calcium levels.
Iiller box arc likely to be contaminated. however, it may be more convenient to store deVIce, bleeding time is highly sensitive in
them with the dispenser in tlle refrigerator. 182. d Anion gap = (Na + K) - (CI + bicarbonate).
133. d Needle puncture of the wall of a diseased. detecting platelet function abnormalities and Tota] CO 2 can be substituted for bicarbonate In
bladde r may predispose 10 bladder rupture. 150. e All other choices listed would tend to cause severe thrombocytopenia. Clot retraction also this case, anion gap = (I 53 + 5) - (87 + 14) = '158
134. d The urine of diabetic patients with excessive blue staining. may be used but is more crude and less - 101 =57.
ketoacidosis may have a ufruity" odor. 15 1. e Additional tests suggested include BUN or quantitative than bleeding time.
183. e Corrected calcium (mg/eIl) = Calcium -
135. b The diet of carnivores usually produces serum creatinine assay, repeat urine specific 167. d Disseminated intravascular coagulation must Albwn..in + 3.5. In this case, corrected calcium =
gravity testing, and a water deprivation test, if first be ruled out before immune-mediated
acidic urine (pH less than 7.0), 7.9 - 1.8 + 3.5 =9.6 mg/d1. Alternatively,

b.c
clinically advised. thrombocytopenia is considered. Ehrlichiosis is Corrected calcium (mg/dl) == Calcium - (0.4 X
136. d Dogs and cats can synthesize ascorbic acid;
152. c Myoglobinuria is best diagnosed when there best diagnosed by serologic testing.
therefore an ascorb ic acid- resistant urine tota] protein) + 3.3, which in tllis case = 7.9 - 1.6
dipstick is best for veterinary use. is clinical or laboratory evide nce of muscle 168. d The preferred specific gravity is between + 3.3 = 9.6 mg/dl.
damage, as indicated by an increase in serum 1. 100 and 1.350.
137. d Nitrate occurs naturally in plants. Dogs and 184. e Certain fu ngal infections, such as
cats are primarily carn ivores. creatine kinase activity. 169. d Results may be positive as early as 5 or 6 blas tomycosis, have been associated wi th
153 . c Mucus and crystals tend to make equine mon ths aft e r infection but usually not until 6 or hype rcalcemia.
138. d ReF refers to relative centrifugal fo rce. An
urine thick and cloudy, especially if the sample 7 months after infection.
RCF of 400 can be obtained using a centrifuge 185. e Urea is easily diffusible across the peritoneal
with a 6-inch- radius arm rotated at 1500 rpm. cools for some time after collection. 170. b Microfilariae of Dirofilaria immitis are membrane and has a low rate of synthesis in

tto
154. a Transitory, mild proteinuria may be related to approximately as wide as the diameter of a fo.als. Creatinine is a relatively larger molecule
139. d Cystinuria occurs almost exclusively in male
extrarenal factors that can affect glomerular cani.n.e erythrocyte. They exhibit stationary With poor diffusion across the peritoneum.
dogs. Dachshunds, basset hounds, Chihuahuas.
Yorkshire terrie rs, and Irish terriers have been permeability. motlhty and have a straight tail and a tapered 186. d Cell cou nts in uncentrifuged cerebrospinal
155. e Bilirubinuria is a common finding with head.
affected. flui d are .usually too low to count accurately;
biliary obstruction. 171. d The direct Coombs' test is perfomled on red preparations must be centrifuged before
140. a Calcium oxalate crystals may also be found in
156. c One of the advantages of cystocentesis is that blood cells collected in EDTA anticoagulant. examination.
small numbers in normaJ urine.
the sample is not co ntaminated by 172. ~ The ~ntinuclear antibody test may be positive
141. c All these kits test fo r the p27 FeLV antigen. 187. c In dogs. mononuclear cells predominate in

o
microorganisms fro m the distal urinary tract. m a .v~ n ety of chronic inflammatory diseases, in transudales.
142. c Some infected cats will die fro m FeLV addition to systemic lupus erythematosus.
infection; however, many more wiU recover and 157. b Calcium carbonate crystals are common in J88. d Hemosiderin-laden macrophages are seen in
the urine of herbivores. 173. b In smears stained with Romanowsky s tains long-standing or chronic hemorrhagic effusions.
become immune.

alk
158. d Ammonium b iurate crystals may be found in formal in fumes cause red blood cells to stain'
143. b Unbound enzyme conjugate was probably 189. d Equi ne joint fluid is normally yellow.
animals with a portosystemic shunt. green ish bl ue. Cellular detail appears indistinct.
left be hind in all three wells. The unbound 174. a Alth ough connective tissue cells may 190. c Hypersegmentalion of neulrophils is an
conjugate reacted with the added s ubstrate to 159. e Bilirubin crystals have this characteristic indication of ch ronicity.
appearance. occasionally exfoliate, the best method to
produce color. 19 L. a Pleocytosis refers to an increased leukocyte
160. a Triple phosphate crystals are the most detect fibro tic change in bone marrow is core
144. b Although several kits on the market have biopsy. coun t in cerebrospinal fluid . Mononuclear cells,
demonstrated enhanced sensitivity and have common crystals in canine and feline urine. such as lymphocytes, are most commonly seen
been approved for testing cats for heartworm 161 . c Calcium oxalate crystals are seen in the urine 175. e A 56°~C ".'ater bath is used for fibrinogen in viral in fectio ns.
of animals that have in gested antifreeze. determlllatlOn by the heat precipitation
infection, detection of s mall numbers of adult w. 192. d A refractometer is not sensitive enough to
heartworms and the low levels of associated method. A high-speed centrifuge is required for
162. b The sulfosalicylic acid p recipitation test detect the small amount of protein in
measuring the packed cell volume.
antigen are still problematic in cats. detects many types of protein, including cerebrospinal fluid.
145. a Many of these animals are transiently globulins and Bence-Jones protein. The reagent 176. c Basophilia is associated with IgE-mediated
disorders. 193. b Alveolar macrophages are more commonly
infected and seroconvert in time. dipstick fo r urine protein only detects albumin. observed in bronchoalveolar lavages.
163. e Frac(ional electrolyte excretion detects 177. d DohJe bodies indicate mild tOxicity.
146. d FeLV is transmitted primarily through the 194. c These are the most common findings in
saliva of infected cats. increased or decreased resorption by the renal 178. d Corrected % reticulocytes = % reticuJocytes
washes from dogs with a collapsed trachea.
ww
tubuJes. X Patient's PCV/Normal PCv. In this dog,
147. c The extent to which an antimicrobial diffuses 195. e Eosinophils are not characteristic of washes
through agar is a property of the individual 164. a Glucosuria may be seen in hyperglycemic corrected reticulocytes = 12% x 15%/45% = 4%.
from dogs with tracheobronchitis.
drug. Drugs tllat diffuse more slowly yield patients with urinary hemorrhage and in cats 179. e CorrectedWBC/J.l.L=(WBCcoum X 100)/ 100
196. c ~is in!ectio n is not necessary for s uperficial
smaller zones of inhibition; however, they may with urinary obstruction. + Nucleated RBCs. In this cat, corrected WBC
count = (9000 x 100)lloo + 50 =6Ooo/J.l.L skin leSions because it interferes with bacterial
still be effective choices for treatment. growth in culture.

C 1998 Mosby-Year Book, Inc. Photocopying is prollibited by law.


www.alkottob.com
62 SECTION 2
www.vet4arab.co.cc SECTION

199. d Sarcoptes miles arc found in the superficial

3
197. b The ova of the lung fluke, Paragonimus
kellicotti, have a characteristic flattened hyperkeratotic portions of the epidermis.
opcrcuJum at one end. 200. a Fluorescence appears best on the deeper hair

om
198. e Antimicrobial growth (lack of a clear zone of shaft portions, especially those below the skin
inhibition) around the antimicrobial disk surface. For this reason , hairs should be
indicates bacterial resistance. plucked and then examined for fluorescence.

Diagnostic Imaging and


NOTES
Recordings

b.c
J.M. Bowen, S.T. Finn-Bodner, JA Hudson, W.R. Klemm,
M.L. Moon, P.w. Pratt, D.E. Thrall, L.P. Tilley, E.R. Wisner

tto
RADIOGRAPHY
Rewmmeluled Reading
Burk RL. Acke rman N: Small animal radiology and ulzrasoTlograp/lY. cd 2, Philadelphia. 1996, WB
Saun ders.
FarrowCS el al: Rndiologyoflilecat, SI. Louis, 1994, Mosby.

o
Morgan JP. Silverman S: Tecliniques of veterinary radiography, ed 5, Davis, Calif, 1993, Velerinary
Radio logy Associales.
Thrall DE: Textbook of veterinary diagnostic radiology, ed 2, Philadelphia, 1994, WB Saunders.

alk
Practice answer sheets are on pages 259-260.

Questions
M.L. Moon
w. 1. The silhouette effect occurs on a radiograph wilen: d. blood vessels are seen "end on," forming fo cal
a. nvo structures are superimposed and are thus densities
doubled in density e. the two crura of the diaphragm are
b. two structures of the same radiographic superimposed on lateral thoracic views
density are in contact and their margins
cannot be distinguished radiographically 2. The silhouette effecr is frequently seen in animals
ww
c, two structures of the same radiographic witll:
density are not in close contact and are
a. pleural effusion
separated by a s ubstance of a differing
radiopacity and their borders can be b. chronic bronchitis
distinguished radiographically c. hearnvorm d isease
d. normal thoracic radiographs
e. age-related changes of the lungs

@1998Mosby-YearBook,Inc.Photocopyingisprohibitedbylaw. Cm7l!ct answers are on pages 92- 100. 63


www.alkottob.com
64 SECTION 3 www.vet4arab.co.cc Diagnostic Imaging and Recordings 65

3. Concerning teclllliqlle for radiography of the 7. Concerning radiograpllicelJaluation of tracheal 10. An air bronchogram is a: c. caudodorsal slant to the caudal vena cava
thorax, which statement is least accurate? collapse, which statemellt is least accurate?
a. thickened, prominent bronchial wall d. caudal lung fields extending caudally to the
a. Routine thoracic films should be made at full a. The intrathoracic trachea collapses most on b. bronchus surrounded by dilated arteries and first lumbar vertebra

om
inspiration, if possible. expiration. veins e. heart appearing relatively enlarged
h. High-kVp techniques produce films \-vith low b. The eXlrathoracic trachea collapses most on c. calcified bronchial wall
contrast and a long gray scale. inspiration .
d. bronchus containing air 15. A 12-year-old cocker spaniel;s seen because of
c. A grid is usually not necessary, even in horses c. Tracheal collapse can only be diagnosed on coughing and a murmur. Thoracic radiographs
e. radiolucent bronchial lumen visible because it
and large dogs. flu oroscopy. reIJeal left atrial and left ventricular enlargement,
is surrounded by fluid-filled lung tissue
d. At least three separate radiographic views are d. In normal animals th e tracheal diameter does pulmonary venous enlargement, and a mixed
needed to completely evaluate the thorax of not change significantly between inspiration alveolar and linear ;rlterstitial infiltrate in the
an aduil horse. and expiration. 11 . An air bronchogram is consistent with which type dorsal and caudallungfields. What is the most
ofluflgpattern?

b.c
e. Low-mAs techniques produce films with less e. Tracheal collapse is most often seen in toy dog likely calise of these findings?
motion artifaci. breeds. a. alveolar a. aspiration pneumonia
b. vascular b. cardiogenic pulmonary edema
4. Radiographic signs a/pleural effusion include all 8. COllcernillgdijferentiatioll Of pneumonia (fobar c. linear interstitial c. chronic bronchitis
of tile following except; consolidation) and col/apse (atelectasis) of lung d. nodular interstitial d. distemper pneumonia
lobes Oil radiographs, which stateme1lt is most e. bronchial
a. a widened, radiopaque pleural space e. metastatic pulmonary neoplasia
accurate?
h. widened, opaque interlobar fissure lines
c. partial collapse of the lung lobes a. Lung lobe atelectasis typically causes a 12. Wl1icl, disorder is not characterized by nodular

tto
mediastinal shift, best seen on a ventrodorsal 16. On a dorsoventral radiographic IJiew, the heart
d. radiolucent air surrounding the lung lobes lesions 011 thoracic radiographs? may bediIJided into segments representing the
or dorsoventral view.
e. blunting of lhe costophrenic angles on 3. primary pulmonary neoplasia face ofa clock. A prominent bulge at the 1- to
b. Pneumonia is characterized by an alveolar
ventrodorsal views b. metastatic pulmonary neoplasia 2-o'cfock position most likely represents the
pattern; atelectasis is not.
elliarged:
c. Lobar consolidation and atelectasis cannot be c. fungal granuloma
5. Pneumomediastinum may be caused by any tile reliably differentiated on radiographs. d. pulmonary abscess a. ascending aorta
fo/lowingexcept: d. Atelectasis always causes a secondary pleural e. chronic bronchitis b. left atriwn

o
a. tracheal rupture effusion; pneumonia does not. c. pulmonary trwa
b. esophageal rupture e. Atelectasis may cause no radiographic 13. Concerni1lg eIJallUlriOIi of peripheral pulmonary d. left auricle
c. pneumothorax changes, but pneumonia is always arteries and /Jeins 011 thoracic radiographs, which e. right atrium

alk
radiographically evident. statement is least accurate?
d. injury to lung parenchyma
e. injury to cervical soft tissues a. Enlarged arteries may indicate heartworm 17. All of the following are consistent with left heart
9. Pneumothorax may cause any of the following disease. elllargemellt except:
radiographic signs except:
6. A lesion ill the right middle lung lobe is best b. Enlarged veins may indicate left-sided a. an elongated, straight caudal border (lateral
visualized on a left lateral radiograph because: a. a widened, radiolucent pleural space congestive heart failure. view)
b. partial collapse of lung lobes c. Enlarged arteries and veins indicate b. elevation of the entire trachea, including the
a. the lesion would be overlying the heart
c. absence of vascular and interstitial markings pulmon ary overperfusion associated with a main-stem bronchi (lateral view)
b. the lesion would be farther away from the w. left-to- right shunt.
outside the lung lobes c. decreased distance between the left ventricle
cassette and hence magnified
d. apparent elevation ofthe heart off the d. Small arteries and veins may indicate and left thoracic wall (ventrodorsal view)
c. the lesion in the nondependent (uppermost), hypovolemia and shock.
sternum on lateral views d. "reversed-D" appearance on ventrodorsal view
better-inflated lung lobe is better visualized
e. visible, opaque fissure lines e. Peripheral arteries are typically larger than e. bulge of left auricle at the 2- to 3-o'c1ock
d. there is less superimposition of ribs in this veins.
position position on ventrodorsal view
e. air in the dependent lung helps to provide
ww
contrast J4. All of ti,e followillg are signs ofexpiration on
thoracic radiographs except:
a. decreased distance between the heart and
diaphragnl
b. small, opaque lung fields

11:1 1998 Mosby-Year Book. Inc. Photocopying is prohibited by law. Correct ariSlwrs are on pages 92-Joo.
www.alkottob.com
66 SECTION 3 www.vet4arab.co.cc Diagnostic Imaging and Recordl1Jgs 67

18. 1710racic radiograpllS ofa puppy witli a murmur 22. Loss o/inrranbdominal C01lrrast on radiographs 26. After bone trauma or infection, periosteal new 30. You note a large crallial abdominal mass 011
show marked left atrial and left ventricular may be ca/lsed by any of the following except: bone formation first becomes radiographically abdominal radiographs ofa cm. The stomach is
enlargement. enlarged pulmonary arteries and evidellt after: displaced dorsalfyand caudallJI by tllis mass.
a. peritoneal effusion

om
veins, and a bulge;n rhe pulmonary trunk and Wllat is the most likely site of this mass?
descending aorta. Wllaf is the most likely calise of
b. lack of imraabdominal fat a. 1 to 2 days
Ihesejindings? c. peritonitiS b. 7 to 10 days a. liver
d. peritoneal neoplasia c. 2 to 3 weeks b. ovary
a. pulmonic stenosis
e. free abdominal air d. 4 to 6 weeks c. kidney
b. patent ductus arteriosus
e. Sto 10 weeks d. prosl'ate
c. aortic stenosis
23. What is the earliest stage ofgestation at which e. bladder
d. tetralogyofFallot
mineralized fetal skeletons may be evident in 27. W11at is all important colltraindication to
e. ventricular seplal defect abdominal radiographs ofa pregnant female dog? imravcllous urography?

b.c
31. W11at is the most common site ofosteocllOndrosis
a. 7 to 9 days a. severe dehydration ill dogs?
19. Causes of generalized cardiomegaly inelude all
the following except: b. 10to 19 days b. pyelonephritis u. distal medial humeral condyle
c. 20 to 30 days c. hyposthenuria b. lateral femoral condyle
a. pericardial effusion d. 40 to 45 days d. absence of one kidney c. medial trochlear ridge of the talus
b. pe ricardial-peritoneal diaphragmatic hernia
e. 46 to 50 days e. renal calculi d. caudal humeral head
c. mitral and tricuspid insufficiency
e. lateral femoral condyle
d. dilative cardiomyopathy
24. Conceming the appearance ofaccumulated fluid 28. Radiographic sig1iS ofacute gastric fOrsion

tto
e. hypovolemia and gas 011 right lateral radiographs of the canine include all tile following except: 32. Collcemi1lgosreocllOndrosis in dogs. which
abdomen. wllich statement is most accurate? statemelll is least accurate?
a. gas and fluid distension ofthe stomach
20. 17lOracic mdiograpllS ofa 4-year-old dog with a. Fluid accumuJates in the pylorus, whereas gas
citron;c coughing reuenf a "reversed-D~ appearance b. d isplacement of the pylorus dorsally and to a. Lesions may heal spontaneously.
accumulates in the fundus. the left
to the hean 011 dorsoventral views. a bulge at the 1- b. Lesions are often bilateral.
to 2-o'clock position all the cardiac sWlOuerteoll the b. Fluid accumulates in both the pylorus and c. compartmentalization of the stomach c. Lesions are caused by a defect in enchondral
dorsovelltrallJiew, and enlarged, tortuous pulmon- fundus. (fold ing of the s tomach on itself) ossification.

o
ary arteries Oil both lateral and dorsoventral views. c. Gas accumulates in both the pylorus and d. displacement of the spleen d. Cartilage flaps or free-floating cartilage
WlIat is the mosllikely cause o/these findings? fundus.
e. accumulation offluid in the pylorus on right fragments may be present.
d. Fluid accumulates in the fundus. whereas gas lateral views
a. pulmonic stenosis e. Lesions are primarily seen in young. small-
accumuJates in the pylorus.

alk
b. heartworm disease breed dogs.
e. A horizontal l1uid-gas interface is evident at
c. ventricular septal defect the pylorus. 29. Concemillg evaluation of the bladder using
d. tricuspid insufficie ncy cystography. which statement is least accurate? 33. A 9-month-old German shepherd has shifting
e. mitral insufficiency a. Positive-contrast cystography is best for forelimb lameness. with pain on palpation over
25. You wam to perform an upper gastroimestinal bot" hllmeral diaphyses. Radiographs reveal
contrast series 011 a dog with suspected detection of bladder rupture.
b. Double-contrast cystography is best for mllltlfocal, ,latchy increases in opacity in the
21. On abdomillal radiographs of dogs. tile normal perforation of the jejunum. W1/at is the contrast medllilarycavity oftlie right and left humeri.
kidney length is approximately: agent of choice for this procedure? evaluating the bladder wall and intraluminal
w. filling defects. What is the most likely cause of these findings?
a. I to 1.5 times the length of the sixth lumbar a. barium sulfate because of its superior coating a. panosteitis
c. Double-contrast cystography uses a large
vertebra properties volume of positive-contrast medium to b. osteochondritis dissecans
b. 2.5 to 3.5 times the le ngth of the second b. barium sulfate because of its lack of distend the bladder. with a small amount of c. hypertrophic osteodystrophy
lumbar vertebra hype rtonicity negative-contrast medium (air. carbon d. ununited anconeal process
c. 4 to 5 times the length of the fourth lumbar c. a water-soluble organiC iodine agent because dioxide. nitrous oxide).
vertebra it will not irritate the pe ritoneum e. fragmented coronoid process
d. When negative· contrast agents are used,
ww
d. half the distance of the combined lengths of d. a water-soluble organic iodine agent because carbon dioxide and nitrous oxide are safer
the third and fourth lumbar vertebrae of its high radiopacity than room air in preventing air embolism.
e. twice the width of the first lumbar vertebra e. room air because it will not irritate the e. Survey abdominal radiographs should always
peritoneum be made before cystography of any kind.

@ 1998Mosby-YearBook,lnc.Photocopyingisprohibitedbylaw. OJrrect answers are on pages 92·100.


www.alkottob.com
68 SECTION 3
www.vet4arab.co.cc Diagnostic Imagillg and Recordings 69

34. A 9-mantll-old miniature poodfeexhibits 38. W1lat is the most consistent radiographic sign of 41. A slab fracture of the dorsomedial aspect of the 45. Concerning radiographic technique. which
progressive rigllt rear limb lnmeness and pain. diskospondylitis? third carpal bone is best evaluated llsing which statement is most accurate?
There;s no IIistoryoftrauma. On radiographs of
a. lysis and irregularity of the vertebral end plates two radiographic views?

om
the pelvis, tile rigllt femoral head is flattened and a. Low kVp results in high contrast and a short
b. ventral spondylosis a. dorsolateral-palmaromedial oblique and gray scale.
irregular. with areas of lysis. and the coxofemoral
joillt space is widened. Mw! is tile mosllikely c. calcification of the intervertebral disk dorsoproximal-dorsodistal oblique b. High kVp results in high contrast and a short
callse afthese findings? d. lytic changes of the dorsal spinous processes b. dorsopalmar and lateral-medial gray scale.
e. dorsal spondylosis c. dorsomedial-palmarolateral oblique and c. When taking a radjograph of extremities
a. hip dysplasia flexed lateral (bone). a low-contrast, long-gray scale
h. osteochond rosis d. dorsopalmar and flexed lateral technique is preferred.
c. Legg-Calve-Perlhes disease 39. Concerning degenerative joint disease of the
equine tarms. which statement is least accurate?
e. fl exed medial and ventromedial- d. When taking a radiograph of the thorax, a
d. panostcitis ventroproximal oblique high-contrast, short-gray scale technique is

b.c
e. degenerative joint disease a. It is most commonly seen in the d istal preferred.
intertarsal and tarsometatarsal joints. e. Radiographic contrast depends only on kVp.
42. Concerning radiographic evaluation ofnavicular
35. Concerning osteosarcoma in dogs, which b. Periarticular osteophyte formation is disease inllorses, which statement is least accurate?
statemellt is least accurate?
common in affectcd joints.
a. Large numbers of lollipop-shaped, dilated 46. When processing x-ray film, the developer:
c. Joint -space narrOwing or fusio n of affected
a. It can occur in young dogs, as young as I or 2 joints may occur. channels in the distal border of the navicular a. removes unexposed silver salts from the film
years of age. d. There is not always a good correlation bone are consistent with navicular disease. emulSion, making unexposed areas clear
b. The radiographic appearance is variable; lysis, between clinical and radiographic signs. b. Cystlike lesions in the medullary cavity are b. converts exposed silver halide to metallic
proliferation, or both may be seen .

tto
e. Extensive lysis of the tarsal bones is a consistent with navicular disease. silver, making exposed areas black
c. It is the most common primary bone tumor in consistent Sign. c. Erosion of the cortical bone on the flexor c. proceeds at a slower rate at higher temperatures
dogs. surface is consistent with navicular disease. d. never becomes exhausted and therefore does
d. It usually occurs in mid-diaphysis aflong bones. d. Horses \vith clinical signs of navicular disease not need replenishment
40. A radiographic view is rlamed by tlte direction in may have normal radiographs.
e. Large-breed dogs are most commonly e. serves p rimarily to harden the film,
whidl the central ray oftile primary beam penetrates
affected. e. Navicular disease is best diagnosed on a preventing sweUing of the emulsion
the body part. from tlte point ofentraru:e to tile poim
single lateral-medial view of the foot.
ofexit. W1lidl of the following most accurately

o
36. W1Ull is tile proper dosage ofbarium sulfate contrast describes passage ofthe x -ray beam in a dorsal 60- 47. After processing a radiograph. you note mUltiple,
medium for a routine upper gasrrointesrirlal series? degn?e lateral-palmaromedial oblique view? 43. Concerning use of grids in veterinary radiology, branching, black lines extending across the film.
which statemem is least accurate? What ;s the most likely cause of this radiographic
a. I to 2 mllkg body weight a. Starting laterally, the beam travels 60 degrees

alk
dorsally, to exit o n the palmaromedial aspect. a. Grids are placed between the x-ray film and artifact?
b. 6 to 12 mllkg body weight
c. 10 to 20 mllkg body weight b. Starting dorsally, the beam travels 60 degrees the screens. a. light leaking into the darkroom
d. 20 to 40 mllkg bodywcight laterally. to exit on the palmaromedial aspect. b. Grids absorb any radiation traveling on a b. static electricity
c. Starting on the palmar aspect, the beam travels course not parallel to the primal)' x- ray beam. c. poor screen-film contact
e. 50 mil kg body weight
30 degrees laterally, to exit dorsolaterally. c. Grids are used to radiograph body parts d. overexposure
d. Starting on the palmar aspect, the beam thicker than approximately 10 cm.
37. COllcerning Salter cfass- V fractures. which e. overly warm developer solution
travels 90 degrees laterally, to exit d. When grids are used, exposure factors must
statement is most accurate? dorsolaterally. w. be increased.
a. They extend across the physis. e. Starling on the lateral aspect, the beam travels e. Grids Illay be stational)' or moving during the 48. What is the maximum annual legally pemJissible
45 degrees dorsally, to exit on the palmaro- cxposure. radiation dose for occupational exposure?
b. They extcnd from the physis into the
metaphysis. medial aspect. a. 5 rem
c. They extend from the physis into the epiphysis. 44. Radiographic density, the degree ofblackness of b. 10 rem
d. They extend across the physis. from the the finished film, can be decreased by: c. 15 rem
ww
metaphysis into the epiphysis. d. 20 rem
a. increasing the milliamperage (mA)
e. They are usually not seen on radiographs. e. 25 rem
b. increasing the time exposure
because they are compression fractures.
c. increasing tile kilovoltage (kVp)
d. decreasing the developer solution temperature
• e. increasing the time in the developer solution

C 1998 Mosby-Year Book. Inc. Photocopying is prollibited by law. Correct answers are on pages 92-100.
www.alkottob.com
70 SECTION 3
www.vet4arab.co.cc Diagnostic Imaging alld Recordings 71

49. All the following are important principles of 50. All of the following are important principles of 57. Detail is rhe degree of sharpness ofan object on a 61. You take radiographs ofllle stifle alan 8-year-old
radiation safety except wllieh one? radiatiOIl safety durillg equille radiography radiograph. Which ofthefoflowillgdoes not affect dog with a 5 -week history ofsrifle swelling and
except wllic11 olle? detail? pai1l. The radiographs show joint effusion and
a. Use optimal filtration of the x-ray beam.

om
b. Use mechanical restraints when possible to a. Use cassette holders. a. roca1 Spot size mulliple subchondral and periarticular erosive
b. Lead gloves protec t hands only from scatter regiollS. Based 011 the radiographic signs. what is
minimize manual restraint of the patient. b. object-film distance
the least likely diagnosis?
c. Collimate to the smallest area consistent with radia tion, not from the primary beam. c. film-screen contact
the clinical requirements. c. AJways ma nually hold the portable x-ray d. foca l spot-object distance a. septic arthritis
d. Use nonscreen film to achieve optimal detail. machine to achieve high-qua1ity radiographs. e. milliamperage b. rheumatoid arthritis
e. Always wear lead aprons and gloves when in d. Minimize personnel in the radiology room c. degenerative joint disease
the radiology roo m during the exposure. during the exposure. d. synovial sarcoma
e. If necessary, sedate the anima1 to reduce the 58. You are presented witlt a 6·momlt-old Irish seuer
e. lupus erythematosus

b.c
need for repeat exposures. with bilareralforelimb lameness persisringfor 1
month. You note swelling of the carpal and tarsal
joints. On taking a radiograph ofone carpus, you 62. Active periosteal reaction 011 the pafmarolateral
note a region ofdecreased opacity in the and palmaromedial aspeC1S oflhe proximal
metaphysis of tile radius and ulna and regions of phalanx in a horse is most caused by abnormal
D.E. Thrall
soft tissue calCification circumferenriallyaround fJlIllingofthe:
51. For which of tile following radiographic 54. Rare-earth intellsifyillg screellS were illtroduced tlte radial and ulnar metaphyses. The most likely
examinations would a grid provide the greatest in the 1970s. Their biggest advantage over cause of these findings is: a. suspensory ligament
improvement in detail? calcium-tungstate intellSifying screens is: b. superficial digital flexor tendon
a. scurvy

tto
c. deep digital flexor tendon
a. lateral view, body, parakeet a. increased detail b. panosteitis
d. impar ligament
b. craniocaudal view, antebrachium, great dane b. increased contrast c. osteosarcoma
e. middle sesamoidean ligament
c. lateral view, abdomen, German shepherd c. increased cost d. hypertrophic osteodystrophy
d. lateral view, morax. 6-week-old Siamese d. increased speed e. blas tomycosis
e. la teral view, metacarpal bones, aduJt e. dec reased cost 63. YOII are presenced with a 6-year-old collie with a
thoroughbred 2-momh history ofa serosanguineous discharge
59. A 3-month -old dogfallsfrom a height of 15 feet from the left naris. Wlliell radiographic view

o
55. You manually develop a 14 x 17-inch sheet ofx- and lands on all four feet. I1lere are 110 fractures , would most accllrareiyassess the nasal cavity?
52. Which technique would produce an abdominal ray film followi"g exposure to x-rays. When the but the injury crushes one of the distal ulnar
radiograph with the greatest contrast? film is dry. you notice a green stripe at the edge of physes. Wlilch ofthefollowing is least likely to be a. ventrodorsal view of the entire nasal region
b. dorsoventral view of the entire nasal region

alk
olleofthe 14·inch ends. What is the most likely observed 011 radiographs made 4 m01l111s later?
a. 1 mAs, 108 kVp
cause of this discoloration? c. rostrocaudal view of the ironta1 sinuses
b. 2 mAs, 90 kVp a. valgus deformity of the manus
a. The fixer solution level is too low. d. dorsoventra1 view of the nasal region using
c. 4 mAs, 72 kVp b. subluxation of the humeroulnar articulation
intraoral film
d. 8 mAs, 58 kVp b. The green region was not exposed to x·rays. c. fragmented medial coronoid process
c. The developer solution level is too low. e. velllrodorsal view ofthe nasal region using
e. 16 mAs, 46 kVp d. ununited a nconeal process intraoral film
d. Both the developer and fixer solution levels e. subluxation of the ulnar-carpal articulation
are too low.
53. A radiograph made using 20 mAs and 80 kVp is 64. The hallmark radiographic sign of
e. The wash water level is too low.w.
too dark. Which technique is reasonable to usefor 60. Which radiographiC sign is wast likely to be diskospondyliris is:
the second attempt? associated wirh afragmellted medial ulnar
56. After processing a radiograph , you notice an coronoid process? n. spondylosis
a. 20 mAs, 160 kVp imellSely black artifact witll the appearance of b. osteophyte formation
b. 20 mAs, 40 kVp a. new bo ne fonnation on the proximal margin
arborized black lines. The most likely cause of this c. venebral body end-plate lysis
c. 10 mAs, 80 kVp of the anconeal process
artifact is: d. narrowed intervenebra1 disk space
b. incongruity (subluxation) of the humeroradial
ww
d. 40 mAs, 80 kVp e. subluxation of the verte bral column
a. underdevelopment articulation
e. 40 mAs, 60 kVp
b. overdevelopment c. a cleavage line between the coronoid process
c. light rog and the ulna
d. developer splashed on the film in the d . degenerative jOint disease of the e lbow
darkroom e. lysis of the proxima1 aspect of the ulna
e. static electricity

C 1998 Mosby-Year Book. Inc. Photocopying is prollibited by law. Correct answers are on pages 92· 100.
www.alkottob.com
72 SECTION 3
www.vet4arab.co.cc Diagnostic Im aging and Recordillgs 73

65. Wh ich althefallowing is not a common 69. You are presented witll a 7-yea r-ofdgofden retriever 74. You are presellled witli a 6-momh-ofd male 77. Which oflhefollowing is the least reliable
radiographic sign o[vertebraf mafalignment- exhibiting weakness, regurgitation, arul coughing. Centum shepherd for vaccination. During routine radiographic sign of pleural effusion?
rnalarticulation (wobbler) syndrome jn dogs? You rake a radiograph ofthe thorax and note physical examinarion)'Ou notea systolic lieart

om
generalized megaesophagus, ventral alveolar lung murmur. On radiographs of the thorax you note a. retraction of the lung away from the thoracic
a. stenosis of the vertebral canal disease, and a cranial mediastinal11UlSS ventral to wall
elongation ofthe cardiac silhouette, enlargemem of
b. spondylosis the trachea. The mediastinal mass is m ost likely: the main pulmonary artery segment, enlargement b. inability to visualize the cardiac silhollctte
c. intervertebral disk mineralization ofthe left atrium, and enlargement ofparenchymal c. scalloped soft tissue opacity dorsal to the
a. an esophageal tumor
d. osteophyte formation on articular processes pulmonary arteries and veins. The most likely calise sternum on the lateral view
b. a thymoma ofthese findings is:
c. vertebral body end-plate lysis d. blunted costophrenicsuJcus
c. a lymphosarcoma
a. patent ductus arteriosus e. elevation ohhe heart from the sternum on the
d. a rib tumor lateral view
66. In dogs, at wllich site is herniation o[an b. subaortic ste nosis
e. a hematoma

b.c
imeruertebraf disk least likely to occur? c. pulmonic stenosis
a. C I-2 d. ventricular septal defect 78. Concernillga dog Wilh a farge voillmeofflllid i/l the
70. Which parenchymal pattern is commonly pleural space, which statement is least accurate?
b. C3-4 observed on radiographs of dogs with bacterial e. congenital mitral insufficiency
c. T13-Ll pllewllonia? a. The fluid should be obvious on vcntrodorsal
d.13-4 Questions 75 and 76 and dorsoventral radiographs ofthe thorax.
a. bronchial b. The fluid should be obvious on eith er a right
e. L7-S1 b. interstitial You are preserlted with a 12-year-old miniature
or left lateral radiograph of the thorax.
c. alveolar poodle witli a 6-month historyofa "honking" cougli.
You SIlSpect collapse of the intrathoracic trachea.
c. The heart should be bette r seen on a
67. Which radiographic sign is least likely to result

tto
d. vascular dorsoventral rathe r than a ventrodorsal
from hemiatioll a/an interuertebral disk? e. mediastinal radiograph of the thorax.
75. To increase )'our clianGeS ofdetecting this
a. vertebral body end-plate lys is a bnormality with conventional radiographs, YOIl d. The thoracic surface of the dia phragm will be
b. narrowing of lhe intervertebral disk space 71. /n dogs the right cran ial lobar pulmonary artery should make the exposure: impossible to assess completely in
c. narrowing of the s pace be tween a rticular and vein are best seen on which radiographic radiographs.
a. at the e nd of expiration
processes projection? e. The lung parenchyma should be increased in
b. at thc e nd of inspiration opacity because of partial collapse.
d. mineralized material in the vertebral canal

o
a. left laleral c. \vith the dog under gene ral anesthesia
c. sclerosis ofvcrtebral body end-plates b. right lateral d. \vith the neck flexed 79. You take a radiograph of tlie thorax ofa lO-year-old
c. dorsoventral e. at both the end of expiration and the end of
68. YOIl are presented with a 2-year-old German dog with a6-week historyofdYSP1lea and weight

alk
d. ventrodorsal inspiration loss. 71le cardiac silliouette is greatly elliarged and
shepherd soon after it is hit by a car. T1lere is
e. standing latera! has a spheric shape. What is the least likely calise of
pelvic limb paralysis, and you suspect a lumbar
spine fracture. You 1I0te riO abnormalities on a 76. 011 thoracic radiographs of this dog,YOIl also !lote this radiographic appearance of the heart?
lateml radiograph made with the dog ill left 72. Which condition results in a rigllt cranial lobar vein gas accllmulation within the midthoracic a. pcricardial -peritoneal diaphragmatic hernia
lateral recllmbencyand a vertically d irected x-ray that is larger than the right craninllobar artery? esophagus. The most appropriate course ofaction b. pericarditis
beam. The most appropriate course ofaction is to: is to:
a. pulmonic stenosis c. heart-base tumor
a. perfonn a hemilaminectomy from Ll through L4 b. pulmonic insufficiency a. perform esophagography using barium paste d. heartwo rm d isease
b. hospilalize the dog for observation
c. use a horizontally directed x-ray beam and
make a ventrodorsal projection of the lumbar
spine with the dog in lateral rC(:umbency
c.
d.
e.
mitral insufficiency
heartworm disease
aortic stenosis
w. b. perform esophagography using liquid barium
c. perform no other diagnostic procedures,
because this is an acceptable finding in this
patient
e. pericardia! tumor

80. Masses within tlte IImg parenchyma are


frequently not apparefll Oil radiographs made
d. gently roll the dog into right late ral d. repeat the thoracic radiographs and see if the with the affected lung in a dependent positioll.
recumbency and make another lateral 73. W1lich ofthe following is not a common radio- gas is stillihere
grapllic sign in dogs with left heart failure caused S ilch obscuring ofa mass is related to:
ww
radiograph using a vertically directed x-ray e. perform esophageal endoscopy
beam by mitral regurgitation? a. increased air volume in the dependent lung
e. use a horiw ntally directed x-ray beam and a. enlarged left atrium b. decreased air volume in the dependent lung
make a nother lateral projection of the lumbar b. enlarged pulmonary veins c. increased flu id volume in the dependent lung
spine after gently rolling the dog into dorsal c. pulmonary edema d. decreased flui d volume in the dependent lung
recumbency e. compreSSion of the mass in the dependent
d. pleural effusion
e. elevated tracheal bifurcation lung

Q 1998 Mosby-Year Book, Inc. Photocopying is prollibited by law. Correct answers are on pages 92-100.
www.alkottob.com
74 SECTION 3
www.vet4arab.co.cc Diagnostlc Imaging and Recordings 75

81. You take a radiograph of tile tflOrax ofa 3-year-ofd 85. Concerning emphysematous cystitis, which 89. COflcerningexcretory urography in a dog with c. negative-contrast cystography
cat with a 2-montIJ "isforya/a productive statemelll is least accu rate? normal renal junction, using water-soluble, ionic, d. double-contrast cystography
(mucous) cough. You observe three cavitary lung a. It is often associated with bacterial cystitis. iodinated contrast medium, which statement is
e. retrograde urography

om
nodules, about 1.5 em in diameter; in the caudal least accurate?
lung lobes. TIle most likely cause a/these findings b. II cannot be detected on survey abdominal
is: radiographs. a. This type of contrast medium is excreted by 93. The contrast medium of clloke for a negatiue-
c. It can be secondary to diabetes mellitus. glomeruJar filtration. COntrast cystogram is:
a. Paragonimus infection d. It can resull in pneumoperitoneum. b. This type of contrast medium is secreted by
b. Aleurostrongylus infection the renal tubuJes. a. room air
e. It is often associated with hematuria.
c. primary lung tumor c. This type of contrast medium is concentrated b. carbon dioxide
d. metastatic lung tumor by tubular resorption of water. c. helium
86. You are presetlted with a lO-year-oldfemale d. argon
e. pulmonary thromboembolism d. Alternate organs of excretion of this type of
German shepherd that straim on attempts to

b.c
contrast medium are the liver and gasno- e. 100% oxygen
defecate. Another veterinarian had removed an
intestinal mucosa.
82. COllcemiflg pulmollary metastasis. which unidelltified mass from the anal region 2 momhs
statemenl is least accurate? previously. On pe/vic radiographs you note a large e, This type of contrast medium is delivered to
94. Itlnegative-contrast cystography, urine should be
mass in the caudodorsal abdomen , ventral to Ule kidney by the renal artery.
a. It often res ults in interstitial nodules. withdrawn before injection of the COntrast
LS-7 and compressing tile cololl. It is most medium to prevent:
b. It may produce an unstructured interstitial appropriate to tell theowIler that the: 90. A 6-year-old male German shepherd, hit bya car
lung pattern. a, bladder ruptu.re
a, dog has cancer 1 hour prelliously, has hematuria. Abdominal
c. It usually does not result in only one visible radiographs show a loss of serosal margin detail b. vesicoureteral reflux
b. mass probably represents an enlarged lymph

tto
mass. and no visible urinary bladder. No fluid is c. vesicourelhral refl ux
d. It often produces coughing early in its node obtained on abdominocentesis. Tile most d. air embolism
development. c. mass probably represents a colonic tumor appropriate course ofaction is to: e, bubbles on the radiograph
e. It is often misdiagnosed when summation d. mass is normal
a. tell the mvner the bladder is ruprured and
shadows created by vessels overlying ribs are e. mass should be treated with radiation therapy
explore the abdomen surgically 95. Extravasation ofCOntrast medium from tile
seen.
b, tell the owner the bladder has not been urethra imo the parellchyma of the prostate gland
87. In cats, which of the following is least likely to ruptured because the dog is urinating and no during retrograde positive-contrast urethrography

o
83. Concerning splenic torsion, which statemem is impair visualization of serosal margin detail on flu id was obtained on abdominocentesis indicates:
least accurate? abdominal radiographs? c. perform excretory urography to assess the a. neoplasia
a. It does not occur wilhout gastric torsion. a. steatitis bladder
b. nothing of diagnostic significance

alk
b. II frequentl y causes splenomegaly. b. right hean failu re d, perform positive-contrast cystography to
c, p rostatitis
c. emaciation assess the bladder
c. II frequently results in an abnonnal splenic d. hyperplasia
shape. d, peritoneal d ialysis e. perform negative-contrast cystography to
assess the bladder e. a normal prostate gland
d. II may resuil in peritoneal effusion. e. ruptured urethra
e. II may result in displacement of the duodenum.
91. Mlat is the most appropriarecontrast medium 96, On radiographs ofa 9-year-old male German
88. TIle major colltrail.dicatioll to negative-contrast shepherd with chronic hematuria, you observe
for positive-contrastcystographx if! dogs?
84. Which ofthefollowing is the least common cystography using room air as thecol/trast two distinct spheric, large (8 cm diameter), soft
radiographic sigll associared with chronic renal
failure?
a. mineralization of the gastric rugae
a.
b,
w.
medium ill a cat witll hematuria is:
the possibility of inducing bladder infection
the possibility of inducing bladder rupture
a.
b,
c,
barium sulfate paste
barium sulfate solution
air
tissue masses if! the caudoventral abdomen. The
most reasonable conclusion is that:
a, the cranial mass is the bladder and the caudal
b. patchy alveolar lung disease c. high cost d, nonionk, iodinated, water-soluble contrast mass is an enlarged prostate gland
medium b. the caudal mass is the bladder and the cranial
c. mineralization of the foot pads d. the possibility of inducing air embolism
e. ionic, iodinated, water-soluble contrast mass is an enlarged prostate gland
ww
d. small, mineralized kidneys e. the possibility of inducing vesicoureteral medium c. exploratory surgery shouJd be performed
e. hydronephrosis reflux
d, retrograde positive-contrast urethrography
92. TIle best radiographic contrast procedure to assess should be performed
tile urinary bladder mucosa is: e. this is a normal finding and there is no need
a. excretory urography for further studies or treatment
b, positive-contrast cystography

@ I998Mosby-YearBook,Inc, Photocopying is prohibited by law. Correct answers are on pages 92-100,


www.alkottob.com
76 SECTION 3
www.vet4arab.co.cc Diagnostic Imaging and Recordings 77

97. What is the major radiographic difference 99. During all IIpper gastrointestintll radiographic
between the canine and feline stomach? examinarioll,you note two crater-shaped
Questions
outpollcl!ings of barium from the Qntimesenteric JA Hudson and S.T. Finn-Bodner
a. more prominent rugal fold s ill dogs

om
surface ollhe descendingduodenlltn ofa 4-year-
b. more prominent rugal folds in cats 101. Concerning choice ofa 7.S-MHz or a S.D-MHz c. The renal medulla is more echogenic than
old German sllepherd willi a history ofchronic
c. more medially positioned pylorus in dogs probe when performing diagnostic ultrasound, the renal cortex and Liver.
melena. These outpouchings most likely
wllicll statement is most accurate? d. The spleen is generally more hypo echoic
d. more medially positioned pylorus in cats represent:
a. Either probe can be used, but the probe than other abdominal organs, such as the
e. more laterally positioned fundus in cats a. a normal finding
should not be changed during the liver and renal cortex.
b. neoplasia examination. e. Echogenicity of structures in the near field
98. You make left lateral and uentrodorsal abdominal c. phycomycosis b. High-frequency probes provide better can be compared with that of structures in
radiographs ofa 3-year-old German shepherd d. peptic ulcers the far field as long as depth gain compen-
penetration but poorer axial resolution than
wilh a J -day history ofabdomjnal pain and

b.c
e. areas of infarction low-frequency probes. sation is used to produce a uniform image.
discemion. YOllnote extreme dilation ofrhe
stomach witllfluid and gas, bill you cannot
c. Low-frequency probes provide more
specifically idemify che fundic and pyloric regions 101). Water-soluble, iodinated, ionic contrast medium penetration than high-frequency probes and 104. The kidney isa complex organ comprised of
of the stomach The most appropriace action at can be used for lipper gastrointestinal may be required to examine deeper capsule, cortex, medullary pyramids, pelvis, and
radiograpl/icexaminatiolls. W1/Ut is the most structures adequately. pelvic diverticula. WI/icll ofthefollowing best
tllis time is 10:
important contraituiication to use of these d. AJways use as Iowa frequency as possible to describes tile relative ecllOgenicity of tile cortex,
a. rehydrate and observe the patient for 24 hours provide better axial resolution. medullary pyramids, and pelvis/pelvic
agents in a dog with sewre diarrhea?
b. give 500 m.I of barium sulfate PO and repeat e. Use lower-frequency probes when possible diverticula, from the most lzyperec/lOic co the
your attempt to identify the fundu s and pylorus a. high cost least Izypereclloic? Assllme tlull the kidney is

tto
because probes with frequenCies greater
c. make right lateral and dorsoventral b. length of time involved than 10 MHz increase the risk of biohazards. normal and the animal is not undergoing
radiographs of the abdomen in an attempt to c. poor mucosal detail physiologic diuresis.
identify the fundus and pylorus d. exacerbation of fluid-electrolyte imbalance a. cortex > medullary pyramids >pelvis and
102. Concerning acoustic impedance as related co
d. perform an exploratory laparotomy e. irritability to the gut mucosa ultrasonography, which statement is most diverticula
e. perform a complete upper gastrointestinal accurate? b. pelvis and diverticula> medullary pyramids
examinatio n > cortex

o
a. Ultrasound is reflected when there is an
c. pelvis and diverticula> cortex > medullary
acoustic impedance difference between two
pyramids
adjacent s tructures.
ULTRASONOGRAPHY b. The acoustic impedance of a tissue can be
d. medullary pyramids> pelvis and diverticula

alk
> cortex
altered by changing depth (time) gain
compensation. e. The kidney has a uniform hypoechoic
Recommended Reading appearance, with differences in echogenicity
c. Structures with a high acoustic impedance
Burk RL. Ackerman N: Small (.IlIimal radiologyalld u/rrasonogrnplly, ed 2, Philadelphia, 1996, WB seen only with pathologic changes.
reflect ultrasound and are anechoic.
Saunders. d. Acoustic impedance describes the intensity
Cartee RE et a1: Practical veterinary ultrasound, Baltimore, 1995, Williams &Wukins. of ultrasound. 105. Concerning the ultrasonographicappearance of
Green RW: Small animal ultrasound, Philadelphia, 1996, Lippincott-Haven. e. Acoustic impedance describes the frequency tile normal canine kidney, which statemerlf is
Nyland TG, Mattoon IS: Veterinary diagnostic ultrasound, Philadelphia, 1995, WB Saunders. d wavelength of ultrasound produced by most accurate?
RcefVB: Equine diagnostic ultrasound, Philadelphia, 1997, WB Saunders.
Reimer 1M: Atlas o/equine ultrasonography, St. Louis, 1998, Mosby
w. an ultrasound transducer.

103. Concerning tile relative differences in


a. The renal cortex is hyperechoic as compared
with the spleen.
b. The renal medulla is hyperechoic as
echogenicity ofabdominal organs in dogs and compared with the renal cortex.
Practice answer sheets are on pages 259-260.
cats, which statement is most accurate? c. The renal cortex of obese cats has reduced
echogenicity.
ww
a. The renal cortex is more hyperechoic than
the spleen. d. Echogenicity of the renal medulla is similar
to that of the liver.
b. Relative echogenicity is affected by such
factors as probe frequency and interposing of e. Echogenicity of the renal cortex is similar to
other organs between the probe and the that of the liver.
organ being imaged.

e 1998 Mosby-Year Book, Inc. Photocopying is prollibited by law. QJrrect answers are on pages 92· 100.
www.alkottob.com
78 SECTION 3
www.vet4arab.co.cc Diagnostic Imaging and Recordings 79

106. Bright dOl$ at thecorticomedullary junction in c. The muscularis is hyperechoic and is the 112. Which statement best describes the appearance c. The ph renicoabdominal vein serves as a
gray-scale images ofa canine kidney usually thickest layer. of the canine prostate gland in ultrasound vascular landmark.
represent: d. The muscularis and serosa can only be images? d. The pancreas can only be seen when imaging

om
a. normal mineral deposits of no clinical distinguished when pathologic changes are a. The prostate gland is a peanut-shaped organ from the left side of the abdomen; the ascending
significance present. best identified by imaging craniomedial to colon prevents visualization on the right.
h. arcuate arteries e. The wall is composed of five equally thick the prostatic artery. e. The pan creas is normally hyperechoic, wilh a
layers. b. Each prostatic lobe appears as a hypoechoic hypoechoic central core.
c. afferent arterioles
oval, with a hyperechoic line located
d. abnormal mineraJ deposits that warrant
110. Which statement best describes the centraUywithin the lobe. 115. COllcerning the lllrrasollograpliic appearance of
further investigation
e. hippomanes ultrasanographic appearance of the luminal c. The prostate gland is less echogenic than the the gallbladderalJd biledllcts inllealthydogs
cOrllents of the small intestine? renal medulla. and cats, which statement is most accurate?

b.c
a. When fluid is present, the mucosal layers of the d. Intraparenchymal prostatic arteries appear
107. You obtain an ultrasound image ofa renal a. The gallbladder has a thick hyperechoic wall.
near and far walls cannot be distinguished; the to have hyperechoic walls, particularly when
calculus. The area deep to the calculm is imaged at right angles to the ultrasound b. The common bile duct is less than 2 mm
completely black. IN'llat term is used to describe black mucosal layers ~b l end" with the black wide and is difficult to visualize on its course
beam.
tltis black artifact? fluid-filled lumen to fonn a single black layer. to the duodenum.
e. The prostate has a uniform, somewhat
b. Mucus in the lumen is hyperechoic and does c. The feline gallbladder is difficult to visualize
a. acoustic enhancement not allow imaging of the far wall. coarse echogenicity.
because it is usually empty.
b. off- incidence angle artifact c. Gas in the lumen causes acoustic d. Cats have large cystic and common bile ducts
c. slice thickness artifact enhancement, a phenomenon in which 113. Concerning the ultrasonographic appearance of that are easily visualized, unlike those of dogs.

tto
d. refraction artifact deeper structures are bright because of the normal canine uterus, which statement is
e. The lumen of the gallbladder is hyperechoic.
e. acoustic shadowing reverberations between gas and the probe. most accurate?
d. Intraluminal gas cannot be differentiated a. During estrus, the uterus in longitudinal
from intraluminal mucus. 116. Concerning ultrasonograplJicartifacts
lOB. Concerning the ulrrasonographic appearance of section is a homogeneous hyperechoic tube
associated with the normal liueror gallbladder,
the normal canine spleen, which statement is e. Fluid and mucus allow imaging of the far \'lith an anechoic lumen.
which statement is most accurate?
most accurate? wall of the intestine. b. The anestrual uterus is difficult to
distinguish from normal small intestine a. Distal acoustic enhancement occurs when

o
a. Echogenic flow in large splenic veins is an because both have a five-layered wall and a sound is repeatedly reflected between the
abnomlal finding requiring further 111. Considering ultrasound imaging of the urinary
fluid-filled lumen. lung/diaphragm interface and the hepatic
investigation. bladder, which statement is most accurate?
c. The walls of the normal small intestine and parenchyma.
b. The splenic capsule cannot be visualized a. A catheter should be placed and the bladder b. Mirror-image anifact causes vertical black

alk
postpartum uterus are identical, with two
because it has the same echogenicity as the maximally distended before imaging. layers evident. streaks extending from the curved surfaces of
splenic parenchyma. b. Acoustic shadowing is frequently seen deep the gallbladder into the hepatic parenchyma,
d. Pregnancy can be diagnosed in dogs by
c. The spleen has the same echogenicity as the to a normal bladder. ultrasonographic visualization of the deep to the gallbladder.
liver. c. The slightly distended bladder wall exhibits chorionic cavity at 17 to 20 days of gestation, c. Distal acoustic shadowing causes an
d. When imaged oblique to the ultrasound five layers of alternating echogenicity: serosa, after the luteinizing hormone peak:. artifactual increase in echogenidty of the
beam, splenic veins are characteristically muscularis, submucosa, mucosa, and e. The uterus can be differentiated from the liver, deep to the gallbladder.
anechoic to hypoechoic, with poorly mucosal surface. aorta because the aorta bifurcates cranially, d. Distal acoustic enhancement occurs deep to
delineated walls. d. Ultrasonography, rather than cystography,
w. whereas the uterus bifurcates caudally. the gallbladder because of minimal
e. Splenic arteries and veins aTe not usually should be used to evaluate the cranial border attenuation of ultrasound by fluid in the
visible. (apex) of the bladder because this area is gallbladder.
clearly visualized with ultrasonography but 114. Concerning the pancreas ofa normal dog, which
e. Artifacts are only seen when the liver or
not with cystography. statement is most accurate?
109. Which statement best describes the gallbladder is abnormal.
ulrrasonographic appearance of the wall of the e. Acoustic enhancement is frequently seen a. The pancreas is easily imaged because it is
small intestine in a normal dagar cat? deep to the urinary bladder. an anechoic organ surrounded by
ww
hyperechoic mesentery.
a. The serosa, submucosa, and mucosal surface b. In acute pancreatitis the pancreas is
are hyperechoic. hypoechoic and enlarged.
b. Only the serosa is hypoechoic; all the other
layers are hyperechoic.

© 1998 Mosby-Year Book, Inc. Photocopying is prohibited by law. Correct answers are on pages 92-100.
www.alkottob.com
80 SECTION 3
www.vet4arab.co.cc Diagnostic Imaging and Recordillgs 81

Questions 117 and 118 d. septal myectomy E.R. Wisner


e. administration of a ~ -b lo cke r or calcium- 121. Ultrasound waves propagating through tissues 124. Wllich of tile following best describes the basis for
You perform an ecllOcardiographic examination on a
channel blocker are reflected to a greater degree as the difference prodl1Ction ofa tIoo-dimensional ultrasound image?

om
2-year-old, S-kg male domestic shortlzair cat with a
grade-ll systolic murmur over the mitra/llalve area in the acoustic impedanceofadjacem tisslles
a. Changes in the ultrasound wave velocity are
and a diastolic gallop rhythm. Findings afM-mode 11 9. You obtain a trallsverse ultrasound image of the increases. Theacollstic impedance ofa tissue
measured as the echoes propagate through
ecilOcardiography include ti,e following: flexor surface ofa horse's forelimb ar rile level of increases with:
different tissues.
tllejunctioll of the proximal and middle thirds a. increaSing tissue density and increasing
Left arrial diameter 15 mm (normal, 8.5 to 12.5 mm) b. Changes in ultrasound wave frequency are
of rile metacarpus. Wlljc/I of the following
uft l!entriCllfar imemal diastolic diameter 10 mm, ultrasound propagation velocity measured as they propagate through
accurately lists tile structures present in the
(normal, 11 to 16 mm) b. decreasing tissue density and increasing d ifferent tissues.
image, ill order from superficial (closest to the
u[t ventricular waU diastolic diameter 6.5 mm ultrasound frequency c. The amplitude of transmitted ultr.lsound
probe) to deep (farthest from the probe)?
(normal. 2.5 to 5 mm) c. increasing tissue density and decreasing waves is measured as the waves propagate

b.c
IflferlJelltriwlar septum diastolic diameter 7 mm a. superficial digital flexo r tendon, deep digital ultrasound propagation velocity through different tissues.
(normal, 2.5 to 5 mm) flexor tendon, interosseous (suspensory) d. decreasing tissue density and increasing d. Changes in ultrasound wave attenuation are
FractiOllal shortening 70% (normal, 29% to 55%) ligament ultrasound propagation velocity measured and are determined by viscosity,
Two-dimensional examinatioll shows asymmetric b. superficial digital fl exor tendon, deep digital e. increasing tissue density and decreasing stiffness, and scattering properties of
tllickeningofthe interventricular septum at che base, flexor tendon, inferior check ligament, ultraso und freq uency different tissues.
narrowing of the left ventricu{aroutftow tract in interosseous (suspensory) ligament e. The time requ ired for reflected ultrasound
systole (subaortic), and cranial molioll aftlle mitral c. superficial digital flexor tendon, deep digital waves to return to the transducer is
flexor tendon, interosseous (suspensory) 122. As transducer frequency increases (e.g., from 5 to
valve crallial leaflet toward the septum during systole. determined, and a constant velocity of wave
10 MHz):

tto
The mitral valve leaflets are thichmed. Color Doppler ligament, inferior check ligament propagation within the tissues is assumed.
eualuatio" sllOws turbulence ifl tile left vemricular d. superficial digital flexor tendon , inferior a. spatial resolution of the image is degraded
outflow velltral to tlleaorcic valve and a mitral check ligament, deep digital fl exor tendon, b. image contrast increases For Questions 125 through 129, select the correct
regurgiltlllrjer illto the left atrium. interosseous (suspensory) ligament c. spatial resolution ohhe image improves answer from the five choices below.
e. superficial d igital flexor tendon. deep digital d. ultrasound propagation velocity decreases
117. Wllar is tile most likel), cause of these flexor tendon, inferior check ligament a. gain
eclwcardiograpllic fi ndi ngs? e. ultrasound propagation velocity increases
b. pulsed Doppler

o
a. mitral valve endocardiosis and regurgitation 120. Concerning the llltrasonograpilic appearance of c. power
and secondary left ventricular hypertrophy 123. Ultrasound images are prodl~ced b)' transmitting d. B-mode
a normal canine eye imaged Willi a 7.5- or 10-
ultrasound waves of a gillen frequency into tissue
b. congenital hean disease, with subaortic MHz trallsdllcerand a standoffpad (orojfset), e. time-gain compensation
in siron bllrsts or pulses. Each pulse con tains a

alk
stenosis and left ventricular hypertrophy which statement is most accurate?
fixed number of ultrasound waves or cycles. Tile
c. thyrotoxic cardiomyopathy a. The lens appears as an elliptiC hyperechoic axial resolution ofall image improves (sma ller 125. Metllod ofde rem lining the velociryofmoving
d. hypertrophic cardiomyopathy (obstructive) structure between the anechoic anterior objects can be detected} wlrell: StruCtllres, sllch as flowing blood, using the
e. mitral valve vegetative endocarditis, chordae chamber and anechoic vitreous chamber. magflimde oftile frequency shift ofreflected SOllnd
a. the transducer frequency is increased and
tendineae rupfUre, and st'Condary left b. The vitreous chamber is hyperechoic, waves
the number of cycles per pulse is decreased
ventricular hypertrophy whereas the aqueous anterior and posterior
b. the sound wave amplitude is increased and
chambers are anechoic.
the number of cycles per pulse is increased 126. Ultrasound control that uniformly regulates tile
118. What is tile most appropriate treatmemfor this c. Severe retinal detachment is manifested as V-
w. amplitude of ultrasound output
c. the transducer frequency is decreased and
condition? shaped echogenic lines with points of
the sound wave amplitude is increased
aHachment at the optic disk or ora serrata.
a. administration of furosemide and digoxin d. the transducer frequency is increased and 127. Uniform electronic amplification ofall echoes
d. The optic nelVe appears as a uniformly
b. adminisrration of methimazole, with the sound wave amplitude is decreased returning to the trallsducer
hyperechoic tubular structure; the optic disk
thyroidectomy or use of radioactive iodine appears as an anechoic structure at the e. the sound wave amplitude is increased and
c. antimicrobial treatment based on urine or caudal aspect of the globe. the number of cycles per pulse is decreased
128. Comparatively greater increase in electronic
ww
blood culfUre and sensitivity tests, and e. The ciliary body cannot be visualized with amplification of sound waves rerumingfrom
administration of an angiotensin-converting ultrasound. deeper (more distallt) structures
enzyme inhibitor to decrease preload and
afterload
129. UltraSOllnd imagegellerared by return illgechoes
as dots wllose brightness is proportional to the
amplitude of the returning echo and whose
position corresponds to the depth at wllich rile
echo origillated

@I998 Mosby-YearBook,Inc.Phorocopyingisprohibitedbylaw. Correcr answers are on pages 92-100.


www.alkottob.com
82 SECTION 3 www.vet4arab.co.cc Diagnostic Imaging and Recordings 83

130. Wlliell transducer is most appropriate for 134. Wlliell of the following accurately lists 138. Which afthefollowing is most likely co be 139. 1n which disorder would echocardiograpllic
examining a small, superficial structure, such as parenchymal organs in normal dogs, from most observed during ultrasound examination o/tlle examination be least we/ul for definitive
a thyroid gland in a dog? echogenic (briglltest) to least eciJOgenic (darkest)? abdomen ofa ctlt with lymphoma affecting the diagnosis?

om
a. renal cortex,. spleen ,. renal medulla,. liver introobdominallympllnodes?
a. 3.0-MHz mechanical seclor-scanning a. lefl -sided congestive heart failure
transducer b. liver ,. renal medulla ,. renal cortex" spleen a. uniformly hyperechoic nodes, with b. pericardia! effusion
b. to-MHz linear-array transducer c. spleen ,. liver,. renal cortex,. renal medulla signi.ficant distal enhancement c. dilative cardiomyopathy
c. 7.S-MHz mechanical sector-scanning d. spleen ,. renal cortex,. renal medulla,. liver b. unifonnly hyperechoic nodes, with linle or d. bacterial valvular endocarditis
transducer e. renal medulla ,. liver " spleen ,. renal cortex no distal enhancement
e. aortic stenosis
d. S.O-MHz phased-array transducer c. uniformly hypoechoic nodes. with little or no
e. 3.5-MHz linear-array transducer distal enhancement
135. Wilich disorder is most likely to produce a 140. Wilat is the 1I0rmalieft IJentricular percelll
d. complex lymph node internal architecture
generalized increase inliverechogenicity

b.c
that is hyperechoic centrally and hypo echoic fractional shortening ill flnsedated dogs?
131. Mlich strJIcture is mosllikely to produce a (brighmess)?
peripherally a. 15%
slladowingarrifacl in arl ultrasound image?
a. acute hepatitis e. moderately echogenic nodes, wi th a coarse. b. 35%
a. a urinary bladder calculus b. hepatic lipidosis heterogeneous echotexture
c. 55%
b. a renal/sple nic interface that is not c. biliary obstruction d.70%
perpendicular to the ultrasound beam d. hepatic venous congestion c. 85%
c. a complex splenic mass e. multiple hepatic cysts
d. a bile-distended gallbladder

tto
c. a fluid -distended small intestine 136. Which disordero/ten produces a marked
illcrease ill renal cortictll echogenicjty ENDOSCOPY
132. Which strrlctllre is most likely to produce distal (brightness)?
enliaflcemem jn all ultrasound image?
a. renal lymphoma Recommended Reading
a. re nal cortical mineralization b. acute leptospirosis BrcarJeyMJ et al: Colorat/aso/smaJianimal endoscopy, St Louis. 1991. Mosby.
b. a gas-filled s tomach c. polycystic renal disease Tams TR: Small animal endoscopy. ed 2. 5t Louis. 1998, Mosby.

o
c. lead buckshot d. chronic obstructive hydronephrosis Traub-Dargatz It. Brown eM: Equine endoscopy. ed 2, 5t Louis, 1997, Mosby.
d. a renal pelvic calculus e. acute ethylene glycol tOxicity
e. a hepatic cyst Practice answer sheeg. .ilre on pages 259-260.

alk
137. Wilich vascular structures within the normal
133. Wlu'cll of tile following best describes the cause of liver have a circular or linear anechoic core
mirror-image artifacts in an ultrasound image? representing the vessel lumen and a hypereclJOic • Questions
margin representing the vessel wall and
a. return of multiple, nonperpendicularuitrasolUld P.w. Pratt
surrollnding connectilJe tissue?
beam reflections to the transducer, resulting in
duplication of structures in the image a. hepatic vei ns 141. In a fiberoptic endoscope: 142. Most pediatric gastrointestinal fiberoptic
b. reflection of excessive ultrasound waves back b. portal veins a. a coherent fiber bundle transmits light from endoscopes designed for use in human infants:
to the transducer by homogeneous £issues in
lhe imaging field
c. mismapping of reflected ultrasound waves
because of excessive fat in the imaging field
c. hepatic arteries
d. bile ducts
e. lymphatics
w. the light source to the distal tip
b. a noncoherent fiber bundle transmits images
from the distal tip to the eyepiece
c. about 80% of the light from the light source
a. have four-way tip deflection. with one angle
deflecting at least 180 degrees
b. are equipped with an operating or accessory
channel 7.8 to 10 mm in diameter for
d. m ismapping of reflected ultrasound waves has been dissipated when it reaches the passage of diagnostic accessories
from previous pulse repetitions when a high distal tip c. have an umbilical cord containing one
pulse repetition frequency is used
ww
d. each glass fiber is clad with a substance of coherent and tw"o noncoherent fiber bundles
e. multiple reflections betw"een nvo closely low refractive index d. have an insertion tube 40 to 50 cm long
spaced tissue interfaces, producing a series e. water is circulated around the fiber bundles e. are not equipped with a vacuum device
of parallel lines within the image to reduce heat

@ 1998 Mosby-Year Book. Inc. PllOtocopying is prohibited by law. Correct answers are on pages 92-100.
www.alkottob.com
84 SECTION 3 www.vet4arab.co.cc Diagllostic Imag/IIg alld Recordillgs 85

143. For Japaroscopy ill which intraabliominai d. Narcotics can be given to decrease pyloriC 151. eoncemingcolonoscopic examination of small 155. eoflcemi"g endoscopic examination of the right
electrocoagulation willlifcely be used, thegas of and cranial duodenal tone. animals using a flexible jiberoptic endoscope, bronchial tree ofdogs, which statemem is mOst
choice for insufflation is: e. Food should be withheld for 72 hours before which statement is most accurate? accurate?

om
a. air endoscopy. a. The distal tip of the endoscope is inserted a. The bronchus to the right cranial lung lobe
h. carbo n dioxide blindly for 10 cm, after which the colon is exits the principal bronchus medially.
148. During gastric endoscopy. the pyloric antrum insufflated with air.
c. nitrous oxide b. The bronchus to the right middle lung lobe
and pylorus are in view but tile endoscope tip b. It is not possible to pass the distal tip of the exits the principal bronchus venrrally.
d. oxygen
cannot be advanced, or it repeatedly flips over endoscope cranial to the ileocolic valve. c. The bronchus to the right caudal lung lobe
e. hydrogen
the angularis toward the cardia. What is a,e c. The e ndoscope should be slowly withdrawn exits the principal bronchus ventromedially,
usual cause of this phenomenon? to fa cilitate careful visual examination. just beyond the middle lung lobe bronchus.
144. Which condition in smaff animals is least likely d. Biopsy of lesions in the hepatic flexu re of the
a. gastric overinsufflation d. Numerous small tertiary bronchi exit the
to he diagnosed by IIpper gastrointestinal

b.c
b. gastric mural disease colon can be done with alligator forceps or length of the right principal bronchus.
endoscopy? uterine biopsy forceps.
c. atrophic gastritis e. Beyond the origin of the caudal lung lobe
a. esophagitis e. General anesthesia is contraindicated bronchus. the right principal bronchus
d. linear forei gn body traversing the pylorus
h. pyloric hypertrophy because it significantly alters the appearance becomes the accessory lung lobe bronchus.
e. anticholinergic overdosage of the colonic mucosa.
c. hypertrophic gastritis
d. abnormal motility 156. Co nceming transurelhral cystoscopic
149. Wllat is tile major reasoll for unproductive or 152. A contrai"dicatiorl for Japaroscopy is:
e. gastric carcinoma unsuccessful colonoscopic examination? examination of small animals, wllich statement
a. diaphragmatic herrtia is most accurate?

tto
a. bowel perforation b. hepatic cirrhosis
145. A coll trail1diootion/or IIpper gastrointestilWl a. A rigid 2.7 mm (diameter) arthroscope can
endoscopy is: b. iatrogenic intussusception c. bladder rupture be used in female cats and female dogs.
c. inadequate bowel cleansing d. renal amyloidosis b. Urine samples for microbiologic culture can
a. hematcmesis d. underinsufflation of the bowel e. suspected or confirmed neoplasia be collecled during cystoscopy.
h. esophageal stricture e. at resia ani c. Gas insufflation of the bladder is
c. bowel perforation
contraindicated if bladder mucosal bleeding
d. chronic vomiling 153. Omceming rhinoscopic examination ofsmall is suspected or confirmed.

o
150. Concerning preparation of small animals for a"imals, which statement is most accurate?
e. hypersecrctory disorder co!Otloscopy. which statemem is most accurate? d. Biopsy of localized bladder lesions is best
a. Flexible pediatric fiberoptic done through a rigid arthroscope.
a. Magnesium citrate should not be given to rhinolaryngoscopes, designed fo r use in
146. Which phenomenon is observed whetl tlte c. The most common complication is urethral

alk
cats fo r colonic cleansing. human infants, are ideal because they allow
endoscope is passed into tile small intestine oJ perforation.
b. Polyelhylene glycol should not be given to large· volume irrigation.
small-breed dogs?
dogs for colonic cleansing. • b. Light sedation usually is sufficient for
a. bradycardia c. Several warm-water enemas are superior to restraint. 157. COllcerning prepubic percutaneous cystoscopy
h. gastric dilatation-volvulus oral electrolyte lavage solutions. using a rigid arthroscope in small animals,
c. Radiographs of the nasal cavity and frontal which statemellt is most accurate?
c. pyloric nutter d. Metoclopramide is given to stimulate sinuses should be made before rhi noscopy.
d. rectal prolapse vomiting and delay gastric emptying. d. Any bleeding of the nasal mucosa during the a. Xylazine sedation is sufficient for restraint
e. gallbladder evacuation e. Enema solution should be repeatedly infused procedure indicates inflammatory disease or and analgesia.

147. Concerning upper gastroilltestillal endoscopy in


small animals. wllicll statement is most accurate?
at 75 ml/kg.
w. a coagulation disorder.
e. The preferred irrigam is 15% dextrose.
b. An incision must be made with a scalpel
th rough the skin and abdominal wall.
c. Penetration of the opposite (dorsal) bladder
wall warrants immediate laparotomy to
a. Physical restraint is usually sufficient. 154. Which condition i"creases tile risk of suture the bladder wall.
complications after tracheobronchoscopy?
b. Anticholinergic premedication can be used d. An indwelling urethral catheter is left in
ww
to reduce gastric motility and secretion. a. Oslerus osled infection place for 48 to 72 hours after the procedure.
c. Patients typically are placed in right latera] b. vegetable-matter foreign body e. A small bladder with a very thick wall is ideal
recumbe ncy so thai gastric juices gravitate c. stabilized congestive heart failure for the procedure.
toward the pyloric antrum. d. chronic cough
e. pulmonary hypertension

@ 1998Mosby-YearBook.Inc.Photocopyingisprollibitedbylaw. Correct answers are Of! pages 92- 100.


www.alkottob.com
86 SECTION 3 www.vet4arab.co.cc Diagnostic Imagin g alld Recordings 87

158. As viewed with tile endoscope in the nasal 161. Colleeming endoscopic examination a/the 164. eorlceming arthroscopic examination 0/ the 165. In detemlillillg the gender o/a bird by
passage ora llealthy horse, during swallowing: duodenllm Ofhorses. which statemem is most equine carpal joint, which statement is most laparascopy:
accurate? accurate?
a. the soft palate is displaced rostrodorsally a. the laparoscope is inserted into the cloaca

om
b. the pharyngeal openings of the auditory tube a. An endoscope that is 100 cm long is a. The stopcock of the egress cannula must be b. the abdominal air sac must be avoided
close and move la terally s ufficient fo r adult horses. kept open at aJl times until the procedure is
c. the bird should be kept anesthetized for at
c. the caudal pharynx can be seen as it dilates b. The normal duode nal mucosa appears soft com pleted.
least 30 minutes after laparoscopy to prevent
and velvety, \vith a gray- red or yellow- red b. The trocar is inserted before the joint is air sac eventration
d . the epiglollis can be seen as it occludes the
color. dis tended with flu.id or gas.
glottiS d. the gonad can be found caudal to the caudal
c. The duodenal diverticuJum can be seen 24 to c. The trocar and arthroscopic shea th should lobe of the kidney
e. the corniculate process of each arytenoid
30 inches distal to the pylorus. be inserted at a 45-degree a ngle to the skin.
cartilage can be seen as it is adducted e. the bird should be placed in right lateraJ
d. Pe ristalsis ceases because of vagovagaJ d. The arthroscopic sheath and enclosed recumbency

b.c
reflexes. arthroscope should be "walked" along the
159. In elldoscOfJicexamiliarioll of tile guttural e. The extreme na ture of the procedure a rticular cartilage to the s ite of suspected
pouclJes: necessita tes general anesthesia. lesions.
a. the endoscope is int roduced into the dorsal e. The joint is incised before the joint capsule is
nasal meatus distended with fluid or gas.
162. COllcemillgcfldoscopic examinatio n 0/ a mare's
h. general anesthesia is necessary genital tmct. which statemeTil is most accurate?
c. the slitlike open ings a re located rostrovemral
a. The vagina and cervix should be thoroughly
to the pharyngeal recess
fl ushed with gl utaraldehyde solution before ELECTROCARDIOGRAPHY

tto
d. the stylohyoid bone can be identified by its endoscope insertion.
mediolateral course, dividing the gunura!
b. The diestrual cervix is flattened, partially
pouch illla dorsal a nd ventral compartments Recommended Readillg
o pe n, and shuated near the floor of the
e. the inte rna l carotid artery can be identified cranial vagina. Keene BW, Hamlin RL: Small animal carclioloKJ\ Philadelphia. 1997, \VB Saunders.
by its rostroventrallocation in the dorsal
c. The estrual cervix is closed and situated near Miller MS. TIUey LP: Mamml afsmall animal cardiology. ed 2. Philadelphia, 1995. WBSaunders.
compartment
the center of the c ranial vagina. Physid::-Sheard PW:ln Colahan P1" et aI: eqUine metiicineand surgery. cd 5, SI. Louis. 1998, Mosby.
d . The endometrium rapidJy becomes

o
Smith FWK. Tilley LP: Rapid imerpretatioll o/llean sounds, murmurs, alld arrhythmias, Baltimore,
160. COllcemillge1ldoscop;cexam;Ilatioll of the hyperemic when the ute rus is insufflated 1992.Wi.lliams & Wilki ns.
esophagusatld stomach ofadult horses, which with air. Tilley LP: Essentials ofcanine alld feline electrocardiography: illterpretation alld treatment. cd 3,
statemem is most accurate? e. The endometrial fo lds arc highly prominent, Baltimore. 1992, Williams & Wilkins.

alk
a. General anesthesia is usuaUy required hyperemic, and edematous during diestrus.
because of the extreme sensitivity of the
Practice answer sheets are on pages 259-260.
esophageal mucosa. 163. COllcemillgdistelltion 0/ajoi1lt capsule/or
b. Insufflation is co nt raindicated because of the arthroscopic examination ofequine join/s,
danger of esophageal or gastric ru pture. whicll statemellt is most accurate?
c. The e ntrance to the cardia from the Questions
a. Distention with carbon dioxide or nitrous
esophagus can be recognized as a tricuspid loP. Tilley
oxide is more practicaJ but also more
valve-like struc ture. w.
damaging than diste ntion \vith fluid . 166. A 10-mm upward deflection a/the ECG stylus in 167. Tile action o/digitalis in treating atrial
d . The sacc us cecus portion of the stomach can response to 1 m V 0/ electrical current is known as:
b. Distention with sterile saline or lactated arrhythmias is best described as:
be seen by directing the e ndoscope tip
Ringer's solution is effective, rela tively a. depolarization
dorsally and to the horse's right side. a. direct action on the atrial tissue to abolish a n
harmless, and very practicaJ. b. pola rity
e. Sufficient warm water is instilled to fully irrita ble focus
c. DistentiOll with ai r offe rs the advantage of
distend the stomach. c. standardizalion b. slowing of the ventricular rate by prolonging
easier particle mobiLi7..ation and removal
d. conductivity the refractory period of the atrioventricuJar
ww
than distention with fl uid.
e. aUlOmaticity node
d. Distention \vith flui d is complicated by the
need for sophisticated pumps, ,vith sensitive c. slowing the atriaJ rate so that the s inus node
can take over as pacemaker
gauges to continually monitor pressure.
d . increasing the s trength of ventricular
e. Distention with fl uid provides a sharper
contrac tion
image a nd more accurate assessm e nt of
cartilage than diste ntion \vith gas. e. slowing the ventricular rate by prolonging
the refr actory period of Purkinje fib e rs

C 1998 Mosby-Year Book. Inc. Photocopying is prohibited by la w. Correct answers are on pages 92- 100.
www.alkottob.com
88 SECTI ON 3 www.vet4arab.co.cc Diagnostic Imaging and Recordings 89

168. Left-axis deviation occurs wilen: c. propranolol, verapamil, and lidocaine 178. ~arasympathetic stimulation of the heart results 183. As the PR inrervallengthens, the RR interval
d. propranolol, d igitalis, and d iltiazem m: shortells before "dropped complexes." 711is
a. lead I is positive and aVF is positive
e. digitalis, enalapril, and lidocaine a. increased heart rate statemetlt best describes:
b. lead I is positive and aV F is negative

om
c. aVF is positive and lead J is negative b. increased force of contraction a. type-II atrioventricular block, with variable
d. aV F is negative and lead I is negative 174. When the sympatlletic nervous system is c. decreased heart rate and slowed block
stimulated: atrioventricular conduction b. third-degree atrioventricular block
e. lead I is negative, aVF is negative, and lead II
is negative a. the heart rate increases d. increased speed of atrioventricular cond uction c. first-degree atrioventricular block
b. the speed of atrioventricular cond uction e. increased atrioventricular conduction d. type-I atrioventricular block (Wenckebach
169. Which rhythm is characterized by slowing and decreases and the heart rate decreases phenomenon)
speeding of the heart race related to respiration? c. the force of contraction decreases 179. The normal PR imerval jn dogs js: e. left bundle branch block
d. the heart rate decreases

b.c
a. sinus arrhythmia a. 0.06 to 0.13 second
e. the speed of atrioventricular conduction b.0.lOtoO.16second 184. The three augmented wtipolnr limb leads are:
h. normal sinus rhythm
increases and the heart rate increases
c. sinus bradycardia c. 0.12 to 0.20 second a. aV!~ aVR, and aVF

d . sinoatrial block d. 0.1 6 to 0.24 second b. I, n, and III


175. A continuous rhythm strip showing each normal e. 0.04 to 0.10 second c. I, aV R, and III
e. sinus tachycardia
complex followed by a premature ventricular
d. I, aVL> a nd aVF
complex is most appropriately interpreted as:
170. The QRS complex 011 the ECG represents: 180. At times tile sinus impulse penetrates and e. I, II, and aVL
a. ventricular trigeminy contributes to ventricu!aractivation, along with

tto
a. atrial depolarization b. irregular rhythm a premature focus in the ventricles. The complex 185. In dogs and cats electrical allemalls is associated
b. ventricular repoiarization c. ventricular bigeminy produced by this collision is referred to as: with:
c. ventricular depolarization d. infrequent premature ventricular
a. R-on -T premature ventricular contraction a. pericard ial effusion o r supraventricular
d. atrioventricular conduction contractions
b. fusion ce mplex tachycardia
e. atrial repoiarization e. ventricular escapes
c. sequential premature ventricular comraction b. ventricular tachycardia
d. ven tricular escape complex c. sinus arrest or atrioventricular block

o
171 . The P wave on the ECG represents: 176. An arrhythmia that can progress to ventricular
e. atrial premature complex
fibrillation and for which the clinician must
d. left ventricular enlargement or pericardial
a. firing of the sinus node effusion
continually monitor is:
h. ventricular repolarization 181. A single ventricular ectopic focus may fire once e. ventricular fibrillation

alk
c. atrial repolarization a. first-degree atriovenrricular block
or may fire in a series of rapid successive
d. atrial depolarization b. sinus tachycardia jmpulses to produce: 186. Sick SifllLS syndrome is a term associated with:
e. ventricular depolarization c. atrial premature complexes
a. ventricular escape rhythm a. pacemaker battery failure
d. sinoatrial arrest
b. ventricular fusion complexes b. ventricular fibrillation
e. ventricular tachycardia
172. The QT interval on the EeG represents: c. ventricular fibrillation c. sinus bradycardia and sinoatrial block
a. ventricular depolarization d. ventricular tachycardia d. heartworm disease
177. Some atrial impulses are blocked in the e. vemricular bigeminy
b. atrial depolarization w.
atrioventricular junction and do nOl reach the c. respiratory disease and cor p ulmonale
c. ventricular repolarization ventricles, but the PR illlerval of those that do
d. conduction through the atrioventricular conduct are identical to each other. This 182. eoncerningfirst-degreeatrioventricular block in 187. Wolff-Parkinson- \.1Ihite syndrome consists ot
node statemerll best describes: dogs, which statement is most accurate?
e. ventricular depolarization and repolarizarion a. sinus bradycardia and ventricular premature
a. type-II atrioventricular block a. There is a 2:1 ratio ofPto QRS. complexes
h. third-degree atrioventricular block b. Conduction through the junction is b. ventricular preexcitation with paroxysmal
ww
173. Drugs most commonly used in treatment of c. first -degree atrioventricular block prolonged more than 0.13 second. supraventricular tachycardia
atrial arrhythmias include: d. type-I atrioventricular block c. The most common cause is heart failure. c. complete atrioventricular block and
a. digitalis, procainamide, and lidocaine e. left bundle branch block d. The rhythm is irregular. ventricular preexcita tion
b. disopyramide, procainamide, and e. The heart rate is rapid. d. a trial tachycardia and heartworm disease
propranolol e. ventricular escape complexes and sinoatrial
arrest

© 1998 Mosby-Year Book, Inc. Photocopying is prohibited by law. Correct answers are on pages 92-100.
www.alkottob.com
90 SECTION 3
www.vet4arab.co.cc Diagtlostic Im aging a /Ill Recordings 91

c. variation in heart rate and rhythm secondary 193. Tile cellular origin of tile EEG is predomillafl tiy 195. The brain structures most directly illvolved in
188. The important features of right ventricular tile result of
etllflrgement are: to increased vagal tone from respiration maintaining tile EEGofalert wakefulness are
d. ventricular fib rillation during cardiac arrest a. algebraic summation of postsynaptic the neocortex and tile:
a. wide and positive QRS complexes in leads I,

om
after repeated a u em pts to convert the membrane potentials a. amygdaJa
II, III , and aVF rhythm b. summation of excitatory effects in cortical b. limbic system
b. S waves in leads 1, II, JII, and aV F and an axis e. sinus bradycardia secondary to hypokalemia synapses c. hypothalamus
of + 150 degrees and decreased a utomaticity c. changes in gl ial me mbrane potentials d. basaJ ganglia
c. S waves in leads I, II , III , and aV F and an axis
of +30 degrees d. summation of action potentiaJs e. brain stem
d. deep Q waves in leads I, II , III , and aVF and 190. Atrial standstill is most likely to be associated e. unknown factors not yet elucidated by
an axis of +30 degrees witll: research evidence
196. The major use of electromyography is in:
e. wide and negative QRS complexes in leads a. hypoadre nocorticism, Iddney failure or

b.c
aV R• aVu and aV F diabetes mellitus 194. Tile EEG is most likely to provide mOrlitoringof a. suppressing excessive electrical activity in
all of tile following except: muscle
b. hypoadrenocorticism, cor pulmonaJe or
diabetes melli tus b. neurologic diagnosis
189. AtrioventriC/llar dissociation ca ll represellt a a. anesthesia
c. hearhvorm disease, hypoadrenocorticism or c. acupuncture
combination of b. sleep
liver disease d. diagnosing electrical diseases of muscle
a. increased ventricular automaticity with an c. dream sleep
d. liver disease, congestive heart failure or e. promoting reinnervation of skeletal muscles
eClOpic ven tricular fo cus in control of the d. coma
diabetes mellitus
ventricles, while the sinoatrial node e. pain
continues to activate the atria e. gastric torsion, hypoadrenocorticism or cor

tto
pulmonale
b. control of atrial and ventricular activity by
the sinoatrial node. with conduction from
the sinoatrial node to the atria,
atrioventricular node, and ventricles
ELECTROMYOGRAPHY
Recommended Readi"g

ELECTROENCEPHALOGRAPHY Braund KG: Clinical syndromes in veterinary neurology. cd 2, SI. Louis, 1994. Mosby.

o
Chrisman Cl: Problems in small animal neurolOID\ ed 2, Ballimore, 1991, Williams &Wukins.
Mayhew IG: I1lrgeanimal neurology. Ballimore, 1989, Williams & Wilkins.
Recommellded Readillg Oliver IE, Lorenz MD: Handbook o/veteri"ary "eurology. ed 2, Philadelphia, 1993, WB Saunders.

alk
Klemm WR: Animal electroencephalography, New York, 1969. Academic Press.
Klemm WR: API)/ied electronics for veterinary medicine and animal physiology, Springfield, Ill, Practice answer sheets are on pages 259-260.
1976, Charles C Thomas.
Klemm Wit [n [ndrieri RJ: Epilepsy. Prob[Vet Med 1:535-556, 1989.
Questions
Practice answer sheets are on pages 259·260. I.M. Bowen

Questions
W.R. Klemm
w. 197. Use of needle electrodes in conducting all
electromyographic examination:
a. prevents insertion activity
198. After nerve injury,fibrillatioll potentials occur in
denervated canine skeletal muscles witllin:
a. I to 2 hours
b. increases serum creatine phosphokinase b. 12 to 24 hours
191. 71leeiecrroencepllalogram (EEG) can most 192. WI,at causes tile major artifact associated witll (CPK) activi£y c. 2 t03 days
accurately be described as: EEG recording in animals? c. inhibits respiration
ww
d. 5 to 7 days
a. the best way to measure intelligence a. epilepsy d. stimulates interference patterns e. 10to 12 days
b. the only way to monitor consciousness b. lighlning e. reduces blood potassium levels
c. a plot of microvolts versus time c. radio interference
d. a refl ection ofthe amount of brain activity d. computer te rminals
e. the best way to detect epilepsy e. muscle spasms and movement

Cl 1998 Mosby--Year Book, Inc. Pilotocopying is prollibiled by law. Correct answers are Of! pages 92-100.
www.alkottob.com
92 SECTION 3 www.vet4arab.co.cc Diagnostic Imaging and Recordings 93

199. If a peripheral motor nerve is partially damaged, 200. Peripllerai nerve conduction velocity. which can 27. a Intravenous contrast agents may cause severe contrast, long-gray scale technique is preferred
which oft/le/ollowing is most likely to be be measured electromyograpllically. is most side effects if given to markedly dehydrated for thoracic films. Many factors affect contrast.
observed Oil eleclromyograplTicexaminarion 10 likely to be slowed in all animal with: patients. The other problems listed are not 46. b Film development is faster at higher

om
days after injury? contraindications for intravenous urography. temperatures. Developer solution must be
a. myopathy
28. e The pylorus is not dependent on right lateral replenished periodically. Developer solution
a. fibrillation potentials only b. neuronopalhy
views of patients with gastric IOrs ion and does not harden the film.
b. motor unit potentials only c. axonopathy therefore does not accumulate fluid. 47. b Static electricity causes such branching
c. combination of fibrillation potentials and d. myelinopathy 29. c On a double-contrast cyslOgram, the bladder is artifacts.
motor unit pote ntials e. uppe r motor neuron disease distended with negative-contrast medium. with 48. a The National Council on Radiation Protection
d. myolOnic discharges a small amount of positive-contrast medium and Measurements recommends a maximum
e. absence of nerve conduction added. permissible dose of 5 rem per year.
30. a Hepatomegaly displaces the stomach 49. d Nonscreen film requires longer exposure times

b.c
caudodorsally. The other organs listed are caudal and should not be used for routine procedures.
to the stomach.
Answers 31. d Osteochondrosis most commonly affects the
50. c Portable x-ray machines should never be hand
held during use, because this increases radiation
I. b This is the definition of the silhouette effect. physical exam ination findings support this caudal aspect of the humeral head. exposure.
2. a Pleural effusion usually s ilhouettes the heart diagnosis. 32. e Os teochondritis dissecans is primarily seen in 5 1. c Grids are indicated for parts thicker than 10
and diaphragm. 16. e Only the pulmonary trunk is located at this large-breed dogs. Cill.
3. c Grids are used ifthe body part to be site. 33. a These signs are typical of panosteitis. The 52. e High-mAs, low-kVp techniques produce the
radiographed is greater than 10 cm thick. 17. d A "reversed- DO. appearance is typical ofright patient characteristics and physical examination highest contrast. All listed techniques are

tto
heart enlargement. findings support the diagnOSis. equivalent in terms of film blackness.
4. d Radiolucent air surrounding the lung lobes is a
sign of pneumothorax. lB. b These cardiac changes are seen with patent 34. c The radiographic findings and patient 53. c Halving the mAs is a reasonable adjustment to
5. c Pneumomediastinum docs not occur
ductus arteriosus. Aortic s te nosis is characleri! tics support a diagnosis of Legg- correct an exposure that is too dark.
secondary to pneumothorax. characte rized by a bulge in the ascending aorta. Calve-Perthes disease. 54. d Rare-earth screens have increased speed as
Ventricular septal defect does not cause a bulge 35. d Osteosarcoma usually occurs in the their main advan tage.
6. c VVhen the soft tissue lesion is in the
in the descending aorta. Pulmonic stenosis and metaphyseal area of long bones.
nondependent lung, it is surrounded by more air, 55. d Unprocessed x-ray film is naturally green to
tetralogy or Fallol cause right heart enlargement. 36. b Barium solution is given PO at 6to 12 m1 /kg.
giving it better cont rast and visibilily. gray.

o
19. e Hypovolemia usually res ults in a small heart. 37. e Salter class-V fractures are frequently not
7. e Tracheal colla pse can be diagnosed on plain 56. e Static often causes this type of artifact.
radiographs. Fluoroscopy is helpful but not 20. b These changes are typical of heartworm evident on radiographs. The other choices 57. e MilJiamperage does not affect detail.
essential in most cases. disease. None of the other diseases listed would describe Salter fractures of class I through rv.
58. d No other option produces these radiographic

alk
have enlarged, tortuous arteries. 36. a Lysis of the vertebral end plates is consistent
B. a Atelectasis (collapse) typically causes a signs.
mediastinal shift to compensate for loss of lung 21. b The normal canine Jddney is 2.5 to 3.5 times with diskospondylitis.
the length of the second lumbar vertebra. 59. c "is not related to insufficient growth of the
volume. There is no loss or lung volume in 39. e Lysis of the tarsal bones is not a s ign of ulna.
pneulllonia. 22. e Free abdominal gas increases contrast in the degenerative joint disease.
abdomen. 60. e It is not found in patients with a fragmented
9. e Visible, opaque fissure lines are a sign of 40. b None of tlle other descriptions is accurate. coronoid process.
pleural effusion. 23. d Felal skeletons can be seen on radiographs by 41. a A dorsolateral-palmaromedial oblique
40 to 45 days of gestation. 61. e Lupus erythematosus is nonerosive in dogs.
10. e Only the bronchial lumen is seen with an air projection delineates the dorsomedial aspect of
bronchogram: the bronchial walls are 24. a Fluid accumulates in the pylorus, which is the third carpal bone, whereas a dorsoproximal- 62. e This ligament attaches at the affected region.
silhouetted against fluid in the lungs.
II. a Air bronchograms are only seen with an
alveolar pattern. 25.
w.
dependent on right lateral views. Gas
accumulates in the nondependent fundus.
c Iodine contrast medium does not cause severe
inflammation/peritonitis if it leaks into the
dorsodistal oblique projection provides an end-
on view of the tarsal bones.
42. e Navicular disease is best diagnosed using at
least three different views of the navicular bone.
63. d This provides an unobstructed view of the
nasal cavity.
64. c The other options are much less specific.
65. e It usually occurs only with discospondylitis.
12. e Chronic bronchitis typically causes a bronchial
pattern, not a nodular pattern. peritoneal space. Barium sulfate causes severe 43. a Grids are placed between the patient and the 66. a There is no disk at CI-C2.
peritonitis if it leaks into the peritoneal space x-ray film. 67. a This usually occurs only with diskospondyliris.
13. e Peripheral arteries and veins are tht same size.
ww
and is contraindicated if bowel or gastric rupture
14. d On an expiratory radiograph, the lung fields 44. d DecreaSing the developer solution 68. c Patients with spinal fractures shouJd not be
is suspected. temperature makes the film lighter
typically do not extend caudal to the l\velfth manipulated. An orthogonal view is necessary
thoracic vertebra. 26. b Periosteal new bone formation is (underdeveloped). All other choices increase the ror complete evaluation.
radiographically evident 7 to 10 days after injury. radiographic density.
15. b The radiographic changes described are 69. b Myasthen ia gravis, megaesophagus, and
typical of left-sided congestive heart failure 45. a High kVp results in low contrast, \vith a long aspiration pneumonia are common in patients
(pulmo nary edemal. The clinical signs and gray scale. A high-contrast, short-gray scale with thymoma.
technique is preferred for extremity films ; a low-

@ 1998 Mosby-Year Book. Inc. Photocopying is pro/libired by law.


www.alkottob.com
94 SECTION 3 www.vet4arab.co.cc Diagnostic Imaging and Recordings 95
70. c Bacterial pneumonia produces an alveolar 94. e Air bubbles compromise the quality of the 104. c The capsule is a bright linear echo when the
infiltrate. allows visualization of the far wall. Mucus and
study, impairing assessment of the bladder wall. ultrasound beam is perpendicular to the kidney. gas have echogenicity s imilar to that of the
7 1. a In the other options the vessels are not clearly 95. b Extravasation occurs in normal and The cortex and medulla are both hypoechoic, mucosal layer (all are hyperechoic). so that the
seen. abnormal prostate glands.

om
but the cortex is always more echogenic than interface between the mucosal s urface and
72. c Enlargement of this vein is consistent with 96. d This procedure allows identification of one the medulla. Echogenicity of the renal cortex either gas or mucus cannot be dis tinguished.
pulmonary venous hypertension. mass as the bladder and more accurate should be approximately the same as that of the Because fluid is anechoic, the mucosal surfaces
73. d Pleural effusion is more common in right assessment of the other mass. liver and less than that of the spleen. The renal are clearly seen as a separate layer.
heart failure . 97. d This is important to know so as not to pelvis is hyperechoic unless distended. Ill. e From the lumen to the external serosal layers,
74. a These radiographic signs are classic for patent misdiagnose stomach displacement in cats. Diverticula are hyperechoic. the four wall layers are as follows; a hypoechoic
ductus arleriosliS. 98. c Attempting to ide ntify the fundus and/or 105. e The renal cortex is more echogenic than the mucosal layer; an irregular, discontinuous
75. e Comparing the tracheal appearance at pylorus by using gastric gas as a contrast renal medulla. Echogenicity of the renal cortex echogenic lamina propria submucosa parallel to
inspiration and expiration is very helpful. medium is the most appropriate next step in an should be apprOximately the same as that of the the mucosa (lamina epithelialis); slightly

b.c
attempt to rule out gastric volvulus. liver and less than that of the spleen. The renal heterogeneous, hypoechoic smooth muscle;
76. c Dyspneic, coughing patients often have gas in
cortex of obese cats has increased echogenicity and a hyperechoic serosa/perivesicular fat
the esophagus. 99. a This is the typical appearance of
because offal in the renal tubules. interface. All four layers may be visible when the
77. e This is more often a sign of pneumothorax. pseudoulcers.
106. b Small bright dots a t the corticomedullary bladder is collapsed, but imaging is suboptimal
78. c Fluid pools around the heart. prod ucing a 100. d The hypertonic nature of the contrast
junction represent a rcuate arteries. because the wall is artificially thickened. When
silhouette sign. medium pu lls the fluid onto the bowel from the
intravascular space. 107. e Acoustic shadowing occurs when a ll or most the bladder is viewed with a high-frequency
79 . d There is no obvious rcason why heamvorrn oCthe ultrasound beam is attenuated (reflected, transducer (7.5 MHz or higher), v.rith slight to
disease would produce a heart with the shape 101. c High.freque ncy probes provide better axial moderate distention, three layers are seen: a
absorbed, or scattered) by a structure so that no
described . resolution hut poorer penetration than low- hype rechoic inner layer representing the
ultrasound is reflected by deeper structures.

tto
80. b The depcndclH lung is partially collapsed and frequency probes. Low-frequency probes submucosa/ mucosallumen interface; a slightly
provide poorer axial resolution and better 108. d The spleen has smooth borders and a well-
exhibits increased opacity but may silhouette a defined capsule that appears as a fine heterogeneous or hypoechoic middle layer
mass. penetration than high-frequency probes and representing the smooth muscle layers; and a
may be needed to image deeper structures. For echogenic line when imaged at right angles to
81. a Of the choices listed, Paragonimus is the most the probe. TIle spleen has a finer and more hyperechoic outer laye r representing the
example, in imaging the liver of a large dog. the serosa/ perivesicular fat inte rface. Stretching the
common cause of cavitary lung lesions in cats. hyperechoic echo pattern than the liver.
sonographer might use a 7.S-MHz probe to bladder wall by maximal distention can mask
82. d Metastasis does not usually produce clinical examine the s upe rficial portion of the liver but Intrapare nchymal splenic venous branches are
s igns until late in the disease. hypo echoic to anechoic when viewed at a n even severe bladder wall thickening. Acoustic

o
might swilch to a S.O-M Hz probe for enhancement enol shadowing) occurs deep to
83. a Splenic torsion can occur as an independent examination of deeper portions of the liver. oblique angle to the ultrasound beam but the
event. walls may appear hyperechoic when imaged at the urinary bladder because there is minimal
102. a Acoustic impedance of a tissue can be anenuation of ultrasound as it passes through a
84. e Hydrone phrosis is not a reliable sign of right angles to the probe. Echogenic flow is a
calculated from the density of the tissue and the cystic structure. The cranial border (apex) of the

alk
chronic renal failure. normal finding in larger vessels and does not
propagation speed of sound in the tissue. bladder is poorly imaged by many ultrasound
indicate pathologic changes. Splenic arteries
85. b Emphysematous cystitis is easily seen in Sound is reflected from an interface behveen probes because of ultrasound refraction in this
are not visible within the parenchyma, although
survey radiographs. two tissues with different acoustic impedances. area. This area can be seen in a cystogram.
arterial rami may uncommonly be found at the
86. b The location of the mass is most consistent The intens ity (viewed as brightness) of sound 112. e The canine prostate gland is a bilobed
hilus. Splenic veins are visible.
\vith external iliac lymphadenopathy. reflected depends on the angle of the structure surround ing the proximal urethra at
ultrasound beam and the difference in acoustic 109. a The gastrointestinal tract has five layers, \'lith
87. e Most of the urethra is extraperitoneal. altemating hyperechoic and hypoechoic echoes. the neck of the bladder. The normal prostate
impedance. gla nd has a uniform, somewhat coarse
88. d The insolubility of air makes it hazardous in From th e lumen to the external serosa, the layers
this situation. 103. b The relative echogenicities of these tissues echogenicity similar in brightness to that of the
89. b The type of contrast medium specified is not
secreted by renal tubules.
90. d This is the most accurate method of assessing
w.
are as follows: splee n > renal cortex and liver>
renal medulla. In comparing echogenicities of
two tissues, the sonographer must e nsure that
the tissues are at the sam e depth (to avoid
are as follows: mucosal surface (hyperechoic),
mucosa (hypoechoic), submucosa Olyperechoic),
muscularis propria (hypoechoic), and serosa
(hyperechoic and includes the subserosa).
spleen. The urethra is a linear structure located
between the lobes and has been described
variously as hyperechoic or hypoechoic. The
description of a hypoechoic oval \'lith a
bladder integrity. artifaci'caused by depth gain compensation), 110. e Mucus and gas are both hyperechoic, but
hyperechoic line located centrally \'Iithin the
91. e This offers the optimal combination of safety that the probe frequency is the same for each mucus allows visualization of the far wall of the
intestine, whereas gas exhibits distal or deep lobe would better apply to the testicle. The
and low cost. organ, and that structures are not present that
ww
acoustic shadowing. Reverberations between testicle is a smooth, homogeneous, elliptic
92. d The other choices lis ted do not allow the might alter the echogenicity of one of the organ, \'lith the rete (mediastinum) testis
mucosa to be visualized in comparable detail. tissues (e.g., if fluid is superficial to the imaged the transducer and gas appear as artifactual
lines \'Iithin the shadow; acoustic shado\'ling appearing as a central hyperechoic line in
93. b The high solubility of carbon dioxide tissue, echogenicity of the tissue is increased). longitudinal images and a hypcrechoic dot in
associated with gas has been called '·dirty."
decreases the chance of fatal gas embolism. transverse images.
Intraluminal fluid is anechoic and (like mucus)

@ 1998Mosby-YearBook, lnc.Pho(ocopyingisprollibiredbylaw.

www.alkottob.com
96 SECTION 3 www.vet4arab.co.cc Diagnostic Imagitlg and Recordings 97

11 3. d The uterus is hypoechoic, not hyperechoic. 11 6. d Artifacts are frequently seen when imaging 119. b The su perficial digital extensor tendon image at the point of reflection . A constant
During estrus the lumen may not be visible or a the liver and ga1lbladder. Mirror-image artifact originates on the media1 epicondyle of the sound wave velocity through soft tissues is
small amount orfluid may be presenl. The occurs when sound is repeatedly reflected humerus and inserts d istally on the phaJanges. assumed even though slight variations in
(An accessory ligament originates on the distal

m
uterus may be differentiated from the aorta and between the lung/diaphragm interface and the propagation speed occur.
caudal vena cava because these vessels hepatic parenchyma. Black streaks originating radius.) Deep to the superficial digital extensor 125. b Pulsed Doppler is a method for determining
bifurcate caudally, whereas the uterus fTom the curved surfaces of the gallbladder and tendon is the deep digital flexor tendon, which the velocity of moving structures, such as
bifurcates cranially. (Also, the vessels are uSlImly extending into the hepatic parenchyma deep to originates on the d istal hwnerus, proximal flowing blood. The technique relics on changes
anechoic with a proper gain selling.) In contrast the gallbladder are caused by refraction of olecranon, and mid-radius/ulna. The inferior in the frequency of ultrasound waves reflected
to the gastrointestinal tract, the uterine wall in sound from a curved object. Distal acoustic check ligament continues as the palmar carpal from objects moving toward or away from the

.co
anestrus does not exhibit layers. Also, enhancement ("through transmission~) often ligament, travels deep to the deep digital flexor transducer. In the example of flowing blood,
hypcrechoic gas or ingesta in the ilHcstinal causes an artifactual increase in echogenicity of tendon, and joins the deep digital flexor tendon ultrasound waves reflected off blood cells
lumen and peristalsis are generally apparent. the live r deep to the gallbladder. in the mid-metacarpal region. The interosseous moving toward the transducer have a higher
Immediately postpartum, the endometrium 117. d Echocardiographic findings in hypertrophic (suspensory) ligament originates on the palmar frequency than the original emitted sound wave
and myometrium can be distinguished. Three cardiomyopathy include left ventricular surface of the third metacarpal bone and lies frequency, whereas sound waves reflec ting off
layers may be visible in the myometrium: an hypertrophy, either symmetric or asymmetric, deep to the flexor tendons and inferior check cells moving away from the transducer have a
inne r hypoechoic circular muscle layer; a behveen the left ventricular wall and septum. ligament. It bifurcates at the level of the lower frequency.
central hyperechoic fibrovascular layer; and an Left ventricular chamber size is reduced and junction of the middle and distal thirds of the
126. c The power control on an ultrasou nd machine
metacarpus.

ob
oLlter h ypoechoic longitudinal muscle layer. the wall is noncompliant, preventing diastolic determines the amplitude or intensity of the
The waU of the small intestine has five layers. filling and causing elevated pressures at the end 120. c Severe retinal detachment appears as V- sound waves produced. As power is inc reased,
Pregnancy can be diagnosed in dogs by of diastole. The high e nd-diastolic pressure, shaped echogenic lines with points of the brighlness of the image is increased
ultrasonographic visualization of the chorionic along with distortion of the mitral valve attachment at the optic disk or ora serrata. The because more sound waves are reflected back to
cavity at 17to 20 days after the luteinizing apparatus, results in mitral regurgitation and lens is anechoic, whereas the anterior and the transducer.
hormone peak. The chorionic cavity appears as left atrial dilation. These changes are posterior lens capsules are hyperechoic linear
127. a The gain control o n an ultrasound machine is
a 1- to 2- mm spheric vesicle. aggravated by ventricular hypercontractility, structUres. The anterior, posterior (when seen), used to electronically amplify the signal
114. b The normal pancreas is difficult to image which increases cardiac workload and and vitreous chambers are anechoic. The optic

ott
produced by reflected ultrasound waves
because it is thin, has an echogenicity similar to tachycardia, decreasing the diastolic time nerve is hypoechoic, whereas the optic disk is
returning 10 the transducer. Amplifying the
tha t of adjacent m esentery, and is proximal available to fill the ventricle. In this patient, hyperec hoic. The iris a nd ciliary body are seen signal increases the brightness of Ule
(cranial) lO gas in the bowel. II has a narrowing of the left ventricular outflow tract as short echogenic lines on each side of the
ultrasound image. Most machines have a gain
homogeneous echogenic texture that is created by disproportional septal hypertrophy lens.
control that uniformly modifies the sound wave
isoechoic to slightly hyperecho ic to live r and is creating functional obstruction to outflow. 121. a Acoustic impedance for a given material amplification.
hypoechoic to spleen. The right extremity is resulting in high-velocity turbulent flow; this (tissue) is defined as the sound wave velocity
128. e Time-gain compensation controls are used to
dorsomedial to the duodenum, the body is further distorts the mitral valve and increases multiplied by the density of the material. As the

alk
e lectronically amplify reflected waves from
associated \vith the cranial duodenal flexure, afterload o n the ventricle. difference in acoustic impedance between £\VO
deeper structures. Because sound waves
and the left extremity is found in lile region 118. e p-blockers slow the h eart rate, which would adjacent tissues inc reases, the proportion of
emitted from a transducer are auenuated and
bordered by the stomach, spleen, and decrease cardiac workload and allow for sound waves reflected at the interface of the
reflected by superficial structures, deeper
transverse colon. The phrenicoabdominal vein inc reased diastolic filling of lile ventricle. This tissues increases. Increased reflection of sound
structures receive comparatively fewer waves.
serves as a vascular landmark for the adrenal increases cardiac output and decreases cardiac waves results in a brighter image at the point of
The time-gain compensation control
glands. The pancreaticod uodenal vein may be muscle hypoxia. Calcium-channel blockers reflection.
compensates for this nonuniform Signal
seen centrally in the pancreas. In acute slow the heart rate and increase ventricular 122. c Both the axial resolution and lateral generated for image formation.
pancreatitis the pancreas becomes thicker and re laxation, which assists diastolic filling and resolution of an image are improved by
129. dB-mode, or brightness mode, is the standard
more hypoechoic. Pancreatic pseudocysls may
be seen as anechoic areas lilal can become
q uite large. Hyperechoic areas may be seen
w.
decreases the workload of the ventricles. increasing the transducer frequency.
123. a Axial resolution improves with increasing
transducer frequency and with fewer cycles per
for nvo-dimensional image generation. The
brightness of a particular point in an image
depends on the intensity of reflected echoes
with chronic pancreatitis. pulse. returning to the transducer from the point of
115. b The normal gallbladder is teardrop shaped, 124. e Production of an ultrasound image reHes on reflection.
with a smooth, uniform wall less than 2 to 3 the transducers receiving sound waves
mm wide. The wall is isoechoic to norma1liver 130. b A IO-MHz linear-array transducer is the best
ww
reflected from tissue interfaces at various choice. High-frequency transducers provide
and is therefore difficu lt to see and measure. distances. The time for a given pulse to be
The gallbladder lumen is rare ly empty and better spatial resolution (better image detail)
emitted, reflected at an interface, and return to than low-frequency transducers. linear-array
usually contains anechoic bile. The common the transducer dete rmines the distance of the
bile duct is less than 2 mm wide and is difficult transducers, which produce a rectangular
interface from the transducer. The intensity of image, provide better image quality in the near
to follow to the duodenum. reflected waves returning to the transducer field (superficially) than do sector-scanning
determines the brightness of the ultrasound transducers. which produce a wedge-shaped
image.

@ 1998 Mosby-Year Book, Inc. Pllotocopying is prollibited by law.

www.alkottob.com
98 SECTION 3 www.vet4arab.co.cc Diagnostic Imaging and Recordings 99
131. a A urinary bladder calculus would be most 141. d Unless the fibers are clad, light tends to leak 162. d Introduction of air into the uterus produces a 178. c The cardiac parasympathetic nerves form the
likely to produce a shadowing a rtifact because as it traverses the fibers toward the distal tip of hyperemic response.
of reflection or refraction of almost all the endoscope. vagus nerves, whose fibers terminate primarily
163. b Both cause minor pathologic changes in in the s inoatrial node, atria, and
ultrasound waves by the calculus. Because very 142. a These features enhance maneuverability of

om
chondrocytcs that resolve in a few days. atrioventricular node, slowing the heart rate
few waves penetrate deep to the calculus, the distal tip. 164. e This preserves the anatomic position of and slowing conduction through the
virtually no signal is generat ed in this region 143. b Air should not be used because of the risk of atrioventricular node.
tendons and their sheaths for accurate incision.
and an anechoic shadow is produced. air e mbolism. Nitrous oxide, oxygen and 165. e Birds should be placed in right lateral 179. a The normal PR interval range is 0.06 to 0. 13
132. e Because cystic snuctures are fluid fill ed, hydrogen can support combustion. seconds.
recumbency for gender determination.
sound waves penetrate these structures to a 144. d Endoscopy can detect morphologic but not
greater degree than for soft tissues. This results 166. c The other possible choices describe electrical 180. b A fus ion complex is simultaneous activation
fu nctional disease. activity within the myocardial cell. of the ventricles by impulses coming from the
in morc sound waves reaching tissues deep to
145. c Surrounding tissues could be contaminated 167. b Digitalis acts directly on the myocardium , s inoatrial node and ventricular ectopic foc i.
the cyst As a result, there is a comparatively
by leakage of ingesta during insufflation. vagus nerve, and atrioventricular junction . 181. d Ventricular tachycardia can be thought of as
greater reflection of sound waves in these

b.c
deeper structures. producing a brighter image 14 6. a It may be caused by vagovagal refl exes 168. b Us ing at least two limb leads (I, aVF) and the a continuous series of ventricular premature
deep to the cyst. stimulated by overdistention of the bowel or by different angles at which they record the heart's complexes resulting from stimulation of an
traction on the mesentery. electrical activity, one can estimate the mean ectopic ventricular focus.
133. a Mirror-image artifacts result from reflected
sound waves traveling by multiple paths back to 147. b Glycopyrrolate is commonly used. electrical axis in the frontal plane. 182. b Delay in conduction of a supraventricular
the transducer. This can delay arrival of some of 148. a The endoscope should be withdrawn to the 169. a Sinus arrhythmia is represented by impulse through the atrioventricular junction is
the reflected waves at the transducer. Because cardia and the stomach partially deflated before alternating pe riods of slower and more rapid called first-degree atrioventricular block.
the ultrasound image is prod uced by mapping further attempts are made. heart rates, usually related to respiration . The 183. d Mobitz type-I atrioventricular block is
structures based on time of travel of reflected 149. c Fasting and administration of oral electrolyte heart rate increases with inspiration and present when the RR interval becomes

tto
sound waves, a delay results in apparent solutions help to cleanse the colonic mucosa in decreases with expiration. progressively shorter as the PR interval
pOSitioning of the structure deeper than its preparation fo r colonoscopy. 170. c Atrial depolarization and repolarization becomes longer, until a P wave is blocked.
actual location. 150. a Some treated cats may develop represent the P wave and subsequent T wave, 184. a Leads I, II, and III represent the s tandard
134. c Spleen is most echogenic, and the renal hypermagnesemia. whereas ventricular repolarization represents bipolar leads.
medulla is least echogenic. 15 1. c The directional control knobs can be the T wave following a QRS complex. 185. a Electrical alternans is diagnosed when the Po
135. b Animals with hepatic lipidosis have a manipulated so as to view the entire 171. d The depolarization waves spread from the QRS, or T complexes alter in configuration on
generalized incrcase in hepatic parcnchymal ci rcumference of the colon. sinoatrial node through the right atrium, every olher complex, every third complex, every

o
echogenicity. 152. a Gas could escape from the a bdominal cavity toward the left atrium and the atrioventricular fourth complex, etc., with each complex
136. e Animals with acute ethylene glycol into the thoracic cavity, caus ing pneumotllorax. node, resulting in a P wave. This indicates originating from the same focus.
intoxication have a marked increase in renal 153. c Preceding rhinoscopic examination may alter depolarization. 186. c Sick sinus syndrome describes a number of
cortical echogenicity. The change in 172. e The QT interval is the summation of electrocardiographic abnonnalities of the

alk
the radiographic appearance of structures.
echogenicity is proba bly the resuJt of ventricular depolarization and repolarization. sinoatrial node, including severe sinus
154. e Biopsy during bronchoscopy could result in
accumulation of calcium oxalate crystals in the serious hemorrhage. 173. d Procainamide, lidocaine, and disopyramide bradycardia a nd severe s inoatriaJ block andl or
re nal tubules. are ventricular antiarrhythmic agents. sinus arrest.
155. b None of the other answer choices accurately
137. b Portal veins have tltis appearance on describes the endoscopic view of the right 174. e Sympathetic stimulation increases the si nus 187. b Wolff-Parkinson-White syndrome consists of
sonograms. bronchial tree. rate and increases tlle force of myocardial ventricular preexcitation, with episodes of
138. c Lym ph nodes are generally uniformly contraction. paroxysmal supraventricular tachycardia
156. a This instrument can be inserted through the associated with an accessory conduction
hypoechoic and produce less distal urethra of dogs and cats. 175. c Ventricular bigeminy occurs when the
enhancement than cystic structures.
w. pathway.
157. d The bladder must be kept undistended to rhythm alternates between a normal sinus
139: a Left ventricular failure resulting in complex and a ventricular premature complex. 188. b In tlle normal canine heart, the axis lies
allow the puncture sites to heal. betwee n +40 degrees and + 100 degrees, with
pulmonary edema would be difficult or 176. e The othe r possible choices are not primary
158. a The view caudally is obscured during right-axis deviation occurring when the axis is
impossible to diagnose us ing cardiac ventricular arrhythmias and are also
swallowing. .. greater than +100 degrees.
ultrasound alone. Diagnosis of pulmonary arrhythmias that usually do not lead to
edema would require thoracic radiographs. 159. c These openings are evident in the ventricular fibrillation. 189. a Atrioventricular dissociation implies that the
Echocardiographic findin gs of the othe r rostroventral portion ofche guttural pouches. atria and ventricles are discharged by two
177. a One or more P waves not followed byQRS.Y
ww
conditions are well described. 160. d The endoscope tip should be directed independent foci of impulse fonnation.
complexes can be classified into two types of
140. b On sonograms of unsedated dogs, the left dorsally and to the right to view this portion of 190. a Hyperkalemia is a very common cause of
second-degree atrioventricular block: Mobitz
ventricular shortening fraction averages the stomach. atrial standstill in animals with
type I and Mobitz type II. With Mobitz type I,
approximately 35%. 161. b Differences in appearance may indicate the RR interval becomes progressively shorter hypoadrenocorticism, kidney failure, or
pathologic changes. as the PR interval becomes longer, until a P diabetes mellitus.
wave is blocked.

C 1998 Mosby-Year Book, 111C. Photocopying is prohibited by law.

www.alkottob.com
100 SECTION 3 www.vet4arab.co.cc SECTION
191 . c All other choices are incomplete or 196. b Electromyography is an important adjunct to
ambiguous. Some of the other choices. such as
band e, are amy half-truths, because there are
other ways to evaluate consciousness and
neurologic examination. By ilSelf,
e lectromyography does not provide a definitive
diagnosis of a nerve or muscle disease. 4

m
epilepsy. 197. b Needle electromyographic e lectrodes cause
192. e All other answers refer to other sources of focal muscle fiber damage, which results in
release of CPK and elevation of serum CPK
artifact, but these problems are not always
present In fact, the problem of muscle spasms
a nd movement is so great that many veterinary
activities proportional to the number of needle
insertions.
Hematology and Cytology

.co
e lectroencephalographers insist that recordings 196. d Fibrillation potentials in canine skeletal
be made with the patie nt under anesthesia. muscles usually appear on day 5 after R.L. Cowell, W.I. Dodds, B.L. Hines
193. a Although the other answers refer to other denervation. A direct re lations h.ip exists
possible sources of EEG signals, these between size of the animal species and the time
influences are probably relatively minor. of appearance of fibrillation potentials.
194. e The EEG becomes activated (low voltage, fast 199. c Fibrillation potentials appear in the
activity) when an animal is in pain. However, it denervated motor units. Motor unit potentials
becomes Similarly activated when the animal is occur in the innervated motor units. Thus both
Recommended Reading

ob
stimulated imensely by stimuli that a re not types of potentials would be recorded from a
painful. Also, the EEG of dream sleep appears partially innervated muscle. Cowell RI.., Tyler RD: Cytology and hematology of tile horse, St Louis, 1992. Mosby.
similar. 200. d Demyelination of a periphe ral nerve results Cowell RI.., Tyler RD, Meinkoth JH: Diagnostic cytologyoftlle dog and cat, ed 2. St Louis. 1998,
195. e Conscious awareness is e nabled by the in conversion of conduction of impulses from a Mosby.
excitatory action of the brain s tem reticular saltatory (nodal) process to continuous Duncan JR et al: Veterinary labomtory medicine, ed 3. Ames, Iowa. 1994, Iowa State Uni\'ersity
form ation upon the neocortex. Lesions or conduction, which is significantly slower than Pre".
stimulation of the brain ste m produces saltatory conduction. fain NC: Essentials of veterinary Ilemat%gy, Baltimore, 1993. \Vllliams & Wllldns.
predicta ble changes in the EEG.

ott
Meyer 0 1 el al: Vererinary laboratory medici/Ie: interpretation and d iagllosis, Philadelphia. i 992,
we Saunders.
Willard MO et al: Small animal clinical diagnosis by laboratory methods. ed 2, Philadelphia. 1994,
NOTES WE Saunders.

Practice answer sheetli are on pages 261-262.

alk
Questions
R.L. Cowell and B.L. Hines
1. Which anticoagulant does not act by binding 3. Which clottingfactors are affected in animals
calcium ? wilh dicumarol toxicosis?
a . heparin a. I, II, V; Xl
w. b.
c.
sodium EDTA
sodium citrate
b. II, V, VII, IX
c. II, IX. X. XII
d. potassium EDTA d. II. VII. IX, X
e. potassium oxalate e. III, VII, IX, X
ww
2. Which type of red blood cell morphology is 4. lNhich factor in Ole clotting cascade has the
compatible with phenothiazine roxicity in horses? shortest half-life?
a. target cells a. I
b. Heinz bodies b. III
c. stomatocylosis c. V
d. basophilic s tippling d. VII
e. nucleated red blood ceUs e. IX
«) 1998 MoslJy-Year Book, In c. Photocopying is prohibited bylaw
Correct aTlSUJers are on pages 124-130. 101
www.alkottob.com
102 SECTION 4 www.vet4arab.co.cc Hematology and Cytology 103

5. A 5-year-old St. Bernard is presenced to your 9. A KurfoJfbody is an eosinophilic-staining 14. All exaggerated response to mild anemia, in the 18. A 6-momll-o/d puppy has had cOllghingand all
small-animal practice because of lethargy and inclusion seen normally witllin the lymphocytes of form of polychromasia, nucleated erythrocytes, OCu/ollasai discharge for the past week. TIle
anorexia 0[2 weeks' duration. Physical and basophilic stippling. is most often seen in puppy was vaccinated at 10 and 12 weeks a/age.
a. rats
examination reveals generalized animals wilh : A blood smear stained Wilh Diff-Quik reveals

om
lymphadenopathy. Microscopic examination ofa b. rabbits
a. iron toxicity homogeneous eosinophilic-staining inciusiom ill
fine-needle aspirate shows a monotonous c. reptiles red blood cells and nemrophils. The most likely
population (>90%) ofround cells with a high d. hamsters b. iron deficien cy
cause ofthis puppy's illness is:
nucleus/cytoplasm ratio and prominent nucleoli. e. guinea pigs c. lead poisoning
These cells are larger than tile rare neutrophils d. folate deficiency a. parvovirus infection
you find. Based on this description. what is the e. copper deficiency b. ehrlichiosis
most likely diagnosis? 10. Wllich of tile following is not classified as a c. borreliosis
round-cell tumor?
a. inflammation d. distempet
15. Larce '/limbers of spherocyres are characteristic of
a. histiocytoma e. pneumonia. \vith normaJ morphologic

b.c
b. osteosarcoma
b. mastocytoma a. anem ia of lead poisoning changes seen in the blood cells of young
c. lymphosarcoma
c. osteosarcoma b. Heinz-body anemia animals
d. benign hyperplasia
d. lymphosarcoma c. iron-deficiency anemia
e. immune stimulation
e. trans missible venereal tumor d. anemia of chronic disease 19. On an impression smear made from an ulcerated
e. immune-mediated hemolytic anemia area on tile forelimb ofa cat. you find multiple.
6. W1Jal is the predominant cell type seen in most small (2 to 4 ~m). basophilic-scainingorganisms
chylous effusions? 11. Which of the following best describes the most
whose shape varies from round to oval to fusiform
likely hematologic findings in a dog given 16. Tn dogs the normal activated clottjng rime is:
•. (cigar shaped). wirh a thin, clear halo. The most

tto
a. mast cell corticosteroids 12 to 24 hours previously?
less than I minute likely calise of the lesion in this cat is:
b. eosinophil
a. lymphoCYlOsis. neutropenia. monocytosis. b. less than 2 minutes a. leishmaniasis
c. neutrophil
d . lymphocyte
eosinopenia
b. lymphopenia, neutropenia, monocytopenia,
, c. less than 3 minutes b. toxoplasmosis
d. less than 4 minutes c. sporotrichosis
e. macrophage eosinopenia
e. less than 5 minutes d. histoplasmosis
c. lymphopenia, mature neutrophilia,
7. Wlljcll oftile following is typical offluid removed monocytosis. eosinophilia e. cytauxzoonosis

o
from tile peritoneal cavityofa cat with effusive d. lymphopenia. mature neutrophilia. 17. Concerning differentiation of warfarin toxicosis
feline infectious peritonitis? monocytosis, eosinopenia from disseminated illtravascularcoagulation
20. In a cat witll a packed cel/volume (PCV) of5%.
(DIC), wh ich statemetlt is most accurate?
a. clear. high-protein fluid containing primarily e. lymphocytosis. mature neutrophilia, an adequate regenerative response is suggested by

alk
lymphocytes monocytosis, eoSinopenia a. Activated c10tfing time is normaJ in both an aggregate reticulocyte cOlmt ofat least:
instances.
b. straw-colored, low-protein fluid containing a. J%
primarily neutrophils 12. In horses, which of the following is the best b. Platelet count is generally elevated in DIC and
b. 2%
indicator ofa bone marrow response to anemia? normal in warfarin toxicosis.
c. straw-colored, high-protein flu id containing c. 5%
primarilyeosinophils c. Platelet count is generaJly elevated in Die and
a. polychromasia decreased in warfarin toxicosis. d.10%
d. straw-colored, high-protein fluid containing b. increased reticulocyte count e. 20%
primarily neutrophils d. Concentrations of fibrin degradation products
c. improved mucous membrane color w. are typically increased in DIC and normal in
e. s traw-colored. high-protein fluid containing increased packed cell volume on serial
d. warfarin toxicosis. 21. Whicll organism commonly affects the central
primarily lymphocytes determinations , e. Concentrations of fibrin degradation products nervous system and might be found on cytologic
e. increased nucleated red blood cell count are typically normaJ in DIe and increased in evaluation ofcerebrospinal fluid?
8. Megakaryocytes are ti,e precursor cells to: warfarin toxicosis.
a. Sporothrix schenckii
a. platelets only 13. Von Wiflebrand's disease is most common in: b. CoccidiQides immitis
b. lymphocytes only
ww
a. boxers c. Histoplasma capslliatum
c. red blood cells only d . Blastomyces dermatitidis
b. Irish setters
d. segmented white blood cells only e. Cryptococcus neoformans
c. Labrador retrievers
e. all circulating blood cells except lymphocytes
d. Doberman pinschers
e. German short-haired pointers

Cl 1998 Mosby-Year Book, Inc. Photocopying is prohibited by law. Corm;t arlSwerSQn- 0" pages 124- 130.

www.alkottob.com
104 SECTION -4 www.vet4arab.co.cc Hematology and Cytology 105

22. A cow in respiratory distress has pale mucous 26. Red blood cell Willi no central pa/lorand much 37. The major role of VOIl Willebrand's factor in the 42. Leukocytosis is all increase in the total white
membranes. Evaluation ofa blood sample reveals smaller thall normal coagulation cascade is ill: blood cell count above the normal range. An
small, round, basophilic-staining inclusions on a. lysis of clots increased number of which cell type most
the margins a/many red blood cells. Based on this

om
27. Differentiated lymphocyte that produces b. formation of platelets commonly characterizes leukocytosis in dogs?
informatioll, what is the most likely diagnosis? immunoglobulin c. activation of factor X a. basophil
a. babesiosis d. activation of factor XII b. eosinophil
h. leptospirosis 28. Eosinophilic inc/usion normally found in e. platelet adherence to damaged tissue c. monocyte
c. molybdenum toxicosis lymphoid cells ofguinea pigs d. neutrophil
d. anaplasmosis e. lymphocyte
38. The normal Ufe span ofa canine red blood cell is:
e. bacillary hemoglobinuria 29. Fragment ofa red blood cell
a. 30 days
43. YOllfind rare, smail, roulld, pale blue- to gray_

b.c
b. 60 days
23. Avian species are hosts to several different blood 30. ImmwlOglobulin -fiffed vacuole ill tile cytoplasm staining. mulberry-like morulae in the cytoplasm
parasites, which can be idelllified 011 examination c. 90 days of neutrophils ofa 3-year-old English poimer
of plasma ceffs
afa blood smear. W1Iich of tile following is not a d. 120 days with polyarthritis. The most likely cause of this
blood parasite a/birds? e. JSOdays finding is:
For Questions 31 through 35, select the correct
a. Prototlleca answer from the five choices below. a. distemper
b. Plasmodium 39. One-slage prothrombin rime (OSPT) is used to b. babesiosis
a. metaplasia evaluate which aspect ofthecoaglllntioll cascade?
c. TIypanosoma c. ehrlichiosis
b. dysplaSia
d. Haemoproteus a. intrinsic pathway only d. leptospirosis

tto
c. crenation
e. Leukocytozoon b. extrinsic pathway only e. histoplasmosis
d. carcinoma
c. common pathway only
e. sarcoma
24. Whicilleukocyce predominates in the peripheral d. intrinsic and common pathways 44. Wllicll of the following is least likely to cause
blood ofhamsters? e. extrinsic and common path,vays macrocytic anemia?
31. Morphologic change of red blood cel~picaffy
a. basophil caused by dehydration and characterized by a. erythremic myelosis
40. TIle activated partial thromboplastin time

o
h. eosinophil multiple indentatiollS in the cell membrane b. folic acid deficiency
c. monocyte (APIT) is used to evaluate which aspect of the c. vitamin 8 12 defiCiency
coagulation cascade?
d. ne utrophil 32. Malignant neoplasm ofepithelial-cell origin d. iron defiCiency
e. lymphocyte a. il1(rinsic pathway only e. cobalt deficiency in ruminants

alk
b. extrinsic pathway only
33. Tissue challge caused by chronic irritation and
25. Which term describes the sex chromatin lobe seen characterized byf'eplacement ofolle mature cell c. common pathway only 45. Which type ofanemia is most likely to be
as a "drumstick" on the nucleus ofa neutrophil type willi allother mature cell type d. intrinsic and common pa£hways macrocytic hypochromic?
from afemaleanimal? e. extrinsic and common pathways
a. regenerative anemia
a. 8arrbody 34. Maligllant neoplasm ofmese1lchymal-cell origin b. anemia associated \vith iron deficiency
b. Heinz body 41. C0 11cern ing corticosteroid-i1lduced neutrophilia, c. anemia associated with copper deficiency
c. Russell body which statemelll is kastaccurate?
d.
e.
Curschmann body
Pappenheimer body
w.
35. Nonneoplastic tissue change associated with
variOlLs factors, such as inflammation, and
characterized by irregular, atypical, and
a. Release of mature neutrophils from bone
marrow is increased.
d.
e.
anemia associated with folic acid deficiency
anemia associated with bone marrow
suppression
proliferative changes in cell populations b. The peak response is seen t2 to 15 hours after
corticosteroid administration.
For Questions 26 through 30, select the correct 46. In a cat with leukocytosis induced by endogenous
answer from the five choices below. 36. Steatitis defines: c. The magnitude of neutrophilia diminishes epinephrine (physiologic leukocytosis), the
with long-term corticosteroid therapy.
ww
a. inflammation or the mammary glands leukocytosis is attributable to:
a. Kurloffbody d . Leukocyte values relUm to pretreatment
b. Russell body b. excessive lipid globules in feces, associated a. neulJ'ophilia only
levels within 24 hours after a single dose of
c. plasma cell with enteritis corticosteroids. b. lymphocytosis only
d. spherocyte c. inflammation of adipose tissue e. Neutrophils remain in circulation longer c. neUlJ'ophilia and monocytosis
e. schistocyte d. excessive conjugated bilirubin in the serum, because of decreased margination of cells. d. neutrophilia and lymphocytosis
associated with hepatitis e. monocytosis and lymphocytosis
e. inflammation of lymph vessels

C 1998 Mosby-Year Book. Inc. Phorocopying is prohibired by law. Correct answers are 0" pages 124·130.

www.alkottob.com
106 SECTION 4 www.vet4arab.co.cc Hema tology and Cyto logy 107

47. Which leukocyte predominates in the peripheral c. Schmauch body 58. Which type of hemorrhage is not typically seen in d. Kupffer cell
blood ofnormal dogs? d. Howell-Jolly body animals with platelet abnormalities or e. T lymphocyte
e. erythrocyte refractile body thrombocytopenia?
a. basophil

m
b. monocyte a. epistaxis For Questions 63 through 67, select the correct
c. neutrophil 53. Which of the following is an inherited disease b. hematuria answer from the five choices below.
causing a defect in leukocyresegme'ltatioll? c. ecchymosis
d. eosinophil a. basophilic stippling
e. lymphocyte a. Evans' syndrome d. petechiation b. Heinz body
e. hemarthrosis

.co
b. Pelger-Huet anomaly c. rouleaux
48. Wllieh leukocyte predominates in the peripheral c. mucopolysaccharidosis d. agglutination
blood ofa normal weanling pig? d. CMdiak-Higashi syndrome 59. Which of the followillg is IlOt a hereditary e. Howell-Jolly body
e. neutTophii granulation anomaly of Birman cats coagulopathy?
a. basophil
b. monocyte a. Pelger-Huet anomaly 63. Random clumping oferythrocytes
c. neutrophil 54. Which oflhe following causes microcytic b. factor X deficiency
hypochromic anemia? c. von WiUebrand's disease
d. eosinophil 64. Organized grouping of red blood cells, resembling
e. lymphocyte a. renal fail ure d. hemophilia A (factor VIII deficiency) stacked COiTlS

ob
b. iron deficiency e. hemophilia B (factor IX deficiency)
49. Which leukocyte predominates in tile peripheral c. estrogen toxicity 65. Mulear remnaflts within mature (nonnucleated)
blood ofa normal adult cow? d. folic acid deficiency 60. In which species has cytauxzoonosis not been red blood cells
e. inflammatory disease reported or described?
a. basophil
b. monocyte a. dogs 66. Clumps of denatured hemoglobin within red
55. Avian tllrombocytes differ from mammalian b. bobcats blood cells

ott
c. neutrophil
d. eosinophil tllrombocytes in that avian tllrombocytes: c. cheetahs
e. lymphocyte a. are phagocytic d. antelopes 67. Ribosomal clumping in red blood cells tliat stains
e. domestic cats basophilic witl! Wrights stain and may appear as
b. are nucleated cells
distinct granules or fine dots
50. Heinz bodies are most likely to be detected on a c. have no role in hemostasis
smear a/blood from healthy: d. can transport oxygen 61. Tn a blood smear from a markedly anemic cal,
youfind large nllmbers ofllucleated red blood 68. A mature red blood cell containing visible iron
e. playa major role in immune responses

alk
a. pigs cells in the absence of polychromasia alld particles (Pappenheimer bodies) is known as a:
b. cats reticulocytosis. The most li/cely cause of this
56. Concerning protamine sulfate, which statement is a. siderocyte
c. dogs finding is:
most accurate f b. Mott cell
d. cattle a. lead poisoning c. Kupffer cell
e. horses a. It is an irreversible anticoagulant. b. erythremic myelosis d. ferrocyte
b. It is a potentiator of sodium citrate in c. Pelger-Huet anomaly
anticoagulation. e. Downey cell
51. Wll ich type o[abnormaJ red blood cell indicates d. lymphoblastic leukemia
microangiopathic hemolysis? c. It is a cause of Heinz-body anemia in cats.
a. target cell
b. lep tocyte
w.
d. I{ is used in an early screening test for
disseminated intravascular coagulation.
e. Chediak-Higashi syndrome

62. A differentiated B lymphocyte develops into a:


69. Which of tile following is sometimes referred to as
plumbism?
e. II is a common cause of autoimmune a. iron deficiency
c. schistocyte hemolytic anemia in dogs. a. rubricyte b. copper toxicity
d. Slomatocyte
b. siderocyte c. lead poisoning
e. Mott cell
ww
57. Which cell type can divide by mirosis? c. plasma cell d. lymphosarcoma
a. myelocyte e. multiple myeloma
52. Which of the following is most difficult to
d ifferentiate from an Anaplasma organism on a b. reticulocyte
Romanowsky-slllined blood smear? c. metamyelocyte
d. band neutrophil
a. Heinz body
e. segmented neutrophil
b. Dohle body

C 1998 Mosby-Year Book, Inc. Photocopying is prohibited by law. Correcranswers are on pages 124-130.
www.alkottob.com
108 SECTION 4 www.vet4arab.co.cc Hematology and Cytology 109

70. Tile hematologic response associated with d. increased erythropoietin production 79. In a vaginal swab from a 2·year-old intact c. leukocytosis caused by neutrophilia and
hypothyroidism in dogs is: e. splenic contraction in stressed animals miniature schnauzer,youfind 96% superficial lymphocytosis
celis, with small pyknotic nuclei. Thisfinding d. leukocytos is caused by neutrophilia and
a. polycythemia

om
indicates that this bitch is most likely: monocytosis
b. erythroleukemia 75. What is the most common cause of polycythemia
c. normocytic normochrom ic anemia wim ;nanimals? a. in estrus e. leukopenia primarily caused by neutropenia
leptocytosis b. in diestrus
a. polycythemia vera
d. microcytic hypochromic anemia with c. in anestrus 84. A Coombs' test is used to help diagnose:
b. familial erythrocytosis
spherocytosis d. in proestrus
c. disorders resulting in hypoxemia a. immune-mediated thrombocytopenia
e. macrocytic hypochromic anemia with e. infected with a fungus
d. abnormal erythropoietin production b. feline infectious anemia
reticulocytosis
e. dehydration and splenic contraction c. equine infectious anemia
80. Which hematologic abnormality is associated

b.c
d. immune-mediated hemolytic anemia
71. Which pair of species can become infected with willI IOllg-term bracken-fern jngestion in cattle?
76. Which of the following does not stimulate e. microangiopathic hemolytic anemia
Ehrlichia? a. aplastic anem ia
erythropoiesis?
a. dogs and pigs b. myeloid leukemia 85. WI/icll abllormality is associated Will, VOII
a. estrogens
b. horses and pigs c. sevcre le ukopenia Willebrand's disease?
b. thyroxine
c. dogs and horses d. erythrOid le ukemia
c. androgens a. thrombocytosis
d. cattle and sheep e. iron-deficiency anemia
d. corticosteroids b. thrombocytopenia
e. horses and canle
e. growth hormone c. thrombocytopathy

tto
81. In a transtraclleal wash from a couglling dog. you d. deficiency of factor IX
72. From which cell rype does multiple myeloma find columnar cells, macrophages, a mixed e. deficiency of factor VI I
arise? 77. eonceming the effect ofcorticosteroids on population of bacilli alld cocci, and numerous
grallulopoiesisand lymphopoiesis. which .. Simonsiella organisms. The most likely cause of
a. astrocytes sMtemeru is most accurate? For Questions 86 through 90, select the correct
these findings is:
b. plasma cells a. 11ley have no effect on granulopoiesis or answer from the five choices below.
c. endothelial cells a. a hypersensitivity reaction
lymphopoiesis. a. major crossmatch

o
d. precursor cells of granulocytes b. oropharyngeal contamination
b. 11ley inhibit granulopoiesis and b. minor crossmatch
e. myelin.producing cells of the central nervous c. a mixed fungal and bacterial infection
lymphopoiesis. c. Russell's viper venom time (Rvvr)
system d. a mixed bacterial infection associated with
c. They stimulate granulopoiesis and d. intravascular hemolysis
immunosuppression

alk
lymphopoiesis.
e. a fungal infection associated with overuse of e. extravascular hemolysis
73. Which ofthefollowillg is not associated with d. They inhibit granulopoiesis but stimulate
antibiotics
Heinz -body formmio,,? lymphopoiesis. ,
e. They stimulate granulopoiesis but inhibit 86. Associated Willi babesiosis in cattle
a. onions
lymphopoiesis. 82. Which cell type may occasionally be observed in
b. red maple syrlOllial flllid and is diagnostic of systemic lupus
c. bracken fern 87. Test using recipient's plasma and donor's cells
erythematosus?
d. new methylene blue 7B. The proteins responsible for blood coagulation are
produced primarily in the: a. lupus erythematosus (LE) cells
e. phenothiazine dewormers

74. Polycythemia uera is associated with:


a.
b.
liver
spleen
w. b.
c.
Mott cells
giant cell
88. Associated with anaplasmosis in cattle

89. Used to differentiate factor VII deficiencies from


d. leplOcytcs common pathway coagillopathies
c. kidney
a. pulmonary hypoxia c. siderocytes
d. pancreas
b. reduced atmospheric oxygen
e. small intestine 90. Test using dOllor's plasma and recipient's cells
ww
c. a malignant stem·cell disorder 83. loW/icll hematologic finding is most likely to be
observed in cottle with severe, acute
inflammation?
a. leukocytosis caused by neutrophilia only
b. leukocytosis caused by lymphocytosis only

@1998Mosby-YearBook,Inc.Photocopyingisprohibitedbylaw. Correcr answers are on pages 124-130.


www.alkottob.com
110 SECTION 4
www.vet4arab.co.cc H ematology and Cytol ogy III

91. Theanemia seen jn chronic Tenalfailure is c. meat 102. A giallC cell isformed by /iJSion of 107. In wliich ofthefollowing is a Diilile body most
associated with: d. eggs a. heterophils likely to befoulld?
a. lack of e rythropoietin production e. legumes b. neutrophils a. eosinophil in a greyhound

om
b. hemolysis caused by electrolyte imbalances c. hepatocytes b. cytoplasm of a toxic neutrophil
c. splenic removal of abnormal red blood cells 97. Mwt protein rransports iron in tile plasma? d. lymphocytes c. cytoplasm of a thrombocyte in a bird
d. loss of red blood cells in the form o f hematuria a. transferrin e. macrophages d. nucle us of a liver cell in a puppy with
c. stress· induced estrogen release and resultant b. lipofuscin infectious canine hepatitis
bone marrow suppression c. hemosiderin 103. III w/tich type of leukocyte is ~tox;c change" most e. cytoplasm of a lymphocyte stimulated to
produce antibodies
d. chromaffin commonly seen?
92. In dogs, w/lere is erythropoietin produced? e. transcobalamin a. basophils
108. In a transtracheal wash from a horse,youfind

.c
a. liver h. eosinophils ciliated and nOllciliated columnar and cuboidal
b. kidney 98. A Serto{j-cell OImor is most likely to occur: c. monocytes cells. alveolar macropl/ages, an occasional
c. spleen d. neutrophils neutrophil. and approximately 20% of cells as
a. in the mouth of a cat
d. thyroid e. lymphocytes eosinop/ti/s. You observe no superficial
b. around the eye of a horse
e. bone marrow squamolJS epithelial cells or Simonsiella

ob
c. on the distal limb of a 6-month-old dog organisms. Which of the following is tile most
d. in an ovary of an 8- to 12-year-old female dog 104. In which dog breed do eosinophilsfrequently appropriate cytologic interpretation?
93. Which organisms are referred to as marginal e. in a retained (abdominal) testicle of a 7-year- have more vacuoles t/Jan granules?
bodies? a. This is a normal ceUpopulation.
old dog a. akita
b. There is evidence of malignant neoplaSia.
a. Babesia b. boxer
c. There is evidence of a hypersensitivity
b. Anaplasma 99. Wlljc/I disorder is least likely to produce c. Samoyed reaction .

ott
c. Eperythrozoon peripheral neutropenia? d. greyhound d. There is evidence of bacterial or fu ngal
d. Hem obarlollclla a. hyperadrenocorticism e. A1askan malamute inflammation.
e. Ehrlichia h. parvoviral infection e. There is evidence of oropharyngeal
c. rupture of the cecum 105. Mlich of fhe following best describes contamination.
94. Which of the [oI/owing does not increase d. overwhelming bacterial infection anisocytosis oferythrocytes?
nwnbers ofcirculating platelets in animals with e. Salmonella- induced endotoxemia in a horse 109. Concemillg the procedure ofpercutaneous
a. fo lded in half
immune-mediated thrombocytopenia? trallStracheallbronchial wash, which statement

alk
h. variable sizes
a. vincristine is most accurate?
100. A general term Ilsed to describe abnormal and c. variable shapes
h. splenectomy bizarre shapes in erythrocytes is: d. organized like a stack of coins a. It cannot be done without general
c. tetracyclines e. s malle r than normal, with no cen tral pallor anesthesia.
a. leptocytosis
d. corticosteroids h. It is preferred in frac tious animals.
b. anisocytosis
e. cyclophosphamide c. It requires use of a sterilized endotracheal
c. poikilocytosis 106. Concemillg histiocytomas. whicll statement is
most accurate?
tube.
d. reticulocytosis
95. Which blood type is most common in cats in the d. It reduces the risk of oropharyngeal
United Stales?
a. F
e. stomatocytosis
w. a. They are relatively more common in beagles.
h. They contain distinct spindle-shaped ceUs.
contamination.
e. It involves passage of a needle and catheter
/01. A heterophil is most likely to befollnd in a: c. They tend to occur in youn g dogs. through the thyroid cartilage.
b. Tr d. They can metastasize to the lungs.
a. pig
c. B e. Azurophilic granuJation can obscure the
h. cat
d.AB nucleus ofhlstiocytoma cells.
ww
c. dog
e. A
d. bird
e. Uama
96. Which of the followin g is a poor source of iron?
a. dirt
b. milk

e 1998 Mosby-Year Book. Inc. PhotOCOpying is prollibiled by law. Correct answers are on pages 124-130.
www.alkottob.com
112 SECTION 4
www.vet4arab.co.cc Hema tology and Cytology 113

110. Which a/the following best characterizes a septic c. monocytic leukemia 117. Which term is used to identify the blue-black c. myeloblast, promyelocyte, myelocyte,
exudate? d. p lasma-cell myeloma iron-containing particulate matter seen in bone metamyelocyte, band, segmented neutrophil
e. reticuloendotheliosis marrow macrophages? d. myelocyte, myeloblast. metamyelocyte,

om
a. low protein conlent (<3.0 g/dl), predominantly
lymphocytes present, no bacteria seen or a. bilirubin promyelocyte, band, segmented neutrophil
cultured 114. Which of tile following best describes an b. melanin e. myelocyte. metamyelocyte. promyelocyte.
h. low protein content (<J.O g/d]), predominandy osteoblast? c. hematoidin myeloblast, band, segmented neutrophil
nondegenerate ncuU'ophils present, no a. arises from the granulocytic cell line d. hemoglobin
bacteria seen or cultured e. hemosiderin 122. In which species are the granules ofeosinophi/S
b. a giant cell containing six to ten randomly
c. high protein content (>3.0 g/dl), predominantly almost exclusively rod shaped?
arranged nucle i
degenerate neutrophils present, intracellular
c. a cell that produces calcilOnin within the 118. W1lich disorder is most likely to cause overall a. dogs
and lor extracellular bacteria present
parathyroid gland lIyperceffularityofhematopoietic tissue in bone b. cats

.c
d. low protein content (<3.0 g/dl), predominantly
nondegenerate neutrophils present, d. the precursor cell to oth er skeletal system marrow? c. cattle
intracellular and/or extracellular bacteria cells, such as chondrocytes and fibrocytes d . sbeep
a. myelofibrosis
present e. an ovoid, p lump cell resembling a large b. erythroleukemia e. h orses
e. high protein content (>3.0 gtdl), predominantly p lasma cell. with a round, eccentric nucleus
c. aplastic anemia

ob
foamy macrophages and nondegenerate and dark b lu e cytoplasm
d. chronic ehrlichiosis 123. Concerning immature canine red blood cells.
neutrophils present, intracellular andlor wllich statement is least accurate?
e. chronic renal failure
extracellula r bacteria present 115. Concemingcytologic evaluation of bone ...
specimens. which statement is wast accurate? a. Polychromatic cells are generally larger than
119. Wllich oftllefollowing is most likely to cause mature red blood cells.
111. Which cell type comprises the liningo/tlle a. Samples are relatively difficult to obtain.
overall hypercellularity of hematopoietic tissue b. All reticulocytes are polychromatophilic with
pleural, peritoneal. and uisceral surfaces? b. Coccidioidomycosis is a common cause of in bone marrow? Romanowsky stains.

ott
a. Kupffer cell osteomyeBtis. c. Polychromatophilic cells contain less
a. irradia tion
b. epithelial cell c. Thmors of bone are difficult to differentiate hemoglobin than mature red blood cells.
cytologically. b. myelofibros is
c. endothelial cell d. Mature erythrocytes stain poorly with
d . The most common finding in osteomyelitis is c. suppression by feline leukemia virus (FeLV) supravital stains, such as new methylene
d. mesothelial cell
a n eutrophilic exudate. d. bone marrow necrosis blue.
e. reticuloendothelial cell
e. The most frequently identified bone tumor is e. anemia of chronic inflammatory disease e. Reticulocyte counts are perfonned on
osteosarcoma. closely followed by smears stained with a supravital stain, such

alk
112. Conceming normal cerebrospinal fluid in dogs. fibrosarcoma. .. as new methylene b lue.
120. What is the correct sequence ofmaturation ;'1 the rOO
whicll statemem is most accurate?
blood cell line,from least mature to most nUllure?
a. It should contain fewer than 8 nucleated 116. Concemingcytologic evaluation of synovial fluid J24. Col1ceming cytologic elltlluation of lymph nodes,
a. reticulocyte, rubriblast, metarubricyte,
cells/~L ofdogs, which statemefll is least accurate? which statement is least accurate?
prorubricyte. rubricyte. erythrocyte
b. Nucleated cells should consist primarily of
a. The presence of lupus erythematosus (LE) b. rubriblast, metarubricyte, prorubricyte, a. Ruptured cells are a common artifact
nondegene rate neutrophils.
cells in synovial flu id is diagnostic of lup u s rubricyte, reticulocyte, erythrocyte becau se of th e fragility of lymphoid tissue.
c. Cell counts should be performed 2 to 24 erythematosus.
hours after collection of Ule sample. c. rubriblast, prorubricyte, rub ricyte, b. There is no true cytologic difference between
d. Normal cerebrospinal flu id is clear and pink,
with a total protein concentration of 50 to
«3000 ce ll s/ ~l).
w.
b. Normal synovial fluid is of low cellularity

c. Osteoclasts are not unusual in synovial fluid


metarubricyte, reticulocyte, erythrocyte
d. rubriblast. rubricyte, metarubricyte,
prorubricyte, reticulocyte, erythrocyte
n ormal and hyperplastic lymph nodes.
c. Small lymphocytes typically comprise at
least 75% of a normal lymph node cell
100 mg/ dL
because these cells are present in articular e. prorubricyte. metarubricyte, rubricyte, population.
e. Normal cerebrospinal fluid is clear and cartilage.
yellow, with a total protein concentration of rubriblast, reticulocyte, erythrocyte d. Lymphadenitis can be reliably diagnosed
d. Organisms are often difficult to identify in when neulrophils make up greater than 10%
50 to 100 m g/dL
inflammatory arthropathies of bacterial
ww
o f the nucleated cell population_
121. Mlat is tile correct sequence of maturation in
origin . e. Aspiration of foamy epithelial cells in the
113. Which neoplasm is most likely toS/IOW the myeloid cell line, from least mature to most
e. An increased nucleated cell count with more mature? area of the mandibular lymph node u sually
radiograph ic evidence ofbone lysis? than 90% mononuclear cells su ggests indicates neoplasia.
degenerative joint disease. a. promyelocyte. myeloblast, metamyelocyte.
a. lymphosarcoma
myelocyte, band, segmen ted neutrophil
b. erythremic m yelosis
b. myeloblast, promyelocyte, metamyelocyte,
m yelocyte, band, segmented neutrophil

@ 1998 Mosby-Year Book, Inc. Photocopying is prohibited by law. OJrrect answers are on pages 124-/30.
www.alkottob.com
114 SECT10N 4 www.vet4arab.co.cc Hematology and Cytology 115

Por Questions 125 through 129, select the correct c. pyriformation 136. Concerning cytologic examination of nasal masses c. They tend to exfoliate only a few individual
answers from the five choices below. d. castellation and exudates, which statement is least accurate? cells.
a. Charcot-Leyden crystals e. coronification a. Most nasal tumors are of mesenchymal origin. d. They show very few characteristics of

m
h. Curschmann spirals b. Rhinosporidium infection in dogs causes a malignancy.
c. tingible bodies 133. Which of the followingis not a cytologic criterion polypoid growth. e. They exfoliate small ce[ls with a scant
o/malignancy for epithelial-cell tumors? c. Transmissible venereal tumor can occur in amount of cytoplasm.
d. hematogones
e. lymphoglandular bodies the nasal areas, particularly in males.
a. angular nucleoli
d. Ciliated pseudostratified columnar epithelial 141. Each hemoglobin molewle is composed of
b. uniform nuclear size

.co
cells originate from the nasal turbinates.
125. Free nuclei shed from maturing red blood cells c. large, prominent nucleoli a. 1\o.ro globin chains and one heme group
e. Cryptococcus neofomums infections produce a
(metarubricytes) d. high nucleus/cytoplasm ratio b. 1\o.ro globin chains and two heme groups
mucoid exudate and are most common in cats.
e. large numbers of abnormal mitotic figures c. 1\o.ro globin chains and four heme groups
126. Round, basophilic-staining structures, similar in d. Four globin chains and one heme group
137. Periallal-gland cells can be/ound if! all o/the
size 10 platelets, produced by cytoplasmic 134. Concerning cytologic examination ofthe external e. Four globin chains and two heme groups
following areas except:
fragmentation in lymphocytes ear canal, wliich statement is least accurate?
a. the tail 142. Which biochemical pathway uses glucose to
a. Cerumen, an oily, yellow secretion, stains

ob
127. Elongated double pyramidal structures/armed b. the ears generateATP in mature canineerylhrocytes?
poorly or not at all.
by coalescence ofeasif/opllil granules c. the thighs
b. A sweet-smelling, pale yellow exudate ismost a. Embden-Meyerhofpathway
d. the prepuce
likely attributed to Pseudomonas infection. b. aminolevulinic acid pathway
128. Prominent dark blue (basophilic) nuclear debris e. around the anus
c. Malassezia is a broad-based, budding yeast c. Luebering-Rapoport pathway
within macrophages that is the most common cause of mycotiC d. hexose monophosphate pathway
otitis externa, 138. COllcerning liposarcomas, which statement is
e. methemoglobin reductase pathway

ott
129. Mucous casts formed in small bronchioles d. The ear mite that produces a dry, black, most accurate?
granular discharge associated with parasitic a. They are benign tumors of adipose tissue.
otitis externa in companion animals is 143. In dogs, what is the last erythrocyte stage in
130. The swollen, eosinophilic, lacelike nuclei from b. They are almost always secondarily infected. which the nucleus is still evident?
Otobius megnirli.
ruptured cells are known as: C. Only very small lipid globules are observed in
e. Some of the most corvmon causes of otitis a. rubricyte
cells aspi rated from liposarcomas.
a. ragocytes externa (Maiassezia and bacteria) are often b. erythrocyte
normal inhabitants of the external ear canal. d. They arise most commonly in subcutaneous
b. schistocytes c. prorubricyte
tissue of the shoulders, thighs, and trunk.

alk
c. Fleischmann's celis e. After alcohol fixation, cytologic samples can d. reticulocyte
d. caprocytes 135. Concerning cytologic examination o/the eye, be stained with special stains, such as Sudan e. metarubricyte
e. basket cells which statement is least accurate? rv, to verify the presence of lipid.
a. Keratoconjunctivitis sicca produces a 144. Which of the following best describes lipemic
131. Neutrophi/S containing small, dark lymphocytic exudate. 139. Concerning cytologic examination of squnmous- plasma from a flormal, flon/asted dog?
intracytoplasmic granules, observed on b. The anterior uvea is a site for metastasis of cell carcinomas, which statement is least accurate?
unstained wet preparations 0/ synovial fluid, are systemic carcinomas. a. clear and yellow
known as: w. a. They are often secondarily infected. b. opaque and white
C. Neutrophilic infiltration of aqueous humor is
characteristic of most cases of anterior b. The [Umor masses tend to yield groups of cells. c. clear and colorless
a. ragocytes
uveitis. C. Perinuclear vacuolation is a common d. opaque and yellow
b. schistocytes cytologic finding.
d. In cats, corneal scrapings containing e. clear and pink to red
c. Fleischmann's cells d. They are fmUld only in poorly pigmented areas
primarily eosinophils indicate eosinophilic
d. synoviform cells keratitis. of the skin exposed to ultraviolet radiation.
145. Which species lias the smallest erythrocyte?
e. procartiblasts
ww
e. Most intraocular tumors, either primary or e. They are tumors of epithelial origin.
secondary, do not exfoliate into aqueous a. cats
132. The linear distriburion of cells seen on pull humor; the exception is lymphosarcoma. b. pigs
140. Concerning tumors o/epithelial origin, which
smears o/fluids o/high viscosity is termed: statemeTil is most accurate? c. dogs
d. goats
a. rouleaux a. They include tumors of endocrine glands.
b. windrowing e. horses
b. When malignant, they are called sarcomas.

© 1998 Mosby-Year Book, Inc. Photocopying is prohibited by law. Correct answers art! on pages 124-130.
www.alkottob.com
1I6 SECTION 4
www.vet4arab.co.cc Hematology and Cytology 117

146. Which species has tile largesl erythrocyte? 150. After a single acute anemic crisis (hemorrhage or 154. Which aftllefollowing is ,wta mechanism by 159. Macropllages are derived from wliiclileukocyte?
hemolysis) in a dog, how many days are which intravascular 1/e1lJolysis may occur?
a. cats a. basophil
generally required for peripheral blood
a. osmotic lysis b. mo nocyte

om
b. pigs reticu/ocytes to become most numerous?
c. dogs b. physical injury c. eosinophil
a. I c. oxidative injury
d. horses d. n e utrophil
b.2 d. complement-mediated destruction
e. sheep e. lymphocyte
c. 3 e. increased macrophage activity
d. 7
147. What term describes tile random clumping 160. Wllich of the following is not afunction of
e. 14
(grapelike aggregation) oferythrocytes tliat 155. Wlilch of tile following best describes the eosinopllils?
occurs in some cases ofantibody-mediated mechanism ofanemia associated witll
a. activation of plasminogen
151. In a 3-year-old dog that was normal 2 months myeloplltlt isic disease?

b.c
anemia?
previously, you detect a heart murmur. The dog b. synthesis of complement components
a. rouJeaux a. lack of erythropoietin c. phagocytic and bactericidal capabilities
also has tachycardia, taC#lypnea, and pale
b. coalescence mucous membranes, with a packed cell voilime b. iron diversion to storage pools d. complement ·dependent parasiticidal
c. poikilocytosis af 10%. W1lDt is the most likely cause afthe heart c. infection with such agents as panleukopenia properties
d. agglutinatio n murmur? virus e. inhibition of chemical mediators released
e. polychromasia d. dietary deficiencies, such as vitamin B l2 and from mast cells in anaphylactic reactions
a. acquired valvular insufficiency
folic acid
b. decreased Oleygen flow to the heart
e. physical replacement of bone marrow by
148. Concerning reticulocytes in the peripheral
, 161. Which cells are the only source ofendogenous

tto
c. a congenital defect not detected earlier abnormal proliferation of cells
circulation a/horses, which statemeTll is most d. occult heartworm disease heparin?
accurate?
e. decreased blood viscOSity, causing a. basophils only
156. All tile following coaglilation factors are
a. Reticulocytes are absent in both health and turbule nce b. mas t cells only
produced in tile liver except:
regenerative anemia. c. eosinophils only
b. RCliculocyrcs are a bscnI in health but a. faClor X
152. Conceming the differences between Ilemalytic d. basophils and mast cells
present in regenerative anemia. anemia alld hemorri!Ogicanemia, which h. factor IX
e. eosinophils and mast cells

o
c. Rcticulocytcs are present in health but morc statement is most accurate? c. factor VII
numerous in regenerative anemia. d. fibrinogen
a. The reticuJocyte response is slower in 162. In reference to the leukogram, which of the
d. Reticulocytes are a prominent feature in e. von Willebrand factor
hemolytic anemia. followillg bestdefilles a left shift?

alk
healthy foals, less numerous in adult horses,
and more numerous in regenerative anemia. b. The plasma prOlein concentration is
a. toxic change in n e utrophils
decreased in hemolytic anemia. 157. About how much time does a mature nelltropliil
e. RCliculocytes are found in two forms: spelld circlilating witllin the peripheral blood? b. inc reased segmentation of all leukocytes
punctale reticulocytes are present in health, c. The plasma protein concentration is normal
to increased in hemolytic anemia. c. increased numbers of mature neutrophils
and aggregate reticulocytes are present in a. 5 hours d. increased numbers of immature platelets
regenerative anemia. d. The plasma protein concentration is normal b. 10 ho urs
to increased in hemorrhagic anemia. e. increased numbers of immature neutrophils
c. 15 hours
e. Hyperbilirubinemia is often present with
149. Which species has two types of reticulocytes d. 24 hours
anemia caused by external hemorrhage.
(aggregate and punctate)?
a. pigs
w.
153. Erythrocytesfrom which species are most likely
e. 72 hours
163. W1licll oflllefollow/ngdoes not cause peripheral
blood lymphopenia?
a. viral infections
b. cats to contaill erythrocyte refractile bodies in health? 158. The circlliating pool of segmented nelitTophils is
approximately one tllird tile size of the b. lohne's disease
c. dogs
a. pigs marginated pool ill: c. epinephrine res ponse
d. cattle
b. cats d. ruptured thoracic duct
ww
e. goats a. pigs
c. dogs e. immunosuppressive therapy
b. cats
d. horses
c. dogs
e. llamas
d. cattle
e. horses

o 1998 Mosby- Year B(}()/c, Inc. Pll otocopying is prollibited by law. Correct answers are on pages 124-130.
www.alkottob.com
118 SECTION 4 www.vet4arab.co.cc Hematology and Cytology 119

164. A neutrophil must contain how many distinct 169. Which ofthefollowing is associated with an 174. Which of tile following cannot be counted !vith a 179. Hematologic examination ofa bouine blood
nllclear lobes to be described as exudate? hemocytometer? sample reveals a packed cell volume of 10%,10
hypersegmented? a. hypoproteinemia nucleated RBCsIlOO WHCs, marked basophilic
a. platelets

m
a. one or more b. right heart failure stippling, and marked polychromasia. What is
b. le ukocytes
b. two or more the most likely cause of these findin gs?
c. lymphatic obstruction c. eryth rocytes
c. three or more d. mycobacterial pneumonia d. reticulocytes a. lead poisoni ng
d. four or more e. congestive heart failure with ascites e. erythrocytes and leukocytes b. micro angiopathy
e. five or more c. regenerative anemia

o
170. Wllich of tile following is tile most defining d. iron-deficiency anemia
175. Which tenn describes the average volume ofa
165. What;s the average life span ofa circulating characteristic ofan exudate? population of red blood cells, expressed in e. nonregenerative anemia
platelet?

b.c
a. ceUcount femtoliters?
a. 1 day b. protein concentration a. hematocril 180. Conceming the coagulation cascade in birds.
b. 3 days which statement is most accurate?
c. eryth rophagocytosis b. packed cell volume
c. 5 days d. cholesterol/ triglyceride ratio >1 c. mean corpuscular volume a. Birds do not have thrombocytes.
d. 10 days e. cholesterol/triglyceride ratio <1 d. mean corpuscular hemoglobin b. Birds lack the extrinsic pathway.
e. 14 days e. mean corpuscular hemoglobin c. Birds lack a complete intrinsic pathway.
171. W1,icll of the following is nota typicalfinding in concentration d. Birds do not have a common pathway.
166. Omcerning thrombin. which statement is least a long-standing or resolving hemorrhagic e. In addition to the intrinsic, extrinsic. and

o
accurate? effusion? 176. Concerning/ibrinogen, which statement is most common pathways, birds have another
accurate? dotting pathway.
a. It accelerates fibrinolysis. a. thrombocytosis
b. It accelerates coagulation. b. erythrophagocytosis a. Plasma fibrinogen levels increase \vith age.

ott
181. Conceming hematopoiesis in auian bone
c. It results in fibrin formaLion. c. hypersegmented neutrophils b. It is produced in the spleen. marrow, which statemellf is most accurate?
d. It is a potent platelet antagonist. d. pink or xanthochromic supernatant c. It precipitates at 50° C.
e. It is inunediately derived from factor II. e. macrophages containing black pigment d. Plasma fibrinogen levels decrease with a. Rubriblasts are not evident.
(hemosiderin) systemic inflam mation . b. Myeloblasts are not evident.
e. Plasma fibrinogen levels cannOl be accurately c. Megakaryocytes are not evident.
167. Whicli ofthe following is an in lJivo test of
platelet Junction? 172. In horses, neonatal isoerythrolysis is most determined on heparinized samples. d. Lymphopoiesis occurs primarily in marrow,

alk
commonly seen:
not in peripheral lymphoid tissue.
a. platelet time e. The huge reserve stores of iron occupy
a. after parturition in primiparous dams 177. An iniJerited, semilethal condition resulting in
b. bleeding time approxim ately 90% of the medullary cavity.
cyclic neutropenia (cyclic hematopoiesis) occurs in:
c. thromboplastin time b. in first-born offspring after ingestion of
d. activated clotting time colostrum a. bull terriers
182. WIJ iclt of the following is a hematopoietic
e. Russell's viper venom time c. during the last half of gestation if the dam is b. Gordon setters neoplasm of lymphoid tissue in chichms?
sensitized to the foal's erythrocytes c. Newfoundlands
d. in the dam after breeding if the dam was
w. d. silver-gray collies a. Marek's disease
168. Which althe/allowing best describes a previously exposed to the stud's erythrocytes b. Jahne's disease
neutrophil that is swollen and exhibiting e. Rhodesian ridgebacks
e. in neonatal foals after ingestion of colostrum c. l'yzzer's disease
karyolysis, loss of nuclear membrane. and
if the dam has been sensitized to fetal d. pullorum diSease
homogeneous pink-staining chromatin? 178. Eosinopllils from which species have a
erythrocytes e. Newcastle d isease
a. band neutrophil characteristic "raspberry" appearance?
b. toxic neutrophil 173. Levels of which plasma protein of large animals a. p igs
183. Which of the following is a hematologic disorder
ww
c. reactive neutrophil are used to detect internal inflammatory lesions? b. cats
seen in jntactfemaleferrers wilh repeated
d. neoplastic neutrophil c. cattle anovulatory cycles?
a. kallikrein
e. degenerate neutrophil d. deer
b. duombin a. aplastic anemia
e. horses
c. serotonin b. erythroleukemia
d. fi brinogen c. lymphOid leukemia
e. plasminogen d. granulocytic leukemia
e. immune- mediated hemolytic anemia

C 1998 Mosby-Year Book, Inc. Photocopying is prohibited by law. Correct anslwrs are on pages 124-130.
www.alkottob.com
120 SECTION 4 www.vet4arab.co.cc Hematology ",nd Cytology 121

184. During the/eta' period ill cats, which structure 189. In a centrifuged microllematocrit tube 194. The formatioll alld development of blood cells d. factor VII
does 1I0( directlycontribllle /0 blood/ormation? containing whole blood. the area containing are termed: e. factorVlII
platelets and leukocytes is called the:
a. liver a. hematuresis

m
b. spleen a. band layer b. hemopexis 198. A prolonged one-stage prothrombin timealld a
c. kidney b. white band c. he malOchromatosis normal StyPlJe1l time (Russell's viper vellom
d . yol k sac c. buffy coat d. hema phe resis time) in a bleeding dog indicate a deficiency of'
e. bone marrow d . white line e. hematopoiesis a. factor V
e. myelochrome band •

o
b. fa ctor X
185. \.VlIicll a/the/allowing is not afunction althe 195. Itl avian species. what hematopoietic neoplasm c. fa ctor II
spleen? 190. In carnivores tile icterus index is a measure of is induced by a DNA Ilerpesvirus? d. factorVlI

b.c
a. reservoir of platelets a. the amount of bilirubin in serum a. leukosis-sarcoma complex e. fa ctorVIII
b. reservoir of leukocytes b. unconjugated bilirubin in the liver b. Pacheco's disease
c. reservoir of erythrocytes c. discoloration of the mucosa c. Marek's disease 199. Which of tile following does not prolong the
d. reservoir of lymphoid cells with immune d. the amount of bilirubin in urine d. Fanconi synd rome thrombin time?
function s e. functional hepatic parenchymal mass e. malignant erythroleukemoid syndrome a. heparin therapy
e. destruction of aged or defective erythrocytes b. dysfibrinogenemia
191. A structure released in clusters from tile 196. A prolonged activated partial thromboplastin c. factor II deficiency
186. Which term describes the complex series of cytoplasm of megakaryocytes is the: time and a normal one-stage prothrombin time d. severe hypofibrinogenemia

o
physical and biochemical events that both iTl a bleeding dog indicate a deficiency of: e. high concentration of fibrin degradation
a. micro karyocyte
promote and inhibit blood clocting? products
b. cytoplasmocyte a. factor V
a. hemoprysis b. factor X

ott
c. thrombocyte
b. hemos tasis d. astrocyte c. fa ctor II 200. In reference to the leukogram, which of the
c. hemos iderosis d. faclOr VII fof/owing best describes a right shift?
c. Russell body
d. he matochezia e. factor VIII a. increased numbers of immature neutrophils
e. hematopoiesis b. increased numbers of hypersegmented
192. A reduction of blood volume below normal levels
is known as: 197. A normal activated partial thromboplastin time neurrophils
187. Which tenn describes an intravasculardeposil and a prolonged one-stage prothrombin rime in c. toxic changes in neuuophils

alk
a. polycythemia
comprised a/fibrin and fomled elements a/blood? a bleeding dog jndicate a deficiellCY of' d. increased numbers ofJymphocytes with
b. relative erythrocytosis basophilic cytoplasm
a. embolus c. compensatory leukocytosis a. factor V
e. increased cytoplasmic vacuolation of
b. thrombin d. hypovolemia b. factor X
monocytes
c. thrombus e. shock c. factor II
d. infarct
e. plaque
193. In aviatl species, what hematopoietic neoplasm
w.
is induced by all RNA retrovirus? W.I. Dodds
188. What;s the larges! aftlle bone marrow
a. leukosis-sarcoma comp lex 201. Viral jTlfectioli is classically associated with: c. small platelets
lIematopoieticcefls?
b. erythremic myelosis d. autoagglutination
a. rubriblast a. lymphocytosis
c. Marek's disease e. platelet clumping
b. myeloblast b. lymphopenia
d. Fanconi syndrome
c. lymphoblast c. eosinophilia
e. malignant erythroleukemoid syndrome
ww
d. neutrophilia 203. Autoagglurinated red blood cells usually
d. metarubricyte
e. target ceUs indicate:
e. megakaryocyte
a. hypoproteinemia
202. The imnllme-mediatedfoml of b. hyperosmolarity
thrombocytopenia is characterized by: c. collection artifact
a. leukocytosis d. erythrocyte fragiJjty
b. giant platelets e. immune-mediated hemolytic disease

C 1998 Mosby--Year Book. Inc. Photocopying is prohibited by /o.w. Correct answers are on pages 124-130.
www.alkottob.com
122 SECTION 4 www.vet4arab.co.cc Hematology MId Cyt ology 123

204. Disseminated illlravascularcoagulation c. chromatid body 215. 71le resflllS of which hemostatic test are usually 22 J. The characteristic type of bleeding in animals
prtXiuces red blood cell fragmentation These d. polychromatophilic cell normal i" allimals with von Willebrand's disease? with von Willebrand'sdisease involves the:
fragments afred blood cefls are called: e. monocyte a. bleeding time

m
a. skin and subcutaneous tissue
a. schislocytes b. platelet retention (adhesion) b. muscles
b. microcytes 210. 71le test for erythrocyte antiglobulin is called the: c. prothrombin time c. mucosal surfaces
c. target cells a. Schmidt's test d. botrocetin cofa ctor activity d. joints
d. leptocytes b. Schirmer test e. von Willebrand factor antigen assay e. medullary cavity of long bones

o
e. Howell-Jolly bodies
c. Cooley's anemia test
d. Howell-Jolly test 216. Hemophilia A is characterized by: 222. Anticoagulant rodellticide poisollingcauses
205. A regenerative response to allemia is indico.ted by: e. Coombs' test bleedillg by:
n. low factorVlJl activity

b.c
a. microcytosis and hypochromia b. lowvonWillebrand factor activity a. inhibiting platelet function and fibrin
b. normochromia and nucleated red blood cells 211. Which blood cells llave the longest life span? c. low factor IX activity deposition
c. hyperchromia and stomalocytes d. prolonged primary bleeding time b. inhibiting platelet function and fibrinolys is
a. neutrophils
d. normocytosis and nucleated red blood cells e. normal fac torVlII activity c. preventing formation of fibrin in the liver
b. erythrocytes
e. macrocytosis and polychromasia d. blocking synthesis of vitamin K-dependent
c. thrombocytes
clotting factors
d. megakaryocytes 2 17. Hemophilia B is characterized by:
206. Red blood cells tilat are polychromatophilic e. inhibiting prothrombin activation of blood
e. monocytes a. lowfactorVIII activity vessels
when stained with Wrights stain arecaffed what

o
when srained with new methylene blue? b. low von Willebrand factor activity
212. Toxicanemia caused by oxidant drugs or toxins c. prolonged primary bleeding time 223. 11Je most common inheritance pattern ill VOrl
a. reticulocytes is characterized by deposits ofdenatured d. normal factor lX activity Wil/ebrand's disease is:

ott
b. poikilocytes hemoglobin called:
e. low factor IX activity a. sex-linked recessive
c. leptocytes
a. Howell-Jolly bodies
d. target cells b. a utosomal recessive
b. Heinz bodies 218. Fibrin clot formation is the result of'
e. acanthocytes c. a utosomal incompletely dominant
c. LE bodies
a. thrombin interaction with fibrinogen d. sex-linked dominant
d. refractile bodies
207. Coagulation profiles ofanimals with liver b. plasmin interaction with fibrinogen e. sex-linked codominant
e. Ehrlichia bodies
disease typically show: c. thrombin interaction with plasminogen

alk
a. low von WLllebrand factor activity d. plasmin interaction with plasminogen 224. The bolle marrow precursor cell of the blood
213. Platelet adhesion, an importam illitial ellellt in
e. prothrombin interaction with thrombin platelet is the:
b. reduced fibrinolysis the control of bleedillg, is primarily mediated by:
c. prolonged prothrombin time a. megakaryocyte
a. fibrin
d. thrombocytosis 219. Vitamin K-dependem clotting/actors are b. macrokaryoblast
b. platelet factor 3 sylllhesized in the:
e. hypercoagulability c. mononuclear giant cell
c. von Willebrand factor
a. spleen d. Reed-Sternberg cell
d. thromboxanes
e. pluripotent stem cell
208. What are typical clinical sigm 0/
thrombocytopenia?
a. hematoma formation and joim pain
e. subendothelial elastin
w.
214. The inherifQnce pattern of hemophilia is:
b.
c.
d.
liver
pancreas
kidney 225. Wllat is the hereditary anemia ofbasenji dogs
e. thymus called?
b. epistaxis and exercise imolerance
a. sex-linked recessive a. glucose-6-phosphate dehydrogenase
c. petechiae and ecchymoses
b. autosomal recessive 220. TIle classic sigll ofbleeding ill animals with deficiency
d. melena
ww
c. sex-linked dominant hemophilia is: b. phosphofructokinase deficiency
e. hematuria
d. autosomal incompletely dominam c. porphyria
a. hematoma formation
e. sex-linked codominam d. pyruvate kinase deficiency
209. A leukocyte with a nucleus in tile shape ofa b. epistaxis
spoked wheel is called a: c. petechiae and ecchymoses e. methemoglobinemia

a. plasma cell d. melena


b. leukemoid cell • e. hematuria

C 1998 Mosby-Year Book. Inc. Photocopying is prohibited by law. Correct allswers are 011 pages 124·130.
www.alkottob.com
124 SECTION 4
www.vet4arab.co.cc Hematology and Cytology 125

Answers 37. e Platelets are formed in the bone marrow by


megalcaryocytes.
58. e Hemarthrosis and bleeding into body cavities
are not typical of thrombocytopenia or
I. a Heparin serves as an anticoagulant by Activated clotting time is prolonged in both 38. d Canine RBCs persist for up to 4 months. thrombopathy.

om
accelerating the activity of antithrombin III. The instances. Platelet counts are decreased in 39. e The OSPT evaluates the extrinsic and common 59. a Pelger-Huet anomaly is an inherited d isorder
other anticoagulants listed chelate or bind disseminated intravascular coagulation and pathways. associated with abnormal neutrophil
calcium. normal in warfarin toxicosis. 40. d The APTr evaluates the intrinsic and common segmentation.
2. b Heinz bodies are clumps of denatured 18. d Respiratory disease is a common presenting pathways. 60. a Cytauxzoonosis has not been reported in dogs.
hemoglobin caused by oxidative injury, as with sign of distemper. Characteristic inclusion 41. b The peak response is seen 4 to 8 hours after 61. b Lead poisoning occurs in cats but has been
phenothiazine toxicity and onion toxicosis. bodies are sometimes seen in erythrocytes, corticosteroid administration. reponed only rarely and causes no anemia or
3. d FaclOrs II, VII, IX, and X are vitamin K neutrophils, lymphocytes, andlor monocytes. 42. d leukocytosis in dogs primarily involves only mild anemia. In add ition, polychromasia
dependent. Vitamin K is inhibited by dicumarol. 19. c Many Spororllrix organisms are found in increased numbers ofneutrophils. would be expected. Erythremic myelosis is

b.c
4. d FactorVII in the extrinsic pathway has the infected cats, but the organisms are difficult to 43. c Pale blue inclus ions in neutrophils are seen reported in cats, and blood smears typically
shortest half-life. Therefore the extrinsic pathway find in infected dogs and horses. Fusiform (cigar) with some Ehrlichia ewingii infections. show large numbers of nucleated RBCs.
clotting time is prolonged before the intrinsic shapes are characteristic of Sporothrix schenckii. Parvoviral infection does not cause inclusion 62. c A plasma cell is a differentiated B lymphocyte. If
pathway is affected. 20. d The aggregate reticulocyte count should be at bodies in circulating cells. Distemper inclusions it contains vacuoles filled with immunoglobulin
5. c St. Bemards are predisposed to lymphosarcoma. least 10%. 1b estimate the minimum regenerative are homoge neous, basophilic- or eosinophilic- (Russell bodies), it is known as a Mort cell.
Finding over SO% lymphoblasts in a cytologic response in cats, the rule of thumb is as follows: staining structures in neutrophils and 63. d Agglutination describes random clumping of
preparation of an enlarged lymph node is PCV Aggngate n?ticulocytes erythrocytes or mononuclear cells. RBCs.
considered diagnostic of lymphosarcoma. 20% 2% 44. d Iron defiCiency causes microcytic 64. c in French the term means rolls.
hypochromic anemia.

tto
6. d Neutrophils can predominate. but this is much 10% 5% 65. e Howell -Jolly bodies are nuclear remnants
less common. 5% 10% 45. a Macrocytic hypochromic anemia is within mature RBCs.
7. d ~Wet" feline infectious peritonitis typically 2 1. e Of the organisms listed, o nly Cryptococcus is characterized by large, pale-staining cells, 66. b Heinz bodies are clumps of denatured
produces a straw-colored effusion containing likely to be found in cerebrospinal fluid. suggesting polychromasia and reticulocytosis. hemoglobin in RBCs.
primarily neutrophils. 22. d Anaplasma marginale causes a hemolytic 46. d in cats, epinephrine release increases 67. a Basophilic stippling ofRBes is evident with
8. a Other precursor cells within the bone marrow crisis in cattle. The organism is transmitted by numbers of circulating neutrophils and Wright's stain.
give rise to other circulating cells. the one-host tick Boophilus annulatus. lymphocytes. 68. a Siderocytes contain iron particles.

o
9. e Kurloffbodies are normal findings in the 23. a Prorotheca is an alga that can cause 47. c Mature neutrophils predominate in the blood 69. c Plumbism is from the Latin term for lead,
lymphocytes of guinea pigs. disseminated disease in dogs but usually only of normal dogs. plumbum.
10. c Osteosarcoma is a spindJe-ceU tumor of causes cutaneous lesions in cats. 48. e Lymphocytes predominate in the blood of 70. c Hypothyroidism typically causes normocytic
mesenchymal origin. All the others listed are 24. e Lymphocytes are the predominant leukocyte weanling pigs.

alk
normochromic anemia with leptocytosis. Answer
round-cell tumors. in hamsters. 49. e Lymphocytes predominate in the blood of e describes anemia that is regenerative (caused
II . d Corticosteroid administration produces 25. a This sex chromatin lobe on the nucleus is normal adult cattle. by hemorrhage or hemolysis). Answer d
lymphopenia, mature neutrophilia, called a Barr body. 50. b Heinz bodies are most common in feline describes anemia caused by iron deficiency
monocytosis, and eosinopenia. 26. d Spherocytes are much smaller than normal blood. (without spherocytosis).
12. d Horses do not release immature red cells from RBCs. 51. c Schistocytes are RBC fragments produced 7 1. c EhrUchia infection has also been reported in
the bone marrow. Therefore polychromasia and 27. c Plasma cells produce immunoglobulins. when membranes are cleaved by fibrin strands in cats and people.
reticulocytes are nO{ seen, nor are nucleated red 28. a Kurloffbodies are found in the lymphocytes of capillary beds. 72. b Multiple myeloma is a tumor of plasma cells,
blood cells seen. Mucous membrane color is a
poor method of determining the response to
anemia.
13. d This disorder is most common in Dobermans.
guinea pigs.
29. e Schistocytes are RBC fragments.
w.
30. b Russell bodies are found in plasma cells.
52. d Howell -Jolly bodies are nuclear remnants
within red blood cells.
53. b The Pelger-Huet anomaly involves defective
WBC segmentation.
which generally produce excessive amounts of a
s ingle type of immunoglobulin. Another
characteristic of this tumor in dogs is bone lysis.
73. c All the other toxicants tend to cause formation
31. c Crenation ofRBCs is seen with dehydration.
14. c Lead poisoning p roduces these hematologic 54. b Iron deficiency causes microcytic of Heinz bodies.
findings. 32. d Carcinomas are of epithelial-cell origin.
hypochromic anemia. 74. c Polycythemias are classified as absolute and
33. a Metaplasia involves a change from one type of
ww
15. e Spherocytosis is characteristic of immune- 55. b Avian thrombocytes are nucleated cells. relative. Absolute polycythemias are further
mediated hemolytic anemia. mature cell to another. divided into primary and secondary.
34. e Sarcomas are of mesenchymal-cell origin. 56. d The protamine sulfate test is used to detect
16. b Activated clotting time in normal dogs is fibrin monomers and early fibrin degradation Polycythemia vera is the only cause of primary
under 2 minutes. 35. b Dysplasia is characterized by proliferation of products. absolute polycythemia.
17. d Concentrations of fibrin degradation products atypical cell populations. 75. e Dehydration and splenic contraction are forms
57. a Myelocytes have approximately two mitoses.
increase in disseminated intravascular 36. c None of the other answers correctly defines The other cell types do not undergo mitotic of relative polycythemia and resolve once stress
coagulation and are normal in warfarin toxicosis. steatitis. is relieved and dehydration is corrected.
• divisions .

C 1998 Mosby-Year Book, 1nc. Photocopying is prohibited by law.


www.alkottob.com
126 SECTION 4
www.vet4arab.co.cc Hematology and Cytology 127

76. a All the orner hormones s timulate 97. a Transferrin is a plasma transport protein. 114. e Osteoblasts are ovoid, plump cells with a 128. c Tingible bodies are nuclear debris in
erythropoiesis. Hemosiderin is a storage form of iron. rounded eccentric nucleus and dark blue macrophages.
77. e Corticosteroids inhibit lymphopoiesis. Transcobalamin is a transport protein fo r cytoplasm. They resemble large plasma cells. 129. b Curschmann spirals are mucous casts in

om
78. a The liver produces coagulation factors. vitamin BIZ' Osteoclasts are large multinucleated cells that small bronchioles.
98. e Sertoli·cell tumors are most commonly arise from the macrophage/monocyte cell line.
79. a These find ings are characteristic of~heat." 130. e Basket cells are not cells but rather are the
associated with feminization in males. There is not a s ingle precursor cell to all other nuclei ofruptllfed WBCs.
80. a Bracken-fern poisoning causes aplastic anemia. skeletal system cells.
99. a Increased release of corticosteroids 131. a Ragocytes may be seen in synovial fluid.
SI. b Oropharyngeal contamination is suggested by 115. e The most frequently identified bone tumor is
associated with hyperadrenocorticism causes 132. b This linear distribution is characteristic of
the presence of superficial epithelial cells and osteosarcoma, closely followed by fibrosarcoma.
peripheral neutrophilia. synovial fluid of high viscosity.
SimOllsiella organisms. $imOllSiella is a bacillus
100. c Poikilocytosis describes variously shaped Although osteosarcoma is the most common
found in colonies formed by organizing in a tumor, chondrosarcoma (not fibrosarcoma) is 133. b Uniform nuclear size is not a criterion.
characteristic pattern. RBCs.
the second most common. ~ic~lly, .malignant neoplasia shows a large

b.c
82. a Motl cells are vacuolated plasma cells. Giant 101. d Heterophils can also be seen in reptiles and variatIOn III nuclear size. An exception is
some laboratory animals. 116. c Finding an osteoclast is not unusual because
cells are seen with chronic granulomatous these cells are present in articular cartilage. lymphosarcoma. Lymphosarcoma is a round-
disease. Siderocytes arc fed blood cells 102. e Giant cells may also be formed by fusion of cen tumor, not an epithelial-cell tumor.
Osteoclasts are present in subchondral bone.
comaining iron panicles. LeplOcytes are thin, monocytes or epithelioid cells. 134. d The ear mite that produces a dry, black,
Their presence in synovial fluid suggests
folded fed blood cells. 103. d Other leukocytes rarely show toxic changes. erosion of articu lar cartilage to subchondral granula r discharge is Otodectes cynotis. Otobius
83. e Cattle have a limited reserve of neutrophils in The most common toxic changes are bone. megnini is the spinose ear tick of food animals.
thc i,r bone marrow and often develop leukopenia cytoplasmic vacuolization, cytoplasmic 135. a Keratoconjunctivitis sicca produces a
basophilia, toxic granulation, and Dohle bodies. 117. e. Hematoidin is a non-iron-containing, golden
dunng severe, acute inflammation. pigment often referred to as tissue bilirubin. neutrophilic exudate, usually containing
104. d The eosinophils of greyhounds tend to be

tto
84. d . N.eonalal isoerythrolysis is diagnosed by bacteria.
118. b Erythroleukemia is the most likely cause. All
clulical behavior and serologic tests. Equine more highly vacuolated. 136. a Most nasal tumors are of epithelial origin.
the other disorders listed cause hypocellular
infectious anemia is confirmed using the 105. b Spherocytes are smaller than normal cells samples. 137. b Pe rianal gland cells can be found in the skin
Coggins test. Feline infectious anemia can be and contain no central pallor. Poilcilocytes are ofthe areas listed. They can also be found in the
confirmed by examination of the blood smear, as 119. e Anemia of chronic inflammatory disease is
of variable shape. Rouleaux refer to stacking of dorsum, but this does not specifically include
can microangiopathies. the most likely cause. Myelofibrosis is the
red blood cells. Stomatocytes are folded red the cars.
displacement of normal marrow e lements by
85. c Thrombocytopathy or abnormal platelet blood cells. 138. d Liposarcomas are malignant, may co ntain
fibrou s tissue. FeLV-induced suppression can

o
function is associated with deficiencies of von 106. c Histiocytomas are more common in young result in a normocellular or hypoceUular large to s mall lipid globuJes, are rarely
Willebrand factor. dogs. sample. secondarily infected, and can be stained with
86. d Babesiosis is characterized by intravascular 107. b Dohle bodies are most likely to be observed 120. c None of the other answers correctly cites the special stains only if alcohol or another lipid
hemolysis. in the cytoplasm of to:oc neutrophils. sequence of erythroid cell maturation. solvent has not been lIsed for fixation.

alk
87. a In a major cross match, the recipient's plasma 108. c If more than 10% of the cells are eosinophils 121. c None of the other answers correctly cites the 139. d Tumors are o nly found in poorly pigmented
is tested with the donor's blood cells. and no criteria of malignancy or oropharyngeal sequence of myeloid cell maturation. areas of the skin exposed 10 ultraviolet
88. e Anaplasmosis is characterized by extravascular contamination exist (superficial squamous radiation. Squamous-cell carcinoma may be
122. b Dogs have pleomorphic granules, but they
hemolysis. cells) . then an allergic or parasitic infection is found in any area of the skin in dogs and cats.
are generally round. Round granules arc seen in
89. c The RVVf is prolonged with deficiency of most likely. 140. a Epithelialtllmors are called carcinomas when
horses, cattle, and sheep. Horses have
fa ctor X orV. 109. d It reduces the risk of oropharyngeal prominent granules that fill the cytoplasm. malignant, and they exfoliate groups of cells
90. b In a minor crossmatch the donor's plasma is COlllamination. The landmark used for this because of formation of cellular "bridges." The
123. b All reticulocytes are polychromatophilic with
tested with the recipient's blood cells. technique is the c ricothyroid ligament. Because
w. cells exfoliated arc large, witlliarge amounts of
Romanowsky stains. Romanowsky s tains
general anesthesia is usually not needed, this cytoplasm and many criteria of malignancy.
91. a Lack of erythropoietin because of renal include Wright's stain and its derivatives.
would not be a good choice for a fractious or 141. d Each hemoglobin molecule comprises a
parenchymal loss is the mechanism underlying 124. e Aspiration of foamy epithelial cells in the area
aggressive animal. heme group and four globin chains.
the anemia in chronic renal failure. of the mandibular lymph node usually indicates
110. c Septic exudates have a high protein content 142. a The Embdcn-Meyerhof pathway uses
92. b In people and animals other than dogs, the neoplasia. The submandibular salivary gland is
and contain many bacteria and degenerate glucose. The hexose monophosphate pathway
liver produces approximately 10% of often aspirated in this area, revealing a uniform
neutfophils. maintains glutathione in a reduced state to
erythropoietin. population of foamy cells in a pink background
ww
111. d These body cavities a re lined with when stained with Wright's stain. neutralize oxidants. The methemoglobin
93. b Allaplasma organisms tend to be located on
mesothelial cells. reductase pathway keeps hemoglobin in a
the periphery of infected RBCs. 125. d Hematogones are nuclei shed from maturing
112. a Normal cerebrospinal fluid is also clear a nd RBCs. reduced state to transport oxygen. The
94. c Tetracyclines are effective against Ehrlichia-
colorless and has a total protein concentration Lucbering-Rapoport pathway allows formation
induced thrombocytope nia. 126. e Lymphoglandular bodies are fragments of
less than 50 mg/dl. of2,3-diphosphoglycerme, which regulates
95. e Most cats in America have type-A blood. lymphocyte cytoplasm. oxygen transport.
113. d Plasmacytomas commonJy cause osteolysis.
96. b Milk has a low iron content. 127. a Charcot·Leyden crystals are formed by 143. e Nuclei are evident in metarubricytes.
coalescence of eosinophil granules.

0 1998 Mosby-Year Book, Inc. Photocopying is prollibited bJ' law.


www.alkottob.com
128 SECTION 4
www.vet4arab.co.cc Hematology and Cytology 129

144. b Clear and colorless plasma is seen normally 160. b Eosinophils have phagocytic properties 178. e The eosinophils of horses tend (0 resemble (hemophilia B), or factor Xl deficiency
in most dogs and cats. Horses and cattle have similar to those of neutrophils but are not plump ras pbe rries. (hemophilia C). A deficiency of factor X11 does
slightly yellow plasma. Icteric samples are protective against bacte rial infection. 179. c Basophilic stippling is common i.1l canlc with not cause a bleeding problem.

om
yellow, whereas hcmoly7..ed samples are pink. to J6l. d He parin is produced by basophils and mast a strong regenerative response to anemia. Lead 197. d Factor VII is the only soluble coagulation
red. cells. poisoning typically produces only slight anemia factor in the extrinsic system.
145. d The average caprine erythrocyte is 4 fJ..m in 162. e Numbers of im mature neutrophils are or no anemia. 198. d Russell 's viper venom activates factor X and is
diameter. increased in a left shift. 180. e Birds have an incomplete intrinsic pathway. used to evaluate the common pathway.
146. e The average canine erythrocyte is 7 fJ..m> in 163. c All the other disorders cause lymphopenia. 181. e If iron occupied 90% of the medullary cavity, 199. e Thrombin time tests fibrinogen level and
diameter. 164. e Hypersegmented neutrophils have five or flight might be difficult. fun ction. Factor II (prothrombin) is not
147. d Rouleaux are groupings of erythrocytes like a more nuclear lobes. 182. a Marek's disease is caused by a DNA herpesvirus involved in thrombin time.
stack of coins. Agglutination pe rsists in a saline 165. d Platelets circulate for apprOximately 10 days. and results in lymphoid proliferation in 200. b Increased numbers of hypersegme ntcd
dilution test, whereas rouleaux do not.

b.c
166. d Thrombin is a potent platelet agonist. It peripheral nerves and central nervous system neutrophils indicate a right shift.
148. a Unlike other species, horses do not show accelerates fibrinol ysis because it activates inflammation. lohne's disease is an infiltrative 201. b Viral infections typically cause lymphopenia.
reticulocytes. plasminogen . It accelerates coagulation disease of the bovine intestine. Newcastle disease 202. c Platelet size is reduced with immune-
149. b Aggregate reticulocytes are similar to those because it activates fa ctors II , IX, and X. is avian pneumoencephalitis. "Jyzzer's disease is mediated thrombocytopenia.
seen in other species and generally increase in seen in many animals and is caused by Bacillus
167. b The other tests are in vitro. 203. e Autoagglutination is characteristic of
number during response to more severe pilifannis. Pullomm disease is Salmonella
168. e Toxic neutrophils exhibil CYlOplasmic pulfarum infection in birds. immune- mediated hemolytic disease.
anemias. Punctate reticulocytes occur during
vacuolization, toxic granulation, and Dahle 204. a Fragments of ROCs are termed sch istocytes.
health, and their numbers remain increased 183. a PerSistently high estrogen levels produce
bodies. Band neutrophils appear normal, 205. e Increased polychromasia indicates
even after numbers of aggregate reticuJocytes aplastic anemia.

tto
except for a broad nucleus without regeneration. Numbers of nucleated RBCs often
return to normal foll owi ng a response. 184. c The kidney is not directly involved in
segmentation. increase with regenerative anemia; however,
150. d Reticulocytosis may nOt be evident until 2 or erythropoiesis in fetal kittens.
169. d Mycobacterial pne umonia is characterized their numbers also increase \vith many
3 days after an anemic crisis. 185. b Although lymphocytes are stored here, that is
by an exudate. The other disorders listed would nonregenerative anemias and therefore do not
151. e Decreased blood viscosity, causing produce a transudate or modified transudate. primarily related to the immunologic fun ction indicate regene ration unless polychromasia or
turbule nce, is the most likely cause. Acquired of the spleen . It is not a leukocyte storage area reticulocytos is is e vident.
170. a Exudates and transudates may have similar
problems could cause this murmur; however, pe r se.
protein concentration. The cholesterol! 206. a Reticlilocyies are young polychromatic RBCs.
given this description, acqui red problems and 186. b Hemostasis is a dynamic series of events that

o
triglyceride ratio is not used to d ifferentiate 207. e Animals with liver disease may have a
congenital defects would be less likely. control clotting.
exudates from transudates. prolonged prothrombin time.
152 . c Plasma protein levels are normal to increased 187. c Thrombin is a coagulation protein. An infarct
171. a Platelets are generally absent. 208. c Petechiae and ecchymoses are common in
in hemolytic anemia. They are decreased in is an area of tissue necrosis, often caused by a
172. e Antibodies in the colostrum cause

alk
hemorrhagic anemia. Reticulocyte response is thrombocytopenia.
thrombus or embolus. An embolus is a freely
more rapid with hemolysis because of the more destruction of the foal's RBCs. 209. a Plas ma cells have a nucleus resembling a
circulating thrombus. A plaque is a flat area or
readily available iron stores. Bilirubin levels 173. d Fibrinogen levels tend to increase with patch. spoked wheel.
cannot increase via hemorrhage if the blood is inflammatory disorders. 210. e The Coombs' test is used 10 detect RBC
18B. e Megakaryocytes are comparatively large.
lost outside the body. 174. d Reticulocytes are immature erythrocytes that antiglobulin.
189. e The microhematocrit tube can be snapped
153. b Heinz bodies, Schmauch bodies, and must be stained with supravital stains, such as 211. b Of all blood cells, RBCs have the longest life
off at the buffy coat and a smear made forWnC
erythrocyte refractile bodies are all the same new methylene blue, and cou nted using light span (up to 120 days).
examination.
thing (clumps of denalUred hemoglobin). microscopy. There are many methods for 212. b Heinz bodies are typically observed in the
calculating reticulocyte pe rcentages.
w. 190. a A high icte rus index may indicate hemolysis
154. e Extravascular hemolysis occurs outside the or dehydration. RBCs of animals with toKic anemia.
vascular system. Increased macrophage activity 175. e Mean corpuscuJar hemoglobin concentration 213. e Platelet adhesion is mediated by von
is a ratio of the weight of he moglobin to the 19l. e Thrombocytes or platelets are derived from
results in extravascular hemolysis. Willebrand factor.
volume of a given population of red blood cells. megakaryocytes.
155. e In myelophthisic disease, hematopoietic cells 214. a Hemophilia is inherited as a sex-linked
Mean corpuscular hemoglobin is the weight of 192. d Hypovolemia can be caused by hemorrhage
in the bone marrow are replaced by abnormal recessive trait.
he moglobin in an average popuJation. Packed or dehydration.
cells, such as fibrous or neoplastic tissue. 215. c Prothrombin time is usually normal in
cell volume and hematocrit are different terms 193. a An RNA retrovirus causes avian leukosis.
ww
156. e Von Wtllebrand factor is produced by animals with von Wtllebrand's disease.
for the pe rcentage of red blood cells in whole 194. e Hematopoiesis describes formation and
endothelial cells. 216. a Animals with he mophilia A have low factor
Glood. maturation of blood cells.
157. b Neutrophils move from tile peripheral blood VIII activity.
176. e Fibrinogen is produced in the liver, and 195. c Marek's d i sea~e is caused by a DNA
into tissue. They do not return to the blood. 217. e Animals with hemophilia B have low factor IX
plasma levels are not affected by age but herpesvirus.
158. b About two thirds of intravascular neulrophils increase during inflammatory processes. activity.
in cats are marginated. 196. e The defect is in the intrinsic system. Possible
Fibrinogen precipitates at 56° 10 58° C. problems include factor VIII deficiency 21B. a Interaction of thrombin with fibrinogen
159. b Macrophages are de rived from monocytes. 177. d Cyclic neutropenia occurs in some gray collies. produces clot formation .
(hemophilia A), factor IX deficiency

e 1998 Mosby- Year Book, Inc. Pllotocopying is prollibited by law.


www.alkottob.com
130 SECTION 4
www.vet4arab.co.cc SECTION

219. b These clotting factors are synthesized in the 223. c Von Willebrand's disease is inherited as an
liver.
220. a Animals with hemophilia easily develop
autosomal incompletely dominant trait.
224. a Thrombocytes are derived from 5

om
hematomas. megakaryocytes.
221. c Animals with von Willebrand's disease show 225. d The anemia is macrocytic hypochromic and
bleeding at mucosal surfaces. may develop before 6 months of age.
222. d After repeated doses, lack of these factors
leads to bleeding.
• Immunology
l.J. Gershwin
NOTES

b.c
Recommended Reading
Cae Clough NE, Roth fA: Understanding immunology, 51 Louis, 1998, Mosby.
Gershwin LJ et a1: Immunology and immunopatllOfogy ofdomestic animals, ed 2, St Louis, 1995,

tto
Mosby.
Lewis RM, Picut CA: Veterinary clinical immunology. Bahimore. 1989. Williams &WIlkins.
T12.ard I: Veterinary immunology: an introduction. ed 5, PhUadelphia, 1996, WB Saunders.

Practice answer sheet is on page 263.

o
Questions
1. MilCh immunoglobulin (lg) is most effective on 3. Wlllch immwlOglobulin (/g) binds to mast cells

alk
mucosal surfaces? and sellSitizes for antigen-induced degranulation?
a.lgG a. IgG
b. lgM b. lgM
c. IgD c.lgD
d. IgE d.lgE
e. IgA e. IgA
w. 2. Mlich immunoglobulin (Ig) is present almost
exclusively in tile intravascular compartmeru?
a. IgG
4. WlIich immunoglobulin (lg) has its major role as
a B lymphocyte differentiation marker?
a.lgG
• b. lgM b.lgM
c. JgD c. IgD
ww
d.lgE d.lgE
e. IgA e. IgA

C 1998 Mosby-Year Book, Inc. Photocopying is prollibited by law. Correcranswers are on pages 139-141. 131
www.alkottob.com
132 SECTION 5
www.vet4arab.co.cc Immull o logy 133

5. Which immullogiobulin (lgJ predominates in d. postcapillary venules 16. Which Tcell cytoldne is considered all autocrine c. in ute ro lysis by the mare's natural killer (NK)
serum dllringa secondaryatltibody response? e. subcapsular vessels growth factor? cells
a. IgG d. des truction in the immediate neonatal period

om
a. interleukin 1
b. lgM 11. Which process occurs ill the thymlts? b. interleukin 2 by antibody and complement
c. 'gD c. interleukin 3 c. consumption of very low levels of maternal
a. B cell maturation
d. IgE immunoglobulins by me neonatal foal
b. antibody production d. interleukin 4
e. IgA c. T cell maturation e. interleukin 5
22. A Holstein calfwitlt persistelllIJelltrophilia and
d. stem-cell he matopoiesis
lack of pus formation at sites of bacterial infection
6. In allergic dermatitis, which cell is the sourceo! e. thrombopoiesis 17. Mlich cytokine is most importam in the immune
histamine? is most IikelyaJJ1icred with:
respoflSe to facultative intracellular bacteria?
a. severe combined immunodeficiency

b.c
a. eosinophil 12. 111e Coggills test for equine infectious anemia and a. interlc ukin J
b. mast cell b. primary agammaglobulin emia
the single radial immunodiffusion method of b. interleukin 2
c. lymphocyte immunoglobulin quantitation have in common
c. Chediak-Higashi syndrome
c. interleukin 4 d. bovine leukocyte adhesion deficiency
d. neutrophil the requirefllellt that the antigen must be:
d. illle rieukin 10 e. cycl ic ne utropenia
e. dendri tic cell a. insoluble c. interle ukin 12
b. solu ble
23. Blood from all Arabian foal with severe combilled
7. What is the most likely calise ofa high titer of c. membrane bound 18. Macrophages are activated by prodllction of immunodeficiellCY is most likely to show:
antinuclearamibodies ill a dog with k idney d. intracellular

tto
failure? e. purified a. interleukin 2 a. normal lymphocyte n umbers. no IgM, and
n. Goodpasture's disease b. inte rleukin 4 variable IgG
b. amyloidosis c. inte rleukin6 b. few to no lymphocytes. nonnal lgM, and
13. Alltigen can be presellted to TcelIs by all of the no IgG
c. leptospirosis d. interferon-a
following except:
e. intcrferon-y c. few to no lymphocytes. no IgM, and
d. systemic lupus erythematosus a. 8 lymphocytes variable IgG
e. nephrotoxic reaction

o
b. T lymphocytes d. normal B lymphocyte numbers, low T
19. Which technique is used to detect monoclonal
c. macrophages lymphocyte numbers, and normallgG
gammopatlly?
8. B /ympllocytes are predominarufy found in which d. dendritic cells and IgM
region of lymph nodes? e. Langerhans' cells a. agglutination e. low 8 lymphocytc numbcrs, normal T

alk
a. cortex b. precipitation lymphocyte numbers, and low IgM
b. medulla c. immunofluorescence
14. Which cytokilles are produced by the eD4' Tcells
c. paracortex
d. immunoelcctrophoresis 24. Bence-fOlies proteins in urine comprise:
called type-l helper T cells?
d. white pulp e. complement fixation
a. interleuldn 4, interleukin 2. tumor necrosis a. immunoglobulin heavy chains
c. red pulp factor-o: #fo b. immunoglobulin light chains
20. Polyclonal gammoparhy is usualfycaused by: c. immunoglobulin J chains
b. interleukin 4, interferon '1, interleukin 2
9. T lympllOcyres are predominan tly [olllld i n which w.
c. interleukin 2. tumor necrosis faclor-o: , a. a malignant plasma-cell tumor d. secrelOry components
region of lymph lIodes? interle ukin 5 b. a benign plasma-cell tumor c. Fub immunoglobulin fragments
a. cortex d. interleukin 4, interleukin 5, interleukin 10 c. hemangiosarcoma
b. medulla e. interleukin 12, interferon-'Y. interleukin 2 d. anal-sac carcinoma 25. Hyperviscosity syndrome is most likely to occur
c. paracortex e. persistent antigeniC stimulation witll a myeloma producing which
d. white pulp 15. Which cytokines are produced by the CD4+ T cells immunoglobulin (IgJ?
ww
e. red pulp called type-2 helper T cells? 21. [n neonatal isoerythrolysis, destructioll of the a. IgG
a. interlcukin 4, ime rleukin 12, interfcron-y foal's erythrocyres is a result of b.lgM
10. Lymphocytes exit Iympllnodes through: b. interleukin 10, interleukin 12, imerleukin 2 a. destruction in utero by antibody and c. IgA
c. interleukin 4, imcrleukin 5, interleukin 10 comple ment d.lgO
a. efferent lymphatics
d. imerlcukin 12, interferon-y, interleukin 2 b. in utero sensitization of the foal 10 the mare's e. IgE
b. afferent lymphatics erythrocytes
c. sheathed arteries e. interleukin 2, tumor necrosis factor-a,
interleuldn 5

C 1998 Mosby-Year Book, Inc. Photocopying is prohibited by law. Correct answers arc 011 pnges 139· 141.
www.alkottob.com
134 SECTION 5
www.vet4arab.co.cc Immullology 135

26. Physiologic effects a/histaminc illcJudeall olllle 31. All ofthefollowi1,g are it, vitro correlates ofcell- 36. In dogs the direct Coombs' rest is performed by 40. 1n autoimmllne myasthenia gravis,
following except: mediated immllnityexcepl: combining rhe: autoantibodies attack:
a. smooth muscle contraction a. delayed hypersensitivity skin test

om
a. patient's washed erythrocytes with guinea pig a. e pinephrine receptors
b. increased capillary permeability b. mixed le ukocyte culture complement b. acetylcholine receptors
c. vasodilation c. lymphocyte stimulation with b. patient's washed erythrocytes with rabbit c. complement receptors
d. increased mucous secretion phytohemagglutinin anticanine IgG
d. Fc receptors
e. chemotaxis of ncutrophils d. NK cell assay c. patient's serum with normal erythrocytes and
e. histamine receptors
e. cytotoxic T cell assay complement
d. patient's serum with normal erythrocytes and
27. Smooth mllscleconCTaction lImc occllrs 12 hours 41. In autoimmune thyroiditis o/dogs, tisslledamage
anticani ne IgG
after exposure ofa seflSitized horse to allergen 32. Qmlacl demlatitis Oil the muzzleofa cat, caused is caused by:
aerosol is caused by: e. patienl's washed erythrocytes with normal

b.c
by expoS/lre to aplastic food dish, is mediated by:
serum a. antibodies to T3
a. histamine a. IgE-triggered mast-cell degranula[ion
b. antibodiestoT~
b. seroto nin b. IgG- and complemelll -rnediated cell damage
37. A calf develops an acute respiratory infection. A c. antibodies to thyroid-stimulating hormone
c. bradykinin c. IgM- and complemen t-mediated cell damage serum sample/or serologic testing is obtained d. lymphocytic infiltration ofthyroid follicles
d. lCllkotriene 8 4 d. immu ne-complex depOSition during tile ilIlless and again after 21 days. Acute e. antibodies to thyroid-stimulating hormone
e. prostaglandin D2 e. T lymphocyte-media ted delayed and convalescent titers for four viruses areas receptors
hypersensitivity /ollows: bovine respiratory syncytial virus 1:16
28. Lesions produced in immullccomplex disease and 1:256; infectious bovine rhinotracheitis virus

tto
1:32 and 1 :64; parainfluenza 3 virus 1:32 and 42. Prevemion of ref aI/us depends on amibodies
involve all of tile following components except: 33. In enzyme-linked immwlOsorbent assay (EUSA) specific/or:
1: 16; and bovine virus diarrhea virus 1:128 and
a. neutrophils to rest for felille leukemia virus (FeLV) antigen in
1:256. WllOt is tlte most fikelycause ofrhis a. Clostridium tetani
a cats serum, the plastic wells o/the test kit are
b. antibodies ill/ection?
coated with: b. Clostridium per/riflgem
c. antigen a. infectious bovine rhinotracheitis virus c. Clostridium botulinum
d. complement a. monoclonal antibody to FeLV
b. bovine respiratory syncytial virus d. botulinus toxin
e. macrophages b. purified FeLV
c_ parainfluenza 3 virus

o
c. horseradish peroxidase-conjugated antibody e. tetanus toxin
to r-eLV d. I10vine virus diarrhea virus
29. Allligen specificity in delayed hypersensitivity e. None of these viruses is likely to have caused 43. An anapl'ylactoid response to a vaccine is:
d. serum from the cat
reactiol/S is con/erred by wllieh celf type? the illness.

alk
e. monoclonal antibody to feline IgG a. an IgE-mediated acute allergic reaction
a. B lymphocyte
b. a shockUke reaction that resembles
b. T lymphocyte 38. Whicll sample is used for the standard
34. Wllicll sample is used in EUSA to test for anaphylaxis
c. macrophage immlUlofluorescem antinuclear alllibody test?
parvollims antigen ill dogs? c. an acute hemorrhagic response
d. NK cell a. skin d. an anergic reaction
a. serum
c. ep ithelioid cell b. Iddney e. an IgE-mediated late-phase response
b. plasma
c. serum
c. fe ces
30. Development alamedar uveitis ("blue eye")
following vaccination wilh a modified-live canille
adenovirus 1 vaccine indicates tltat:
a. adenovirus-specific antibody is presenl in
d.
e.
saliva
whole blood
w. d.
e.
peripheral blood leukocytes
joint fluid

39. WI'icll sample is llsedfor the immunofluorescent


44. Which pair o/cross-reactingantigens has been
used in a vaccination strategy?
a. group-B streptococci and myocardium
the eye 35. lmmunoflilorescellce srainingofa skin biopsy wirh b. feline infectious peritonitis virus and enteric
test for anlimegakaryocyte antibodies?
antica1line IgG shows illlercellular staining of the coronavirus
b. the virus has become virulent and damaged epithelial cells. 17,is pattem is consistent with: a. peripheral blood c. human ABO erythrocyte antigens and
ww
the cornea
a. discoid lupus erythematosus b. lymph node intestinal fl ora
c. there were contaminants in the vaccine
b. chronic allergic dermatitis c. bone marrow d. human and bovine strains of respiratory
d. the dog is allergic to some component of the syncytial virus
c. pemphigus vulgaris d. spleen
vaccine
e. kidney e. canine distemper virus and human measles
e. there is hemagglutination of the virus in the d. scleroderma virus
cornea e. vasculitis

C 1998 Mosby-Year Book, Illc. Photocopyi"g is prohibited by law. Correcra"swersuroon pages 139· 141.
www.alkottob.com
,
136 SECTION 5
www.vet4arab.co.cc Immunology 137

45. A baboon heart grafted imo a human infant is an d. cats 55. The complement-fixation test uses hemolytic c. very specific for particular antigeniC epitopes
exampleo! e. nonhuman primates complement and antibody-sensitized sheep d. present in patients with monoclonal
erythrocytes as an indicator system. In a gammopathies

om
a. an autograft complement-fixation test to tktennine the titer of
50. 111 foals and calves, colostral antibodies are e. present in patients with polyclonal
b. an allograft antibody against Sa1monella antigen, the test tubes
transported across the intestinal epithelium: gammopathies
c. a xenograft contai" serum dilutions starting at 1:10 in tube 1,
d. a syngraft a. only during the first 6 hours of life with twofold dilutions through tube 12. Wllfch
serum sample has the highest antibody titer? 60. In horses, wllich blood group antigens are most
e. an isograft b. during the first 12 to 24 hours of life
immunogenic to horses lacking these
c. during the first week of life a. sample A, with hemolysis in all 12 tubes determinants?
46. Vasculitis detected with immwlOfluoresamceof d. during the firSl2 weeks of life b. sample B, with hemolysis only in tubes 6
deposits of immunoglobulin and complement in through 12 a. Cand U
e. during the first 3 weeks of life
b. Kand P

b.c
the vascular endothelium and tissue spaces can c. sample C, with no hemolysis in any tube
occur with all ofthe following except: d. sample D, with hemolysis only in tubes 2 c. DandQ
51. Allthefollowingare mediated by immune
a. booster immunization for hepatitis through 12 d.AandC
complexes except:
b. infection with a virus that causes chronic e. sample E, with hemolysis only in rubes II e. Aand Q
a. hypersensitivity pneumonitis and 12
viremia
b. anterior uveitis caused by can ine adenovirus I
c. allergic desensitization 61. Congenital deficiency of which complemem
c. flea-bite hypersensitivity compOTlellt would have the greatest negative
d. acute rhinovirus infection 56. Grafting which type of tissue is most likely to result
d. glomerulonephritis caused by viral infection in graft-versus·llOst disease? effect on a puppy's ability to kill pathogenic
e. inhalation of fungal spores by a horse with

tto
e. Arthus reaction at the site of vaccination bacteria (i.e., make the animal most susceptible to
high IgG titers a. skin bacterial infection)?
b. kidney
52. Plasma-cell tumors are most common in the: a. C l q
47. A small-molecular-weigll! compound thar binds c. bone marrow
roa hOS1 protein orcelf and rllell becomes a. skin b. C2
d. heart
immunogenic is called: b. lymph nodes c. C3
e. liver
a. an antigen c. liver d. CB

o
b. a hapten d. bone marrow e. C9
57. Class-I major histocompatibUitycomplex
c. complement e. thymus antigens are commonly typed using:
d. an idiotope • a. lymphocyte stimulation
62. For some respimrory diseases, intranasal

alk
immuni:Ultion is prefemble to intramuscular
e. a paratope 53. What is myeloma protein, the product of plasma- b. mixed lymphocyte reaction immllfli:Ultion because it stimulates production ot
cell tumors? c. agglutination
48. An acute, shocklifce episode following a. cytotoxic T ceUs
a. an immunoglobulin d. complement fixation
consumption o/a drug is most likely to bea drug- b. opsonizing antibodies
b. an albumin e. rnicrocytotoxicity
induced hypersensitivity reaction caused by: c. mucosal IgE antibodies
c. a cytokine
a. IgG d. mucosallgA. antibodies
d. an a-globulin 58. Class-II major histocompatibility complex
b. lgM e. complement-fixing antibodies
e. a j3-globulin antigens are commonly typed using:
c. IgE
d. IgG and IgM
e. immune complexes monoclonal gammopathy except:
w.
54. All of the following SIlggest a diagflOsis of
a.
b.
c.
lymphocyte stimulation
mixed lymphocyte reaction
agglutination
63. Diagnosis of some fungal infections can be
facilitated by injecting an extract of the organism
intradermally and observing the injectioll site for
a. hyperproteinemia d. complement fixation swelling after:
49. In which species is failure of passive b. hypercalcemia e. microcytotoxicity
immunoglobulin tramfer most common? a. 30 minutes
ww
c. hyperglycemia b. 4 hours
a. horses d. anemia 59. Monoclonal antibodies are useful in diagnostic c. 8 hours
b. dogs e. immunodepression tests because Ihey are: d. 24 hours
c. people
a. broadly reactive against many antigenic e. 72 hours
determinants
b. inexpensive and easy to produce

C 1998 Mosby-Year Book, Inc. Photocopying is prohibited by law. Comet answers are on pages 139-141.
www.alkottob.com
138 SECTIO N 5
www.vet4arab.co.cc Immunology 139

64. TI,e reaction described in Question 63 is all d . Three loci code for class-I, -II , and -III 74. A veterinariall sllared ill tile 1996 Nobel Pri2.efar 75. III treatillg patiellts with some types of tumors, rhe
example ofa IrypersensirilJity reaction mediated by; antigens. Medicine, based Oil his research on: patient's T cells arecOlwerted to lymphokille-
e. There are many alleles for each loclls. acci(I(Ited killercelfs by in vitro exposure to:
a. IgE a. simian immunodeficiency virus

om
b. immune complexes b. sequencing the human genome a. inte rferon -a
c. IgG and complement 69. What is tile function of tile class-/l major c. major hi stocompatibility complex restriction b. interferon--y
h istocompatibility complex a migen ? c. interle ukin 3
d. NK cells d. bovine leukocyte adhesion deficiency
e. T lymphocytes a. serves as a complement receptor e. foot-and -mouth disease d. interleukin 4
b. presents antigen to T helper cells e. interle ukin 2
65. Advantages ofa noninfectious subunit vaccine c. presents antigen to T cytotOxic cells
versus a killed whole-organism vaccine include d. serves as an Fc receptor
all of tile following except:
Answers

b.c
e. presents antigen to B cells
a. immune response is to only one protein
b. no adjuvant is necessary 70. Which disorder can have either an autoimmune 1. e IgA is a secretory antibody. It is protected fro m 12. b Both of these are immunodiffusion tests
c. antibodies are not made to non protective or a congeniral etiology? digestion by its secretory component, a peptide requiring soluble antigen and antibody so as to
antigenic epitopes produced by epithelial cells on the mucosal form insol uble complexes at the equivalence
a. myasthenia gravis surface. point.
d. it decreases adverse reactions to vaccination b. pemphigus foliaceus 2. b IgM has a molecular weight of about 900,000 13. b T lymphocytes are incapa ble of presenting
e. it allows for d iscrimination between c. Waldenstrom's macroglobuline mia daltons and is too large to easily d iffuse through antigen.
vaccinated and infected animals
d. bovine leukocyte adhesion deficiency the vascular endothelium.

tto
14. e Interleukin 12 and interferon gamma are both
e. Cushing's disease 3. d IgE is the only immunoglobulin that binds to T helper type- l cytokines.
66. All of the following are amigenic differences high-affinity Fc receptors on mast celis. Some 15. e Interleukins 4, 10. and 5 are all T helper type-2
between t1Imorcefls and " ormai cells in the same species have an IgG subclass thai binds to mast cytokines.
71. A dog deficient inIgA would be most likely to
tissue or individual except: cells with lower affinity, but this is inconsistent.
develop disease related to which body systems? 16. b Interleukin 2 is produced byT cells and binds
a. feline oncornavirus cellmcmbr3 ne antigen Therefore IgE is the best answer. to interle ukin 2 receptors on T cells. This binding
(FOCMA) expression on fcline cells a. musculoskeletal and nervous systems 4. c IgD is the only immunoglobulin for which stimulates T cells to divide; thus interlcukin 2 is
b. cardiovascular and respiratory systems

o
b. loss of histocompatibility antigens the re is no identified function other than as a an aUlocrine growth factor.
c. gas(fointestinal and respiratory syste ms receptor in B cell development. IgM also serves 17. e lnte rleukin 12 induces T helper 1 cells to
c. a-fetoprotein expression on liver cells
d. urinary and reproductive syste ms as a B cell receptor during development, but this produce ...,-interferon, which is also known as
d . carcinoembryonic antigen expression on is not its major role.
e. lymphoid and hematopo ietic systems macrophage-activating factor. Macrophages must

alk
colon cells
5. a IgG predominates in the seco ndary antibody be activalCd to kill facultative intracellular bacteria.
e. addition of new histocompatibility antige ns response, whe reas IgM predominates in the 18. e -v- inte rferon increases ruffled me mbrane
72. Tile barrier function of the skin asan innate primary response.
defense mechanism is best illustrated by: activity. activates e nzyme systems, and enhances
67. Vaccinia virus is all excel/em vector fo r 6. b A mast cell degranulated by an IgE-antigen the killing ability of macro phages.
recombinant vaccines because it: a. allergic dermatitis reaction on its surface releases histamine (type-I 19. d Im munoelectrophoresis distributes the serum
b. discoid lupus erythematosus hype rsensitivity reaction). protein fractions. Then, using agar gel diffusion
a. has a large genome
c. contact dermatitis 7. d Antinuclear antibodies are present in patients with antiserum to specific immunoglobulin
b. does not integrate into the host genome
w. with syste mic lupus erythematos us. One of the heavy chains, one can identify the de nse arc
c. is atten uated and sta ble d. Fllsobacterium necroplJOrum infection ohhe
complications associated with this diso rder is characteristic of the monoclonal antibody
d. does not multiply in nonhuman species equine frog
development of immune complexes and (paraprotein).
e. is easy 10 administer e. hypersensitivity to Culicoides species subsequellt type-III hypersens itivity reaction in 20. e Polyclonal gammopathy can occur with
the glomeruli, which ultimately results in pyometra, chronic abscess, dirofilariasis, and
73. Indigenous flora are desirable in tile glomerulonephritis and kidney failure. syste miC lupus erythematosus. These conditions
68. Concerning the major II istocompatibility
complex, which SUllemem is least accurate? gasrroimestinal tract because they: 8. a Germinal centers that produce B lymphocytes all produce ch ronic antigenic stimulation.
ww
are located in the cortex of lymph nodes. Plasma-cell tumors produce monoclonal
a. Inheritance of class- I antigens is recessive. a. prevent adherence of pathogenic bacteria
9. e T cells are mainly found in the para cortex of gammopathy.
b. Inheritance of class-J antigens is codominant. b. decrease the transit time of ingesta
lymph nodes. 21. d After a foal consumes colostrum that contains
c. Alleles from both parents form the haplotype. c. promote secretion of digestive enzymes
10. a Lymphocytes exit the lymph node through the antibodies 10 the foal's e rythrocyte antigens,
d. stimulate IgA production efferent lymphatics. complement is fIxed and the foal's erythrocytes
e. act as mitogens for Peyer's patch lymphocytes II. c T cells mature and acquire theirT cell markers are lysed or removed by the reticuloendothelial
in the thymus. system .

e 1998 Mosby-Year Book, Inc. PllOfocopying is prohibited by law.


www.alkottob.com
140 SECT ION 5
www.vet4arab.co.cc
• Immunology 141

22 . d This calf has an inherited trait that makes it than one dilution for it to be a significant 52. d Bone marrow is most commonly affected. hypersensitivity reaction 48 to 72 hours after
unab le to make the COI l/COlS adhesion increase. A decreasing titer indicates normal 53. a Plasma-cell tumors produce monoclonal antigen injection .
molecule; thus the leukocytes cannot adhere to metabol ism of preexisting antibody. immunoglobulin. 64. e Sensitized T lymphocytes react with antigen

om
the vascular endothelium and canoO[ undergo 38. c Senun is diluted and used as primary antibody 54. c Myelomas do not cause hyperglycemia. and produce cytokines that attract macrophages
diapedesis and move into the tissues. on a fixed cell line (usually Hep-2 cells) on slides to the area.
55. c Hemolysis indicates that no antigen-specific
23. c Foals with combined immunodeficiency lack that are later labeled with anti-IgG fluorochrome antibody is present to fix the complement. 65 . b Some type of adjuvant is needed to increase
lymphocytes and therefore cannot make dye (species specific). Sanlple A has no Salmonella-specific antibody the immunogenkity of subunit vaccines.
antibody. Affected foal s have some IgG from the 39. c Megakaryocytes are found in bone marrow. (hemolysis in aJll 2 tubes). Sample C has the 66. e Items listed in answers a through d are found
maternal antibodies in lhe colostrum but have 40. b Antibodies to the acetylcholine receptors on most antibody because, even at the greatest with some tumors, but new histocompatibility
no IgM because that must be made by the foaL the motor end plate prevent the signal from dilution (tube 12), there is no residual antigens are not expressed because the genetic
24 . b Bence-Jones proteins are light chains. crossing the neuromuscular junction and complement (no hemolysis). The other samples capability is present only fo r those normally
25. b IgM molecules are very large and impart very in itiating a muscle contraction; thus the patient fall between these two extremes. expressed by that individual.

b.c
high viscosity to the blood. IfIgA is bivalent, it appears weak. 56. c Bone marrow contains immunocompetent cells 67. d Vaccinia virus does multiply in nonhuman
can also increase viscosity. The best answer, 41. d Al though autoantibodies to thyroid that respond to foreign major histocompatibility species.
h owever, is IgM. components are formed, pathologic changes are complex antigens on the host's cells. 68. a Inheritance is codom inant.
26. e Histamine does not cause chemotaxis of thought to be caused by the cellular infiltrate. 57. e Antisera for class-I major histocompatibility 69. b Class- II antigens present antigens to helper cells.
neutrophils. 42. e Tetanus toxin causes the damage, and complex antigens are combined with the cells, 70. a Myasthenia gravis may be congenital and
27. c Histamine. serolOnin, and bradykinin act therefore it is the target for the immune response. followed by complement and a blue dye. inherited.
during the immediate response. Leukotriene B4 43. b An anaphylactoid response resembles Exclusion of the dye (no microcytotoxicity)
and prostaglandin O2 are part of the late-phase means that the cell does not have that particular 71. c Affected animals would most likely develop
anaphylaxis, but it is not initiated by binding of

tto
type- I hypersensitivity response. Only determinant because the complement failed to gastrointestinal or respiratory infection because
IgE antibody to mast cells. It is usually initiated by
prostaglandin D z causes smooth muscle lyse the cell. these have mucosal surfaces where secretory 19A
mast-cell degranulation via other mechanisms,
contraction. is important in preventing infection.
such as the alternate complement pathway. 58. b Class-II antigens are typed using a cellular
28. e Macrophages are not a part of rype-III reaction, that is, response of host lymphocytes w 72. d ~ Thrus h" develops when moist conditions
44. e Distemper in dogs and measles in people are
hypersensitivity reactions, although they may donor (or known type) cells. compromise the barrier function of the kerati n
both caused bypammyxoviruses.lnduction of
move into the area later to help clean up the covering the frog, allowing entry and growth of
antibodies in pups by vaccination with measles 59. c This answer choice is most accurate. Answer d the causative bacteria.
tissue damage. virus can protect against distemper caused by is true but not correct because monoclonal

o
29. b Sensitized T lymphocytes are responsible for cross-reacting epitopes. Fortunately, maternal antibodies detected in patients' sera are not 73. a By occupying a niche in the enteric
antigen specificity. antibodies w distemper virus do not interfere useful in diagnostic tests. environment, these organisms prevent adhesion
\vith immunization. and subsequent infection by pathogens.
30. a This is an Arthus reaction that occurs when 60. e Groups A and Q are most often associated with
74. c The Nobel Prize for Medicine was presented to

alk
antigen and antibody form complexes in the 45. c A xenograft is a graft performed between two neonatal isoerythrolysis.
anterior chamber of the eye. different species. Drs. Zinkernagel and Dougherty for their work
61. c A defi ciency ofC3 inactivates both the classical on major histocompatibility complex restriction.
3 1. a A skin test is an in vivo test, not an in vitro test. 46. d The antigen is limited to the respirawry tract. and alternate pathways. Dr. Doughe rty is a veterinarian.
32. e Contact dermatitis is a T ceU- mediated type-IV High levels of precipitating aO(ibodies would not 62. d IgA is the secretory antibody found on
be present. 75. e Lymphokine-activated killer cells are
hypersensitivity reaction. mucosal surfaces.
produced by stimulating the patient'sT cells in
33. a Most antigen-catching EUSAs use a bound 47. b A small-molecular-weight compound that is 63. e In animals with a fungal infection , injection of vitro with interleukin 2 and the n returning them
virus antigen-specific antibody to "catch" any not immunogenic by itself may become the fungal antigen typically causes a type-IV to the patient.
antigen in the sample. The antigen is then immunogenic by binding to cells or serum
detected by an enzyme-conjugated virus-specific
a ntibody (double antibody sandwich).
34. c The virus is passed in the feces of infected
dogs.
48. c An IgE response to a drug produces
anaphylactic shock.
w.
proteins that become carrie r molecules and
induce an immune response to the hapten.
NOTES
35. c Pemphigus vuJgaris is caused by antibody to 49. a Horses must obtain all their maternal
the intercellular material. antibodies from colostrum, whereas people and
ww
36. b If the test is positive for autoantibody, the cells nonhuman primates obtain it by placental
are agglutinated when combined with rabbit transfer. Dogs obtain most oftheir antibodies
anticanine IgG. from colostrum, with some placental transfer.
37. b The titer for this virus has increased more than 50. b After 24 hours of life, antibodies can no longer
fourfold. Any increase of titer between acute and be transported across intestinal epithelial cells.
convalescent serum samples must be greater 51. c Flea-bite hypersensitivity is rypically a type-I
reaction, with or without a rype-IV reaction.

© 1998 Mosby-Year Book, Inc. Photocopying is proll/bleed by law.


www.alkottob.com
142 SECTION 5 www.vet4arab.co.cc SECTION

NOTES
6

om
Nutrition
L.D. Baker, EA Moser

b.c
Recommended Reading
Case LP el a1: Canine and feline nutrition, 51 Louis, 1995, Mosby.
Guilford WG: Strombeck's small animal gastroenterology, cd 3, Philadelphia. 1996, WE Saunders.
KeUy NC, Wills JM: Manualofcompanion animal !llIfririo" and feeding, Ames, Iowa, 1997, Iowa

tto
State University Press.
Lewis In: Equine clinical nutrition, Baltimore. 1995, Williams & Wilkins.
Lewis LD el a1: Small animal d inirol nutrition 111, Topeka, Kan, 1987, Mark Morris Associales.
Naylor , N, Ralston 51.: Large animal clinical nutrition. 51 Louis, 1992, Mosby.
Nutrient requirements of domestic animals, Was hingto n , DC. Na tional Academ y Press. (TItles in
series: Cats 1986; Dogs, 1985; Dairycattle, 1989; Bee/amle. 1996; Horses, 1989; Sheep, 1985:
Swine, 1988; Goats, 1981.)

o
Practice answer sheet is on page 265.

alk
Questions
L.D. Baker
1. 111e approximate energy content of hay is beSt 3. Whicll feed has the greatest energy density?
indicated by:
a. shelled corn
a. stem size b. barley
w. b.
c.
d.
color
smell
number of weeds present
c. whole cottonseeds
d. soybean meal
e. cottonseed meal
e. number ofleaves present
4. Which ofthe followillg is considered a protein
2. Nonprotein nitrogen is used least efficiemiy in a:
ww
supplement?
a. high-producing dairy cow a. shelled corn
b. low-producing dairy cow b. corn gluten meal
c. pregnant ewe c. soybean hulls
d. gelding on pasture d. oats
e. feedlot steer e. barley

C 1998 Mosby-Year Book,Inc. Photocopying is prohibited by law. Correct answers are on pages 151-153. 143
www.alkottob.com
144 SECTION 6 www.vet4arab.co.cc Nlltritlon 145

5. WI,jel, vitamin is stored in the liver ill significant 10. VVh icll feed has tile lIigllest percentage of rumen IS. In ruminants what is the primary source of c. increase the caloric density of the diet by
amoullts? escape (bypass) protein ? glucose? e levating the protein concentration in the grain
a. fo late a. distillers' dried grains a. am ino acids mix without changing the grain/ fo rage ratio

om
b. cobalamin b. blood meal b. acetate d. increase the caloric density by inc reasing the
c. whole cotto nseeds amount of grain and adding fat to the ration
c. thiamin c. lactate
to maintain forage levels at 40% or more of
d. pyridoxine d. urea d. glycerol dry matter
e. niacin e. roasted soybeans e. propionate e. reduce the caloric density by increasing the
amount of forage in the ration to 60% or m ore
6. WI/jell afthe/ollowing is not used by rumen 11. Wllicllfeed has the lowest percentage of rumen 16. Which of the following is not a nonnal end product of dry ma tter to supply sufficient d ie rary fib e r
microbes as an energy source? escape (bypass) protein ? ofctlrbohydrate femlentation in the rumen?

b.c
a. ammonia a. distillers' dried grains a. lactate 19. Tile revised ration should befed:
b. cellulose b. blood meal b. methane a. ad libitum to a ll cows in early lactation,
c. peptides c. whole cottonseeds c. carbon dioxide regardless of milk production levels
d. fat d. urea d. glycerol b. ad libitum only to cows with high milk
e. carbohydrate e. roasted soybeans e. acetate produc tion
c. ad libitum only to thin cows
7. When rumen unprotected fats are added 10 dairy 12. A nonpregnantiactating dairy cow is producing 17. In a well-managed dairy operation, Holstein d. in limite d amounts in early lactation until
cattle rarions. which minerals should be 45 L of milk per day, with a calcium content of 1.3 heifers are reared to adljew what ideal body cows prove the ir milk production potential

tto
supplemellted? gIL This cow excretes 1.5 gofcalcium daily in her weight at breeding (12 to 14 momlrs ofage)? e. in limited amounts to all cows, regardless of
urine, and her endogenous fecal ctllcium loss is 6 milk production or stage of lactation
a. cal cium and phosphorus
glday. If dietary calcium is 40% aooilable, what is a. 270kg
b. calcium and magnesium this cows daily dietary ctllcium requirement? b. 320 kg
20. Excessi/)(! j"takeofllOllStrucwral carbohydrate is
c. calcium and potassium c. 350 kg
a. 36g conducive to all of the following except:
d. magnesium and phosphorus d. 385 kg
b.72g a. increased milk volume
e. magnesium and potassium e. 400 kg

o
c. 80g b. decreased milk fat content
d. I18g c. decreased milk protein content
8. A producer wall15lOfeed 0.1 kg ofcalcium to his Questions 18 and 19
e. l 65g d. decreased rumen pH
cow. How milch Iimesrolle con raining36 %

alk
A dairy farmer has ctllled on you to evaluate tire e. increased rumen turnover
calcium should be/ed?
13. The volwne of milk produced is determined nutritional status of early-lactation cows. At the farm
a. 0.036 kg you determine Ihat this group ofcows has access to a
mainly by tlie mammary glands uptake of: 2 1. Which cow is 11I0silikely to develop milk fever ?
b. 0.28 kg total mixed ration at all times. However, the cows
c. 0.36 kg a. acetate appear very thin and milk production is suboptimal. a. 2-year-old Jersey fed alfalfa hay before calving
b. glucose The forage/grain ratio, on a dry-mauer basis, is 50:50.
d. 2.8 kg b. 5-year-old Holstein fed timothy hay during
e. 3.6 kg c. propionate the d ry period
d. amino acids 18. To minim ize bodyconditionioss and improve
w. milk production, tire most appropriate c. 2-yea r-old Holstein fed alfalfa hay before
c. lactose recommendation is to: calving
9. A prodllcer is / eeding 21 kg of corn silage to his
cow daily. The dry-matter content is 30%. How d. 5-year-old Holstein fed alfalfa hay during the
a. increase the caloric density of the diet by dry period
muell dry mOiler is being/ed daily? 14. Which o/the/ollowing is most resistant co reducing the amount of forage in the ration to
ferm elllative digestion in the rumen? e. 3-year-old Jersey fed timothy hay during the
a. 70 kg 30% of dry m atte r so that m ore grai.n can be fed
dry period
b. 0.63 kg a. cellulose b. maintain the same caloric density by
ww
c. 7.0 kg b. hemicellulose substituting better-quality forages in the ration,
d. 6.3 kg c. pectin allO\ving the amount of grain to be reduced
e. 0.7 kg d. lignin
e. starch

@ 1998 Mosby-Year Book, Inc. Photocopyi/lg is prollibited by law. Correct answers are on pages 151- 153.
www.alkottob.com
146 SECTION 6
www.vet4arab.co.cc Nutrition 147

22. Which pastllTecomposition grazed by cattle and d, 133% of its requirement for protein 32. Wllat is the primary copper-containing protein in d. vitamin K
sheep is mosllikeiy to predispose to grass tetany? c. 150% of its requirement for protein mamnmIian plasma? e. choline
a. high phosphorus and calcium. low pmassiulll a. plasferrin

om
and sod ium 24. lWlicllfeedstllfffor IlOrses lias tile highest amount b. "y-globulin 38. Which vitamin isa constituent of rhodopsin and
b. high potassium and c..'l1cium, low phosphorus ofdigestiblee"ergy per u"it I/Olullle? c. albumin is involved in mucopolysaccharide synthesis and
and sodium d. ceruloplasmin maintenance ofepithelial tissues?
a. rolled oats
c. high nitrogen and calcium, low phosphorus e. transferrin
b. timothy hay a. vitamin A
and sod ium
c. alfalfa hay b. vilaminC
d. high nitrogen and phosphorus, low potassium
and calcium d. shelled corn 33. TIle branched-chain amino acids include: c. thiamin
e. high nitrogen and potassium, low sodium and e. beet pulp a. arginine, lysine, and glycine d. folic acid

b.c
calcium e. vitamin D
b. valine. leucine, and isoleucine
25. Witll adequate "utrition, a "ormai400- to 550-kg c. tyrosine, tryptophan, and arginine
23. A race horse is beingfed 10 lb of 18% cnlde protein mare call be expected to produce how much milk d. taurine, methionine, and lysine 39. Wllich vitamin promotes growth and
alfalfa Ill/yand 121bofa 10% sweet feed. The dailydllring thefirsI3 months oflactatio,,? mineralization of bones and increases calcium
e. carnitine and lysine
absorption from tile gut?
horse requires 2.51bofprorein per day. In relation
to recommellded levels. tlJis horse is being/ed:
a. 80% of its requirement for protein
a. 5 to 8 kg/day
b. 7to 10 kg/day
c. 9to 12 kg/day
34. What is the protein equivalent of urea? •. vitamin A
b. vitamin D
a. 16%
b. 100% of its requirement for protein d. II to 14 kg/day c. thiamin

tto
b. 100%
c. 120% of its requi rement for protein e.15toI8kg/day d. choline
c. 281% e. niacin
d. 0.015%
e. 625%
LA. Moser Questions 40 and 41
26. A diet contains nutrient A in tile amount of5 29. WJ,ic" amino acid is esselltial for cats but not for A dog owner comes to your clinic with the following
35. lActose is a disaccharide made up of which cwo
mglkg dry matter. flow many parts per million of dogs?

o
,nonosaccharides? infomUltion about a dogfood she obtained from a pet
nlltrierll A are illcluded in tile diet? food company represemative:
a. valine
a. 0.0005
a. glucose and fructose
b. threonine Moisture 75.0%
b. glucose and glucose
b. 0.05

alk
c. lysine Crude protein 9.0%
c. 0.5 c. fructose and raffinose
d. methionine Crudefat5.0%
d. sucrose and maltose
d.5 c. taurine Crude fiber 1.0%
e. 500 c. glucose and galactose

30. Which amino acid contains sulfur? 40. MJat is the approximate crude protein content on
36. Whicll of the following are B vitamins?
27. Tile most abundanl fauyacid ill com oil is: a dry-matter basis?
a. glycine
3. tryptophan aJld thiamin
a. stearic acid b. valine a. 6.75%
w. b. thiamin and vitamin K
b. palmitic acid c. leucine b. 12.0%
c. riboflavin and choline
c. oleic acid d. isoleucine c. 22.5%
d. thiamin and pantothenic acid
d. linoleic acid e. methionine d. 36.0%
e. arachidonic acid e. pantothenic acid and choline e. 63.0%
31. Wlmt is tIle majorcarboltydrate-digesti"genzyme
37. Which vitamin is involved in formation ofactive
28. flow is 10,000 parIS per million expressed as a secrered in pallcrearicjuice? 41. What is the percent dry marter in the food?
ww
percentage (parts per lIundred)? prothrombin and plays an important role in
a. a-amylase blood clorting? a.90.0%
a. l % b. chyme b. 25.0%
a. vitaminA
b. 10% c. deXlrinase c. 75.0%
b. vitamin B, (pyridoxine)
c.O.I% d. trypsin d. 10.0%
c. niacin
d.l OO% e. chymotrypsin e. 60.0%
e. 10,000%

e 1998 Mosby- Year Book, Inc. Plwtocopying is proll/'bilcd by law. Correct omwers are 011 pages 151 -153.
www.alkottob.com
148 SECTION 6
www.vet4arab.co.cc Nutrition 149

47. "Vllat is the longest portiorl of tile small intestine 52. During lactation in queeru,[ood imake typically: 57. In animals what is the predomirumrform of
42. lonophores are low-molecular-weight molecules
in companiorl animals? provitamin D, wllich is, through the action of
tlult hind ions of various minerals and modulate a. stays the same
sunlight on skin, changed to the active form?
rlleir movement across membranes. A major effect b. increases to about 1.5 times normal

om
a. duodenum
QII mmen/ermenration ofionophores is: c. increases to about 2 to 3 times normal a. ergosterol
b. pylorus
a. increased methane production d. increases to about 5 to 6 times normal b. 7-dehyd rocholesterol
c. jejunum
b. decreased molar ratio of acetate [Q propionate e. decreases to a bout 50% of normal c. cholesterol
d. ile um
c. increased molar ratio of acetate to propionate d. cholecalciferol
e. ileocolic
d. decreased production of propionate e. zearalenone
53. Halogeton and greasewood are poisonous plants
e. decreased feed efficiency 48. For wll ich of the three classes of nutrients that grow in the arid and semiarid saline regions
(carbohydrates, proteirlS,[at) are digestion and of the westem United States. I1IeY are toxic to 58. Which of the following most accurately describes
sheep because of their high content of: the/undamenral role o/vitamin E?

b.c
43. When dairy cattle are fed unprotected long-chain absorption most complex and most efficient?
fOlty acids, precautions must be taken to provide a. protein a. copper a. antirachitic
supplemental sources of b. oxalate b. precursor of coenzyme E
b. fat
a. calcium and phosphorus c. carbohydrate c. calcium c. antioxidant
b. zinc and copper d. protein and fat d. selenium d. antired uctant
c. manganese and magnesium e. carbohydrate and fat e. ketone bodies e. required for prothrombin formation
d. iron and phosphorus
e. calcium and magnesium 54. Based on digestive anatomy and physiology. 59. What is the primary nutritional cause of simple

tto
49. Mlich IlOrmone stimulates pancreatic enzyme
secretion and causes contraction o/the rabbits are classified as: goiter (enlargement of the thyroid gland)?
44. Digestible etiler extract fat cotltains gaflbladder, aflowing lipase and bile to enter the a. ruminants a. iodjne deficiency
approximately lIow much energy per unit weiglu, diloderillm? b. carnivores b. iodine toxicity
as compared with carbohydrates? a. cholecystokinin c. nonruminant herbivores c. vitamin K deficiency
a. about the same b. insulin d. obligate omnivores d. vitamin K toxicity
b. about 6.25 times as much c. chymotrypsin e. variable omnivores e. sodium toxicity

o
c. about 2.25 times as much d. glucagon
d. the energy from carbohydrate and fat cannot e. trypsin 55. Gross energy minus energy lost infeces, urine, and 60. What is the generally recommended minimum
be compared on a per unit weight basis gaseous products ofdigestion is equal to: rlutricnt standard for crude protein for canine

alk
e. about half as much SO. Chylomicroru exit the intestinal mucosal cells adult maintenance (units = dry-matter basis at
a. digestible energy
into the lamina propria and are transported: 3.5 kroVgJ?
b. metabolizable energy
45. Wllich ofthefoflowing most acc:urately indicates a. to capillary beds a. 10%
c. net energy
tlie prolein and fat cOlllerlts (80% moisture as fed) b. 15%
b. to the centrallacteals of the villi d. available energy
of tile milk ofcats? c. 18%
c. into the bowel lumen e. gross energy
a. 10 g protein and 6.5 g fat per 100 g milk d. into the mucosal lining d. 23%
b. 15 g protein and 7.5 g fat per 100 g milk e. into the brush border region w. e. 25%
56. The standard metl/Od of diagnosing food
c. 4 g protein and 6.0 g fat per 100 g milk hypersensitivity in dogs involves:
d. 8 g protein and 5.0 g fat per 100 g milk 61. MUll is the generally recommended minimllm
51. Unlike most ocher mammals, cats cannot convert a. serologic examination
e. 45 g protein and 45 g fat per 100 g milk ~-carotene to: nutrient standard for crude protein for canine
b. fasting, purgation, and challenge growth arid reproduction (units = dry-matter
a. vitamin C c. elimination and sequential challenge basis at 3.5 kroVg)?
46. faIry acids are classified by the number of double
b. vitamin B\ d. enzyme-linked immunosorbent assay
ww
bonds they contain. Afalty acid with one double a. 15%
bond is classified as: c. vitamin A e. intradermal testing
b.23%
d. folacin
a. saturated c. 28%
e. arachidonic acid
b. polyunsaturated d.30%
c. unsaturated e. 43%
d. essential
e. nonessential

C J998 Mosby-Year Book, Inc. Pllotocopying is prohibited by law. Correct answers are on pages 151·153.
www.alkottob.com
150 SECTION 6 www.vet4arab.co.cc Nutrition 151

62. In dogs, tlJeanemiQ associated with copper 67. In wllich disease ofdogs can serum folate and 72. Cobalt is a component of 74. What is the calciumJpllOsplJOrus (CalP) ratio in
deficiency is classified as: vitamin B,z levels be used as screening tests? mammalian bone?
a. folic acid
a. hypochromic microcytic a. intestinal malabsorption syndrome b. biotin a. 1:2

om
b. hyperchromic microcytic b. hepatic steatitis syndrome c. camiti ne b. 201
c. normochromic normocytic c. hyperthyroidism d. vitamin B il c. 3: 1
d. hypochromic m acrocytic d. diabetes m ellitus c. taurine d. 0.5: 1
e. hype rchromic macrocytic e. diabetes insipidus e. 5: 1
73. Iii which cwo dog breeds does primnry hepatic
63. 111eftrst step in metabolicacrivation a/vitamin D 68. MlQt are tile most common nutritional deficits in copper storage disease occur as a result ofa 75. A dog has a thin hair coat, scaly dennatitis
is conversion of cllOfecalciJerol to 25- fed home-cooked diets for dogs atld cats? hereditary autosomal recessive disorder? (especially around the eyes, mouth, and ears),
hydroxycllOlecalciferol in the:

b.c
a. protein and fat a. collie and spitz pamkerarosis, and hair depigmematioli. l1,ese signs
a. kidney are most likely related to a dietary deficiency of
b. vitamin A and copper b. dac hsh und and golde n retriever
b. hypothalamus c. vitamin A a nd essen tial fatty acids c. Bedlington terrier and West Highland white a. zinc
c. brush border of the small intestine d. salt and protein terrier b. iron
d. liver e. fat and copper d. bulldog and pug c. sele nium
c. skin e. borzo i and beagle d. calcium
69. The toxic methylxallllline derivative, e. cadmium
64. Oftllefollowingfeedstl1/fs, which isgenemlly rich tlleobromine, m ay cause vom iting, diarrhea,

tto
in camitille? sudden collapse, and death in dogs and cats.
Mlat common hOlLselloldfoodstuffcontains this
a. meat
marerial? Answers
b. algae
c. legumes a. cocoa I . a Larger s tem s izes refl ect forage maturity, high 16. d Glycerol is produced from triglycerides a nd
b. onions fiber levels. and lower digestibility and available not from carbohydrates.
d. yeasts
c. whole wheat Oour energy. 17. c This is the ideal body weight fo r heifers to be
e. fat

o
d. cornstarch 2. d Nonprotein n itrogen has little benefit for bred.
hindgut fe rmenters, such as horses. 18. d Th is strategy sho uld red uce body condition
65. MlQt is the primary site ofcholesterol syn thesis in
e. vanilla
3. c This feed contains the greatest amount of fa t. loss and improve milk production.
most animals?

alk
4. b This feed contains more than 20% of crude 19. a The ai m is to maximize caloric intake to
70. How many carIS ofdogfood (SOO-kcal protein.
a. brush border o f the small intestine s uppor! early-lactation milk production and to
metabolizable energy per can) are needed da ily for
b. adipose tissue 5. b This is vita min 8 ,2 , lim it mobilization of adipose tissue reserves.
maintenance ofan auerage 13.6-kg (30-/b) dog?
c. kidney 6. d Rumen microbes cannot use fat as an e nergy 20. c Increased fermentable carbohydrate in the
a. 2 source. rumen generally increases microbial yield and
d. panc reas
b.3 7. b Calcium a nd m agnesium form insoluble soaps increases milk protein content.
e. liver
c. 4 with rumen unprotected fats. 2 1. d Excessive calcium fed during the dry period
d.5 8. b 0.1/0.36 =0.277 kg. plays tlle largest role in development of milk
66. In normal dogs, ingestion a/which type of
commercially form ulated dogfood results in the
highest immediate postprandial p lasma glucose
and insulin concemrations?
e. 6
w.
71. An animal will die most quickly from iackofwhar
9. d 2 1 kg X 0.30 = 6.3 kg.
10. b Approximately 80% of the protein bypasses the
rumen.
fever in multiparous cows.
22. e These factors inhibit the availability of
magnesium.
23. c I.B + 1.2 =3 1b of protein consumed ; 3/2.5
nutrient? 11. d Urea is 100% degradable in the rumen.
a. canned required = 1.2 = 120%.
12. e 45L x l.3g/L=58.Sg+L5g+6g= 66g/0.4
b. dry (lObble) a. water 24. d Corn is more dense tha n oats a nd weighs more
= l 6Sg.
ww
c. dry (kibble) soaked in wate r b. protein per volume.
13. b Lactose is the osmotic regulator of m ilk
d. semimoist c. carbohyd rate volume and is synthesized from glucose in the 25. d Mares in early lactation can produce 25 to 30
e. a mixture of canned and dry (equal calories d. fat mammary gland. lb of milk daily.
from each) e. e ne rgy 14. d Lignin is not a carbohydrate and is not 26. d 5 mgl kg = 5 ppm (pans per million).
fennentab le. 27. d More tha n 50% o f the fatty acids in corn oil is
15. e Glucose is produced from propionate via linoleic acid .
gluconeogenesis. 28. a 10,000 ppm = 1%.

@ 1998 Mosby-Year Book, Inc. Pllotocopying is prollibited by klw.


www.alkottob.com
152 SECTION 6
www.vet4arab.co.cc Nlltrition 153

29. e Taurine is an essential amino acid for cats but 49. a Cholecystokinin , aJongwith secretin, is released 71. a Fat-free muscle is composed of75% to 80% 73. c Bedlingtons and Westies may inherit copper
not for dogs. into the blood when add and partly digested food water. Water is required for essentially all body storage d isease.
30. e Methionine, cysteine, and cystine arc sulfur- from the stomach e nte r the duodemun. functions. 74. b There is little variation in the elementary

om
containing amino acids. stimulating secretion of pancreatic juice. 72. d Cobalt is a component of vitamin BI2 composition of bone ash from the 2: 1 Ca/P ratjo.
31. a a-Amylase is the primary carbohydrate 50. b Chyiomicro ns exit intestinal mucosal cells. (cyanocobalamin). 75. a A deficiency of zinc would cause these clinical
digestive enzyme secreted in pancreatic juice. enter ]acleals, and then move into larger signs.
32. d Ceruloplasmin is the primary copper- lymphatic channels.
containing protein in mammalian plasma. 51. c Cats cannot convert j3-carotene to vitamin A.
33. b Valine, leucine. and isoleucine are branched- 52. e During lactation a Queen's food intake typically
chain amino acids. increases 2 to 3 times normal.
=
34. c 45% nitrogen x 6.25 about 281 % protein 53. b These plants contai n a high level of oxalates.

b.c
equivalent.
35. e Lactose is comprised of glucose and galactose.
54. c Rabbits are cecal fe rmenters.
=
55. b Gross energy - Fecal energy Digestible
NOTES
36. d Thiamin and pantothenic acid are B vitamins. energy. Gross e ne rgy - Fecal/ urinary/gas energy
37. d Vitamin K is involved in formation of active = Metabolizable energy.
prothrombin and is important in blood clotting. 56. c Food hypersensitivity is diagnosed by feeding
3B. a Vitamin A is a constituent of rhodopsin, is
an elim ination diet, followed by challenge with
involved in maintenance of epithelial tissues, individual fo od ingredients.
and has a role in mucopolysaccharide synthesis. 57. b In animals, provitamin D is 7-

tto
39. b Vitamin D promotes growth and mineralization dehydrocholesterol. In plants, provitamin D is
of hones and increases calcium deposition. e rgosterol.
=
40. d 9.0 divided by 0.25 36% protein on a dry- 5B. e Vitamin E acts as an antioxidant in food and in
matter basis. Divide the as-fed protein by the the body.
percentage of dry matter to get the percentage of 59. a Goiter is primarily caused by insufficient
protein on a dry-matter basis. iodine.
=
4 1. b 100 minus water (75) dry matter of 25%. 60. c The generally recommended level of crude

o
42. b A decreased molar ratio of acetate to protein for maintenance is 18%.
propionate decreases methane production and 61. b The generally recommended level of crude
increases propionate production. protein for growth and reproduction is 23%.

alk
43. e Dietary fat may form a complex with calcium 62. c Copper deficiency causes normochromic
and magnesium, making them unavailable for normocytic anemia.
absorption by the gastrointestinal system. 63 . a The principal active metabolite is formed in
44. c Digestible ether extract fat contains about 9 the kidney.
kcal/g, whereas digestible carbohydrate contains 64. a Meats are a rich source of carnitine, whereas
about 4 kcal/g. plan ts are a poor source.
45. a On a dry-malter bas is, this equates to about 65. e The liver is the primary site of cholesterol
44% protein and 28% fat. Compare this with cow's synthesis from acetate.
milk, which contains about 26% protein and 36%
fat and bitch's milk, which contains about 35%
protein and 43% fat.
46. c Fatty acids wim no double bonds arc
simple carbohydrates.
w.
66. d Semimoist dog food co ntains large amounts of

67. a Low serum levels of these nutrients indicate


intestinal malabsorption.
saturated. Those with one or two double bonds 68. b Most home-cooked diets have high levels of
are unsaturated, whereas those with more than protein and fat but insufficient vitamin A and
ww
!\vo double bonds are polyunsaturated. copper.
47. c The jejun um is the longest part of the small 69. a Cocoa and related products (chocolate)
intestine and fi Us the mid-abdomen. contain theobromine.
48. b In good-quality diets. fats are usually digested 70. a A 13.6-kg dog requires 900 to 1000 kcal of
wilh more than 90% efficiency in dogs and cats. metabolizable energy per day and so would
require two cans of this food.

e 1998 Mosby-Year Book, Inc. Photocopying is prollibited by law.


www.alkottob.com
154 SECTION 6
www.vet4arab.co.cc SECTION

NOTES
7

om
Pharmacology
T.P. Clark, L.E. Davis, S.H. Duran, J.E. Riviere

b.c
Recommended Reading
Adams HR: Veterinary pharmacology and therapeutics, cd 7, Ames, Iowa, 1995, Iowa State
University Press.
Allen DG: Handbook o!veterinary drugs, Philadelphia, 1993, Lippincott,
Barragry TC: Veterinary drug therapy. Baltimore. 1994, Williams & Wilkins.

tto
Duran SH , Lin He: Mosby's IIf!rcrinarydrug re/erenre, 51 Louis, 1998, Mosby.
Ha rdman IG et aI: Goodman alld Gilman's rIle pJwrmarologica/ basis olrlierapeutics, cd 9, New
\ork, 1996, McGraw-Hill.
Me lmon KL et al: Melmon and Morrelli's clinical pharmacology: basic principles in therapeutics.
cd 3, NcwYork, 1992. McGraw·HilI.
Physicians'desk reference, Oradell, NJ. 199B, Medical Economics.
PI!}'sician's genRx, 5t Louis, 1998, Mos by.

o
Plumb DC: Veterinary drug handbook, ed 2, Ames, Iowa, 1995, Iowa Stale University Press.
Prescott IF. Baggol ID: Antimicrobial /hempy in veterinary medicine, cd 2, Ames, Iowa, 1993, Iowa
Slate University Press.

alk
Ve/erinQrypllllnnaceuticalsand biologicals 199711998, ed 10, Lenexa, Kan, 1997, Ve lerinary
Medicine Publishing.

Practice answer sheets are on pages 267-268.

w. Questions
T.P. Clark
1. A partial agonist is a drug tllat: 2. Five drugs are known to bind to a,-adrenergic
receptors. Using radiolabeled drug-binding
a. blocks the response produced by an agonist
studies, equilibrium dissociation constants (KrJ
b. produces a maximal response by occupying are determined for each of rile drugs. Which
all or a fraction of the receptors hypothetic drug has the highest affiniry for tile
ww
c. produces a half-maximal effect aI-adrenergic receptor?
d. produces less than a maximal response even
a. drug 1_ KD = 10 nanomolar
when it occup ies all of the receptors
b. drug 2 _ Ko = 55 nanomolar
e. produces half-maximal inhibition
c. drug 3 _ ~ = 0.21 micromolar
d. drug 4 _ Ko = 5.5 micromolar
e. drug 5 _ Ko = 0.12 molar

01998 Mosby-Year Boolc,lnc. Photocopying is prohibited by law. Correct answers are on pages 178· 184. 155
www.alkottob.com
156 SECTION 7
www.vet4arab.co.cc Pharm4cology 157

3. A newllnaigesic compound is a weak acid (pK" = 6. Drug X and drug Y have identical whole-body 10. A" epinephrine dosage of0.01 mg/kg;s indicated 15. W1lnt is a primary metaboliteo/methoxyfluralle.
4) and must penetrate the blood-brain-barrier to clearance rates (4 ml/min· kg). However. tile ;n a dog. Using a J:1000 epinephrine solution, and where does this metabolite exert its toxicity?
exert its effects. WI,e" used in a dog Wilh plasma eliminatioll/ialf-lifefor drngX is I hour; how sllOuld tliis beadmjllistered toa 2S-kgdog?
a. bromine; kidney

om
respiratory acidosis (blood pH 6.8), how should whereas the plasma elimination half-life for drug
Y is 48 hours. W1Jat accounts for these drugs
a. DiluteO.1 ml of the l:IOOOsolution to I mI, b. bromine; liver
the drug dose be adjusted. compared with the
and inject 0.25 ml. c. fluorine; kidney
dose givell to a normal dog (blood pH 7.4), to havillg identical whole-body clearance?
achieve tiJerapemic levels ill the brai,,? b. Dilute 0.1 mJ of the 1:1000 solution to 10 mJ.
a. Drug X is metabolized. and drug Y is not. d. fluorine; liver
and adm.inister 0.25 mt
a. Increase the dose because more of the drug is b. Drug X is eliminated from the kidney and e. methanol; centraJ nervous system
c. Dilute 0.1 Illl of the 1:1000 solution to 100 mi.
ionized. drugY in the bile. and administer 0.25 mJ.
b. Decrease the dose because more of the drug is c. Drug X has a smaller volume of distribution d. Administer 0.25 ml of the I: 1000 solution. 16. W1Jat effects does halothane hatJe all the
ionized. (han drugY. c. Administer 2.5 ml of the 1:1000 solution. cardiovascillar system?

b.c
c. No dose adjustments are necessary. d. Drug X is more lipophilic than drugY. u. depresses myocardiaJ contractility but
d. Increase the dose because more of the drug is e. Drug X is an acid, and drug Y is a base. maintains normal blood pressurc by
11. Epinephrine is used with local anesthetics
nonionized. s timulating the sympathetic nervous system
because a/its;
e. Decrease the dose because more of the drug is 7. Drug X undergoes phase-[J metabolism by being b. maintains normal blood pressure by
nonionized . a. a-adrenergic receptor agonist propenies stimulating the baroreceptor rcflex
conjugated with glucuronic acid. Drug X carmot
be used ill which species because the glucurollide b. 132-adrenergic receptor agonist properties c. protects the heart from the arrhythmogenic
4. A cow has mastitis caused bya Staphylococcus sylllhetic pathway functions at a very slow rate? c. 131 -adrenergic receptor agonists properties effects of epinephrine
species. Based on bacterial culture and sensitivity d. muscarinic receptor agonist properties d. can induce spontaneous arrhythmias,
a. horses

tto
testillg. the following antibiorics are indicated. c. nicotinic receptor agonist propenies especially at light surgical anesthesia (stage
Based on rile prillciples o/ioTlic trapping, which b. cattle
III, plane I)
one will achieve the hig11est concentration ill milk c. pigs e. causes dose-dependent cardiomyopathy
d. calS 12. Wliich drug would potentiate the effects 0/
(pH 6.8) when used systemically (blood pH 7.4j?
succillylc/lOlille?
a. cloxacillin (acid, pK. 2.7) e. dogs
a. methacholine 17. W11a' is a primary metabolite a/halothane. and
b. ampicillin (acid, pK. 3.4) IlOw is tile toxic effect o/this m etabolite
8. A drug tllat mimics parasympatheticstimulatioll b. neostigmine
c. cephaloridine (aCid. pK. 4.4) l7lallifested?

o
o/thegastrointestinal tract would cause: c. atropine
d. sulfadimethoxine (acid. pK. 6.1)
e. erythromycin (base. pK. B.B) ..
b.
increased motility and sphincter constriction
decreased motility and sphincter constriction
d. scopolamine
c. pilocarpine
a.
b.
bromine: lethargy and ataxia
bromine; renal damage

alk
c. chlorine; metabolic alkalosis
5. A highly polar, parenterally administered alltibiotic c. increased motility and sphincter relaxation d. fluo rine: hepatic damage
13. Tile effect a/an intravenous injection of
lias a sllort plasma eliminatioll half-life because 0/ d. decreased motility and sphincter relaxation e. fluorine; re naJ damage
rapid renal excretion. Peak blood levels after a acetylcholine all blood pressure call be blocked by:
e. no effect
single injection are 16 W1ml. l/tlleal1tibiotic has a a. metoclopramide
Iialflife 0/ 1.5 1I0urs. what would the predicted 18. All tlle/ollowing are associated with routi/Ie.
9. What is the order of potency/or I3radrenergic b. propranolol
blood levels be 7.5 hours after an injection? properly administered halothane anesthesia
receptors, from most potent to least potent? c. d-tubocurare except:
a. 12fJ.g /ml d. atropine
a. norepinephrine> isoproterenol > epine phrine
w. a. transient decrease in the ability to
b. 8 ILg/ ml c. nitroprusside
b. isoproterenol > norepinephrine> epinephrine concentrate urine because of mild renal
c. 3IJ.g/ ml hypoxia
c. All have equal potency.
d. 1.5ILg/ml 14. III dogs tile vasodilator effect 0/ a low-dosage b. transient increase in blood urea nitrogen and
d. epinephrine> norepinephrine> isoproterenol
e. 0.5 jl.g/ml infusion a/dopamine Oil the renal vasculature is creatinine because of decreased glomerular
e. isoproterenol> epinephrine> norepinephrine caused byacrivotioll of filtration rate during anesthesia
c. transient liver damage because of mild
ww
a. dopamine receptors
b. a-adrenergic receptors hepatic hypoxia
c. 132-adrenergic receptors d. increased cerebral blood flow
d. muscarinic receptors e. decreased intracranial pressure
e. nicotinic receptors

C 1998 Mosby-Year Book, Inc. Photocopying is prollibited by law. Q)rrecranswers are on pages 178- 184.
www.alkottob.com
158 SECTION 7
www.vet4arab.co.cc Pharmacology 159

19. Concerning nitrous oxide, which statement is d. distribution across the placenta and into fetal 27. Concerning primidone, which statement is most 31. Which of the following most accurately explains
most accurate? tissue accurate? why second-generation competitive H, receptor-
e. prolonged recovery in obese animals blocking drugs do not cause drowsiness?
a. Most dogs can be successfulJy anesthetized a. It is metabolized to phenobarbital and

om
using only a mixture of80% nitrous oxide and phenylethylmalonamide. a. They compete for H2 receptors as well as HJ
20% oxygen. 23. What is a cardiovascular side effect of most b. It is the anticonvulsant drug of choice in cats. receptors.
b. Nitrous oxide causes distention and increased barbit1lrates? c. It is a potent inhibitor of hepatic microsomal b. They increase cardiac output and oxygen
pressure inside any closed, air-containing enzymes. levels in the brain.
a. bradycardia in dogs, caused by stimulation of
space within the body. d. It causes none of the side effects reported with c. They have serotonin agonist properties.
the vagus nerve
c. Nitrous oxide slows uptake of other anesthetic phenobarbital. d. They have caffeinelike effects.
b. tachycardia in dogs, secondary to peripheral
gases from the alveoli to the blood by e. Serum primidone levels can be used to guide e. They are less lipophilic and do not cross the
competing for anesthetic transport molecules. vasodilation and decreased arterial pressure
c. prevention of catecholamine-induced dosage regimens. blood-brain barrier.

b.c
d. Because of its excitatory effects on the central
arrhythmias
nervous system, nitrous oxide use increases
the dose of other anesthetic agents required d. increased mean arterial pressure 28. Conceming phenytoin, which statement is most 32. Which drug would be most appropriate to give
to attain surgical anesthesia. c. increased cardiac output accurate? before surgical removal of a mast-cell tumor in
e. Nitrous oxide decreases activity of the a dog?
a. It is 100% bioavaiJable.
sympathetic nervous system and therefore 24. Mlich one of the following most accurately b. In dogs its elimination half-life is longer than a. epinephrine
has desirable antiarrhythmic effects. describes ketamine anesthesia? that of phenobarbital. b. prostaglandin F2n
a. Ketamine causes rapid induction to stage III c. It induces hepatic microsomal enzymes. c. cimetidine

tto
20. Prop%l causes centrallleruous system of anesthesia. d. It is metabolized to phenobarbital and d. neostigmine
depression and: b. Peripheral reflexes are lost at surgical planes phenylethylmalonamide. e. aspirin
a. is an anticonvulsant of anesthesia. e. It is the anticonvulsant drug of choice in cats.
b. does not cause respiratory depression c. Ketamine has a wider therapeutic index 33. Omeprazole blocks gastric acid secretion by:
c. decreases intracranial and intraocular (LD 50 / ED 50 ) than barbiturates. 29. Benzodiazepines exert their amicorlvulsam
a. competitively blocking gastrin receptors
pressure d. Ketamine is a potent visceral analgesic. effect by:
b. competitively blocking H2 histamine

o
d. is metabolized in the liver e. Ketamine induces potent muscle relaxation. a. blocking neuronal fast sodium channels receptors
e. is a glutamate receptor agonist b. enhancing the inhibitory effect of GABA c. noncompetitively blocking acetylcholine
25. Concerning ketamine, which statement is least c. inhibiting the excitatory neurotransmitter receptors
accurate?

alk
21 . Barbiturates produce anesthesia through: glutamate d. inhibiting the gastric parietal cell H+-
a. Ketamine is derived from phencyclidine. d. decreasing influx of calcium into neurons K+ATPase pump
a. increased synaptic transmission
b. Ketamine is a GABA agonist and glutamate e. blocking neuronal potassium channels e. inhibiting gastrin release from pyloric glands
b. increased seroton in release
receptor antagonist.
c. action at the -y-aminobutyric acid (GABA)
receptor (GABA mimetic) c. Ketamine stimulates £he cardiovascular 30. Concerning use ofbromide as an anticOlllmlsant, 34. Effects of antiinflammatory doses of
d. depression ofthe thalamoneocortical system system through a sympathomimecic effect. which statement is most accurate? glucocorticoids include all the following except:
in conjunction with activation of the limbic d. Ketamine increases cardiac output.
a. It hyperpolarizes neurons by replacing a. binding membrane glucocorticoid receptors
system e. Ketamine is excreted in the bile; therefore
w. intracellular chloride. b. inducing cellular lipocortin synthesis
e. decreased oxygen uptake by neurons poor renal function does not preclude its use.
b. Its elimination half-life is 7 days. c. interfering with cellular phospholipase
c. Without a loading dose, steady-state activation
22. All thefollowing are associated with 26. Concerning phenobarbital, which statement is dynamics can be reached within I week. d. decreasing cellular leukotriene synthesis
thiobarbiturates except: most accurate? d. It has no side effects. e. decreasing cellular prostaglandin synthesis
a. rapid distribution across the blood-brain a. It is metabolized by hepatic microsomal e. It competitively blocks fast sodium channels.
ww
barrier enzymes.
b. initial rapid distribution to highly perfused b. It is not metabolized but rather is excreted
tissues, followed by redistribution to other unchanged in the urine.
tissues c. Long-term therapy with phenobarbital
c. rapid recovery because of redistribution of the decreases its rate of metabolism.
anesthetic into adipose tissue d. It is a potent inhibitor of hepatic microsomal
enzymes.
e. It is excreted in the bil e.

© 1998 Mosby- Year Book, fllc. Photocopying is prohibited by mw. Correct answers are on pages 178-184.
www.alkottob.com
160 SECTION 7
www.vet4arab.co.cc Pharmacology 161

35. Concerning aspirirl, which statement is most d. It is a synthetic compound not present in 44. Digitalis therapy is beneficial in treatingarrial 49. Angiotensin-converting enzyme (ACE) inhibitors
accurate? normal animals. fibrillnrion because it: may cause renal failure in dogs wilh congestive
e. It requires calcium to exert anticoagulant heart/ailure through:
a. After absorption, it is metabolized to salicylic a. decreases the refractory period in atrial

om
acid.
activity. muscle a. afferent glomerular arteriolar constriction
h. It is excreted unchanged in the bile. b. slows atrioventricular conduction b. afferent glomerular arteriolar dilation
c. It is a weak base with a pKa of 8. 40. A dog has difficulty breathing secondary to c. converts the rhythm to normal sinus rhythm c. efferent glomerular arteriolar constriction
pulmonary thromboembolism. Which treatment d. increases atrial muscle contractility d. efferent glomerular arteriolar dilation
d. There is no difference in oral absorption of
would have the most rapid effect in preventing
regular and buffered aspirin. e. blocks sympathetic input to the heart e. constriction of both afferent and efferent
furrherdeposition offibrin in the clot?
e. It blocks leukotriene synthesis. glomerular arterioles
a. intravenous heparin 45. In which situation is digitalis contraindicated?
b. oral synthetic coumarin derivative

b.c
36. In which species does aspirin have the longest a. congestive heart failure 50. Which of the following does not increase the risk
elimination half-life? c. intravenous protamine sulfate of renal failure associated with angiolensin-
d. fresh whole blood transfusion b. first- or second-degree atrioventricular block converting enzyme (ACE) inhibitor therapy?
a. ruminants c. atrial fibrillat ion and atrial flurter
e. plasma transfusion a. hypovolemia
h. horses d. supraventricular tachycardia
c. dogs e. concurrent use with diuretics b. vomiting
d. cats 41. Cardiac glycosides exert their cellular effects on c. hypotension
the heart by: d. diuretic therapy
e. There are no species differences in 46. Which drug dilates blood vessels by competitive
elimination half-life. 3. blocking fast sodium channels antagonism of(XI-adrenergic receptors?
e. oxygen therapy

tto
b. inactivating troponin and tropomyosin
c. inhibiting membrane vohage-dependent a. nitroglycerin
37. Which drug inhibits the lipoxygenase pathway as 51. Conceming enalnpril. which statement is most
calcium channels b. nitroprusside sodium accurate?
well as the cyclooxygenase pathway?
d . inhibiting membrane sodium-potassium c. prazosin
a. acetaminophen d. diltiazem a. Enalapril is the active form of the drug.
ATPase
b. aspirin e. captopril b. It is associated with a Significant first-pass
e. inhibiting membrane calcium ATPase
c. flunixin effect.

o
d. naproxen c. It has a more rapid onset of action than
42. At therapeutic levels. cardiac glycosides produce 47. The intravenous infusion rate/or nitroprusside is captopri!.
e. ketoprofen all the following effects except: 5/Jog/kg/min. Using the 35 jJ,g/ml solution and a
d . It has a longer duration of action than captopril.
15 drop/ml drip set, how many drops per minute

alk
a. increased cardiac output at all levels of e. It is not approved by the U.S. Food and Drug
38. A dog has a recent history of spontaneous should beadministered to a 22-lb dog?
ventricular end-diastolic volume Administration (FDA) for use in dogs.
epistaxis, ecchymosis, and petechial hemorrhage a. 12 drops/min
secondary to warfarin i1ltoxication. Which drug b. reflex withdrawal of sympathetic vasomotor
is most appropriate for therapy? lone (vasodilation) b. 21 drops/min 52. Concerning the action ofverapamil on the heart.
c. increased myocardial oxygen consumption c. 47 drops/min which statement is least accurate?
a. heparin d. 84 drops/min
d. slowed heart rate
b. vitamin K] (phytonadione) a. It is a negative inotrope.
e. slowed atrioventricular conduction e. 116drops/min
c. vitam in K2 (menaquinone) b. It is a negative chronotrope.
d.
e.
vitamin K3 (menadione)
protamine sulfate
in patients with heart failure?
w.
43. How does digitalis therapy increase urine output 48. Blood volume is reduced after therapy with
angiotensin-converting enzyme (ACE) inhibitors
because of
c. It speeds conduction through the
atrioventricular node.
d. It decreases the rate of sinoatrial discharge.
a. direct blockade of sodium resorption in the e. It decreases coronary vascular resistance.
39. Concerning heparin, which statement is most a. inc reased aldosterone secretion
proximal tubule
accurate?
b. competition for antidiuretic hormone b. decreased aldosterone and antidiuretic 53. What is the predominant mechanism ofaction of
ww
a. When given intravenously, it can directly receptors in the collecting duct hormone secretion class-1 antiarrhythmics?
break down an existing clot. c. stimulation of central thirst centers. with c. increased aldosterone and antidiuretic
b. It requires antithrombin III to have functional secondary polydipsia hormone secretion a. f3-adrenergic receptor blockade
anticoagulant activity. d. secondary to increased renal perfusion d. decreased antidiuretic hormone secretion b. calcium channel blockade
c. It prevents synthesis of some clotting factors because of improved hemodynamics e. decreased renal potassium reabsorption c. potassium channel blockade
by the liver. e. renal medullary washout d. sodium channel blockade
e. sodium-potassium ATPase blockade

© 1998 Mosby-Year Book, Inc. Photocopying is prohibited by law. Correct answers are on pages 178-184.
www.alkottob.com
162 SECTION 7
www.vet4arab.co.cc Pharmacology 163

54. What is the predominant mechanism afaction of c. competitive inhibition of antidiuretic 62. What is the mechanism ofaction offurosemide? 67. Concerning the effects of narcotic antitussive
class-3 antiarrhythmics? hormone receptors agents, which statement is least accurate?
a. competitive inhibition of antidiuretic
a. l3-adrenergic receptor blockade d. inhibition of chloride transport in the thick hormone receptors a. They cause constipation.

om
ascending loop of Henle
h. calcium channel blockade b. decreased bicarbonate resorption in the b. They depress peripheral cough receptors in
e. decreased sodium resorption in the distal tubule the bronchi.
c. potassium channel blockade tubule
d. sodium channel blockade c. competitive inhibition of aldosterone c. They are indicated for nonproductive cough .
e. sodium-potassium ATPase blockade receptors d. They are contraindicated when copious
59. Aside from diuresis, carbonic anhydrase d. inhibition of chloride transport in the thick respiratory secretions are present.
inhibitors result in: ascending loop of Henle e. They can cause respiratory depression.
55. What is the major electrophysiologic effect of
a. hyperchloremic metabolic alkalosis and e. decreased potassium secretion in the
quinidine on atrial and lIelUriwlar muscle that
alkaline urine collecting duct

b.c
accounts for its antiarrhythmic properties? 68. Sulfollamides are strucfUra/ analogs of
b. hyperchloremic metabolic acidosis and
a. decreased phase 0 depolarization alkaline urine a. tetrahydrofolate reductase
63. Effects ofmetoclopramide include all the
h. decreased phase I depolarization c. hypochloremic metabolic alkalosis and acidic following except: b. folic acid reductase
c. decreased phase 2 depolarization urine Q c. para-aminobenzoic acid (PABA)
a. increased lower esophageal sphincter tone
d. decreased phase 3 repoiarization d. hypochloremic metabolic acidosis and d. pteridine
b. increased force of gastric contraction
e. increased spontaneous phase 4 alkaline urine e. glutamic acid
depolarization c. dopamine receptor antagonism
e. hyperchloremic metabolic alkalosis and acidic
urine d. increased colonic motility

tto
69. The 2,4-diamillopyrimidille compounds,
e. peripheral antiemetic action
56. In which condition is use ofmannirol trimerhoprim and ormetoprim, exert their
contraindicated? 60. What is the mechanism ofaction of thiazide antimicrobial effecrs through:
diuretics? 64. Which antacid increases the risk ofgastric ruplllre?
a. acute oliguric renal failure a. competitive inhibition of dihydrofolate
b. intracranial hemorrhage a. competitive inhibition of antidiuretic a. aluminum hydroxide reductase
c. acute glaucoma hormone receptors b. magnesium hydroxide b. decreased dihydropteridine uptake
d. cerebral edema b. decreased sodium and chloride resorption in c. magnesium sulfate c. competitive inhibi('ion or para-aminobenzoic

o
e. increased intracerebral pressure the distal tubule d. sodium bicarbonate acid
c. competitive inhibition of aldosterone e. sodium sulfate d. irreversible binding to transpeptidase
receplOrs e. irreversible binding to the 305 ribosomal
57. By what mechanism does ammonium chloride

alk
d. inhibition of chloride transport in the thick subunit
cause diuresis? 65. Concerning apomorphine, which statement is
ascending loop of Henle
least accurate?
a. increased tubular ammonium load and e. increased potassium secretion in the
a. Its emetic effects are countered by opiate 70. Sulfonamides may produce any of the following
increased osmotic force in the tubular lumen collecting duct side effects except:
b. sodium channel blockade in the proximal antagonists.
tubule b. It is a synthetic derivative of morphine. a. hypoprothrombinemia
61. What is the mechanism ofaction of
c. decreased chloride resorption in the loop of c. It exerts its effects at the chemoreceptor b. keratoconjunctivitis sicca
spirorlOlactone?
Henle trigger zone. c. aplastic anemia and thrombocytopenia
d. ionic interference with aldosterone receptor
binding in the distal tubule and collecting
duct
hormone receplOrs
w.
a. competitive inhibition of antidiuretic

b. decreased sodium and chloride resorption in


the distal tubule
d. It is a dopamine agonist.
e. It produces emesis within 20 minutes of
administration.
d.
e.
yellow staining of dental enamel
hypothyroidism

e. increased glucose secretion in the proximal


c. competitive inhibition of aldosterone 71. What is the mechanism ofactio1l ofpenicillills?
tubule 66. Jmestinal effects ofanticholinergics include all
receptors a. irreversible binding to cell wall porins
the following except:
ww
d. inhibition of chloride transport in the thick b. irreversible binding to the 505 ribosomal
58. By what mechanism do carbonic anhydrase a. decreased intestinal secretions
ascending loop of Henle subunit
inhibitors cause diuresis?
e. increased potassium secretion in the b. decreased segmental intestinal movements c. prevention of peptidoglycan synthesis by
a. increased potassium secretion in the collecting duct c. decreased propulsive intestinal movements binding to transpeptidase
collecting duct d. antagonism of the purgative effects of d. interference with folic add synthesis
b. competitive inhibition of aldosterone neostigmine e. competitive inhibition of dihydrofolate
receptors e. emesis reductase

© 1998 Mosby-Year Book. Inc. Photocopying is prohibited by law. OJrrect answers are on pages 178-184.
www.alkottob.com
164 SECTIO N 7
www.vet4arab.co.cc Pharmacology 165

72. Concerning penicillin G, which statement is most d. acute collapse following intravenolls injection 79. Con cerning metabolism and disposition of 83. Mlich of the following is a toxic effect of
accurate? e. pancytopenia erythromycin, which statement is least accurate? /luoroqllinolones?
a. degenerative chondrocyte changes in young

om
a. Its spectrum of activity is limited to gram- a. Oral erythromycin base must be enteric
negative bacteria. 76. What is llJeantimicrobial mechanism ofaction of coated to prevent acid-induced inactivation. animals
b. It is resistant to 13-lactamases and is effective chloramphenicol? b. It is well distributed to the brain and b. staining of dental enamel
against Staphylococcus species. cerebrospinal fluid. c. cardiotoxicity and arrhythmia
a. binding 10 the bacte rial 50S ribosomal
c. II is rapidly destroyed by gastric acid. subunit and inhibition of peptidyltransferase c. It is primarily excreted in the bile as an d. fatal d iarrhea
d. It is combined with procaine to prevent pain activity inactive metabolite. e. pancytopenia
at the injection site. b. reversible binding to the bacterial 50S d. It must be administered 3 to 4 times daily
e. It is a semisynthetic penicillin. ribosomal subunit and inhibition of because of its shon half-life.
84. Concem ingaminoglycosideantibiotics, which
ribosomal translocation e. It causes severe diarrhea in horses.

b.c
statemeTll is most accurate?
73. eoncem ingcepltaiosporins, which srateme1lt is c. binding to the bacterial30S subu nit, resulting
in interference with aminoacyl tRNA binding a. They are highly lipid soluble.
least accurate? 80. Which drug should not be placed in an
to the acceptor (A) site automatic, powered syringe for injection because b. They easily cross biologic membranes.
a. They are bactericidal. c. They are poorly absorbed following oral
d. irreversible binding to the bacterial 305 and of the risk of cardiovascular roxicity from
b. They penet'rale gram- negat ive bacteria better possibly 50S subunit, resulting in abnormal accidental self-injection? ad ministration.
than penicillins. codon:anticodon recognition d. They are highly metabolized in the liver.
c. They are more s usceptible to j3-lactamases a. tylosin
e. inhibhion of DNA synth esis by disruption of e. They easily cross the blood-brain barrier.
produced by Staphylococcus species than are b. e rythromycin
DNA gyrase activity

tto
penicillins. c. lincomycin
d. T hey are produced from Cephalosporium d. tilmicosin 85. Mlat is the antimicrobial mechanism of action of
acremonium. 77. All the following are associated with aminogiycosides?
chloramphenicol toxicity except:
e. clindamycin
e. They interfere with peptidoglycan synthesis. a. binding to the bacterial 50S ribosomal SUbWlit
a. reversible dose·dependent bone marrow and inhibition of peptidyltransferase activity
81. Which drug causesfaral diarrhea secondary 10
suppression b. reversible binding to me bacterial 50S
74. WI/ar is theamimicrouiaf mechanism a/action of Clostridium overgrowth in people, horses. and
b. reduced gonadal function in males and ribosomal subunit and inhibition of

..
tetracyclines? rabbits?

o
females ribosomal translocation
a. binding to the bacterial 50S ribosomal c. dose-dependent leukopenia in neonates tylosin c. binding to the bacterial 30S subunit. resulting
subunit and inhibition ofpeptidyhransferase b. erythromycin in interference with aminoacyl tRNA binding
d. irreversible dose- independent bone marrow
activity

alk
suppression c. lincomycin to the acceptor (A) site
b. reversible binding 10 Ihe baClerial 50S d. tilmicosin d. irreversible binding to the bacterial 305 and
e. cholestasis and arthropathy
ribosomal subunit and inhibition of possibly 50S subunit, resulting in abnormal
ribosomal translocation
e. clindamycin
codon:anticodon recognition
c. binding to the bacterial 305 subunit. resulting 78. Mlat is the Q/ltimicrohialmechallism of action of
e. inhibition of ON A synthesis by disruption of
in interference with aminoacyl tRNA binding erythromycin? 82. WIJar is the antimicrobial mechanism ofaction of
DNA gyrase activity
to the acceptor (Al si le /luoroquinololle antibiotics?
a. binding to the bacterial 50S ribosomal
d. irreversi ble binding to the bacterial30S and subunit and inhibition of peptidyltransferase a. binding to the bacterial 50S ribosomal
possibly 50S subunit. resulting in abnormal w. 86. COflcemillgaminogiycoside-induced
activity subunit and inhibition ofpeptidyltransferase neuromuscular blockade, which statemem is
codon:anticodon recognition b. reversible binding to the bacterial 50S activity least accurate?
e. inhibition of DNA synthesis by disruption of ribosomal subunil and inhibition of b. reversible binding to the bacterial 50S
DNA gyrase activity ribosomal translocation ribosomal subunit and inhibition of a. It results from inhibition of prejunctional
c. binding to the bacterial30S subunit. resulting ribosomal translocation ace£y1choline release.
75. Possible toxic effects of tetracycline include all tile in interference with aminoacyl tRNA binding c. binding to the bacterial 305 subunit. resulting b. Paralysis of the diaphragm has been
associated with intrapleural injections.
ww
following except: to the acceptor (A) site in interference with aminoacyl tRNA binding
d . irreversible binding to the bacterial30S and 10 the acceptor (A) s ite c. Of all aminoglycosides, neomycin is most
a. tympanitis or otitis media likely to cause neuromuscular blockade.
possibly 50s s ubunit. resulting in abnormal d. irreversible binding to the bacterial 305 and
b. staini ng of dental enamel codon:anticodon recognition pOSSibly 50S subunit. resulting in abnormal d. Calcium infusion can reverse neuromuscular
c. photo(Qxicity and photosensitization e. inhibition of ON A synthesis by disruption of codon:anticodon recognition blockade.
DNA gyrase activity e. inhibition of DNA synthesis by disruption of e. Aminoglycosides competitively inhibit
DNA gyrase activity nicotinic receptors.

CI 1998 Mosby-Year Book. Inc. Photocopying is pro/libited by law. UJrrect answers are on pages 178-184.
www.alkottob.com
166 SECTION 7
www.vet4arab.co.cc Pharmacology 167

87. W1llch aminoglycoside is most nephrotoxic? 92. Cyclic amines, such as amantadine and 96. Concerningcyc/opJlOsphamide, which statement c. melphalan
rimantadine, exert their antiviral activity by: is most accurate? d. thiote pa
a. gentamicin
e. carmustine

om
b. kanamycin a. blockade of viral fusion and release a. It cannot be administered orally.
c. neomycin b. incorporation into growing mHNA b. It must undergo hepatic activation.
d. s treptomycin c. inhibition of RNA polymerase c. It is a severe vesicant. 99. \oWlic/1 drug has been associated with direct
e. amikacin d. inhibition of viral e nzymes d. It is cardiotoxic. mast-cell degra"uitltion, independent of IgE?
e. inhibition of DNA polymerase in e. It is cell cycle phase specific. a. cytosine arabinoside
herpes viruses b. taxol
88. Concerning vancomycin, which semement is most
accurate? 97. WlII·cll drug is particu/arlyeffective in treating c. vincristine
93. Amiviml 'Iucleoside analogs are usually in the cllro"icgranuiocytic leukemia? d. bleomycin
a. It is bacteriostatic.

b.c
form of prod rugs and must be activated by: e. doxorubicin
b. Its use should be limited to penicillin- a. mechlorethamine
resistant Staphylococcus infections. a. incorporalion iOlo growing polypeptide b. cyclophosphamide
c. Only oral formulations are used clinically. chains c. melphalan 100. Which drug has acute cardiac arrest and
d. It is hepatotoxic. b. phosphorylation d. busulfan cardiomyopatllyas toxic effects?
e. It is eliminated through biliary excretion. c. glycosylation e. carmustine a. etoposide
d. incorporation into viral DNA
b. doxOfubicin
e. protease inhibitors
89. Concerning rilampin, which statement is most 98. Which drug is characterized bya high degree of c. cisplatin

tto
accurate? lipid solubilitya"d is used to treat brain and d. l- asparaginase
94. eoncerning amphotericin B, which statement is meningeal tumors? e. tamoxifen
a. It is highly lipid soluble.
most accurate?
h. It is degraded in an acidic environment. a. chlorambucil
c. It does not penetrate neutrophils or a. Alopecia is the most serious common adverse b. busulfan
macrophages. effect and is self-limiting.
d. Its spectrum of activity is limited to gram- b. Nephrotoxicity is the most serious common
adverse effect and is predictable and dose

o
negative bacteria.
related. l.E. Davis
e. It should not be used in horses.
c. Nephrotoxicity is the most serious common 101. Large doses ofcorticosteroids may produce: d. urticaria
adverse effect, is not reversible, and is usually
e. hepatic lipidosis

alk
90. Concern ing melTonidaz<)le, which statement is fatal. a. nephritis
kast accurate? d. Hepatocellular toxicity is the most serious b. hepalOpathy
a. It is effective against anaerobic bacteria. common adverse effect and is more severe in c. encephalomalacia 104. Prochlorperazine is least effective in prevention
dogs than in cats. d. neutropenia ofuomitingassociated with:
b. It has an extended antimicrobial spectnun
against protozoa, trichomonads, and e. Hepatocellular toxicity is the most serious e. amaurosis a. vestibular stimulation
amoebae. common adverse effect and is usually fatal. b. uremia
c. It does not penetrate the central nervous 102. Excretion of drugs by the kidneys is: c. gastroenteritis
system. 95. eoncerningantifungals and their effects on
w. d. stimulation of the chemoreceptor trigger zone
d. It is potentially mutagenic and teratogenic. steroid biosymilesis. which statement is most a. unaffected by variation in renal blood flow
e. radiation sickness
e. It is clinically useful in hepatic accurate? b. generally more rapid if the dmg is not
encephalopathy. extensively protein bound
a. Ketoconazole is also used to treat 105. Urinary pH might be intentionally altered as
c. more rapid with dmgs that are actively
hyperadrenocorticism. part ofa therapeutic regimen to accomplish all
secreted by tubular cells
91. Which class ofantiviral compounds prevents h. Altered steroid biosynthesis is considered a the following except to:
d. always increased by administration of diuretics
attachment ofviruses to "receptors" on tile host cell? serious adverse effect of miconazole.
ww
e. a minor pathway for elimination of dmgs a. prevent formation of some calculi
a. nucleoside analogs c. Ketoconazole causes increased testosterone
production . b. decrease irritant properties of urine
b. interferons c. increase the effectiveness of certain
d. Potassium iodide causes hypothyroidism. 103. A toxic manifestation ofdibucai"e toxicity is:
c. cyclic amines chemotherapeutic drugs
e. ltraconazole causes diuresis. a. agranulocytosis
d. synthetic pep tides d. modify the pH of extraceliuJar fluid
e. interleukins b. coma
e. alter the rate of urinary excretion of another
c. seizures drug

C 1998 Mosby-Year Book, Inc. Photocopying is prohibited by law. Correct answers are on pages 178- 184.
www.alkottob.com
168 SECTION 7
www.vet4arab.co.cc Pharmacology 169

106. Morphine sulfate: c. an emetic response 117. Burow's solution is employed in wet dresSings d. can cause renal necrosis
d. suppression of conditioned avoidance behavior because of its: e. is expensive to use
a. stimulates the respiratory center
e. hypertension when given in large doses

om
h. dilates the bronchioles a. emollient effect
c. depresses the spinal cord b. depilatory action 123. Overactivityand intolerance to restrairll
d. stimulates intestinal smooth muscle 112. Long-term cllronic administration ofbarbiturates c. demulcent effect (hyperkinesis) in a dog are appropriately
to an epileptic dog may reduce the respome to d. astringent action created with:
e. dilates the pupil in dogs
otller anticonvulsant drugs because of e. effect on pigmentation a. acepromazine
107. You wish to dilute strong tincture of iodine (7%) a. tolerance caused by decreased receptor b. methylphenidate
with 70% ethanol to prepare 4 oz a/tincture of sensitivity 118. Which of tile following is not produced by c. phenytoin
iodine USP (2%) for use on small animals. How b. increased rate of excretion glucocorticoid administration? d. mephenesin

b.c
much ethanol and 7% iodine must be used? c. accelerated rate of biotransformation e. valpro ic acid
a. increased gluconeogenesis
a. 3 fl oz of ethanol a nd 1 fl oz of 7% iodine d. increased volume of distribution
b. depressed protein catabolism
h. 50 ml of e thanol and 45 ml of 7% iodine e. impaired intestinal absorption 124. 1,25-DihydroxycI10Iecalciferol, the actilleform of
c. eosinopenia
c. 160 ml of ethanol and 80 m1 of7% iodine d. monocytopenia viUlmin 0, is synthesized in the:
d. 86 ml of ethanol and 34 ml of 7% iodine 113. What is tile effect of propantlieline on tile gut? e. neutrophilia a. liver
e. 45 ml of ethanol and 55 mI of 7% iodine a. spasmogenic b. skin
b. antimuscarinic and ganglioplegic 119. Which catl1artic may be given safely toa c. kidney

tto
108. Abnormal elimination behavior in a dog, c. stimula tes motility lactating sow nursing a litter of piglets? d. gut
caused by separation anxiety, may be effectively d. increases secretion by intestinal glands e. bo ne
a. cascara
treated with: e. evacuates the colo n b. aloe
a. estradiol c. castor oil 125. Heartworm disease can he prevented in dogs
b. amphetamine 114. An anticoagulant chat is suitable for systemic d. liquid petrolatum living in enzootic areas by administering, at
c. phenobarbital administration is: monthly intervals, the larvicidal drug:
e. croton oil
d. primidone

o
a. calcium EDTA a. dithiazanine
e. amitriptyline b. dicumarol b. ivermectin
120. Tile renal toxicity associated witl1 long-term
c. sodium fluoride sulfollamide therapy is: c. thiacetarsamide
J09. VitaminKis:

alk
d. thromboplastin d . diethylcarbamazine
a. related to water solubility at a particular
a. synthesized in the small intestine e. sodium oxalate urinary pH e. levamisole
b. found in significant quantity in cereal grains b. diminished by using a single sulfonamide
c. absorbed from th e gut in the absence of bile 115. Norepinephrine and phenylephrine decrease tile c. independent of urinary volume 126. A drug tllat has affinity for and inceracts with a
d. essential for thrombin formation in blood heart rate by: d. related to neural function cell receptor 10 elicit a response is called:
vessels a. directly depressing the cardioaccelerator e. least common with sulfathiazole a. an agonist
e. found in alfalfa as phytonadione center in the medulla b. a competitive antagonist

110. Which dog breed is most likely to exhibit an


idiosyncratic reaction to ivermectin?
b. decreasing peri pheral blood flow
c. activating a baroreceptor reflex
d. decreasing bronchial muscle tone
w.
e. exerting a direct negative inotropic effect
121. Aspirin:
a.
b.
has no antiinflammatory effects
produces hypothermia with overdosage
c.
d.
e.
an incomplete antagonist
a potentiating agent
an enzyme inducer
a. great Dane c. is eliminated more rapidly by dogs than by pigs
b. golden retriever d. is effective in reducing visceral pain 127. A cationic detergent is one in which the
c. dalmatian 116. Which eicosanoid induces aggregation of hydrophobic portion is positively charged. An
ww
e. inhibits cyclooxygenase
d. cocker spaniel placelets in tllecirwlation? example is:
e. collie a. prostacyclin a. sodium orthophenylphenate
122. Although halothane is a widely used inhalant
b. prostaglandin ~ anesthetic, a disadvantage is that it: b. tincture of green soap
111. Tranquilizers produce: c. leukotriene C4 c. benzalkonium chloride
a. can depress the medulla
d. thromboxane A2 d. sodium hypochlorite
a. anesthesia when given in large doses b. has powerful hypotensive effects
e. prostaglandin H2 e. sapinated cresol
b. decreased seizure activity c. is a respiratory depressant

© 1998 Mosby-Year Book, Inc. Photocopying is prohibited by law. OJrrect answers are on pages 178-184.
www.alkottob.com
170 SECTIO N 7
www.vet4arab.co.cc Pharmacology 171

128. Atropine exerts its main therapeutic effect at d. incompatibility 137. Which dassof drugs lowers the seizure 142. Use of which antimicrobial drug ill food-
receptors in the: e. drug interaction threshold and may induce extrapyramidal producing animals is specifically forbidden by
motor effects? U.S. Food and Drug Administration regulations?

om
a. central nervous system
h. neuromuscular junction 133. A drug that stimulates gastric emptying, a. phenothiazines a. oxytetracycline
c. autonomic ganglia relaxation of the pyloric sphincter, and increased b. benzodiazepines b. ampicillin
d . parasympathetic effector organs tone ofrhe esophageal sphincter is: c. monoamine oxidase inhibitors c. chloramphenicol
e. postganglionic parasympathetic nerves a. metodopramide d. butyrophenones d. sulfamethazine
b. a tropine e. tricydics e. tylosin
129. To dispense or prescribe controlled substances i" c. morphine
tlte U"itedStates, a veterinarian must be d. epinephri ne 138. Ifa drug has a b iologic half-life of 17 hours in 143. Rapid induction ofanesthesia by administration

b.c
licensed to practice and be registered with tile e. haloperidol cattle, approximately how long should the drug ofan inltalalll anesthetic is primarily caused by
Drug Enforcemem Administration (DEA), wi/jell be withheld before sillughter, so that the meat the low:
is a division of the: does not contain drug residue?
134. Furosemide indl~ces diuresis by: a. potency of the gas
a. U.S. Department ofJustice a. 17 hours b. partial pressure of the gas in the central
a. inhibiting carbonic anhydrase
b. U.S. Food and Drug Administration b. 8.5 hours nervous system
b. antagonizing aldosterone
c. U.s. Department of the Treasury c. 34 hours c . solubility of the gas in blood
c. inhibiting resorption of chloride in the loop
d . Customs Department d. 7 days d. rate of metabolism of the gas
of Henle
e. U.S. Department of Agriculture e. 85 hours e. respiratory rate

tto
d. decreasing permeability of the renal
collecting ducts
/30. Eicosanoid,s are involved in all the following e. inhibiting sod ium resorption in the proximal 139. Which cholinergic alkaloid is commonly used 144. Phenylpropanolamine is often effective in
pl1ysiologic /llnctiollS except: tubule topicaJJy to produce miosis? control of urinary incontinence because it:
a. regulation of heat production a. arecoline a . stimulates release of vasopressin by th e
h. regression of the corpus lu teum J 35. Bone marrow suppression produced by b. muscarine pituitary gland
methotrexate can be ameliorated by b. inhibits contraction oCthe detrusor muscle

o
c. blood dotting c. pilocarpine
d. renal failure administration of d. methacholine by blocking cholinergiC receptors
e. hepatic failure a. stanozolol e. neostigmine c. increases tone of the internal urethral
sphincter by stimulating a-receptors
b. daunorubicin

alk
d. decreases renal blood flow
131. Whitfield's ointment is used as a: c. leucovorin 140. In which species does salicylate exhibit the
e. blocks serotonin receptors in the hypothalamus
d. prednisolone shorlest elimination half-life?
a. protective
e. cobaltous chloride a. cattle
b. sunscreen 145. A drug used to treat atrial fibrillation is:
c. keratolytic b. rabbits
136. A horse that has been treated with chloral hydrate c. dogs a. aminophylline
d. depilatory
may subsequently show glucosuria because: d. cats b. lidocaine
e. disinfectant
w.
a. chloral hydrate stimulates release of e. people c. nitroglycerin
epinephrine from the adren al gland d. epinephrin e
132. The phenomenon caused by long-term
b. chlo ral hydrate decreases glucose u tilization e. procainamide
administration of certain drugs and 141. The best route for administration ofa drug that
characterized by hepatomegaly, increased by the tissues forms a very alkaline solution is:
prolein content of the liver, and increased c. the metabolite urochloralic acid is a reducing 146. Most drugs are absorbed into the circulation by:
substance a. intravenous
microsomal enzyme activity is known as:
b. subcutaneous a. facilitated d iffusion
ww
d. chloral hydrate decreases the renal threshold
a. induction c. intramuscular b. pinocytosis
for glucose
b. inhibition d. intraperitoneal c. simple diffusion
e. the renal tubules are damaged by chloral
c. enhancement hydrate e. intrathecal d. active transport
e. osmosis

© 1998 Mosby-Year Book, Inc. PllOtocopying is prohibited by law. Correct answers are on pages 178-184.
www.alkottob.com
172 SECTION 7
www.vet4arab.co.cc Pharmacology 173

147. Which condition is an absolute 149. \Nhich preparation has the longest duration of 156. Gentamicin and amikacin are antibiotics with a c. thiamylal sodium
contraindication to use ofcorticosteroids? action? na"ow therapeutic index, meaning the dosage d. guaifenesin
must be carefully controlled to prevent toxicity.

om
a. ketos is a. posterior pi tuitary injection USP e. delOmidine
What is the safest way to monitor the response to
b. ulcerative keratitis b. vasopressin injection USP these antibiotics?
c. allergic dermatitis c. vasopressin tannate injection USP 161. If you wish to prepare 1 L ofa 5% guaifenesin
a. Measure levels in serum to make sure they
d. arthritis d. arginine vasopressin solution in 5% dextrose, how much ofeach
are within the safe range.
e. lupus erythematosus e. lysine vasopressin ingredient will the solution c01ltain?
b. Give very low dosages.
a. 100 g of dextrose and 100 g ofguaifenesin
c. Use these drugs for no more than I day.
148. Dose-related cardiac damage is an important 150. What is II complication of insulin therapy in b. 5 g of dextrose and 5 g of guaifenesin
d . Use these drugs for 3 days, withh old the
toxic effect oJ lite chemotherapeutic agent: diabetic patients? c. 50 g of dextrose and 50 g of guaifenes in
drugs for another 3 days, and then resume

b.c
a. mithramycin a. hypokalemia treatment for 3 more days. d. IO g of dextrose and IO g of guaifenesin
b. doxoru bicin b. metabolic acid osis e . Keep the animal well nourished while the e. 0.5 g of dextrose and 0.5 g of guaifenesin
c. 5-fluorouradl c. dehydration drugs are be ing given.
d. chlorambucil d. hyponatremia 162. Cimetidineand ranitidineare:
e. cytosine arabinoside e. acute renal shutdown 157. A dog has a serum creatinine level of3 mgldl.
a. an tacids used for reflux esophagitis
You p lan to begin ampicillin therapy using a
dosage of25 mglkg body weight. How often b. anthelmin tics used for hookworm infections
should you give the drug? c. antibiotics used against gram-positive

tto
S. H. Duran bacteria
a. every 4 to 6 hours d. H2 antagonists used for gastrointestinal ulcers
151. All the following are examples of extra label use 153. A vinl ofsodium penicillin G powdercontairu 5 h. every 6 to 8 h ours
e. injectable laxatives
ofa veteriflary drug except: million units. Ifyou add 18 ml ofsterile water to the c. every 8 to IO hours
vin/, the concentration will be 250,000 UllitsJmL d. every 1 to 2 hours
a. drug used exac!ly as indicated on the label
What volume ofpowder was originally in the vial? 163. When injected, oxytetracycline should be given:
h. d rug used for a species other than those e. every 12 to 18 hours
indicated on the label a.4ml a. with calcium products

o
c. route of administration different than that b.2ml b. very slowly intravenously
158. What key words should you look for when
indicate d on the label c. I m l purchasingfluids for intravenous injection? c. by any convenient route
d. disease treated not indicated on the label rl.20m] d. with blood products
a. sterile, distilled

alk
e. dosage interval d ifferent from that ind icated e. 5 ml e. very rapidly as an intravenous bolus
b. distilled, bacteriostatic
on the label
c. sterile, pyrogen free
154. You plan to dispense an external parasiticide to 164. Aminophylline, prednisone, and terbutalineare
d. autoc1aved, distilled
152. A mi/liequivalem (mEq) is calculated by dividing a eiient.ln wha.t type olcontainer should the used in combination to treat:
e. bacteriostatic, irrad iated
the milligram (mgJ molecular weight by the insecticide be dispensed?
a. infections with gram-positive bacteria
valence, whicli gives mg/mEq. Ifpomssium
a. the original container, with a safety cap b. intestinal malabsorption
chloride (KC/) has a molecular weightof74.5, 159. Lactated Ringer's injection USP is not
how many milliequivalents are in 1 gofKCn b. a clear glass bottle, ap propriately labeled c. squamous-cell carcinoma
w. compatible with:
c. an amber glass bottle, ap p ropriately labeled d. chronic obstructive pulmonary d isease
a. 74.5 mEq a. calcium chloride
d. a plastic I-gallon cOnlainer e. diabetes insipidus
b. 7.45 mEq b. potassiu m chloride
e. a plastic container with a wide mouth
c. 13.4 mEq c. sodium chloride
d. 20 mEq 165. Whole blood, plasma, and blood substitutes
d. sodium bicarbonate
155. What is the osmolarity ofan isotonic solution? should:
e. 134 mEq e. potassium acetate
ww
a. 750 mOsm/L a. be kept at room temperature for 24 hours
b. 300 mOsm/L before use
160. An animal has an allergic reaction to xyiazine.
c. 1000 mOsm/ L What other drug should not be administered to b. be given in the same intravenous line as
d. 200 mOsm/L this animal? other medications
e. 150 mOsm/L c. not be used to dilute any drugs
a. ketamine
d. be mixed with calcium products before use
b. acepromazine
e. be given with dextrose solutions

© 1998 Mosby-Year Book, Inc. Photocopying is prohibited by law. Correct answers are on pages J 78-184.
www.alkottob.com
174 SECTION 7
www.vet4arab.co.cc Pharmacology 175

166. Afreratimillistralioll ofheracillin porassiwn, the c. It is permitted if they have been employed by 175. Therapeutic drug monitoring involves: c. measurement of serum or plasma levels of a
drug is meUlbofized and appears in tissue and the hospital for al least 3 years.
a. periodic physical examination of animals drug to maintain levels in the optimal range
blood as: d. It is permiued if lhey are blood relatives. d. close visual observation of animals in the

om
that have been treated with drugs
a. ampicillin and amoxicillin e. It is nOI pe rmitted under any circumstances. first 8 hours after a drug is administered
b. testing of animals for possible anaphylactic
b. hetacillin and carbenicillin reaction before treatment with a drug e. periodically obtaining tissue or exudate
c. ampicillin and hetacillin 171. Whic" drug is used (a treat gastric ulcers in dogs specimens for culture during a course of
d. amoxicillin and penicillin and cats bill is flat absorbed? drug therapy
e. penicillin and carbenicillin a. cimetidine
b. ranitidine
167. Strong tinclIIreo/iodine USP is comprised ot c. neomycin J.E. Riviere

b.c
a. 2% iod ine in water d. sucralfate
176. All the fol/owing physiologic processes result in 180. A IO-kgdog is gillen an antibiotic at a dosage of
b. 2% iod ine in alcohol e. omeprazo[e decreasing plasma concentrations ofdrug 5 mglkg. If the drug's half-life is 30 minutes, how
c. 7% iodine in water except: long will it takefor the animal to have less thm!
d. 7% iodine in alcohol 172. A ciiem calls and says she left tire container of a. absorption I mgofdrug remaining in its body
e. 15.6% iodine in water 4iethylcarbamazine citrate oral liquid inlrercar a. 30 minutes
b. distribution
for 3 weeks during the summer. Concerning the
patency aft/!edrugafter such storage, which c. metabolism b. 60 minutes
168. O/tlle/allowing, with which drug can statement is most accurate? d. elimination c. 90 minutes

tto
epinephrine be giuell safely? e. tissue sequestration d. 120 minutes
a. It has probably lost potency.
a. dextrose 5% injection USP e. 180 minutes
b. It has probably gained potency.
b. sodium bicarbonate c. It will regain any losl potency if placed in the 177. Which factor is most important in determining
c. warfarin sod ium refrigerator. the extent and rate ofdrugdisrributian? 181. All animal is brought to yourciinic in renal
d. ascorbic acid d. It \villiose any excessive potency if placed in a. route of elimination failure. It is severely dehydrated, and its
e. hetaSlarch the refrigerator. b. route of administration glomerular filtration rate is only one quarter of
nonna/. 71!e animal reqllires antibiotic trr!O.tment.

o
e. It is probably unaffected and can be used c. drug's physiochemical properties l1!e antibiotic)'Ou select is cleared solely by renal
169. Records Oil use of COli trolled substances in a \vithout concern. d. size of the dose administered glomerular filtration; its ooiume ofdistribution is
veterinary practice should be maintained by: e. dosage fo rm used limited to extracellular fluid such that its oolwne

alk
a. a veterinary technician with a license in that Slate 173. Mlat is the main adverse effectof ofdistribution is halfofnormal. If the drng's half
phenylbutazone? life in a normal animal is 60 minutes, what would
b. a clerk with experience in inventory control 178. Which of the fallowing sites has the lowest ratio
its half-life be in this animal?
c. a hospital administrator with a key to the a. pUlmonary edema ofblood flow to tissue mass
controlled substances cabinet b. crystals in the urine a. 30 minutes
a. brain
d. an office manager c. gastric ulcers b. 60 minutes
b. muscle
e. a veterinarian registered with the Drug d. cardiac decompensation c. 120 minutes
c. skin
Enforcement Administration e. dryness of the mouth d. 240 minutes
d. bone
w. e. 360 minutes
e. kidney
170. COllcernillga ucterinariall'sdispemingof 174. A dehydrated animal is best treated by
medication (0 1I0spifai staff members for use in intravenalls infusion of 182. If a drug's volume ofdistribution is doubled in a
treating tlwtnselves, wllicll statement is most 179. The rate afelimination of drugs with a high
diseased animal, which a/tire following would
accurate? a. dext rose 15% inj ection USP hepatic extraction ratio is limited by: flat occur?
b. dextrose 10% injection USP a. rate of hepatic blood flow
a. h is permitted if they present a prescription a. Half-life would double.
c. dextrose 20% injection USP
ww
written by a physician. b. rate of bile flow
d. dextrose 50% injection USP b. Serum concentrations would be halved.
b. It is penniued if the drugs are used on the c. liver size
c. Clearance would double.
premises. e. acetated Ringer's injection USP d. extent of serum protein binding
d. Elimination rate constant would be halved.
e. glomerular filtration rate
e. Plasma protein binding would double.

@ 1998 Mosby-Year Book, Inc. Photocopying is prollibiled by law. Correct answers are on pages 178-184.
www.alkottob.com
176 SECTION 7
www.vet4arab.co.cc Pllarmacology 177

183. All the followingfactors would double the free c. increased by 50% 192. A lactating Holstein cow is presented to your c. reduced minimum inhibitory concentration
(non-protein bound) effective plasma d. increased by one half-life clinic with afever, regenerative neutrophilia, (MIC) for an organism after prolonged
concentration ofa drug except: e. unchanged and "harsh" lung sounds. Diagnostic tests suggest therapy

om
bacterial pneumonia. Culture ofa transtracheal d. development of bacterial resistance after
a. doubling the dose
wash yields Pasteurella sensitive to all the administration
b. halving the rate of hepatic metabolism 188. If a disease process doubles the half-life ofa drug antimicrobials listed below. Which of these
c. halving the volume of distribution in a food animal, the withdrawal time should be:
e. persistent antimicrobial effect after drug
would be the drug ofchoice in this case? concentrations have declined below the MIC
d. halving the extent of protein binding a. doubled for the bacteria being treated
a. gentamicin
e. doubling the extent of protein binding b. halved b. polymyxin B
c. increased by 50% c. chloramphenicol 197. Which antimicrobial is most nephrotoxic?
184. Ifol/ edoes not want a drug to accumulate in the d. increased by one half-life d. cephalexin a. ampicillin

b.c
body, the: e. unchanged e. sulfadimethoxine b. erythromycin
a. dosage interval should be less than or equal
to the half-life c. doxycycline
189. From tile sfandpoinr of increased antibacterial 193. Which antimicrobial would be optimal for d. gentamicin
b. dosage interval should be much longer than efficacy, which dmg combination is least initial treatment ofa urinary tract infection in a
the half-life . e. enrofloxacin
rational? cae? Assume that the bacteria involved are
c. dose given should be as small as possible equally sensitive to all the drugs listed.
a. ampicillin with kanamycin
d. dosage should be divided into smaller units 198. What is the bacterial site ofaction for penicillin
b. ampicillin \'lith gentamicin a. ampicillin antibiotics?
and administered more often

tto
c. chloramphenicol with oleandomycin b. doxycycline
e. dosage form should be of the sustained- a. 30S ribosomal subunit
release type d. penicillin with streptomycin c. erythromycin
e. clindamycin with gentamicin d. enrofloxacin b. 50S ribosomal subunit
e. lincomycin c. DNA gyrase e nzyme
185. Whjcl, factor influences the magnitude of the c. dihydrofolate reductase enzyme
steady-state concentration achieved after drug 190. Which ampicillin dosage regimen would be
e. transpeptidation enzymes
administration? optimalfor surgical prophylaxis? 194. Which antimicrobial undergoes enterohepatic

o
a. one dose the night before surgery and one recycling?
a. dose 199. What is the most common adverse effect of
b. route of administration after skin closure a. carbenicillin penicillin antibiotics?
c. dosage form (capsule, tablet, injection, e tc.) b. a 5-day regimen beginning during surgery b. oxytetracycline
a. drug allergy

alk
d. drug potency c. a single dose after skin closure c. colistin
d. one dose at anesthetic induction and one d. cephalexin b. kidney damage
e. route of elimination
after skin closure e. penicillin G c. liver damage
e. Ampicillin is never indicated for surgical d. bone marrow suppression
186. What is the most important factor in determining prophylaxis. e. inner ear damage
how long it will take a drug to achieve steady- /95. Which faeror is least important when selecting
state serum concentrations in a multiple-dose an antimicrobial for clinical use?
regimen? 191. After administering penicillin G for 5 days CO an 200. When selecting a generic drug to substitute for
anjmal with a Staphylococcus infection, the a. spectrum of antimicrobial activity an original proprietary (brand name) drug that
a.
b.
c.
size of the dose
number of doses
route of elimination
is now indicated?
a. ampicillin
w.
clinical condition has not improved. Which drug b.
c.
d.
safety index (toxic versus effective dose)
potency
presence of tissue residues after use (food
a nimals)
you have used for years, what is the most
important factor to ensure that the generic drug
will be as effective as the original drug?
d. route of administration a. dosage forms available in same sizes as the
b. amoxicillin
e. half-life e. cost proprietary drug
c. cloxacillin
b. of comparable cost to ensure comparable
ww
d. phenoxymethyl penicillin
187. If you double tile dose of drug given to afood J96. What is the "postantibiotic effect" in quality
e. pe nicillin V
animal, the normal withdrawal time should be: antimicrobiallherapy? c. same drug concentrations in various dosage
a. doubled a. persistent tissue residues after forms to ensure equal potency
b. halved administration d. dosage forms certified as bioequivalent to
b. reduced drug clearance with each successive the proprietary drug
dose e. manufactured under sanitary conditions

© 1998 Mosby.- Year Book, Illc. Photocopying is prohibited by law. Correct answers are on pages 178-184.
www.alkottob.com
178 SECTI ON 7
www.vet4arab.co.cc Pharmacology 179

Answers 29. b Benzodiazepines exert their anticonvulsant


activity by enhancing the inhibitory effect of
44 . b Cardiac glycosides do not convert atrial
fibrillation to normal sinus rhythm. Rather, they
I. d UnJike a full ago nis t, a partial agonist 17. a The bond for fluorine is much stronger than GAllA on excitatory neurons. slow the ventricular rate by decreasing

om
produces less than a maximal response, even for bromine or chlorine; thus little fluorine is 30. a As an anticonvulsant, bromide hyperpolarizes atrioventric ular conduction.
when all the drug receptors are occupied. released, as with methoxyflurane. neurons by replacing intracellular chloride. Its 45. b Because cardiac glycosides decrease
2. a The lower the Ko• the higher a drug's affinity IS. e Halothane has both direct and indirect effects elimination half-life is approximately 1 month; atrioventricular conduction. they are
fo r a given receptor. on he patic and renal function. It also increases therefore, loading doses are recommended to contraindicated in first- or second-degree
cere bral blood flow and intracranial pressure. achieve steady-state levels within I week after atrioventricular block.
3. e Nonionized drugs morc easily cross the blood-
beginning therapy. 46. c Prazosin is a prererential ai-adrenergic
brain barrier; therefore this acidic drug will be 19. b Nitrous oxide can accumulate in any air-
morc nonionized at a more acidic pH and the containing space in the body. 3 1. e Newer HI-specific antihistamines are less receptor antagonist.
dose must be reduced to avoid overdosage. lipophilic and therefore poorly cross the blood - 47. b 22 lb -;- 2.2 Ib/kg = 10 kg. 10 kg X 5 JJ.g/kg/min =
20. c Propofo! decreases cerebral perfusion,
brain barrier.

b.c
4. e The weak base, erythromycin, is more ionized resulting in decreased intracranial and 50 JJ.g/min. 50 ....g/min x I ml/35 JJ.g = 1.43 m1/min.
and the refore ionically trapped in milk, which is intraocular pressures. It is a GABA agonist. lacks 32. c Patients with mast-ceil tumors have high 1.43 m1/min x 15 drops/ml = 21 drops/min.
more acidic than plasma. anticonvulsant properties, causes short periods endogenous histamine levels and are prone to 4S. b Blood volume decreases after therapy with
of apnea. and is metabolized extrahepatically. gastric hyperacidity and gastric ulcers. An ACE inhibitors because of natriuresis secondary
5. e Five half·lives would pass after 7.5 hours, so
appropriate presurgical drug is an Hz blocker, to decreased aldosterone secretion. Natriuresis is
peak drug levels would be halved five times. 21. c Barbiturates bind to the barbiturate portion of such as cimetidine. followed by decreased ADH secretion and
6. c Body clearance is di recdy proportional to the GAllA receptor complex, increasing chloride
conductance in ne urons. 33. d Omeprazole blocks the parietal cell pump in decreased renal water resorption.
volume of distribution; therefore drug X has a
gastric oxyntiC glands. 49. d Angiotensin II causes constriction of efferent
smaller volume of distribution than drug Y. 22. e The high lipid solubili ty ofthiobarbiturates
results in rapid d istribution into the brain and 34. a Glucocorticoids bind to cytosolic and nuclear glomerular arterioles. ACE inhibitors prevent this

tto
7. d The glucuronide synthetic pathway of cats
redistribution to othe r tissues. Fat animals are glucocorticoid receptors. where they induce constriction by decreasing angiotensin II levels.
functions much more slowly than that of other
quick to recover. whereas lean animals cellular Iipocortin synthesis. lipoconin interferes thereby reducing the glomerular filtration rate.
species.
experience prolonged recovery. with phospholipase activation, reducing both 50. e An swers a though d tend to lower blood
8. c The parasympathetic nervous system favors leukotriene and prostaglandin synthesis.
digestion. Therefore a drug that mimics it effects 23. b Tachycardia is a common finding of volume and result in prerenal azotemia. Volume
barbiturate anesthesia, as well as ventricular 35. a Aspirin is metabolized to salicylic acid, which contraction predisposes animals to ACE
will increase gu t motility and decrease sphincter
fibrillation when the animal is hypothermic. in turn blocks the cyclooxygenase inflammatory inhibitor- induced renal failure.
tone.
Barbiturates potentiate catecholamine- induced pathway, leading to reduced prostaglandin 5 1. d Because enalapril has a slower onset of action

o
9. e Norepinephrine has the least affinity for 132- syntheSis.
adrenergic receptors. arrhythmias. Mean arterial pressure and cardiac but a longer duration of action than captopril,
output decrease. 36. d Because cats cannot metabolize and excrete once-daily dosing is possible.
10. d AI: 1000 solution is equivalent to I mg/ml. aspirin as well as other species, aspirin has the
The refore 0.25 011 of th is solution will deliver 24. c The therapeutic index of ketamine is wider in 52. c Calcium channel blockers slow conduction

alk
most species than that of barbiturates. It longest half-life in cats. through the atrioventricular node and are
0.25 mg.
produces a cataleptic anesthetic state in which 37. e Unlike other nonsteroidal antiinflammatory therefore indicated in treatment of
11. a The vasoconstricting effects of epinephrine. drugs, ketoprofen appears to inhibit both the supraventricular tachycardia.
periphe ral reflexes are maintained. Ketamine is a
through its ("I-adrenergic action. prolong the lipoxygenase and cyclooxygenase pathways.
poor visceral analgesic. 53. d The predominant mechanism of action of all
duration of local anesthesia by preventing
25 . e Ketamine is highly metabolized in the liver and 3S. b Vitamin K] is the treatment of choice in c1ass- l antiarrhythmics is blocking of sodium
absorption of the drug.
excreted in the urine. Its use is contraindicated in coumarin intoxication. channels.
12. b Neostigmine is an acetylcholine esterase
animals with he patic or renal disease. 39. b For full anticoagulant activity, heparin 54. c The predominant mechanism of action of all
inhibitor that allows accumulation of
26. b Phenobarbital is metabolized by hepalic requires antithrombin III. class-3 antiarrhythmics is blocldng of potassium
acetylcholine at the neuromuscular junction. This w.
microsomal enzymes. It is a potent inducer of 40. a Heparin is the most rapid-acting channels.
potentiates the action of succinylcholine, which is
a depolarizing neuromuscular-blocking agent. these enzymes; continued administration anticoagulant. Coumarins have a lag pe riod 55. a Quinine is a c1ass-IA antiarrhythmic that
increases the rate of metabolism. Metabolites because they exert their effect by inhibiting blocks sodium channels. This blockade
13. d Intravenous acetylcholine reduces blood
and unchanged drug are excreted in the urine. synthesis of endogenous clotting factors. decreases the rate of phase-O depolarization in
pressure through muscarinic receptors on the
vascular endot helium. Atropine is a muscarinic 27. a Primidone is metabolized to phenobarbital 41. d Cardiac glycosides bind to and inhibit the atrial and ventricular muscle.
antagonist. and phenylethylmalonamide in the liver. sodium-potassium ATPase pump. 56. b Mannitol, as do all other osmotic diuretics,
ww
Because phenobarbital has the longer half-life, it 42. c Cardiac glycosides increase cardiac output. acutely expands extracellular fluid volume. Such
14. a In small doses, dopamine is specific for
tends to accumulate and blood levels are used as withdraw sympathetic tone, slow the hean rate. an expanded volume would exacerbate
dopamine receptors. In larger doses. it binds to
a guide to design dosage regimens. and reduce myocardial O2 consumption. intracranial hemorrhage.
0;- and p-adre nergic receptors.
2S. c Phenytoin , like some other anticonvulsants. is 43. d Cardiac glycosides increase urine output in 57. c Chloride dissociates from the ammonium
IS. c Fluorine metabolites impair the kidney's
a potent inducer of microsomal enzymes. This patients with heart failwe by improving renal chloride molecule in the blood, increasing the
ability to concenlrate urine.
drug has a very short half-life in dogs, and perfusion secondary to improved chloride load delivered to the kidney.
16. d Animals subjected to light planes of halothane continued use further shortens its elimination hemodynamics. Diuresis is not caused by a
anesthesia are more prone to spontaneous half-life via enzyme induction. direct effect on the kidney.
arrhythmias.

@ 1998Mosby-YearBook,Inc.Photocopyingisprohibitedbylaw.
www.alkottob.com
180 SECTION 7
www.vet4arab.co.cc Pharmacology 181

58. e In part, sodium is absorbed in the renal tubule 74 . c Tetracyclines interfere with protein synthesis 89. a Rifampin is a highly lipid-soluble, acid-stable 105. d Alteration of urinary pH has a negligible
in exchange with protons. When protons are not through binding to the bacterial 305 ribosomal antibiotic that penetrates neutrophils and effect on the pH of extracellular fluid.
available through inhibition of carbonic subunit, resulting in interference with aminoacyl macrophages. Its spectrum of activity includes 106. d Morphine arrests peristalsis by its spasmodic

om
anhydrase, sodium is not resorbed. (RNA binding lO the acceptor (A) site. gram-positive bacteria and mycoplasmas. It has action on smooth muscle in the gut
59. b Chloride ions are co-secreted with protons in 75. e Pancytopenia has not been associated with been used to treat gram-positive infections in 107. d 40z X 30 mlloz =120 ml total, comprised of
the coUecting duct. When protons are not tetracycline therapy. horses. 5 parts 70% ethanol and 2 parts 7% tincture.
available through inhibition of carbonic 76. a Chloramphenicol inhibits peptidyltransferase 90. c Metronidazole penetrates the central nervous 108. e Amitriptyline appears to diminish
anhydrase, chloride ions accumulate in the activity at the bacteriaJ 50S ribosomaJ subunit system well and is indicated in treatment of apprehension and alter the related behavioral
blood and the urine becomes alkaline. and inhibits elongation of the growing peptide anaerobic bacterial meningitis. response.
60. b Thiazide diuretics are secreted into the chain. 91. d Synthetic host-cell protein receptor anaJogs 109. e Alfalfa meal is an important dietary source of
proximal tubule by an organic acid transport 77. e Cholestasis and arthropathy have not been attach to the viral coat and prevent viral vitamin K.
system, where they inhibit both sodium and associated with chloramphenicol toxicity. attachment to host-cell receptors.

b.c
chloride resorption in the distal tubule. 1l0. e Some collies develop blindness, coma, and
78. b Erythromycin prevents translocation of amino 92. a The antiviral activity of amantadine and death in response to ivermectin.
6 1. c Spironolactone is a competitive inhibitor of acids to the growing peptide chain by reversibly rimantadine involves inhibition of late-stage
aldosterone receptors. IlL d None of the other choices is accurate.
binding to the bacterial 50S ribosomaJ subunit. assembly of virus.
62. d Furosemide is actively secreted into the 112. c Barbiturates cause induction of hepatic
79. b Erythromycin is well distributed to most tissues, 93 . b These compounds must be converted to the
proximal tubule and inhibits chloride resorption microsomal enzymes. This is an imporlant
with the exception of the brain and spinal corel. triphosphate form to compete for incorporation
in the thick ascending loop of Henle. mechanism of drug interactions.
80. d Cardiovascular toxicity has been observed in into growing DNA strands.
63. d Metoclopramide is not associated with altered 113. b Propantheline is a quaternary ammonium
primates, swine, and horses. 94. b The major toxicity associated with
colonic motility. antimuscarinic compound. This confers
81. c Serious or fatal diarrhea has been associated with amphotericin B therapy is dose-dependent

tto
appreciable activity at ganglionic and
64. d Oral sodium bicarbonate results in liberation lincomycin use in horses, humans, and rabbits. nephrotoxicity.
muscarinic sites.
of CO 2 , which can result in gastric rupture, 82. e Unlike other classes of antibiotics, 95. a Ketoconazole has been successfully used in
particularly at the site of gastric ulceration. 114. b Oicumarol is a vitamin K antagonist within
fluoroquinolones work through interfering with dogs to treat Cushing's disease caused by
the liver. The other compounds listed are
65. a Apomorphine is a pure dopamine receptor bacterial DNA synthesis. inhibition of corticosteroid synthesis and,
unsuitable for such usc.
agonist, and its effects are not countered by 83. a Fluoroquinolone use is associated with
consequently, cortisol+lowering properties.
opiate antagonists. llS. c Stimulation of the carotid sinus by increased
arthropathy in young animals. 96. b The nitrogen mustard, cyclophosphamide, is
blood pressure slows the heart rate.
66. e Anticholinergic drugs, such as non-cell cycle specific and can be administered

o
84. c Aminoglycosides are highly polar compounds 116. d Thromboxane is synthesized in platelets and
aminopentamide, prevent vomiting and and consequently are poorly absorbed orally and orally or intravenously. It must undergo hepatic
decrease intestinal secretions and segmentaJ and activation. Its major toxicity is sterile is a powerful inducer of platelet aggregation.
poorly cross biologic membranes.
propulsive intestinal movements. Neostigmine is hemorrhagiC cystitis. 117. d This solution of aluminum subacetate is used
85. d Aminoglycosides result in production of to treat acute, oozing skin lesions.

alk
a parasympathomimetic, and its effects are 97. d Busulfan is particularly effective for chronic
nonsense proteins through interference with
opposed by anticholinergic drugs. granulocytiC leukemia but is not effective in 118. b Glucocorticoids increase protein catabolism.
reading the mRNA transcript.
67. b Narcotic cough suppressants work through acute leukemia. 119. d Pharmacologically active components of the
86. e Aminoglycosides inhibit prejunctional
depression oCthe medullary and pontine centers, 98. e The high lipid solubility of carmustine aids in other four compounds are excreted in milk and
acetylcholine release; paraJysis of the diaphragm may cause diarrhea in the pigs.
where the cough reflex is controlled. its transport into the central nervous system,
has been associated \vith intrapleural injections.
68. c Sulfonamides are structural analogs of PABA Neomycin is most likely to cause neuromuscular making it effective for treating brain and 120. a Some sulfonamides calise lower-nephron
and prevent conversion of PABA to blockade; calcium infusion can reverse the meningeaJ tumors. nephrosis by forming crystals in the renal
dihydrofolate. neuromuscular blockade. Aminoglycosides have 99. e In addition to cardiotoxicity, doxorubicin can wbules.
69. a Trimethoprim and ormetoprim are
competitive inhibitors of dihydrofolate
reductase. Their combination with sulfonamides
renders them bactericidal.
no effect on nicotinic receptors.
87. c Neomycin is the most nephrotoxic
w.
aminoglycoside and consequently is not
cause acute tOxicity associated with mast-cell
degranulation.
100. b The major toxicity of doxorubicin is acute
cardiac toxicity or cardiomyopathy associated
12 1. e The principal action of aspirin is to decrease
synthesis of prostaglandins by inhibiting
cyclooxygenase.
122. b Halothane depresses the myocardium and
recommended for parenteral use.
70. d Tetracyclines, not sulfonamides, arc 88. b Vancomycin is a complex peptide antibiotic. It
with cumulative doses. blunts the baroreceptor-mediated tachycardia
associated with enamel staining. is bactericidal, poorly absorbed orally, 101. b Corticosteroids cause hepatomegaly and in response to hypotension.
ww
71. c Penicillins are bactericidal because they potentially nephrotoxic, and eliminated in the elevated serum alkaline phosphatase activity. 123. b Hyperkinesis is paradoxically improved by
disrupt bacterial cell wall syntllesis. urine. Its use should he limited to penicilIin- 102. c Such drugs have a large renal clearance treatment with a central nervous system
72. c Penicillin G cannot be given orally because of resistant Staphylococcus infections. limited primarily by renal blood flow. stimulant.
rapid degradation by gastric acid. 103. c Local anesthetics may cause convulsions. The 124. c Vitamin 0 3 is converted to 25-
73. c CephaJosporins arc less susceptible to 13- other answers are unrelated to dibucaine. hydroxycholecalciferol in the liver and is
lactamases than are penicillins. 104. a Antihistaminics (e.g., diphenhydramine) are hydroxylated at the one-position by renaJ
much more effective for prevention of motion mitochondria.
sickness.

© 1998 Mosby-Year Book, Inc. Pilotocopying is pro/zibiced by law.


www.alkottob.com
184 SECTION 7
www.vet4arab.co.cc SECT ION

183. e Increasing the fraction of drug bound to 191. c Gram-positive staphylococci should be
proteins would reduce the free concentration of
drug.
susceptible to any lactam antimicrobial unless the
bacteria produce lactamase. This is the only drug 8

om
184. b A drug"accumulates in (he body unless the listed that is resistam to inactivation by lactamase.
dosage interval is significantly longer than the 192. e This sulfonamide has the optimal
half-life. pharmacologic profile for treating this
185. a The dose directly determines the magnitude
of the steady-slate concentration seen after
drug admin istration.
condition. The other compounds are too toxic,
are not approved for use in food animals, or
would produce persistent residues.
Principles of Surgery
186. e The time it takes to reach the steady-state 193. a This broad-spectrum antimicrobial is
T.P. Colville
concenlration is solely a function of the drug's excreted by the kidney into the urine.
hatf-life. 194. b This tetracycline is the onl y drug listed that is

b.c
187. d Jf you double the dose, after one half-life the excreted into the bile, a prerequisite for
drug remaining in the animal is approximately enterohepatic recycling. All the others are
the same as if the original dose were given . excreted into the urine.
Thlls the withdrawal time need only be 195. e Drug potency can be improved by
increased by one half-life. form ulat ion and thus is not a limiting factor. Recommended Reading
188. a In cont rast, iflhe half-life is doubled, it will The other factors are inherent properties of the
Auer JA: Equine surgery, Philadelphia, 1992, WB Saunders.
now lake twice as long to eli minate the drug and drug and cannot be manipulated. Economics
(he wilhdrawal time should now be doubled. should always be a factor in selecting therapy. BOjrab MJ: Current techniques in small animal surgery, ed 4, Baltimore, 1998, Williams &Wilkins.

tto
189. c Both these drugs have an identical 196. e If a drug has a significant postantibiotic Colahan PI el al: Equine medicine and surgery, ed 5, St. Louis, 1998, Mosby.
mechanism of action. For optimal amibacterial effect, therapeutic effects may persist even after Fossum TE: Small animal surgery, 51. Louis, 1997, Mosby.
efficacy. drugs used together should have drug concentrations fal l below minimum Jennings PB: The pracrice of large animal surgery, Philadelphia, 1984, we Saunders.
diffe rent mechanisms of action. inhibitory concentrations. Slatte r DH: Textbook of small animal mrgery, ed 2, Philadel phia, 1993, WB Saunders.
190. d A prophylactic antibiotic should be 197. d Gentamicin is an aminoglycoside antibiotic Swaim SA, Henderson RA: Small animal wound managemellt, Philadelphia, 1990, Lea & r-ebiger.
administered according to this protocol so that that can cause nephrotoxicity.
maximum concentrations are present at the 198. e Penicillins inhibit bacterial cell wall synthesis.

o
surgical site when organisms are introduced ..._ _-,p~,,~ctice answer sheet is on page 269.
199. a Penicillins have minimal adverse effects,
into tissues by surgical manipulation. The drug except for sensitization.
should not be present before or after this 200. d By law, bioequivaJent drugs have the same
"window of opportunity. ~ Questions

alk
therapeutic efficacy and thus can be used
interchangeably.
1. Wllich surgical instrument is priman'ly used to 3. Wllich of the following is the correct surgical term
hold orgam and tissues out ofrhe way to for declawing?
facilitate exposure of the operative field?
a. celiotomy
NOTES a. elevator b. cystotomy
b. forceps c. onychectomy
w. c. retractor d. hysterectomy
d. rongeur e. colpotomy
e. hemostat
4. Which type of needle is most appropriate for
2. Wllich orthopedic instrument has sharp, suturing intemal organs, such as gastrointestinal
opposing. cup-sl/aped jaws used to shape bone by structures?
Mcliewing" out small pieces?
ww
a. blunt
a. chisel b. cutting
b. curette c. reverse cutting
c. osteotome d. tapered
d. rongeur e. trocar
e. trephine

e 1998 Mosby-Year Book, Inc. Photocopying is pro/libited by law. Correct answers are on pages 195- 198. 185
www.alkottob.com
182 SECTION 7
www.vet4arab.co.cc Pharmacology 183

125. b Iverrnectin is approved in the United States 146. c Most drugs are relatively small molecules that 158. c Only fluids that are sterile and pyrogen free 171. d Sucralfate is an alkaline aluminum complex
for prevention ofhearrworm infections in dogs. diffuse across membranes in response to should be given intravenously. of sucrose sulfate that acts locally rather than
126. a Agonists interact wilh the ceU receptor to concentration gradients. 159. d Lactated Ringer's USP contains calcium systemically. It fonus a barrier at the ulcer site

om
elicit a response. 147. b Corticosteroids inhibit healing, and this may chloride. Mixing with sodium bicarbonate and protects the ulcer from further damage
127. c Benzalkonium chloride is the only cationic lead to corneal perforation and loss of aqueous forms an insoluble p recipitate, calcium caused by pepsin, acid, or bile.
detergent in the group lis ted. humor. carbonate. 172. a Diethylcarbamazine citrate, used for
128. d Muscarinic siles are blocked by atropine. 148. b Anthracycline antibiotics, such as 160. e Detomidine and xylazine are chemically heartworm prevention in dogs, decomposes
129. a The OM is a division of the Department of doxorubicin, cause cumulative dose-related related. Animals that have an adverse reaction when exposed to temperatures over 300 C (86-
Justice. cardiomyopathy. to xylazine may also react to detomidine. F) and when exposed to light. Always dispense

130. a Heat production is a resuh of metabolic 149. c Vasopressin tan.nate is more slowly absorbed 161. c A 5% dextrose-5% guaifenesin solution this drug in an amber bottle.
activity, which is not affected by eicosanoids. than the other preparations listed. contains 5 g of dextrose and 5 g of guaifenesin 173. c The major toxicity associated with
phenylbutazone is gastric ulcers. The dosage

b.c
131. c Whitfield's ointment contains benzoic and 150. a Insulin and glucose d rive potassium per 100 ml, which is equal to 50 g of each per
salicylic acids. These compounds are fungistatic intracellularly, causing hypokalemia. 1000 ml. s hould be tapered to small amounts or the drug
and keratolytic. 15 1. a Extra-label use involves use of an approved 162. d Cimetidine and ranitidine are H2 antagonists should be used for short periods only, because
drug in a manner not consistent with the drug that block gastric acid secretion, which aids it accumulates.
132. a Induction of microsomal enzymes increases
the rate of biotransformation of the inducing container label or package insert treatmen t of ulcers. Because the re are no H2 174 . e Acctated Ringer's injection is the fluid of
agent and lo r other drugs. 152. c There are 13.4 milli equivalents (mEq) in I g antagonists approved for veterinary use, these cho ice. Hypertonic dextrose solutions would
(1000 mg) of potassium chloride. The equation drugs can only be used in accordance with act as an osmotic diuretic and further
133. a This combination of actions is unique to dehydrate the animal.
is as follows: extralabellaws.
metoclopramide and prevents esophageal
163. b Oxytetracycline chelates (binds) calcium. The 175. c Therapeutic drug monitoring involves

tto
reflux of gastric fluid.
134. c This action results in increased excretion of
mEq =___---'m""' -_ __ vehicles used in some oxytetracycline products, measuring the amount of drug in the serum or
Molccular weightlValence such as propylene glycol, can cause anaphylaxis plasma of the diseased animal to achieve the
sodium, chloride, and water. best therapeutic dosage, with the least toxicity.
153. b First divide the initial amount (5 million if these drugs are given by rapid intravenous
135. c Leucovorin is a reduced, functional folate injection. Because of their pH or vehicles, some 176. a This is the only pharmacokinetic process
coenzyme. units) by the final amount (250,000 units/ ml).
5,000,000 + 250,000 = 20 m!. Then subtract the products cannot be given intramuscularly. listed that increases drug concentrations in the
136. c Urochloralic acid gives a false-positive result volume of diluent added (18 ml). 20 - 18 = 2 ml 164. d Aminophylline (a bronchodilator), central companment. The others remove drug
in tests for reducing sugars. of powder originally in the vial. prednisone (an antiinflammatory from this compartment and thus result in

o
137. a These effects are associated most commonly 154. a All inseClicides s hould be dispensed in the glucocorticoid), and terbutaline (a selective P2- decreas ing plasma concentrations.
\vith the phenothiazines but could also be seen original container, with all EPA in formation on adrenergic agonis t) can be given in 177. c The p rimary determinant of dis tribution is
with the butyrophcnoncs, because these effects the label and in a safety-cap container or spray combination to treat chronic obstructive the drug's ability to cross biologic membranes.
are associated \vith dopaminergic receptors.

alk
container. pulmonary disease. 178. d The bulk of parenchymal [issue of bone is
138. e Virtually all the drug would be eliminated 155. b An isotonic solution has an osmolarity of300 16S. c Whole blood, plasma, and blood substitutes calcified and not perfused, thereby restricting
from the body after five half-lives have elapsed. mOsm/L, which is the same osmotic p ressure should not be used as diluents of any drugs, the distribution of drug.
139. c Pilocarpine is well absorbed from ophthalmic as for body fluid s. A solution with an osmolarity and they can be infused only by piggyback with 179. a Clearance of drugs with a high hepatic
solUlions and ointments and causes less th an 300 mOsm/L is hypotonic and could sodium chloride injection 0.9% USP. extraction ratio is limited by the rate of hepatic
contraction of the pupillary sphincter. cause lysis of red blood ceUs. A solution with an 166. c Hetacillin potassium is ch emically related to blood flow.
140. a The half-life for salicylate is only 30 minutes osmolarity greater than 300 mOsm /L is ampici lli n. Some of the drug is metabolized to 180. e The initial amount of drug in the animal is 50
in cattle. hyperlonic. ampicillin, and some rem ains as hetacillin. mg (5 mg/kg X 10 kg). After one half-life, 50%
14 [. a Irritating drug solutions are infused slowly
into a vein to allow for dilution.
142. c Use of chloramphenicol in these animals is
prohibited because of the fear that residues in
w.
156. a The safest way to monitor patients receiving
aminoglycosides is to obtain serum samples and
measure peak and trough levels to make sure
drug levels are within the safe, therapeutic range.
167. d Strong tincture of iodine USP is a 7% tincture,
which has an alcohol base. It is commonly used
to dip the umbilical cord of newborn calves.
Mild tincture of iodine is a 2% solution and can
is eliminated, leaving 25 mg. After two half-
lives, 12.5 mg remains; after three half-lives,
6.25 mg remains; after four half-lives, 3.12 mg
remain; after five half-lives, 1.6 mg remains; and
food products might cause aplastic anemia in 157. e Ampicillin is elim inated primarily by the be used to dip the umbilical cord of foals. after six half-lives. 0.8 mg remains. Six half-lives
susceptible people. kidneys and normally administered every 6 168. a Epinephrine is usually given by intravenous equal 180 minutes.
bolus injection. If it must be diluted, it should 181. c Half-life is proportional to the clearance
ww
143. c Equilibration occurs more rapidly when the hours for 7 to 10 days. If an animal has a serum
agent is not soluble in blood. creatinine value of3 mg/ d1 (normal, I to 2 be added to dextrose 5% injection USP. divided by the volume of distribution. If the
144. c Motor activity of the internal urethral mg/dl), this indicates reduced renal function, 169. e A veterinarian registered with the DEA and clearance is one quarter of normal but the
sphincte r is mediated by sympathetic nerves. so the drug should be given less frequently prescribing controUed substances should keep volume of distribution is doubled, the resulting
Receptors are of the a-adrenergic type. because it is not being eliminated at a normal accurate records. half-life is twice normal, or 120 minutes.
145. e Procainamide is the only drug listed that is rate. A good rule of thumb is as rouows: Serum 170. e A veterinarian is not permitted to dispense 182. c Clearance is independent of the volume of
effective for treatment of s upraventricular creatinine X Normal dosage interval = Adjusted any drugs to people. Severe penallies are distribution.
arrhythmias. dosage interval. 3 X 6 =18 hours. associated with this offense.

C 1998 Mosby-Year Book, Inc. Photocopying is prollibited by law.


www.alkottob.com
186 SECTION 8
www.vet4arab.co.cc Principles a/Surgery 187

5. Wlliell type of needle is mast commonly Ilsed for 10. Whic/I oftllefollowing best describes the location 16. What is the proper term for entrance of 21. Which suture material shows the lease
sltCllring in general surgery? ofan incision extendingfrom the xiphoid process microorganisms into an incision during a conductiOIl offluid bycapi/laryaction?
to tile umbiliCllS ofan animal? surgical procedure?
a. straight a. braided colton

om
b. " circle a. dorsal midline a. contamination b. braided polyglycolic acid
c. ~ circle b. flank b. debridement c. braided silk
d ~ circle c. paracostal c. dehiscence d. monofilament nylon
e. ~ curved d. paramedian d. infection e. chromic catgut
e. ventral midline c. septicemia
6. WI,;ell of the following is "ot considered an 22. All tile followillg are lIonabsorbabfe suture
elective surgical procedure? For Questions 11 through 13, select the correct 17. Whidl of the following items does not have to be materials except:

b.c
answer from the five choices below. sterile during a surgical procedure involving
a. correction of a an eye with proptosis a. conan
aseptic technique?
b. dew claw removal in a field trial dog a. autoclave b. nylon
c. mastectomy to remove a benign tumor b. boiling a. drapes c. silk
d. ovariohysterectomy in a healthy 6-momh-old c. dry heat b. gloves d. stainless-steel wire
cat d. ethylene oxide gas c. gown e. chromic catgut
e. tail docldng in 3-day-old boxer puppies e. liquid chemical disinfectant d. mask
e. suture material
23. Wllich type ofsurgical gut is absorbed m ost
7. Healing ofa properly sutured surgical wound is

tto
11 . Most appropriate for sterilization ofan electric rapidly from tissues after surgery?
most appropriately termed: driflto be Wied in an ort/lOpedic surgical 18. Wilich o/the following is tile agem for
a. extrachromic
procedure sterilization byautoclaving?
n. first-intention healing b. heavy chromic
b. granulation a. chemical disinfectant solution c. medium chromic
c. secondary union 12. Most appropriate for sterjfjzation of a needle b. dry heat d. mild chromic
d. second-intention healing holder to be used in a surgical procedure c. ethylene oxide gas e. plain gllt
d. ionizing radiation

o
c. wound contraction
13. Most appropriatefor sterilization of dissecting e. steam
24. Wllich of tile following is not an effective/arm 0/
scissors to be Wied in a surgical procedure
8. Wllieh of tile following lias tile poorest potential surgical hemostasis?
for healing and relllm (0 normal jimccioll after 19. Which size ofelectrical clipper blade is most

alk
a. crushing
damage and effective surgical repair? 14. Which of the following describes the minimal commonly used/or clipping the hair from a
surgical site? b. curettage
a. bone exposure rime and temperature for aucoclaving a
c. electrocoagulation
surgical pack? a. #10
b. intestine d. ligation
c. liver a. 121 C for 15 minutes
0
b. #20
e. pressure
d. nervous tissue b. 121" F for 15 minutes c. #30
e. uterus c. 250 F for 5 minutes
0
d. #40
25. Wilich a/the/allowing is nota likely cause of
d. 2500 C for 20 minutes w. e. #50
dehiscence ofan abdominal incision ?
9. Wl!ich term describes removal of necrotic tissue e. 250 0 F for 20 minutes
a. chronic vomiting
from a wound? 20. Which suture size is smnlkst in diameter?
h. excessive physical activity
a. debridement J5. Mtich of the following is tile most effective and a. 0 c. s tormy recovery from anesthesia
b. inspissation immediate indicator that the conditions for b. 0000
sterilizao·on have been met in an autoclaved d. surgical wound infection
c. lithotripsy c. 2-0 e. suture maleriallarger than needed
ww
surgical pack?
d. retraction d. .2
e. baJlo(tcment a appearance of the instruments e. 3-0
b. autoclave tape
c. chemical indicator
d. culture results
e. melting pellet

<0 1998 Mosby-Year Book. Inc. Photocopying is prohibited by law. Correct answers are on pages 195-198.
www.alkottob.com
188 SECTION 8
www.vet4arab.co.cc PrincIples o/Surgery 189

26. Concerning aseptic surgical technique. which c. granulation tissue 35. Wlt;ch factor related to infection ofa surgical d. orthopedic equipment
statemem is U!astaccllrate? d. wound contraction wound is most sigtlificantlyaffected by aseptic e. surgical drapes
e. wound flushing technique?
a. A sterile item touched by a nonsterile item

om
becomes nonsterile. a. number of microorganisms entering the 40. TIte lime necessary to achieve disinfection of
b. Irlhe sterilify of an item is in doubt, consider 3/. \!VI/jcll oftlte following is nor a characteristic of wound surgical instruments with liquid chemicals can be
it sterile. first-jntemionlvowld Ileating? b. pathogenicity of microorganisms entering the shortened by:
c. Nonscrubbed personnel can touch only wound
a. minimal contamination a. agitating the solution
nonslerile items. c. species of microorganisms entering the
b. minimal tissue damage wound b. cooling the solution
d. Only sterile items can contact exposed patient
c. minimal role of wound contraction d. route of exposure to infectious microorganisms c. using a lower concentration than
tissue.
d. wound edges not approximated recommended
c. Only sierile items can contact other sterile e. susceptibility of the patient

b.c
e. wound edges sutured d. using a higher concentration than
items.
reco mmended
36. The effectiveness ofa bactericidal surgical scrub of e. warming the solution
27. WI/iell a/the/allowing is an everting suture 32. Which illcision is most appropriate for ones hands and arms depends on tlte:
pattem rltat should only be used to close skin exploratory surgery i1l a dog's abdomen, in which
incisions? the precise location of the problem if not known? a. combination of contact time and scrubbing 41. Which surgical wire size is smallest in diameter?
action
a. Cushing a. dorsal midline b. le ngth of time the soap is in contact with the a. 40 gauge
b. Lembert b. flank ski n b. 10 gauge
c. 26 gauge

tto
c. Parker-Kerr c. paracostal c. pH ofthe skin surface
d. simple interrupted d. paramedian d. scrubbing action of the brush d. 32 gauge
c. vertical mattress e. ventral midline e. temperature ofthe water c. 20 gauge

28. Which D/the/allowing is an inverting suture 33. \W,ich ofthefollowing is 1I0t an early sigtl of 37. Wltich ofthe following, when used alone as a 42. Wlliclt surgical drape material prevents passage
pattem used mailily tosulure hollow imernai wOlllld dehiscence tll~ring tlte first 24 hours after surgical scrub soap,forms a bacteriostatic film of bacteria through the drape to tlte patient's skin
organs? abdominal surgery? over the skin? by capillary action when rite top surface of the

o
drape becomes lvet?
a. horizonlal maUress a. body temperalUre elevation of]" to 2~ F a. chlorhexidine
b. change in texture of the wound edges b. chlorpheniramine a. cloth
h. Lembert
c. serosanguineous discharge from the incision c. hexadimethrine b. fenestrated paper

alk
c. purse string
d. simple interrupted or continuous d. swollen incision d. hexachlorophene c. musl in
e. very warm incision e. povidone-iodine d. paper
e. vertical mattress
e. plastic
29. Which of tile following is most effective in 34. Ti,e main goal ofaseptic surgical technique is to 38. Which of the followillg tloes not normally have to
minimizing the scarringfrom skin sutures and prevem cOlllamillatioll of be sterilized as part ofgootl aseptic surgical 43. Which suture size is smaller in diameter than 3-D?
achieving good wolmd healing? a. operative personnel technique? a. 2-0
a. leaving sutures permanently in place h. sterile fields w. a. cap b #1
b. removing the sutures 2 days after insertion c. slCrile zones b. drapes c. 113
c. removing the sutures 7 days after insertion d. surgical instruments c. gloves d.1I4
d. using large-diameter suture material e. surgical wounds d. gown e. 4-0
e. using suture material that produces e. scrub brush
significant inflammation 44. WIljel, suture material is absorbable?
ww
39. Liquid chemical sterilization is used primarily for: a. cotton
30. Which of tile fol/owing does not enhance healing a. electrical equipment b. nylon
ofall open wound?
b. hemostatic forceps c. polypropylene
a. d~bridement c. instruments with sharp edges d. silk
h. exuberant granulation tissue e. chromic catgut

1.0 1998 Mosby-Year Book, Inc. Photocopying is prollibited by law. Correct answers are on pages 195-/98.
www.alkottob.com
190 SECT ION 8
www.vet4arab.co.cc Principles o/Surgery 191

45. Wlzieh surgical instrument should not be d. both nonscrubbed and scrubbed personnel 56. Whic/l suture size is 1n.rger ill diameter than size 112? d. no significance
routinely steam sterilized? e. neither nonscrubbed nor scrubbed personnel •. 0 e. serious but not an acute emergency
a. Backhaus lowel clamp b. #1

om
h. Halsted mosquito forceps 5/. Wllat is tile significance ofdehiscellce of the c. 2-0 62. Which suture pattern is neilJuran inl!erring nor
c. Mayo-Hegar needle holder muscle, subcutaneous tissue, alld skin la}'ers ill a d. #3 an el!erting pattern?
ventral midlille surgical wou"d?
d. Mctzenbaum scissors e. 3-0 a. Cushing
e. Bard-Parker scalpel handle a. acute e mergency b. horizontaJ mattress
b. cosmetic proble m only

..
57. Wllicll suture material is synthetic? c. Lembert
46. Castration o/a healtlly6-month-old cat is an c. minor significance d. simple interru pted or continuous
chromic catgut
exampleo/: d. no Significance e. vertical mattress
b. cotton

b.c
a. cosmetic surgery e. serious but not an acute e mergency
c. nylon
b. elective surgery d. plain catgut 63. Willa! is the healing potential ofa fractured bone
c. emergency surgery 52. Why is a rece"t surgical wound usually slightly that is properlya/igned and kept immobile?
e. silk
warmer lllan the su rrounding normal tissues?
d. exploratory surgery a. excellent
e. first-i ntention surgery a contamination b. good
58. Which of the following is not a typical sign of
b. debridement c. fair
hemorrhagic shock in a postsurgical patient?
47. WllI'ell solution callses the least tissue damage c. infecl"ion d. poor
a. deep, slow breathing
and is most oppropriate[or woulldfIushing? d. inflammation

tto
b. pale mucous membranes e. very poor
a. hydrogen peroxide e. septicemia
c. slow capillary refill
b. isotonic sal ine d. tachycardia 64. Whidl ofthefollowingindicates thebest blood
c. povidone- iodine scrub 53. How 10llgafter sllrgerydoes a sutured surgical sllpplyto theedgesofa U/Owld in unpigmenU!d skin?
e. weakness
d. povidone- iodine solution
wound hegillto gaill significant strellgrhjrom
production ofcoUagen strands, so that tile wound a. black wound edges
e. tap water edges are begi"ning to be held together by tissue 59. What is the most appropriate suture pattern to b. bluish-purple wound edges

o
as well as sutures? use in closing an incision of the urinary bladder? c. gray wound edges
48. Which incisioll provides the best ouerall exposure
of the abdominal cavit),?
a.
b.
6 to 8 hours
4 to 6 days
•. Cushing
b. horizontal mattress
d.
e.
pink wound edges
white wound edges

alk
a. dorsal midline c. 12 to 14 days c. s imple continuous
b. flank d. 24 to 26 days d. simple interrupted 65. What portiol! ofa surgical gown is considered
c. paracosIaI e. 28 to 30 days e. vertical mattress sterile during stlrgery?
d. paramedian a. e ntire o utside of the gown
e. vent ral midline 54. WOllnd cOrUrac(iofl is produced hy: b. front and sides of the gown, from the neck to
60. Suture material used to close a surgicaluJOurld
represents what kind of irritant to hody tissues? the bottom, including the arms
a. movement of only the dermis
49. What is (lie most appropriate suture pattern to a. chemical
c. fron t of the gown, from the neck to the
close an incision ill all allimal's stomach? b. movement of only the epidermis w. bottom, including the arms
•. horizontal mattress
b. Lembert
c.
d.
movement of a ll layers of the skin
reproduction of epidermal cells
b. infectious
c. photic
d. front and sides of the gown, from the neck to
the waist
e. reproduction of all skin cells d. phys ical e. front of the gown, from the waist up,
c. purse-suing e. thermal including the arms
d . simple interrupted
55. As a part ofeffective aseptic technique, surgical
e. vertical manress
ww
gowns: 61. Mlal is the usual sigttificance ofa small seroma 66. Which bacterial form is most easily destroyed by
deep to (beneath) the skin suture line after aseptic commoll sterilization methods?
a. are commonly made of cloth or paper
50. Wllich operating room persOlmel should try to face surgery?
alOOY from sterile fields durillg a surgical procedure? b. a re put on by touching only the outside a. spores of aerobes
c. are routinely sterilized by ethylene oxide gas a. acute emergency b. hyphated form
a. all personnel b. cosmetic problem only
d. do not need to be sterile, only clean c. dormant form
b. nonscrubbed personnel only c. minor significance
e. protect against contamination from the waist d. spores or anaerobes
c. scrubbed personnel only down e. vegetative form

e 1998 Mosby-Year Book, Inc. Photocopying is prohihited hy law. Correct answers are on pages 195-198.
www.alkottob.com
192 SECT ION 8 www.vet4arab.co.cc Principles of Sll rgery 193

67. What is theftrsl phase a/wound healing? c. gastroscopy 78. When not otherwise occupied, scrubbed surgical 83. Wllicll type of needle is most appropriate for
a. epithelial phase d. gastrostomy personnel should stand with their: suturing a ligament?
b. fibrob last phase e. gastrotomy a. arms folded across the chest a. blunt

om
c. innammalory phase b. hands held apart and above shoulder level b. cutting
d. malUration phase 73. W11a1 is tile correct surgical tenn for creation ofa c. hands clasped between waist and shoulder c. reverse cuning
e. scarring phase pemJQnellt arrificial opening illto the esophagus? level d. tapered
a. esophagectomy d. hands held down and to each side e. trocar
68. In theftrsl 24 hOllrs o/primary union wound b. esopbagopexy e. hands on the surgery table
healing, most of the resistance co opening a/the c. esophagoscopy 84. Wlliell type of needle is most appropriate/or
sutured WQund is provided by: d. esophagostomy 79. During surgery, when is it permissible for suturing tile uterus?

b.c
a. collagen strands c. esopbagotomy nOllScrllbbed surgical personnel to pass between
scrubbed personnel and the patient?
a. blunt
b. fib rin strands b. cuning
c. fibroblasts 74. With which type a/abdominal incision can the a. at any convenient time c. reverse cutting
d. granulation tissue abdominal wall be most effectively closed using a b. never d. tapered
c. sutures single layer a/sutures? c. when opening suture material c. trocar
a. high flank d. when adjusting the anesthesia machine
69. If there are 110 complications, how long after b. low flank e. when adjusting the intravenous drip
85. Surgical removal o/a ruptured spleen is an
surgery should skin sutures generally be removed? c. paracostal

tto
example of
a.2to3days d. paramedian 80. Wllell aseptically opening a sterile surgical pack
a. cosmetic surgery
b. 4 10 5 days e. ventral midline 0" an instrument stand, it is not proper for
1I0nscmbbed surgical personnel to toucll tile: b. elective surgery
c. 71010 days c. eme rgency surgery
d. 1510 17 days 75. Scrubbed surgical personnel become a. autoclave tape
d. exploratory surgery
e. 18102 1 days contaminated iftlley touch: b. conte nts of the pack
e. first- intention surgery
a. objects in sterile fie lds c. corners of the wrap

o
70. What is the correct surgical term for incision of b. objects outside the sterile zone d. instrument stand
e. outside of the wrap 86. Removal ofa large skin tumor has left a large skill
the urinary bladder? c. properly stcrilized s urgical instruments defect to be closed. Which suture pattern is least
d. sterile objects

alk
a. cystectomy likely to cause skin tearing when large wounds
b. cystopexy e. fresh ly exposed tissues of the patient 81. Wlljcll characteristic applies to ethylene oxide are closed under tension?
gas?
c. cyslOscopy a. Cushi ng
d. cystostomy 76. NOr/scrubbed surgical personnel may properly a. flammable b. horizontal mattress
e. cystoto my touch anything that is: b. Exposure is not considered a health hazard. c. Lembert
a. contaminated c. noncombustible d. purse-string
71. What is ti,e correCl surgical term for removal of b. inside the patient d. nontoxic to tissues e. simple interrupted
the kidney? c. inside the stcrilc zone w. e. safe to breathe
a. nephrectomy d. part of a sterile field 87. Wllicll of tile following is lIot a likely contributor
b. nephropexy e. sterile 82. WI/icll type of needle is most appropriate for to dehiscellceof an abdominal surgical incision?
suturing muscle?
c. nephroscopy a. chronic vomiting
d. nephrostomy 77. How should scrubbed personnel pass each other a. blunt b. internal su ture ends cut too short
e. nephrotomy in the operating room? b. cutting c. infection
ww
a. any way that is convenient c. reverse cutting d. skin sutures left in place too long
72. What is the correct surgical tenn for sUlUring the b. back to back d. tapered e. suture material of too-small diameter
stomach to the body wall to fIX the stomach in c. back to fronl e. trOcar
place? d. front to back
a. gastrectomy e. front to front
b. gastropexy

@ 1998 Mosby-Year Book, Inc. Pllotocopying is prohibited by law. OJrrect answers are on pages 195-198.
www.alkottob.com
194 SECTI ON 8
www.vet4arab.co.cc Principles o/Surgery 195

88. Which type ofdressing, when removed, provides 93. Concerning the principles ofcryosurgery, which 98. The use afextreme cold to destroy unwanted c. nylon
tile least traumatic and least irritating means of statement is least accurate? tissue is termed: d. polypropylene
debriding a wound with extensive tissue damage? e. s ilk
a. Frozen tissues should be thawed slowly. a. cosmetic surgery

om
a. dry gauze b. Uttle aftercare is required. b. cryosurgery
b. dry nonadhesive pad c. Multiple freeze-thaw cycles should be applied. c. elective surgery 100. To be classified as nonabsorbable, suture
c. gauze dressin g with an oily antiseptic d. Tissues should be frozen to - 25 0 C. d. orthopedic surgery material must maimain its tensile strength in
d. gauze dressi ng with a water-soluble an tiseptic e. Tissues should be frozen rapidly. e. prophylactic surgery tissue for longer than.'
e. wet saline dressing a. 30 days
94. How long after surgery does the strength ofa 99. Which suture material is badly damaged by steam b. 60 days
89. Most o/che clinical signs seen in animals in shock sutured surgical skin wound return to its original sterilization? c. 90 days
related to excessive blood loss are attributable to: preoperative strength?

b.c
a. polyglycolic acid d. 120 days
a. acidosis a. 7 to 10 days b. polyester e. IBO days
b. alkalosis b. 2 1 to 28 days
c. cell dealh c. 60 days
d. redistribution of blood flow d. 2 to 3 years
e. tissue hyperoxia e. never
Answers
I. c The instrument is named for its fun ction of 12. a Steam sterilization in an autoclave is most
90. W1lich ollhe/allowing is a noncapillary suture 95. In second-imention healing, which of the refracting organs and tissues oul of the way. commonJy used for instruments and equipment

tto
material suitable for skin closure? following must he present before wound 2. d None ofthe other instruments listed has not damaged by moisture or heal.
contraction orepirheUal regeneration can occur? opposing jaws. 13. e The sharp edges of scissors are dulled by
a. braided conon
a. collagen fibers 3. c Declawing involves removal (-ectomy) ofthe steam in an a utoclave. Boiling and dry heat are
h. braided potyglycolic acid
nails or claws (oflych -). not sufficiently effective. The expense and
c. braided silk b. exudative tissue
4. d A tapered-po int needle easily passes through hazards of ethylene oxide are not warranted.
d. monofilament stainless steel c. fi brin clot
soft organs, making a tunnel through which the 14. a The minimal standard for sterilization of
e. monofilament coUon d. granulation tissue

o
suture material is drawn. surgical instruments in an autoclave is 121 0 C
e. scar tissue (250° F) for at least 15 minutes.
5. c Th e ~- circle needle offers the best compromise
91 . The main goal of surgery to remove a pus-filled of shape. allowing use in both shallow and deep 15. c Chemical autoclave ind icators are the only
uterus (pyometra) is to: 96. When putting on sterifeglovesJor aseptic surgery, incisions. type listed that can give inunediate information

alk
which of the fallowing is not permitted? 6. a An eye with proptosis must be returned to the on all three basic criteria for autoclave
a. prevent subsequent pregnancy sterilization (presence of steam at the proper
a. touching one gloved thumb with the other eye socket quickly to prevent permanent damage
b. alter the behavior of the animal combination of exposure time and temperature).
gloved thumb to the eye.
c. make a diagnosis 16. a Microorganisms in a wound during surgery are
b. touching the outside of the glove with 7. a The basic requirements for healing by first
d. restore the animal to a normal reproductive intention are minimal tissue damage and considered contaminants until, or unless, they
state scrubbed fingers multiply and cause damage.
apposition of the edges of the wound. usually
e. restore health despite loss of normal c. touching the outside of the gown cuff with the 17. d A surgical mask does not come in contact with
with sutures.
reproductive funct ion inside of the glove cuff anything sterile during a surgical procedure, so it
w. B. d The basic functional unit of the nervous
d. touching the outside of one glove with the need onJy be clean.
system. the neuron. is incapable of reproduction.
outside of the other glove lB. e An autoclave sterilizes by exposing packs to
92. With what kind oJknol should sutures be so damage to the nervous system is often
e. touching the ins ide of the glove cuff with repaired by scar tissue. The other organs and steam under pressure.
routinely tied?
scrubbed fingers tissues listed have excellent healing pOlential. 19. d A #40 clipper blade is a "surgical" blade. It clips
a. bowline the hair off at the skin surface.
9. a Debridement of a wound facilitates healing by
b. granny knot 97. How should packs be placed in an autoclave for minimizing the amount of inflammation 20. b Numbered suture sizes (e.g., #2) decrease in
ww
c. square knot sterilization? necessary before filling of the defect can begin. size as the number gets smaller. down to size O.
d. slip knot 10. e The xiphoid process and umbilicus are both From that p oint on the sizes get smaller as the
a. diagonally
e. half hitch on the animal's ventral midline, number of Os (or the number in front.ofthe 0)
b. horizontally increases.
1 L d An electric drill would be damaged or
c. tightly packed 21. d All the other suture materials listed are either
inadequately sterilized by any of the other
d. unwrapped methods. braided or twisted and have the potential for
e. vertically considerable capillary action.

© 1998 Mosby-Year Book. Inc. Ph otocopying is prohibited by law.


www.alkottob.com
196 SECTION 8
www.vet4arab.co.cc Principles ofSllrgery 197

22. e Surgical gut is absorbed by the body. 38. a The surgical cap does not come in contact 54. c Wound contraction represents movement of 73. d The suffix -ostomy means to create an artificial
23. e Treatment with chromic acid delays with tissues of the patient direcdy or indirectly, the entire thickness of the skin toward the center opening in an organ or tissue.
absorption of surgical gut by the body. so it need only be clean, not sterile. of the wound. 74. e The linea alba, on the ventral midline afme

om
24. b Curettage involves the scraping of a tissue or 39. c Uquid chemical sterilization does not dull 55. a All other choices are incorrect. abdominal muscle wall, is me tendinous
cavity. sharp edges. 56. d All other choices are smaller. attachment of the ventral abdominal muscles.
25. e Use of overly large suture material would not 40. e Warming the solulion accelerates the chemical 57. c All other choices are from natural sources. One layer of sutures in this area effectively closes
cause wound dehiscence. It would actually reactions necessary to kill microorganisms. 58. a A patient in shock would show rapid, shallow the whole thickness of the abdominal wall, after
prOvide greater holding power than smaller 41. a The relative size of the wire, as measured by breathi ng in an effort to oxygenate the blood as which the skin is closed.
suture material. gauge, is inversely proportional to the gauge rapidJy as possible. 75. b Anything outside the s terile zone in an
26. b Contaminated items appear identical to sterile number. For example, 40-gauge wire is smaller 59. a The Cushing pattern is the only one listed that operating room is considered contaminated.
items. If there is any doubt about the sterility of than 32-gauge wire. is an inverting pattern appropriate for closure of 76. a Nonscrubbed personnel should only touch
an item, it must be considered contaminated. 42. e Cloth and paper drapes are subject to capillary things thai are not sterile.

b.c
a hollow organ.
27. e All the other patterns are inverting or action. Plastic drapes are not. 60. d Suture material aClS as a physical irritant until 77. b Passi ng back to back prevents accidental
appositional. 43. e From largest to smallest, these sizes are ranked it is absorbed, removed, or encapsulated wilh contamination oflhe front, sterile portions of
28. b The other patterns are inverting (mattress as follows: #4, H3, HI, 2-0, and 4-0. 3-0 is m idway scar tissue. their gown.
patterns), appositional (simple pattern), or used in size between 2-0 and 4-0. 61. b Unless they are very large or ruptured, 78. c The bands of scrubbed personnel should always
only to close an orifice (purse-string). 44. e All the other su ture materials listed are postoperative seromas are unsightly but of little be held between waist level and shoulder level to
29. c All the other choices promote increased nonabsorbable. other importance to the animal's health. help prevent inadvertent contamination. Clasping
scarring or early disruption of the wound. 45. d Steam dulls the sharp edges of scissors. 62. d The simple pattern is an appositional pattern. the hands, when not mheIWise occupied, helps
30. b Exuberant granulation tissue (proud flesh) 46. b Elective surge ry is done by choice, so it can be It brings the incision edges together without prevent fatigue from compromising the position of

tto
acts to block wound healing and epithelial performed when condjtions are most inverting or everting them. the hands and arms.
regeneration. The other choices would likely appropriate. 63. a Bone has excellent healing capadties, 79. b Nonscrubbed personnel should never violate
enhance wound healing. 47. b The other listed solutions are irritating to the provided the fracture fragments are properly the sterile zone in which scrubbed personnel are
31. d One of the most important characteristics of tissues or are not isotonic with the patient's aligned and movement is kept 10 a minimum . working.
wound healing by first intention is tissue fluid s. 64. b Bluish-purple wound edges indicate that 80. b The sterility of the pack contents would be
approximation of the wound edges. 48. e The ventral midJine approach provides the blood vessels in and under the skin are des troyed if touched by a nonscrubbed person .
32. e The ventral m idline approach to the abdomen congested with blood. 81. a Ethylene oxide gas is very flammab le.

o
most elCtensive access to the abdominal cavity.
gives the most extensive access to the abdominal 49. b Incisions in viscera are best closed with 65. e This is the only portion of a surgical gown that 82. d A tapered-point needle easily passes through
cavity. inverting suture patterns. The other listed is considered sterile during surgery. muscle, making a tunnel through which the
33. a Slight elevation of body temperature for 1 or 2 patterns are everting, appositional or 66. e The vegetative bacterial form is the actively suture material is drawn.

alk
days is nonnal after major surgery. The other inappropriate for wound closure. feeding, growing, reproducing form. It is most 83. e A reverse-culting needle cuts a tunnel through
choices are all early indicators of wound 50. e All personnel in the operating room should easily destroyed by common sterilization and the tough tissue of a ligament that is less likely to
dehiscence. face toward sterile fields so they are aware of disinfection methods. tear through than the tunnel created by a
34. e Prevention of surgical wound contamination is their relationship to them. 67. c Inflammation is the first step in wound standard (inside-curve) cutting needle. The other
the whole purpose of aseptic technique in the 5 1. a Dehiscence of all layers of the body wall healing. It "cleans up" the damage so the defect needle points would not easily pass through this
operating room. exposes abdominal viscera. Repair must be can be repaired by the balance of the healing tough tissue.
35. a This is the only choice that can be influenced immediate to prevent serious damage to process. 84. d A tapered-paint needle easily passes through
by aseptic technique. The others are inhe rent to abdominal structures. 68. e Other than su tures, a surgical wound has no muscle, making a tunnel through which the
the patient, the surgical procedure being
performed, or the microorganisms in the
environment.
36. a The antimicrobial effect of a surgical scrub
w.
52. d Inflammation results from any insult to the
body, whether intentional (surgical) or
unintentional {traumatic, infectious}. Increased
appreciable strength until significant numbers of
collagen fibers arc produced at 4 to 6 days.
69. c Before 7 days, the wound may not have
suture material is drawn.
85. e Splenic rupture is a potentially life-threatening
condition. If surgery is indicated, it must be
performed immediately.
blood supply to an inflamed area produces the enough strength to resist separation. After 10
depends on sufficient exposure of the skin to the increased warmth of the area. Good surgical days, inflammatory reaction to the sUlUre 86. b Mattress suture patterns spread the tension
soap, as well as the scrubbing action that loosens technique minimizes inflammation but does not material may cause significant scarring. created by each suture over a broad area and are
ww
dead skin and debris and works the soap down eliminate it. 70. e The suffix -oromy means to male an incision less likely to tear out because of tension on the
into the cracks and crevices of the skin. 53. b It takes 4 to 6 days for production of collagen into something. suture line.
37. d Hexachlorophene fonns a bacteriostatic film strands in a wound to reach a significant level. 71. a The suffix -ectomy means to s urgically remove 87. d Leaving skin sutures in place too long
on the skin if used exclusively to wash the hands Until that time, tlle wound is held together by something. increases scarring but does nOl directly
and anns. Other soaps remove the protective film. sutures. contribute to breakdown of the surgical wound.
72. b The suffix -pexy means to fix something in
place.

e 1998 Mosby-Year Book, Inc. Photocopying is prollibited by law.


www.alkottob.com
198 SECTION 8
www.vet4arab.co.cc SECTION

9
88. e Wet saJine dressings are useful to help debride 94. e The strength of the scar that res ults from
wounds with extensive tissue damage. They heaJing of a surgicaJ skin wound never reaches
absorb and remove inflammatory products from that of the normal skin around it.

om
the wound. 95. d After dead and damaged tissue has been
89. d Redistribution of blood flow results in the pale removed from a wound by inflammation, a bed
mucous membranes, poor capiUary refill, and of granulation tissue, consisting primarily of
cold extremities seen in shock.
90. d The other suture materials listed are braided
or twisted and can conduct fluid and
collagen fibers and capillaries, must form on
the floor of the wound so that the processes that
reduce the size of the wound can begin.
Theriogenology
microorganisms by capillary action from the 96. b If the outside of the glove is touched by W.F. Braun, Jr., T.J. Burke, l.A. Dierauf, M. Drost, B.E. Eilts,
surface of the sldn to the deeper layers. Cotton anything that is not sterile, including freshly
scrubbed fingers, it becomes contaminated and
F.l. Frye, T. Gemeinhardt, S. Hudelson, D.G. Huff, J.D. Letcher,
(answer choice e) is not available as

b.c
monofilament. must not be used for surgery. S. Mobini, D.H. Nielsen, M.B. Paster, D.O. Schaeffer, R.C. Tubbs
91. e Removal ofthe ut erus may restore health, but 97. e Packs placed verticaJly in the autoclave
it rusa precludes future breeding. receive the best circulation of steam around
92. c A square knot is conve niently tied and their contents.
provides a very secure knOl. 98. b The prefix cryo- m eans cold. Recommended Reading
93. b The principal action of cryosurgery is 99. a Polyglycolic acid should never be sterilized in Altman RB et al: Avian medicine and surgery, Philadelphia. 1997. WB Saunders.
destruction of unwa nted tissue by freezing. This an autoclave.
Feldman EC, Nelson RW: Canine and feline endocrinology and reproduction . cd 2. Philadelphia,
leaves dead tissue that must be liquefied and lOll b Suture material is considered nonabsorbable 1996. WB Saunders.

tto
removed by inflammation. Such areas must be if it retains its tensile strength for at least 60
monitored closely and kept clean, and frequently Frye FI.: Biomedical and surgical aspects o/captive reptile lIusbandry, ed 2, Melbourne. Fla. 1991,
days after implantation in tissue.
Krieger Publishing.
they require bandaging.
Hafez ESE: Reproduction in /ann animals. ed 6, Baltimore, 1993, Williams & \Vilkins.
Johnston SD. Olsen PN: Canine&/eline reproduction, Philadelphia. 1997. \VB Saunders.
Leman AD et a1: Diseaseso/swine. ed 7. Ames, Iowa, 1992. Iowa State University Press.
NOTES Mader DR: Reptile medicine and surgery, Philadelphia. 1996. WB Saunders.

o
McKinnon A, Voss JI.: £quille reproduction. Baltimore, 1992, Williams & WLikins.
Morrow DA: Current tllerapy in theriogenology, cd 2. Philadelphia, 1986, WB Saunders.
Rosskopr n, Woerpcl R: Diseases 0/ cage and aviary birds. cd 3. Baltimore, 1996. Williams & Wilkins.

alk
Varner DD et a1: Manual 0/ equine reproduction, St. Louis. 1998, Mosby.
PetcrsAR, Ball PJ: Reproduction in cattle. Ames, Iowa, 1995. Iowa State University Press.
Radostits OM et a1: Veterinary medicine, ed 8, Philadelphia. 1994. WB Saunders.
Youngquist RS: Current therapy in large animal theriogello/Qgy, Philadelphia. 1997. WB Saunders.

Practice answer sheets are on pages 271 ·272.

w. DOGS AND CATS


T.J . Burke
Questions
ww
J. The estrous cycle oftile auerage bitch is best c. nonseasonally polyestrous
classified as: d. nonseasonally monestrous
a. seasonally polyeslfous e. a blend of seasonally and nonseasonally
b. seasonaJly monestrous polyestrous

10 1998 Mosby-Year Book, Inc. Photocopying is prolJibited by law. Correct answers are on pIlges 234-244. 199
www.alkottob.com
200 SECTION 9
www.vet4arab.co.cc Tlleriogeno logy 201

2. The estrolts cycle of the average queen exposed to 7. In tile hitch, which hormone is present during 12. Which o/the/ollowing is approved in tile United 17. In vaginal cytology, which cell predominates
natural daylight is best cllillified as: pregnancy but Itot during pseudopregnancy? States for prevention 0/estrus in cats? during standing heat (estrus) in the bitch?
a. seasonally polyestrous a. estradiol·1 7j3, a. mibolerone a. para basal epithelial cell

om
b. seasonally monestrous b . relaxin b. megestrol acetate b. uncornified epithelial cell
c. nonseasonally polyestrous c. prostaglandin Fz.. c. diedlylstilbestrol c. s uperficiaJ epithelial cell
d. nonseasonaJly monestrous d. progesterone d. methyltes tosterone d. erythrocyte
e. a blend of seasonally and nonseasonally e. luteinizing hormone e. None of the drugs listed have been approved e. polymorphonuclear cell
polyestrous for this purpose in cats.
8. Among bitches ofall breeds, including mongrels, 18. Estrogenic hormones are frequently used in
3. Mll'e/l dog breed normally exhibits only one which stage ofrheestrous cycle is tile most 13. In dogs, final maturation o/spermatozoa occurs treatmem ofmismating in the bitch. Concerning
estrus per year? variable in lellgtM ill the: such treatment, which statement is least

b.c
accurate?
a. German shepherd a. proestrus a. seminiferous tubule
b. minia ture poodle b. estrus b. rete testis a. Bitches under 4 years of age are less likely to
c. basset hound c. metestrus c. epididymis su ffe r bone marrow toxiciry.
d. basenji d. diestrus d. vas deferens b. This treatment increases the incidence of
uterine infection by about 30%.
e. great Dane e. anestrus e. seminal vesicles
c. Treated bitches remain in heat longer than
normal.
4. Wlljeh of the following is a reliable indication of 9. Which of rile following is least likely co terminate 14. Theoptimllm time to inseminate a bitch is:

tto
d. The treatment causes no adverse side effects.
ovulation ill the birch? standing heat;1I a queell?
a. 2 to 3 days before ovulation e. The success rate is directly related to dle time
a. drop in rectal temperature below lOlO F a. mating with a fCltile tom b. 4 to 6 days before ovulation between mismating and the onset of therapy.
b. change in color of the sanguineous vulvar b. mating widl an infertile tom c. I to 2 days after ovulation
discharge c. injection of 0.25 mg of es tradiol cypionate d. 4 to 6 days after ovulation 19. III medical treatment of pyometra, which of the
c. glucose in Ihe vaginal [Juid d. oral administration of 5 mg of megestrol e. 8 to 9 days after ovulation following has shown the best results in emptying
d . rise in serum progesterone above baseline acetate daily for 10 days the uterus o/pllrulent contents?

ko
levels e. obtaining a deep vaginal sample fo r culture,
15. A normal stud dog of which breed produces all a. estradiol-17j3
e. standing for mounting by the male using a sterile swab
ejaculate containing the grea:est Ilumberofspeml? b. progesterone
a. miniature schnauzer c. oxytocin
5. Which of the following isa reliable indication 10. Wllieh oftllefollowillg is approved in the United d. prostaglandin Fz..
that wllelpirlg will ensue within 24 hours? States for prevelltion ofestrus ill the bitch, once b. German shepherd
c. basenji e. calcium gluconate
proestrlls has begun?
a. rise in rectal tcmperature above 102.5 F
0

al
d. great Dane
b. drop in rectal temperature to 100.50 F a. m ibolerone
e. Sperm numbers are about the same in all 20. Wh ich of the following is not a usual/eature of
c. drop in serum progesterone to baseline levels b. megestrol ace1"ate Brucella canis infection ill dogs?
these breeds.
d. clear mucoid vulvar discharge c. estradiol cypionate
a. feve r
e. food refusal d. methyltcstostcrone
16. A normal stlld dog o/whicll breed produces all b. prolonged bacteremia

6. Which of the following is mosllikely to detect


pregllallcyat 18 days in the bitc1t?
e. prostaglandin F2"
w.
11. Wh ich of tile following is approved ill the United
ejaculate concaining tile greatest concentration
sperm?
0/ c.
d.
epididymitis
abortion
a. miniature schnauzer e. mild generalized lymphadenopathy
States for prolonged lise (lip to 2 years) in
a. B-mode ultrasonography b. German shepherd
preventioll ofestr·us ;n the biecl/?
b. Doppler ultrasonography c. basenji
c. routine radiography a. mibolerone
ww
d. great Dane
d. palpation b. megestrol acetate
e. Sperm concentration is about the same in all
e. serum progesterone determination c. estTadiol cypionate these breeds.
d. methyltestosterone
e. diethylstilbestrol

0 1998 Mosby-Year Book, Inc. Photocopyillg is pro/dbited by Inw. Conect answers are on pages 234-244.

www.alkottob.com
202 SECTION 9
www.vet4arab.co.cc Theriogenoiogy 203

21. Which of the following, used alorle, can confirm a c. a freemartin 30. The ownerofa 3-year-old masti/fbitch says, "The d. testicular Jamshidi needle biopsy
diagnosis a/brucellosis in dogs? d . a monorchid dog has been in heat for the last 9 weeks." The dog e. testicular Franklin-modified Vim-Silverman
has a slightly edematous vulva. Vaginal cytologic needle biopsy
e. infertile

om
a. history of abortion during (he last trimester of examination reveals no erythrocytes or leukocytes,
pregnancy and the epithelial cells are all intermediate to
h. blood culture 26. Cryptorchidism in dogs is: superficial cells. The most likely cause of these 35. Radiography may help determine the viability of
c. tube agglutination test findings is: late-term fetuses. Which of the following is not a
a. often a cause of colic in affected neonates radiographic sign offetal death?
d. tube agglutination test using 2-mercaptoethanol b. a recessive genetic defect a. normal estrus
e. rapid slide agglutination test a. intrauterine gas
c. unrelated to other health problems later in life b. normal metestrus
b. absence of molar teeth in the fetal jaws
d. less common than in cats c. pituitary adenoma
22. Concerning brucellosis in dogs. which statement c. malalignment of cranial bones, with overlap
e. readily diagnosed at birth in large breeds d. vaginitis
d. fetal subcutaneous gas

b.c
is most accurate? e. cystic ovaries
e. hyperextension of the fetal bodies
a. A combination of tetracycline hydrochloride 27. A l-year-oldAKC-registered male German
and dihydrostreptomycin will cure an infected shepherd is presented to you. On physical 31. Which of the following is not a common cause of
dog. examination, you can locate only one scrotal testis. abortion, stillbirth, or fetal mummification in the 36. A 2B-momh-old beagle bitch has had three
b. Vaccination of at least 85% of the dogs in a The dog has had no previous surgery. The most queen? normal heats. At the last two heats, the owner
kennel will halt an outbreak. appropriate recommendation to the owner is to: attempted to mate her with a proven male on the
a. feline calicivirus infection thirteenth and fifteenth days, respectively, after he
c. All dogs exposed to the organism become a. have the scrotal testis removed
infected. b. feline herpesvirus infection first noticed blood drippingfrom the vulva. When
b. have the cryptorchid testis removed the male mounted the bitch, she yelped, bit the

tto
d. Infected dogs may show no clinical signs. c. feline parvovirus infection
c. have both testes removed d. feline leukemia virus infection male, and ran behind the owner. The most likely
e. Blood cultures within 2 weeks of infection will cause of these nonproductive breedings is:
detect infected dogs. d. replace the cryptorchid testis to a scrotal e. feline corona virus (nonemeric) infection
position by orchiopexy a. inexperience of the male
e. replace the cryptorchid testis to a scrotal b. preputial adhesions
23. In assessing sperm morphology, which of the 32. Concerning uterine subinvolution, which
position with gonadotropin therapy c. uterine hypoplasia
following is not considered a primary defect? statement is most accurate?
d. vaginal stricture

o
a. tightly coiled tail a. It is effectively treated with several
28. A 2-year-old miniature schnauzer has an empty e. breeding too early in the cycle
b. double head progesterone injections.
scrotum and an underdeveloped prepuce. During
c. balloon head exploratory celiotomy, you discover a uterus and b. It is common in primiparous queens.
c. H can be prevented by injection of oxytOCin 37. Transmissible venereal CIImors ofdogs:

alk
d. double midpiece two gonads that, on histologic examination, are
identified as ovaries. Tllis dog is most \vithin 24 hours after parturition. a. are caused by a virus
e. detached head
appropriately classified as a: d. It is common in primiparous bitches. b. are nearly always malignant
a. female pseudohermaphrodite e. It is the first sign of pyometra. c. are transmitted only during coitus
24. fnthe bitch. protrusion ofthefloor of the caudlll
vagina and vestibule through the labia during b. male pseudohermaphrodite d. sometimes spontaneously regress
proestrus/estrus: c. bilateral cryptorchid 33. Bacterial prostatitis: e. do not respond to radiation therapy
a. is not unus ual and is clinically normal d. hermaphrodite a. is uncommon in older male cats
b.
c.
d.
indicates the bitch is ready to breed
will not regress during metestrus/diestrus
indicates uterine prolapse
e. semihermaphrodite w.
29. The most common endocrine cause of subfertility
and infertility in the birch is:
b. produces azoospermia
c. produces oligospermia
d. interfe res with copulation
e. has little effect on fertility
3B. An B-year-old cryptorchid standard schnauzer has
had no previous surgery other than standard
puppy cosmetic procedures. The dog has shown
abdominal discomfort for "about a dlly." Physical
e. indicates vaginal hyperplasia examination reveals a firm, nonpainful
a. hypogonadism abdominal mass about 6 cm in diameter and a
25. A lO-month-old male toy poodle with only one b. hyperadrenocorticism 34. Which technique yields the best results in softer B-cm mass near the pelvic inlet. You suspect
ww
palpable testis in the scrotum is best described as: c. hypothyroidism determining the cause ofazoospermia in a dog neoplasia involving a retained testis. Which tumor
d. hyperthyroidism with bilateral testicular atrophy? is most likely to befound in this dog?
a. normal
b. a unilateral cryptorchid e. cystic ovaries a. testicular fine -needle aspirate a. Sertoli-cell tumor
b. testicular wedge biopsy h. interstitial-cell tumor
c. testicular Vim-Silverman needle biopsy c. Leydig-cell tumor
d. granulosa-cell tumor
e. seminoma

© 1998 Mosby-Year Book, Inc. Photocopying is prohibited by law. Correct answers are on pages 234-244.
www.alkottob.com
204 SECTION 9
www.vet4arab.co.cc Theriogenology 205

39. Which tumor is most likely to produce bone 44. On microscopic examiflation of vaginal smears 48. Properly frozerl canine semen can still fertilize ova c. 48 hours
marrow dysplasia? from a birch, erythrocytes: after storage in liquid nitrogen for: d. 96hours
a. Sertati-cell tumor a. are present only during proestrus a. up to 6 mo nths e. I week

om
b. interstitial-cell tumor b. are no longer seen after ovulation occurs b. up to 1 year
c. Leydig-cell tumor c. are present throughout the cycle c. up to 3 years 50. Concerning cats with a tortoiseshell hair-coat
d. granulosa-cell tumor d. do not indicate the stage of the cycle d. up to 6 years color pattern. which statement is least accurate?
e. seminoma e. are nucleated until ovulation e. longer than 10 years a. Most of them are females.
b. Most males with a tortoiseshell coat are fertil e.
40. Routine aerobic cultures of swab samples taken 45. You spay a 6-month-old mixed-breed bitch. and 49. Sperm remain viable in fresh chilled-extended c. Orange and black colors are related to alleles
from the caudal vagina ofa normal bitch are 4U years lacer tile angry owner presents the dogfor canine semen for approximately: found only on X chromosomes.
examination. The owner insists the dog had a

b.c
least likely to reveal: a. 6 hours d . Ma1es with a tortoiseshell coat are usually XXY.
heat cycle about J month previously and may e. Chimeric males may be fertile.
a. Escherichio. coli have had another heat during the past year. b. 24 hours
h. Streptococcus [ecaUs Assuming the owner is correct, which stateme1lt is
c. j3-hemolytic Streptococcus most accurate?
d. Mycoplasma
e. Proteus vulgaris
a. The patient is likely to have an elevated serum
estradiol level.
CATTLE
b. The patient is likely to have an elevated serum M. Drost
41. Which afthe/allowing is not a sign afmale

tto
progesterone level.
For Questions 51 through 55, select the correct 57. Thecircufating level of which hormone rises
feminizing syndrome? c. The patient is likely to have answer from the five choices below. acutely during each estrous period?
a. pendulous prepuce hyperadrenocorticism.
d. Vaginal cytologic examination is likely to show a. anestrous a. luteinizing hormone
b. gynecomastia
a preponderance of supe rficial cells. b. estrous b. estrogen
c. hypopigmentation of the skin
e. Ultrasonography is likely to reveal a cystic c. metestrous c. prostaglandin F2"
d. fem ale posture (squatting) when urinating
ova'Y. d. m onestrous d. progesterone

o
e. symmetric alopecia
e. polyestrous e. fo llicle-stimulating hormone
46. A bitch that was presumably spayed but rlOW
42. Which of the following is a cricerion indicating shows signs of heat mostUkely has: 51. Having a single estrus during a breeding season 58. An advamage of nonsurgical collection of bovine
dystocia?

alk
a. hyperadrenocorticism embryos oller surgical collection of embryos is that
a. 1 hour since birth of the previous fetus nOrlSllrgical collection:
b. a pituitary adenoma 52. Not cycling
b. brachycephalic breed a. requires little skill
c. aright ·sided ovarian remnant
c. 72 days since the last breeding, with b. can be done repeatedly in the same donor,
d. a left-sided ovarian remnant 53. Having recurring periods ofestrus during a
pregnancy confirmed without complications
e. an ectopic ovary breeding season
d . strong co ntractions for 15 minutes c. can be done earlier after the onset of estrus in
e. male was larger than the female the do nor
47. Which of the following will riot reliably termiflate
w. 54. Recently in heat
pregflancy in the bitch? d. requires a smaller volume of flushing medium
43. A dog with bilateral aplasia of the vasa deferentia e. permits accurate counting of corpora lutea
is likely to have: a. progesterone 55. In heat per rectum
b. ovariectomy
a. azoospermia
c. prostaglandin Fz., 56. Estrogen is the predominant hormone during: 59. Progesterone is the predominant hormone during:
b. oligospermia
d. dexameth asone
c. necrozoospennia a. pregnancy a. anestrus
ww
e. hysterectomy
d. low volume of ejaculates b. anestrus b. estrus
e. decreased libido c. diestrus c. proestrus
d. me testrus d. diestrus
e. estrus e. metestrus

© 1998 Mosby-Year Book. Inc. Photocopying is prohibited by law. Correct answers are on pages 234·244.
www.alkottob.com
206 SECT ION 9
www.vet4arab.co.cc Tll eriogenology 207

6G. During prolonged dystocia j" a cow, tile calfs 64. As a species, cattle are: 69. Prostaglandin Fa. induces estrus in cows Dilly c. check the dam for trauma to the birth canal
pelvis becomes lodged in file dam's birt" canal wilen givell during: d. treat the calf's umbilical cord with tincture of
a. monestrous
(hip lock), and the cal/dies. Tile most appropriate a. anestrus iodine

om
courseofactioll is to: b. polytocous
b. diestrus e. check all four teats o n the dam for normal
c. uniparous
a. perform a cesarean section flow of colostrum
d. primiparous c. the immediate postpartum period
b. repel the calf and then then rotate its hips for e. multiparous d. metestrus
extraction e. proestrus 74. 111ere is 110 commercially auailable vaccine for
c. use the calf puller to forcibly extract the dead prevefl tiof! ofabortion caused by:
calf 65. Hydramllios differs from lIydrallamois in tlJQI
witli liydramnios: 70. During dystocia in a Holstein heifer, the delay if! a. brucellosis
d. pe rform a pubic symphysiotomy on the dam b. bovine virus diarrhea
spomafleous deliueryofa large calfis least likely
e. using a felotome. remove the craniaJ tw'O- a. there is more severe uterine and abdominaJ

b.c
to be attributable to: C. infectious bovine rhinotracheitis
thirds ofthe calf, and then divide the calf's
pelvis
distention
b. fetaJ deglutition or renal dysgenesis is the
underlying cause
•.
b.
the horizontaJ d iameter of the bony pelvis
incomplete dilation ofthe cervix
d. listeriosis
c. trichomoniasis
61. A farmer has a Hereford cow tllat cycles regularly. c. adventitious pl acentation or placental c. a co nstricted vagina
His cows rim witll a bull at all times. 011 rectal dysfun ction is the underlying cause d. a constricted vestibulovaginal junction 75. Postcoital pyometra is a virtually patllognomolll'c
palpation to check tile cow for pregnancy, you are
III/able {O retract the Ilterus caudally into the
pelvic catwl. The most likely explanation is that
d. excessive fetal fluid is quickly replaced after
percutaneous am niocentesis
e. circulatory collapse is a major risk on
e. constricted labia
..
sigl/ ofil/fectioll with:
Escherichia coli

tto
71. Euen practitioners experienced in bovine embryo b. Actillomyces pyogenes
the cow: cesarean section
transfer Iwuedifficulty with: c. Listeria monocytogenes
a. is 5 mon ths pregnant d. Campylabacter fetus var. fetus
a. achieving high fertilization rates
b. has pyometra 66. A deji"itiue sign ofestrus in cows is: c. Trirrichomonasferus
b. achievi ng a consistent superovulatory
c. has adhesions that are preventing movement a. mucous vulvar discharge response
of the uterus b. ruffled hair on the tail head c. recovering at least 60% of the embryos 76. 011 rectal examination ofa grade Jersey cow that
d. is carrying a mummified fetus c. a large follkle on the ovary has been lactaringfor 280 days, you detect a
d. cryopreservarion of quality embryos

o
e. is carrying a macerated fetus d. vulvar swelling mummified fews. T1le most appropriate course of
e. synchronizing the estrous cycles of donor and
e. standing to be mounted recipient cows action is to:
62. [n commercial esrablisl""eflts, embryos are a. infuse the uterus with mineraJ oil

alk
routinely recovered by nonsurgical methods. After b. administer 50 IU ofoxylocin intramuscularly
67. Teaser bulls witll surgically created lateral 72. The most common cause of poor reproductive
tlleOllse! ofestrus ill tlledollor. tile embryos are
deviation of the penis are least likely co transmit: performance in cows on large dairy farms is: twice daily for 3 days
recovered on day:
a. anestrus c. administer 25 mg of prostaglandin F2"o
a. II a. campylobacteriosis
intramuscularly
b. leptospirosis b. cystic fo llicles
b. 7 d. send the cow to slaughter
c. infectious bovine rhinouacheitis c. improper semen handling and poor artificial
c. 3 c. re move the fetus by hysterotomy
d. tuberculosis insemination technique
d.5
e. bovine virus diarrhea w. d. poor estrus detection
e. 9 77. A cow in late gestation has a 360-degree uterine
e. uterine infeC(ions
torsio". The most appropriate treatment is:
68. Wlla! is tile least developed accessory sex gland of
63. WlIa! is tile shortest pllase of tile estrous cycle a. cesarean section
bulls? 73. After assisting a heifer in delivery of a live calf, tile
ofcows? b. fetotomy
most appropriate next step is to:
a. estrus a. prostate gland rolling the cow (Schafer method)
c.
b. bulbourethral gland a. examine the dam for another calf
ww
b. metestrus d. detorsio n with a detorsion rod
c. seminal vesicular gland b. make sure the calf is breathing
c. diestrus e. uterine relaxant therapy (clenbuterol)
d. anestrus d. ampulla
e. proestrus e. body of the epididymis

10 1998 Mosby-Year Book, Inc. Photocopying is prohibited by law. Correct answers are on pages 234-244.
www.alkottob.com
206 SECTION 9
www.vet4arab.co.cc TlJeriogeno logy 209

78. There is an 80% chance that a cow is pregnant 83. Wllat is tile most dependable indication of 88. Tile corpus hemorrhagicum ofcows is: 93. fertilization takes place in the:
when progesterone levels in the milk are: pregnancy 011 rectal examination ofa cow 75 days
a. small and of a soft consistency a. uterine body
after artificial insemination?
a. high at the time of artificial insemination and

om
b. large and of a firm consistency b. cervix
high 21 days later a. gross asymmetry of the uterine horns c. small and of a firm consistency c. oviduct
b. low at the time of artificial insemination and b. palpation of a fetus d. large and of a soft consistency d. tip of the uterine horn
low 21 days later c. palpation of an amniotic vesicle e. larger than a corpus luteum e. tubo-uterine junction
c. high at lhe time of artificial insemination and d. fremitus in the uterine artery
low 22 days later e. fluctuation in the uterine hom ipsilateral to
d. low at the time of artificial insemination and 89. On any givell day, approximately how many 94. The adverse effecls ofsummer heat alld humidity
the corpus luteum
h igh 22 days later heifers ill a group of 100 Ilormaliycyclillg 15- Oil fertility in dairy cattle are most deleterious:
month·old heifers are ill heat?
e. low at the time of artificial insemination and a. during the first week ofgestation

b.c
84. To ensure maximal profit for tile producer, dairy
low 30 days later a. 5 b. at the time of fertilization
heifers sllould be bred:
b. 15 c. during the second trimester of gestation
a. to calve at 24 months of age c. 25
79. What is the incitienceoffreemarrinism in heifer d. during the last 2 weeks of gestation
callies born as a twirl with a bull calf? b. at 24 momhs of age d.35 e. in 14- to 17-month-old heifers
c. during the second or third heat after the onset e. 45
a. 25% of puberly
b. 40% d. to Angus bulls to minimize the incidence of 95. A cow witl! uterus unicorn is:
c. 55% dystocia 90. On reclal palpation the uterus of a cow pregnant
a. is sterile

tto
d.70% for 2 or 3 months call bedifferemiacedfrom a
e. no later than 9 months of age uterus of comparable size in a cow Wifh b. has long periods of anestrus
e.85% c. also bas only one ovary
pyometra by:
85. Aggressive rectal palpation can result in abortion
a. slipping the chorioallantoic membrane d. has an insufficient number of carunc1es to
80. Scrotal circumference is an indicator of by: sustain a pregnancy
b. asymmetry of the uterine horns
a. serving capacity of the bull a. causing one or more placentomes to e. is likely to develop hydrallantois
c. the presence ofa large corpus luteum
b. fertility level of lhe bun hemorrhage d. fluctuation of the larger uterine hom

o
c. early onset of puberty in female offspring b. slipping the chorioallantoic membrane 96. I" dairy cows the onset ofestrus:
e. the absence of follicles or a corpus luteum on
d. libido a f the bull c. damaging the amniotic vesicle the ovaries a. usually occurs between midnight and 6 AM
e. temperature of the testicles d. traumatizing the young fetus b. usually occurs between 6 PM and midnight

alk
e. triggering regression of the corpus luteum 91. Schistosomus refle.:r.us is characterized by: c. occurs at any time of day or night
81. Wllat is the characteristic location ofrupture and d. usually occurs between noon and 6 PM
a. a split cranium and torticollis
hematoma ollhe bovine pen is? 86. In cows a dominant follicle: e. usually occurs between 6 AM and noon
b. a duplicate caudal vertebral column, with
a. in the glans penis a. may fo rm on any day of the estrous cycle bent rear legs
b. dorsal to the attachment of the retractor penis b. ovulates when the corpus luteum regresses c. cleft palate and a laterally deviated head 97. llllhefirst postpartum insemination ofa cow,
muscles c. interferes with successful superovulation d. an underdeveloped spine and severely optimal fertility is achieved by depositing tile
c. just proximal to the sigmoid flexu re d. is refractory to luteinizing hormone contracted tendons semen ill tile:
d . in the perineal region , dorsal to the scrotum
e. at the preputial reflection
e.
w.
produces extra fo llicle-stimulating hormone

87. Wllal is tile longest pllase oftlte bovine


e. a ventral abdominal defect and a
ventrodorsally curved spine
a. cervix
b. cranial vagina
c. uterine horn contralateral to the graafian
82. Wllich disease causes abortion primarily from esrrous cycle? 92. Prolapse ofthe uterus is frequently associated with: follicle
felal disease ralher than placental disease? d. uterine horn ipsilateral to the graafian follicle
a. proestrus a. hypocalcemia
ww
a. brucellosis b. twin pregnancy e. body of the uterus
b. metestrus
h. campylobacteriosis c. estrus c. an emphysematous fetus
c. aspergillosis d. diestrus d. prolonged gestation
d. infectious bovine rhinotracheitis e. anestrus e. hydrops allantois
e. trichomoniasis

C 1998 Mosby-Year Book, Inc. Photocopying is prohibited by law. Correct answers are on pages 234·244.
www.alkottob.com
210 SECTION 9
www.vet4arab.co.cc Theriogenology 211

98. TIle cliallce for pregnancy in rile near future is c. 8 months of gestation 105. III the Northern Hemisphere during the late Jaff, J09. In mares theovur1I:
best for a cow: d. 6.5 months of gestation a mare may show signs ofestrus Jar as iong as 10
to 14 days when leased bya stallion. TIlis type of a. enters the uterus within 20 minutes after
a. carrying an emphysematous fetus e. 5 months of gestation

om
estrual behavior is: fe rtilization
b. a boning a 7-month-old fetus because of b. enters the uterus about 5 days after ovulation
leptospirosis 100. Mlat is the mQst appropriate specimen to a. prolonged but probably normal during the c. undergoes a mandatory embryonic d ia pause
c. carryi ng a macerated fetus examine to confirm suspected mycoricabortion transition from cycling to anestrus before entering the uterus 14 days after
d. aborting a 6-month-old fetus because of ill a cow? b. normal for mares in the summ e r, but ovulation
aspergillosis prolonged for mares in the fall d. enters the uterus abOlIl 10 days after
a. fetal lung
e. carrying a mummified fetus c. prolonged and abnormal, and should be ovulation
b. fetal abomasal contents treated with 100 JJ.g of gonadotropin-
c. cotyledon e. remai ns in me oviduct until me pregnancy
releasing hormo ne
signal is received from the u lerus at 14 days

b.c
99. Most mummified bovine fetuses Ilaue died at d. fetal spleen d. prolonged and probably the result of cystic after ovulation
about: e. fecal liver ovarian degeneration
a. 3.5 months of gestation e. normal
1 JO. Mu ltipleovu/ario/lS in mares:
h. 2 months of gestation
106. Ovulation in mares occurs: a. are extremely rare
b. occur in about 75% of all mares and are
a. 18 hours after the end of standing heat generally dual ovulations
HORSES b. 2 days before the end of estrus c. are generally triple ovulations occurring 1

tto
c. on the first day of estrus day apart
B.E. Eilts d. the day after estrus ends d. are generally triple ovulations occurring
101. Mares are best described as: d. exposing the mares to 16 hours of light and 8 e. shortly after coitus because the mare is an within 1 102 hours
hours of darkness about 2 months beforc the induced ovulator e. occur in about 20% of mares and are
a. nonseasonally monestrous
estrous cycles are scheduled 10 commence generally dual ovulations
b. seasonally monestrous
e. exposing the mares to 8 hours of light and 16 107. To establish the approximate time ofoVlIlation
c. nonseasonaUy polyestrous hours of darkness a bout 2 months before the by rectal palpation, a mare must be palpated: For Questions III through 115, select the correct

o
d. seasonally polyestrous estrous cycles are scheduled to commence
a. 2 days after coitus because me mare is an answer from the five choices below.
c. nonseasonally induced ovulators
induced ovulator a. prostaglandin
104. Itl the Northern Hemisphere during the month of
b. every 48 hours

alk
b. progesterone
102. In tile Northern Hemisphere. mares normally February. a mare may show signs of estrns
have estrous cycles: (teasing, winkillg, squatting) for as long as 10 to c. every 72 hours c. equine chorionic gonadotropin (formerly

•. in the spring
b. in the fall
14 days when teased bya stallion. This duration
ofestrual behavior is:
d. every 12 hours
e. I day after coitus because the mare is an
induced ovulator
p regnant mare serum gonadotropin)
d. estrogen
e. human chorionic gonadotropin
c. in the winter a. normal
d. at any time of the year b. prolonged but probably normal during the
transition from anestrus to cycling J08. The drug oJcllOice to induceovuiation ill an 111. Produced by the endometrium and ca uses lYSis
e. in the fall and winte r estrual mare with a 35-mm Jollicle on the of tile corpus luteum
c. prolonged and abnormal and should be
w.
trea ted wilh 10,000 IU of human chorionic ovary is:
103. The most effective method to induce estrous gonadotropin a. progesterone 112. Produced by the corpus luteum
cycles ill anestrous mares is by: d. prolonged and proba bly the result of cystic h. prostaglandin
a. administering altrenogest atO.044 mg/ kg PO ovarian degeneration c. equine chorionic gonadotropin (formerly 1J3. Produced by the endometrial cups
fo r 14 days e. normal for marcs in the s umme r but pregnant mare serum gonadotropin)
prolonged for mares in the spring or late
ww
b. administering 3000 IU of human chorionic d. human chorionic gonadotropin
go nadotropin 1M once winter 1/4. Has luteinizing ho rmone activity
c. estradiol
c. adminis tcring 100 JJ.g of gonadotropin-
releasing hormone 1M once 115. Produced by the granulosa cells in theJollicle

C 1998 Mosby-Year Book, Inc. Photocopying is prollibired by law. Correct answers are on pages 234-244.
www.alkottob.com
212 SECTION 9
www.vet4arab.co.cc TheriogenoJogy 213

116. 71,e mobility pllase of the equine embryo lasts: 120. To maintain pregnancy in mares, the ovaries: 124. After parturition and placental expulsion in a c. does not occur in mares because of the
mare, YO" examine the placenta and find red, normalluteolytic process in nonpregnant
a. I day a. are needed throughout pregnancy
velvety tissue rhat comprises one wall of the mares

om
h. 4 days b. are needed untiJ about 35 days of gestation, placerua. Tllis red, velvety layer is the: d. occurs when the endometrial cups produce
c. 16days when the placenta begins producing
a. amnion equine chorionic gonadotropin (pregnant
d. 20days
progestogens
mare serum gonadotropin)
c. are needed until about 120 days of gestation, b. uterine side of the alIantochorion
e. 24 days e. occurs when the mare retains the corpus
when the placenta begins producing c. fetal side of the allantochorion
luteum, does not return to estrus, and
progestogens d. allantois
117. After the mobility phase. a s;ngfe equine embryo maintains tone in the uterus and cervix
d. are needed until about 120 days of gestation, e. umbilical stalk. despite not being pregnant
usually becomes attached: when the uterus begins producing
a. ipsilateral to the corpus luteum to prevent progestogens

b.c
125. After parturitioll in mares, the placenta is 129. In mares, stage /I of parturition normally lasts:
the local countercurrent exchange of e. are not needed after ovulation because the
normally expelled within:
prostaglandin uterus produces progestogens throughout a. 30 minutes
b. contralateral to the corpus ]uleum to be as gestation a. 10 minutes b. 8 hours
far away from the local countercurrent b. 90 minutes c. 12 hours
exchange of prostaglandin as possible 121. Rectal palpation of a mare ill diestrus is most c. 24 hours d. 16 hours
c. o nly in the body of the uterus likely to reveal: d. 48 hours e. 24 hours
d. in either horn or the body of the uterus, e. 60 hours
a. a fl accid uterus and a cervix with tone
regardless or tile location ohile corpus

tto
iuteum b. a uterus with tone and a flaccid cervix 130. To induce luteolysis in a mare, rile best metllod is
c. a flaccid uterus and a flaccid cervix J26. Aside from milk calcium levels, whaltliree to administer:
e. only until the endometrial cups are formed criteria are used to determine readiness for
d. a uterus with tone and a cervix with tone a. prostaglandin at least 5 days after the end of
induction 0fparturition in mares?
e. an edematous uterus and a flaccid cervix estrus
J 18. If twin embryos are diagnosed by a. at least 300 days of gestation. cervical
ultrasollogmphyat /4 days after ovulation and b. prostaglandin 1 to 4 days after the end of
relaxation, and correct positioning of the foal estrus
are in tile same uterine horn, the most 122. Rectal palpation ofa mare in estrus is most
b. at least 300 days of gestation, cervical

o
appropriate course ofactiotl is to: likely to reveal: c. human chorionic gonadotropin at least 5
relaxation, and "waxing" of the teats days after the end of estrus
a. move one of the embryos to the opposite a. a flaccid uterus and a cervix \vith tone c. at least 330 days of gestation, cervical
uterine horn and crush it d. human chorionic gonadotropin 1 to 4 days
b. a uterus with tone and a flaccid cervix relaxation, and "waxing" ofthe teats after the end of estrus

alk
b. crush one of the embryos without moving c. a flaccid uterus and a fla ccid cervix d. at least 310 days of gestation, correct
either one e. ahrenogest at least 5 days after the end of
d. a uterus with tone and a cervix with tone pOSitioning of the foal, and ~waxing" of the estrus
c. wait until day 20 before moving one of the e. an edematous uterus and a cervix with tone teats
embryos to the opposite uterine horn and e. at least 330 days of gestation, "waxing" of the
crushing it teats, and correct positioning of the foal 131. Superovulatio n in mares is:
d. wait unlil day 20 and crush one of the 123. Qmcemingdetection of the corpus luteurn in a
mare by a single rectal palpation, which a. best attained using follicle-stimulating
embryos without moving either one hormone
sta/emelll is most accurate? 127. The dmgofclloice to induce parturition in
c. reduce the feed intake of the mare so one of b. best attained using equine chorionic
w. mares is:
the embryos starves a. The corpus lute um is not readily identifiable gonadotropin (formerly pregnant mare
by a single palpation. a. prostaglandin Fz« serum gonadotropin)
b. The corpus luteum is easily palpable, b. oxytocin c. best attained using altrenogest
J 19. TI,e endometrial cups in a mare's uterus are of:
primarily because of the "crown" that c. dexamethasone
3. maternal origin and secrete equine chorionic d. best attained using human chorionic
extends beyond the ovarian surface. d. estradiol gonadotropin
gonadotropin c. The corpus luteum is easily palpable, e. bromocriptine e. not readily attainable
ww
b. maternal origin and secrete prostaglandin primarily because of the line of demarcation
c. fetal origin and secrete prostaglandin berween luteal tissue and ovarian stroma.
d. fetal origin and secrete progesterone d. The corpus luteum is palpable on the ovary 128. Pseudopregnancy ill mares:
e. fetal origin and secrete equine chorionic contralateral to the ovulatory follicle. a. occurs only when the mare retains the corpus
gonadotropin e. The corpus luteum is palpable, primarily luteum because of early embryonic death
because of the way it changes the shape of b. occurs when the corpus luteum persists for
the ovary. about the duration of gestation because the
mare is not pregnant

e 1998 Mosb~Year Book, Inc. Plzotocopying is prolzibited by law. Correct answers are on pages 234-244.
www.alkottob.com
214 SECTION 9
www.vet4arab.co.cc Tlleriogenology 215

132. Embryo collection for embryo transfer in mares d. 38 to 40 days 140. 711e stallion's penis is best c11aracterized as a: 145. Tlze stallion's peflis is best described as:
is routinely performed: e. 55 to 60 days a. high-volume. low-pressure erectile system a. musculocavernous
a. 3 days after ovulation by placing a catheter in

om
b. high-volume, high-pressure erectile system b. fibroelastic
the uterine body and flushing both horns 136. During natural breedillg.lhe staffion ejaculates c. low-volume. low-pressure erectile system c. fibrocavernous
simultaneously into tlte mare's: d. low-volume. high-pressure erectile system d. musculoelastic
b. 7 days after ovulation by surgically flushing
a. vestibule e. high-volume. high-pressure fibroe lastic e. musculofibrous
the oviduct ipsilateral to the ovulatory follicle
b. caudal vagina system
c. 7 days after ovulation by placing a catheter in
the Ulerine body and flushing hoth horns c. cranial vagina 146. Spermatogenesis in stallioTlS requires:
simultaneously d. oviduct 141. What are tile primary stimuli that cause a
stallion to ejaculate?
a. 12 days
d. 7 days after the end of estrus by placing a e. uterus
b. 24 days

b.c
catheter in the uterine body and flushing a. Sight and smell c. 55 days
both horns simultaneously 137. 111ecorrect method to artificially inseminate a b. s mell and temperature d. 75 days
e. 14 days after the end of estrus by placing a mare is to: c. temperature and sight
catheter in the ulerine body and flush ing e. 100 days
a. place one hand in the rectum, insert the d. pressure and smell
both horns simultaneously
insemination pipette into the cranial vagina, e. pressure and temperature
147. As compared with ejaculates during periods of
and deposit the semen there
133. In trails/erring an equine embryo into a recipient long day length (16 hOllrs light. 8 hours dark).
b. place one hand in the rectum, insert the 142. In tile stallion testis. the ifllerstitial cells of during periods o{shon day length (8 hours light,
mare, it is 1Iecessary to: insemination pipette into the cervix, and

tto
Leydig produce primarily: 16 hours dark) the ejaculate of stallioTlS lias:
a. place the embryo into the uterine body deposit the semen there
c. place one hand in the rectum, insert the a. follicle-stimulating honnone a. more volume and more cells
h. place the embryo as far as possible into the
horn ipsilateral to the ovulating ovary insemination pipette into the uterine body, b. luteinizing hormone b. more volume and fewer cells
c. place the embryo as far as possible into the and deposit the semen there c. estradiol c. less volume and fewer cells
horn contralateral to the ovulating ovary d. place one hand in the vagina, carry the d. testosterone d. less volume and more cells
d. surgically implant the embryo in the horn insemination pipette into the cranial vagina. e. progesterone e. an equal volume and equal number of cells
and deposit the semen there

o
ipsilateral to the ovulating ovary
e. surgically implant the embryo in the hom e. place one hand in the vagina, insert the 143. YOIl palpate the scrotum and testes ofa stallion 148. Concerning the accessory sex glaruls of stallioTlS.
contralateral to the ovulating ovary inseminatjon pipette through the cervix, and with colic to determine i{there is torsion of the wlzich statemellt is most accurate?
deposit the semen in the uterus
spermatic cord. The normal oriefltatiOIl of the

alk
epididymis is witll the: a. The stallion has bulbourethral and vesicular
134. Ifl mares. unfertilized oua are: glands but not a prostate gland.
138. Mlat is the minimum dose of normal, motile
a. commonly found in the uterine hom a. head cranial. tail caudal, and body ventral b. The stallion has a prostate gland but no
sperm cells {or successful artificial insemilUltiOIl
ipsilateral to the ovulating ovary ofa mare? b. head caudal. tail cranial. and body ventral bulbourerluai or vesicular glands.
b. commonly found in the uterine horn c. head caudal, tail cranial. and body dorsal c. The stallion has prostate and bulbourethral
contralateral to the ovulating ovary
a. 500 million d. bead cranial, tail caudal, and body dorsal glands but no vesicular glands.
b. 1 billion d. The stallion has prostate, bulbourethral, and
c. expelled from the cervix I to 2 days after e. head cf<mial, tail caudal, and body lateral
ovulation c. 10 billion w. vesicular glands.
d. retained in the uterus for months d. 50 billion e. The stallion has no accessory sex glands.
144. TlleSertoli cells o/the testis are thollght to
e. retained for months in the oviduct ipsilateral e. 100 billion produce:
to the ovulating ovary 149. Daily sperm output can be accurately
a. testosterone
139. Stallion spermatozoa cafl survive ifl the mare's determifled in a stallion by collecting and
utermfor up to: b. estradiol analyzing semen once daily for at least:
135. After parturition, mares normally resume fertile
c_ progesterone
ww
estrous cycles within approximately: a. 20 minutes a. 3 days
d. follicle-stimu lating hormone
a. 5 to 10 days b. 2 hours e. luteinizing hormone b. 7 days
b. 18 to 21 days c. 1 day c. 21 days
c. 26 to 30 days d. 2 days d. 28days
e. Sdays e. 35days

@ 1998 Mosby-Year Book. Inc. Photocopying is prohibited by law. Correct answers are on pages 234-244.
www.alkottob.com
216 SECTION 9
www.vet4arab.co.cc Therlogeno logy 217

150. You collect semen [rom a stallion once and then c. the same volume and twice the number of 156. Stiffbirths in swine: 160. What is the most accurate method to detect
again J IlOllr later. As compared with thefirsc spermatozoa pregnancy in swine?
ejaculate, the second ejaculate is likely to have: a. usually are caused by physical abnormalities
d. half the volume and half the number of

om
ofthe sow's reproductive tract a. measurement of estrone sulfate levels in
a. half the volume and the same number of spermatozoa
b. are primarily caused by infectious agents maternal serum 25 to 30 days after breeding
spermatozoa e. the same volume and the same number of
that kill the fetuses before panurition begins b. using a mature boar and an experienced
b. the same volume and ha1f as many spermatozoa
c. are difficult to differentiate from early breeding manager to check for estrus 18 to 24
spermatozoa postpartum dealhs days after breeding
d. are primarily from noninfectious causes, and c. measurement of progesterone levels in
primarily occur during farrowing maternal serum 17 to 20 days after breeding

P IGS e. occur more often in middle-parity sows


(three to five parities) lhan in older sows
d. measurement of prostaglandin Flo levels in
maternal seru m 12 to 15 days after breed ing

b.c
R.C. Tubbs (more than six parities) e. amplitude-depth ultrasonographic
examination 30 to 45 days after breeding
151. What is the most importam [acror influencing 154. Mummifiedfetuses:
tile time ofollset ofpuberty ill gilts? 157. COllcerning abortion ill swine, which statemem
a. are usually caused by in utero viral infectio ns is most accllrate? 161. 71Je normal sequence of endocrinologic ellellts
a. environment that occur after day 35 a nd before day 70 of tllat occurs at tile onset of puberty in gilts is:
u. The cause is easily di agnosed by serologic
b. housing gestation
exam ination of m aternal serum. a. increased pulsatile release of gonadotropins
c. nutrition at the time of expected onset of b. are associated wllh a speCific n utritional leads to progressive development of ovarian
b. About 60% to 70% of a bortions have
puberty defiCiency

tto
noninfectious causes. follicles. increased levels of estradiol-I 7[3, the
d. transport c. occur primarily in litters from sows with preovulatory luteinizing hormone surge, and
more than five parhies c. About 60% to 70% of abortions have
e. exposure to a mature boar near the time of infectious causes. ovulation
expected onset of puberty d. occur because of prolonged duration of b. increased pulsatile release of progesterone
farrowing d. The cause is almost never diagnosed.
causes release of prostaglandin Flo. from the
e. Abortions occur in about 5% of all
J52. Amplitude-depth (A-mode or pulse-echo)
e. may result from a high level of carbon uterine endometrium, stimuJating lysis and
p regnancies in most swine herds.
ultrasonography for pregnancy detection in sows:
monoxide in lhe farrowing house ovulation

o
c. high levels of luteinizing hormone cause
a. is 100% accu rate between days 30 and 75 of 158. Seasonal infertility in swine: pulsatile release of oxytocin, leading to
gestation 155. Tile regimen for inducing parturition in sows
iTivolves useot a. is primarily caused by heat stress ovulation
b. detects a fluid-filled object and emits an d. high levels of progesterone cause release of

alk
audible signal, visible light, or both a. 20 mg of dexamethasone on day 112 of b. occurs primarily after June to September
matings relaxin, which triggers lhe preovulatory
c. converts reflected uhrasonic beams into two- gestation; parturition occurs in 2 days luteinizing hormone surge and ovulation
dimensional images of the interior cross b. 20 mg of dexamethasone on day 112, c. affects older sows more severely than
younger sows e. cystic ovarian follicles are sensitive to the
section of the animal followed by 20 IV of oxytocin on day 113; effects of pulsati le releases of gonadotropin.
d. is o nly applicable after calcification of fetal parturition occurs in 24 hours d. is less severe in outdoor breeding herds than
resulting in ovulation
skeletons c. 10 mg of prostaglandin F2Q on day 112, in indoor breeding herds
e. detects fetal heart movements and fo Uowed by 5 IU of oxytocin 24 hours la ter; e. is less severe when sows are housed in
parturition begins 24 to 30 hours after groups rather than in crates 162. What is the optimum daily caloric iTJIake
movement of fluid lhro ugh umbilical vessels w.
prostaglandin injection (megacalories of metabolizable energy per day)
or uterine arteries
thal results in the shortest mean weanillg-to-
d. 20 mg of estradiol cypionate on day 112, 159. Lordosis ill swine is characterized by: service interval?
followed by 20 IU of oxytocin on day 113;
153. Lactation lengths of less than 21 days in sows a. rolling, high-pitched squealing, and frequent
parturition occurs on day 114 a. 20
have been associated with: defecation
e. a combination of equine chorionic b. 16
a. cystic ovaries in 25% of affected sows gonadotropin (pregnant mare serum b. frequent urination, squatting, and polydipsia
c. 12
ww
b. more rapid return to estrus and improved gonadotropin) and human chorionic c. erect hair, frequent rubbing, and rooting
d. 6
reproductive performance gonadotropin on day 113 of gestation; d. swelling and reddening of the vulva,
e. not known
c. persistent anestrus in up to 40% ofsows parturition occurs 24 hours later immobile stance, arched back, and erect ears
d. increased litter size and weaning weights in e. a cool nose, straight tail, and frequent
the next liuer sweating
e. prolonged weaning-to-service intervals and
decreased size of the next litter

C 1998 Mosby-Year Book, Inc. Photocopying is pro/libited by law. Correct answers are on pages 234-244.
www.alkottob.com
218 SECTION 9
www.vet4arab.co.cc Th erlogenology 219

163. Altering tile suckling pattern aftlle litter by d. Increased pulsatile secretion of ovarian 169. Prostaglandins are not usedforestrus d. testosterone
weaning IIa/flitters 2 days early decreases the estrogens stimulates the preovulatory synchronization in swine because: e. adre nocorticoids
weaning- fo-service intenlDf: progesterone surge and onset of estrus

om
a. prostaglandins are too expensive
within 5 to 15 days in most sows.
a. in high-producing sows but not in low- b. the porcine corpus luteum is unresponsive to 172. For pregnancy to be maintained, the minimum
producing sows e. Increased pulsatile secretion of adrenal the luteolytic effects of single doses of number ofembryos necessary iutlle uterus all
steroids is followed by ovarian hypertrophy, prostaglandins until day 12 of the cycle, and day 12 after matillg is:
b. in sows of some breeds but not in others
estrogen and progesterone secretion, luteolysis begins normally on day 15 of the
c. in any sow under all circumstances increased prostaglandin F:z., levels, estrus, a.
cycle; therefore the time fram e when it is
d. only whe n the weaning-to-service interval is and ovulation. b.3
effective is too narrow to be of practical use
already prolonged c. 4
e. whe n the weaning-ta-service interval is c. the porcine corpus luteum is not affected by
166. A producer wil/I a 100-sow farrow-to-feeder pig prostaglandins d. 6
normal

b.c
operation caflsyou because hefound some d . pigs have 12-day cycles. so synchronization e. B
aborted fetuses ill tile gestation unit 011 hisfarm. is not needed
164. Optimal fertility is attained wlien theftrsl YOIl measure tile crowll -to-rump lellgth of the e. the porcine corpus luteum secretes high J 73. Wlljcll set of boar sem en characteristics is
postweallillg service occllrs: fetuses and determine that they are about 45 levels of estrogens, which negate the effects
days'gestatiollal age. W1lich sample would be lIormal?
a. 12 to 14 days after weaning of prostaglandins
most IIseful in determining the cause ofabortion a. volume= 100 ml; morphology = 87%
b. 7 to 10 days after weaning when submitted to a diagllostic laboratory? abnormal; concentration = 2.2 million sperm
c. 3 to 6 days after weaning 170. Wllat is tile normal sequence ofendocrinologic cells/ mi
a. maternal serum events tllat occur on days 13 to 15 ofa normal
d. 3 to 6 days after weaning in summer months b. volume = 50 mI ; morphology = 9% abnormal;

tto
and 6 to 10 days after weaning in winter b. feed (nonpregna nt) estrous cycle;lI sows? co ncentration = 26 million sperm cells/ Illl
months c. fetal flu ids and tissue
a. Levels of prostaglandin secreted by the c. volume = 100 mI; morphology = 3%
e. at any time; weaning-ta-service interval has d. culture swabs from the sow's reproductive uterine endomenium increase, causing lysis abnormal; concentration = 13.8 million
no influence on fertility tract of the corpus luteum; ovarian progesterone sperm cells/ mJ
e. seme n samples from boars levels decrease; luteinizing hormone and d. volume =100 mI: morphology = 3 1%
/ 65. Norma l mating bellavior ill swine is controlled fo llicle-stimulating hormone increase; estrus abnormal; concentration = 3.3 million sperm
and ovulation follow 5 to 8 days later.

o
by pituiraryand o/Jarian hormones. During 167. In fa te summerulId early fall, tllere is: ce ll s/ mI
normal lactation, serum levels offollicle- b. Progesterone increases, estrogen decreases. e. volume = 250 mI; morphology = 12%
a. improved re productive performance because and ovulation occurs.
stimulating hormone, luteinizing honnone, and abnormal ; concentration::: 55 million sperm
of decreased stress during lactation
progesterone are low. TIle suckling reflex blocks c, Progesterone secreted by the ovary decreases cells/ml

alk
lllleinizing 1I0rmone secretion, and inhibin b. a decreased number of stillborn pigs and prostaglandin levels, causing uterine
blocks fofficle-stimllfating hormone secretion. mummified fetuses involution, estrus, and ovulation.
What sequence ofevents normally occurs on c. a decreased number of sows that remain 174. W1lat is the theoretic adlJantage ofmating sows
d. Progesterone causes the uterine
remollal of tile sucklillg reflex and removal of anestrous for 30 days or longer two or more times durillgone estrous period, as
endometrium to contract, releasing opposed to breedillg a single time during an
negative f eedback by illliibill ? d. a decreased percemage of sows that return to prostaglandin, which stimuJates oxytocin
estrus within 7 days of weaning estrous period?
a. A 30-day postlactational anestrus occurs in release and the onset of estrus.
most sows. e. increased expression of libido by mature e. Follicular estrogens stimulate lysis of the a. The increased physical contact induces more
boars but not by immature boars corp us lute um and release of prostaglandins ovulations.
b. Incrensed pulsat ile secretion of luteinizing w.
hormone, increased follicle -stimulating from the ovary, which causes the onset of b. More matings tend to settle sows, so they are
hormone levels, and increased follicular 168. In sows, ovulation occurs: estrus and ovulation. less likely to fight and disrupt pregnancy.
development lead to production of ovarian c. The likelihood of mating at the optimum
a. at the time estrus begins lime relative to ovulation is increased.
estrogens, the preovulatory luteinizing 171. Pregnancy is signaled by maternal recogllition of
hormone surge, and onset of estrus within 5 b. 12 hours after estrus ends d. There are no advantages to mating twice
fetal:
to 15 days in most sows. c. 36 to 40 hours after the onset of standing versus mating once.
ww
estrus a. prostaglandin
c. Increased pulsatile secretion of progesterone e. More matings keep boars in better physical
and increased ovarian estrogens inhibit d. 6 hours befote estrus begins b. estrogen condition, with bette r semen quality.
secretion offollicle-stimulating hormone and e. 18 to 24 hours after the onset of standing c. progesterone
luteinizing hormone by the hypothalamo- estrus
hypophyseal-ovarian axis and prevent estrus
in most sows fo r a minimum of 15 days.

10 1998 Mosby-Year Book, Inc. Pholocopying is prohibited by law. Correct answers are on pages 234-244.
www.alkottob.com
220 SECTION 9
www.vet4arab.co.cc Theriogenology 221

175. The most common recommendatiofls[or using 179. An owner of a 450-sow feeder pig herd calls yort J 82. What is the most common cause ofdystocia 186. What is the best biologic measurement of sow
mature (older than 12 months) and immature in January because his farrowing rate has fallen in sows? herd efficiency?
boars (younger than 12 months) are to: to 68% and preweaning mortality has risen from
a . eclampsia

om
16% to 20% in the past 2 months. During the a. pigs weaned per farrowing crate per year
3. use mature boars for 5 to 7 matings per week; b. cranial fetal presentation b. pigs weaned per inventoried sow per year
initial visit you observe that tile next group of
use immature boars for 3 to 5 matings per c. caudal fetal presentation
pigs due to be weaned has 15% mortality c. cost to produce a weaned pig
week
(normal for the herd is 15%) and that the d. breech fetal presentation d. feed cost per pound of pork produced
b. use mature boars fo r 3 to 5 matings per week; farrowing rate in the next 2 groups is projected
use immature boars for 1 or 2 matings per e. uterine inertia e. feed cost per ton of sow feed
at 78% (normal for the herd is 80%). W1Jat is the
week most likelycallse of the problem?
c. lise mature boars for 10 to 12 matings per 183. A producer calls you because a group of4-month- 187. W11ich Leptospira serovar is most commonly
week; use immature boars for 12 to 14 a. porcine reproductive and respiratory old gilts on the finishingfioor have red, swollen identified in serologic surveys of swine in the
syndrome

b.c
matings per week vulvas and developing mammary glands. What is midwestern United States?
d. use mature boars for 1 or 2 matings per week; b. pseudorabies the most likely cause of these sigru?
a. Lpomona
use immature boars for 1 or 2 matings per c. seasonal fluctuations in fertil ity and
a. The p roducer has developed a new line of b. L bratislava
month preweaning mortality early-maturing gilts.
d. transmissible gastroenteritis c. L IIardjo
c. use mature boars without restriction, since b. exposure to zearalenone
they cannot be overused; use immature e. subclinical infections with multiple d. L icterohaemorrhagiae
c. exposure to aflatoxin e. L. canicola
boars for 6 matings per week pathogens
d. exposure to a mature boar
e. pseudopregnancy

tto
176. Mummifiedfetuses usually indicate that 180. Infectious late-term abortions in swine herds are 188. W1Jat are tlJe most common infectious
infection occurred at which stage a/pregnancy usually associated with infection by: reproductive disorders in swine herds?
and witl! which agelll? 184. By definition, "nonproductive days" are days on
a. parvovirus which sows: a. porcine reproductive and respiratory
a. before day 35 with Leptospira b. Streptococcus species syndrome, eperythrozoonosis, and erysipelas
b. before day 35 with pseudorabies virus a. do not eat b. toxoplasmosis, streptococcal infection, and
c. encephalomyocarditis virus
c. after day 35 with ActinobacjlJus b. are not in the farrowing house staphylococcal infection
d. Leptospira species
c. are not gestaling or lactating c. brucellosis, pseudorabies, and Eubacterium

o
pleuropneumoniae e. pseudorabies virus
d. after day 35 with parvovirus d. are being held to ~dry up" after lactation sllis infection
before they are sold as culls d. leptospirosis, parvoviral infection, and
e. after day 75 with the agent of porcine
181 . A producer has an average swine herd. One e. are in the breeding area but not in estrus pseudorabies
reproductive and respiratory syndrome

alk
group of breeding-age gilts is seronegativefor e. leptospirosis, parvovirai infection, and
porcine parvollirus. With regard to the campylobacteriosis
177. What approximate percentage ofmalllre seronegative gilts, what is the most appropriate 185. Development oflean replacement breeding
(multiparous) sows are normally in estrus advice for tile producer? females that reach puberty at a young age has
within 7 days after weaningfollowingan been associated with: 189. What is the normal duration of ejaculation in
a. Because these gilts have not been ex-posed to boars?
approximate 21-day lactation? a. small litter s ize in gilt litters, reduced second-
the virus, they are unlikely to show clinical
a. 75% to 80% signs of the disease. litter size, prolonged weaning-to-estrus a. 18 seconds
intervals after the first lactation, and high b. I to 2 minutes
b. 65% to 70% b. Because these gilts a re unprotected, they
w. subsequent culling rates
c. 35% to 40% should be exposed [Q the field virus or c. 4 to 6 minutes
vaccinated before breeding, to protect the b. large litter size a nd superior rebreeding d. 10 minutes
d.l00% perfonnance
fetuses from parvovira! infection. e. 11.5 minutes
e. 85% to 90% c. higher lifetime productivity
c. These gilts should be bred only to
seronegative boars. d. bigger appetites, higher feed intake, and
178. Sexual maturity ofboars is indicated by age and: d. These gilts should be bred only to better milking performance
ww
a. testicular circumference seropositive boars. e. higher prevalence of diarrhea and slow
e. These gilts should be culled. growth of offspring
b. carcass composition
c. carcass length
d. testicular weight
e. back-fat level

© 1998 Mosby-Year Book, Inc. Photocopying is prohibited by law. Correct answers are on pages 234-244.
www.alkottob.com
222 SECTION 9
www.vet4arab.co.cc Theriogenology 223

190. The primary purpose of medicated early 195. In what part afthe sow's reproductive tract does 199. What is the typical sow culling rate in well- 200. What is the mean duration of second-stage
weaning and modified medicated early weaning fertilization occur? managed, intensive confinement swine labor and the average interval between
technologies is to: production units? individual births, respectively, during normal
a. uterus

om
a. 10% to 11% parturition in sows?
a. improve sow reproductive performance b. vestibule
h. produce weaned pigs free of specific diseases c. uterotubular junction b. 15% to 18% a. 2 to 3 hours; 15 to 16 minutes
c. eliminate all disease from a swine herd d. isthmus c. 20% to 30% b. 6 to 8 hours; 8 to 10 minutes
d. decrease the cost to produce a weaned pig e. oviduct d.40%t055% c. 2 to 3 hours; 20 to 35 minutes
e. decrease feed costs associated with e. 65% to 80% d. 2 to 6 hours; 35 to 40 minutes
producing a weaned pig through increased 196. What is the most common physical injury to the e. I hour; 10 minutes
use of medication reproduClive tract of boars?

b.c
a. testicular frostbite
191. A client witll 300 sows historically has low
farrowing rates in December. A review afhis records b. laceration of the distal end of the penis SHEEP
reveals that December farrowing rates for {he past c. testicular trauma
5 years ranged from 5 7% to 68%. He has 30 d. frostbite of the distal end of the penis S. Mobini
farrowing crates available per sow group. To ensure e. laceration of the prepuce 201. How long is the estrous cycle ofsheep? c. in the summer
that farrowiflg crates are kept fILII in December, a. 16 days d. during estrus
IlOw many sows must be mtlted per group? e. immediately after lambing
197. During which developmental period is the b. 19 days
a. 53 sows in July matings

tto
porcine fetus most susceptible to the effects of c. 21 days
b. 53 sows in August matings teratogenic agents? d. 28 days 205. What type of placentation do sheep have?
c. 30 sows in all groups a. placental e. It varies because sheep are seasonal a. cotyledonary
d. 44 sows in August matings b. neonatal breeders.
b. discoid
e. 44 sows in September matings c. embryonic c. zonary
d. pre-attachment 202. Ewes are seasonally polyestrous. In the Northern d. diffuse
192. The head of the epididymis in boars is located: e. fetal Hemisphere they normally cycle:

o
e. chorioallantoic
a. at the dorsal pole of the testis a. in the fall
b. at the ventral pole of the testis 198. A cliem calls you about an award-winninggilt b. in the spring 206. What is the most significant cause ofabortion in
Ile recently bought at the state fair. The gilt c. in the winter

alk
c. craniomedial to the testis sheep in the United States?
d. within the parenchyma of the testis farrowed yesterday and only had five pigs. He d. in the fall and winter
wants to know what to do with tile animal after a. toxoplasmosis
e. within the inguinal canal e. the year around
the litter is weaned. Your advice is to: b. chlamydiosis
a. cull the animal after weaning, because litter c. Q fever
193. The boar penis is: 203. What is the predominant hormone during d. campylobacteriosis
size is unlikely to increase in subsequent diestrus?
a. hemodynamic matings e. brucellosis
b. cartilaginous, with a flared glans peniS b. breed the animal after weaning and hope for a. progesterone
c. fibroelastic, with the distal end coiled in a
counterclockwise direction
d. about 20 cm long and 5 cm in diameter subsequent litter sizes
w.
a larger litter, because there is little
relationship between first-litter size and

c. use gonadotropin-releasing hormone to


h.
c.
d.
e.
estrogen
luteinizing hormone
follicle-stimulating hormone
testosterone
207. Which method is most widely usedfor estrus
synchronization in ewes during the breeding
season?
e. retractable even though lacking a sigmoid a. introduction of a ram to the flock
flexure superovulate before the next breeding
d. cross-foster the pigs to another sow and sell b. prostaglandin F20. administration
204. Administration of prostaglandin F2tJ. induces c. use of progestogen-impregnated vaginal
ww
the gilt immediately
194. What is theduratiOll of spermatogenesis in boars? estrus in ewes only when given: pessaries
e. perform a full serologiC profile and uterine
a. 20 days culture and breed the gilt after weaning if the a. outside the breeding season d. use of controlled intravaginal drug-releasing
results of the diagnostic tests are negative b. during the breeding season devices (CIDRs)
b. 18 days
e. adding progestogens to feed
c. 45 days
d. 35 days
e. 60 days

© 1998 Mosby-Year Book, Inc. Pllotocopying is prohibited by law. Correct answers are on pages 234-244.
www.alkottob.com
224 SECTION 9
www.vet4arab.co.cc TlleriogerlOlogy 225

208. What is the most common and predictable way 213. What is a sign ofestrus in ewes? 218. What is tlTe major cause ofepididymitis in rams? 222. In ewes, parturition can be induced in tlTe last
to induce and synchroni2.e estrus in mature ewes few days ofpregnancy by administration ot
a. mucous discharge from the vulva a. brucellosis
duringanesrrus?
b. vulvar swelling

om
b. actinomycosis a. dexamethasone
3. prostaglandin Fla administration c. bleating and tail wagging c. campylobacteriosis b. prostaglandin Fz.:.
b. use of progestogen-impregnated vaginal d. "winking" of the vul va d. chlamydiosis c. p rogesterone
pessaries for 12 to 14 days
e. standing to be mounted e. salmoneUosis d. ovariectomy
c. artificial lighting programs
d. progestogen administration for 12 to 14 days,
e. oxytocin
followed by 400 to 500 IU of pregnant mare 214. Examination ofa ewe with dystocia reveals a 219. COllceming artificial insemination ofewes,
serum gonadotropin live fetlls in anterior presefltotioll, with one which statemem is most accurate? 223. ArtlJrogryposis and hydranencepltaly in fetal
forelimb retained within the birth canal. This lambs are caused by:
e. melatonin adm inistration a. It is simple because the structure of the
[erus is most appropriately delivered by:

b.c
cervix alIO\'IS easy passage of the pipette. a. toxoplasmosis
209. What is the normal length afgestation in ewes?
a. cesarean section b. Correct p laceme nt of the piperte is very b. brucellosis
b. pubic symphysiotomy of the dam d ifficult because of the structure of the
a. 58 to 62 days c. Akabane virus disease
c. felolo my cervix.
b. 14510155 days d. border disease
d. judicious traction on the head and exposed c. It is complicated because a spiral p ipette
c. 278 to 290 days e. enzootic abortion of ewes
forelimb must be carefully manipulated through the
d. 11 2 toi l Sdays e. repulsion, followed by repositioning of the cervix.
e. 335 to 345 days retained forelimb d. It is easily done transvaginally using a 224. Rams are evaluated for breeding sOlllldness by:

tto
speCUlum. a. a thorough physical examination, \vith no
210. TIle optimum time for insemination ofa ewe is: 215. TIle penis oftlJe ram is most accurately e. It is diffic ult but can be learned quickly by other examin ations or tests
described as: owners and handlers b. semen analysis, with no other examinations
a. 9 hours after the onset of estrus, to the end of
or tests
estrus a. fibroelastic, with a corkscrew shape
220. W.hen embryos are surgically recovered from c. a thorough physical examination, with
b. 15 to 30 hours after the onset of estrus b. fibroelastic, with a filiform process
donor ewes, on which day after ovulatioTl should emphasis on the reproductive organs, and
c. 12 10 IS hours after the onset of estrus c. fibroelastic, with an upturned tip embryos be recovered? seme n analysis

o
d. every other day, beginning on the second day d. musculocavernous, with a bell-shaped glans d. measurement of scrotal circumference, with
of eslrus a. day 1
e. muscular, with a flared glans no other examinations or tests
e. 6 to 8 hours after the end of estrus b. day3
e. test matings

alk
c. day6
216. What is tile major cause ofmortality in neonatal
211. What is the average volume of semen ejaculated
d. day9
lambs? 225. Scrotal circumference in rams is directly
by rams? e. day II
a. hypothermia and hypoglycemia correlated witli:
a. I to 1.5 ml b. birth inj ury a. economic value
b. 5 to 10 rnl 221. What is the most accurate means ofdetecting
c. infec tions early pregnancy ill ewes? b. libido
c. 30to 100 mi d. predation c. testicular texture
d. 150 to 300 ml a. Doppler ultrasonography
e. congenital defects w. d. serving capacity
e. 300 to 400 ml b. radiography
e. sperm production
c. palpation of the abdomen
217. What is tile most common defect of the penis
d. B-mode ultrasonography
2 J2. A major consideration in an artifICial and prepuce oframs?
insemination program is semen collection. While e. serum progesterone determination
a. spiral deviation of the penis
still maintaining adequate spenn concentrations,
semen can be collected from rams: b. ventral deviation of the penis
ww
c. corpus cavernosal shunt
a. 3 times daily, 2 days per week fo r 1 week
d. failure of the retractor penis muscle to relax
b. once daily, every other day for 2 weeks
e. balanoposthitis
c. many times per day for severa) weeks
d. many times per day for I week
e. twice daily, 3 days per week fo r 6 weeks

@ J 998 Mosby-Year Book, Inc. Pliotocopying is prollibited by law. Correct answers are 011 pages 234·244.
www.alkottob.com
226 SECTION 9
www.vet4arab.co.cc Therlogenology 227

233. To optimize pregnancy rates in caprine embryo 240. Retained fetal membranes (placenta) in goats
GOATS transfer, how many embryos must be transferred are associated with all the following conditions
to the recipient? except:
W.F. Braun, Jr.

om
a. one a, selenium deficiency
226. Wlwt is the best method to detect esrrus in the doe? 230. m/iell organisms are the most common
b. two b. dystocia
infectious causes ofcaprine abortion in the
a. observing standing and mounting behavior c. three c. mannosidos is
United States?
among does d. four d, abortion
b. observing a reddened, swollen vulva, with a. Chlamydia psirtaci. Brucella melitensis. and
e. five e. metritis
vaginal mucus secretion Campylabaeter jejun i
c. use of a teaser or breeding buck b. Chlamydia psircaci, Coxiella burnetii, and
Brucella melitcnsis For Questions 234 through 238, select the correct 241. Pregnancy ketosis in the doe is the result ofan
d. use of a "huck rag" impregnated with the answer from the five choices below.

b.c
c. Ccunpyfobacrer jejuni, Coxiella burnerii, and energy deficiency during ltlte gestation. although
scent of a buck
Toxoplasma gOlldi; the exact etiology is unknown, which of the
c. serum progesterone assay a. habitual abortion
following does "ot predispose to the condition?
d. C/Ilamydia psittaci, Toxoplasma gondU, and b. campylobacteriosis
Coxiella bumeti; c. toxoplasmosis a. mu ltiple fetuses
227. Assays of wlliell of lhe following is not used for
c. Cflfamydia psirraci, Campylabaeter jejuni, d. listeriosis b. fatty liver
pregnancy detection in the doe?
and Brucella abortlls c. overconditioned doe
e. chlamydiosis
a. pregnancy-specific protein B d. suboptimal feed intake
b. chorionic gonadotropin 23 1. What is tile SOl/rce ofprogesterolle that e. high dietary energy levels in early lactation

tto
234. Abortions caused by this disease may be
c. estrone sulfate maifltains pregnancy in goats?
diagnosed on the farm by the presence of
d. placental lactogen macroscopic, calcified foci of necrosis jn the
a. corpus luteum until day 55 of gestation, then 242. Pseudopregnancy is commol! in goats and is
e. progesterone the placenta p/acemal cotyledons. characterized bya persistent corpus [uteum,
b. primary corpus luteum , roUowed by anestrus, and a variableaccumultltiOIl offluid
228. The microscopic structure of the cnprine accessory corpora lutea, then the placenta ill tile uterus. Concerning pseudopregnancy ill
235. Abortions caused by this disease may be
placenta is classified as: c. placenta only goats, which statemem is least accurate?
preceded by reduced appetite and milk

o
a. hemochorial d. placenta and adre nal glands production and signs ofsepticemia. a. Does may develop an enlarged udder and
b. syndesmochorial e. corpus luteum only abdomen.
c. endotheliochorial 236. Abortions caused by this disease are rare in the b. The treatment of choice is a luteolytic dose of
prostaglandin.

alk
d. epitheiiochorial 232. Omcemingabortion induced with United States; the organism is ingested and
prostaglandin administration, which statement about one third of infected fetuses have grossly c. Diagnosis is based on ultrasonographic
e. discoidal examination of the uterus.
is most accurate? visible fiver necrosis.
d. Diagnosis is based on radiographic
229. Artificial insemination ill the doe is commonly a. It can be performed anytime during examination of the uterus.
gestation. 237. This is the most common cause of infectious
performed by: e. Does may deliver a "cloudburst" of fluid at
b. Prostaglandin administratio n does not caprineabortion ill the United States.
a. transcervical insemination using a vaginal the end of a period equal to normal
induce abortion in goats. gestation.
speculum
c. It is usually performed between day 7 and
w. 238. This is most often seen in older Angora does.
b. transcervical insemination via rectal
the end of gestation.
manipulation 243. W1,at is the normal duration ofgestation in
d. It is usually performed after day 55 of 239. W1lat is the most common cause ofdystocia in
c. transabdominal insemination via a goats?
gestation. the doe?
laparoscope a. 3 months
e. It is usually performed from days 1 to 35 of
d. cranial vaginal deposition of semen a. fetomaternal disproportion
gestation. b. 4 months
e. transabdominal insemination using a b. failure of the cervix to dilate c. 5 months
ww
syri nge and 4-inch needle c. attempted expulsion of more than one fetus d. 6months
at the same time
e. 7 months
d . posterior presentation of the fetus
e. uterine inertia

C 1998 Mosby-Year Book, Inc. Photocopying is prollibited by law. Correct answers are on pages 234-244.
www.alkottob.com
228 SECTION 9 www.vet4arab.co.cc Theriogenology 229

244. Concerning urolithiasis (urinary calculi) in male d. volume of 5 to 10 ml, concentration of 1.5 to
goats. which statemelll is least accurate? 4 X 106 sperm/ml BIRDS
a. It is an important problem in Pygmy goat e. volume of25 to 50 ml, concentration of 1.5 to
M.B. Paster

om
wethers. 4 X 106 sperm/mI
251. Follicle-stimulating hormone induces all ofrhe c. alert when hatched
b. Management factors associated with the
following except: d. hatch with thermoregulatory ability
condition are prepubertal castration, high- 248. What are the accessory sex glands ofbucks?
concentrate diets, and insufficient water a. development of ovarian follicles e. are exothermic, with a metabolic rate lower
a. ampulla, seminal vesicles, disseminated
intake during very cold weather. b. secretion of estrogen by the ovaries than that of adults
prostate, and bulbourethral glands
c. Common sites of obstruction are the base of c. feather growth
b. ampulla, seminal vesicles, compact prostate,
the urethral process and the distal bend of d. spermatogenesis 256. Which hormone induces the preovulatory surge
and bulbourethral glands
the sigmoid flexure. of luteinizing hormone?
c. ampulla and seminal vesicles only e. tubular growth of the testes

b.c
d. Catheterization of the urinary bladder is an
imponant pan of therapy. d. seminal vesicles, disseminated prostate, and a. triiodothyronine
bulbourethral glands only 252. The serum level of which hormone is generally b. progesterone
e. Ultrasonography may be used to confirm the
presence of bladder lesions and calculi in the c. compact prostate only higher in male tllan infemale birds during tile c. estradiol
urethra. breeding season? d. prolactin
249. Concerning the intersex condition in goats, a. adrenocorticotropic hormone e. arginine vasotocin
245. During the hreeding season, ovulation in does which statement is least accurate? b. luteinizing hormone
occurs: a. An intersex is an animal that externally c. triiodothyronine 257. Which hormone induces expulsion of tile egg

tto
a. 18 hours after the end of standing heat shows both male and female characteristics. d. mesotodn from the oviduct?
h. 12 to 36 hours after the onset of standing b. It is more prevalent in dairy goats than in any e. tetraiodothyronine a. thyroxine
heat other domestic species. b. estradiol
c. 24 hours after intromission c. The gonads (testes or ovotestes) may be 253. Prolactin induces all of the following except: c. arginine vasotocin
scrotal, inguinal, or abdominal in location.
d. 18 hours before the onset of stand ing heat a. nesting behavior d. prolactin
d. Most polled intersexes are karyotypically
e. 24 to 36 hours after the end of standing heat h. incubation e. progesterone
male (XY).

o
e. It is more prevalent in dairy goats that are c. cessation of egg laying
246. What is tile duration ofesrrus in goats? polled (hornless) than in horned goats. d. caring for the young 258. Which hormone stimulates development of "crop
a. 21 days e. increased production of luteinizing hormone milk" in pigeons and doves?

alk
b. variable because of induced ovulation 250. Which of tile following most accurately describes and follicle-stimulating hormone in both a. tetraiodothyronine
the caprine estrous cycle? sexes b. estradiol
c. J2 to 24 hours
d. 8 to 12 hours a. seasonally polyestrous, cycling induced by a c. prolactin
e. 36 to 48 hours decreaSing photoperiod 254. The offspring ofaltricial species are d. progesterone
characterized by all tile following except: e. arginine vasotocin
b. seasonally polyestrous, cycling induced by
247. Which afthefollowing most accurately describes an increasing photoperiod a. hatch naked
buck ejaculates? c. nonseasonal polyestrous, cycles 21 days long h. hatch blind 259. The testes have all thefollowingfunctions
a. volume of 0.5 to 2 ml, concentration of 1.5 to
4 X 109 sperm/ml
b. volume of5 to 10 ml, concentration of l. 5 to
photoperiod
w.
d. monestrous, cycling induced by a decreasing

e. monestrous, cycling induced by an


increasing photoperiod
c. are exothermic, with a metabolic rate lower
than that of adults
d. hatch with thermoregulatory ability
e. hatch with a low metabolic rate that
except:
a. production of male gametes
b. secretion of steroid hormones thai control
4 X 109 sperm/ml development
increases during development and usually
c. volume of 0.5 to 2 mI, concentration of l.5 to c. secretion of steroid hormones that control
4 X 106 sperm/ml exceeds that of adults by fledgling time
function of accessory sex organs
ww
d. secretion of steroid hormones that suppress
255. Tile offspring of precocial species are song behavior
characterized by all of the following except: e. secretion of steroid hormones that influence
a. hatch with their eyes open secondary sex characteristics
b. covered with down when hatched

© 1998 MosUy-Year Book. Inc. Photocopying is prohibited by law. Correce answers are on pages 234-244.
www.alkottob.com
230 SECTION 9
www.vet4arab.co.cc Tileriogenology 231

260. 771e bursa of Fabricius is associated with all the d. The ovaries periodically hypertrophy and 268. Concerning the blood chemistry of laying birds, c. Progesterone stimulates secretion of
/ollow;ngexcept: atrophy on alternate sides throughout life. wllich statement is least accurate? luteiniZing hormone.
e. The ovaries become especially active during a. Serum levels of phospholipids, fatty acids, d. Progesterone is the principal hormone
a. adrenal gland development

om
molting. and neutral fats increase by 3- to IS-fold in exerting a positive-feedback effect on release
b. development of immunity through of luteinizing hormone in hens.
production of humoral factors birds laying eggs.
264. Which hormone regulates I/le pattern ofgrowth b. Ovarian hormones increase serum levels e. Estradiol, produced by ovarian follicles,
c. production of ted blood cells induces the physiologic changes associated
and maturation ofovarian follicles? offalS.
d. testicular growth with egg laying.
e. development of immunity through a. prolactin c. The blood glucose level in layers is double
production of cellular units that of nonlayers or males.
b. foUicle-stimulating hormone
d. The blood calcium level is doubled in laying 270. Reproductivefailure in captive birds is usually
c. estradiol associated willi:
birds.
261 . Conceming the relations11ip between molting d. progesterone

b.c
e. Plasma cholesterol levels are increased 3-fold a. failure of the ovary to develop follicles that
and reproduction, which stalemeflt is least e. luteinizing hormone in layers. ovulate
accurate?
b. failure of males to develop normal testes
a. Androgens and estrogens may delay or 265. Mlich hormone decreases production offollicle- 269. Concerning hormone interactions during the c. failure of the pituitary gland to respond to
inhibit molting. stimulating hormone and luteinizing hormone ovulatory cycle, which statement is least hypotllalamic stimulation
b. The molting cycle is often associated with a and stimulates brooding behavior? accurate? d . failure of the ovary to respond to
marked increase in serum levels of gonadotropin stimulation
a. prolactin a. Estrogen never stimulates release of
triiodothyronine and thyroxine.
b. estradiol luteinizing hormone. e. stress, which causes release of

tto
c. Sex hormones have a protective effect on adrenocorticotropic hormone
mature feathers. c. progesterone b. Testosterone is rarely found in the plasma of
d. Molting is associated with decreased levels of d. cholecalciferol laying hens.
testosterone and estrogen. e. mibolerone
e. ThyrOid hormones stimulate early
development of new feathers. 266. A hen thaI lays a cerlain number ofeggs during
a specified period and then ceases laying eggs is
ZOO AND EXOTIC ANIMALS

o
262. In male birds, wllicll cells secrete testosterone? best described as: L.A. Dierau!, F.L. Frye, T. Gemeinhardt, S. Hudelson,
a. 13 cells of the islels of Langerhans a. a polyovous layer D. Huff, J.D. Letcher, D.H. Nielsen, D.O. Schaeffer
b. ScrtoHcells b. imennittently polyovous 271. Wllich stage is nol included in thedevelopmem 273. Reptile species that [ayeggs that subsequently

alk
c. chromaffin cells c. a determinate layer ofreptiles? hatch outside the body are described as:
d. argentaffin cells d. seasonally fertile
a. larva a. nulliparous
e. Leydig cells e. seasonally subfertile
b. juvenile b. oviparous
c. adult c. viviparous
263. Concerning tile reproductive tract offemale 267. A hen tllat con tinues laying eggs until a specific d. embryo d. polyploid
birds, which statement is most accurate? number afeggs has accumulated in the nest and
e. fetus e. monoecious
then ceases laying eggs is best described as:
a. The left ovary is more well developed than w.
the right ovary. a. a determinate layer
272. The mealls by which an all-female line of 274. A hermaphrodite:
b. The right ovary is more well developed than b. a clutch-limited layer reptiles call develop is called:
c. a polyovous layer a. is sexually attracted to members of the
the left ovary.
a. androgenesis same sex
c. The ovaries are equally well developed d. an indetermina te layer
b. misanthropism b. is sexually attracted to members of the
throughout life. e. an environmental layer
c. parthenogenesis opposite sex
ww
d. gynogenesis c. has only one type of gonad, either ovaries or
e. he rmaphroditism testicles
d . has both testicular and ovarian tissue
e. lacks gonadal tissue altogether

C 1998 Mosby-Year Book, Inc. PhOTocopying is prollihired by law. Correct answers are on pages 234-244.
www.alkottob.com
232 SECTION 9 www.vet4arab.co.cc Theriogenology 233

275. Difficulty in fayingeggs or giving birth is called: d. occurs when the animal reaches about 45 283. Amphigonia retardata refers to: 288. III most reptiles the erectile portion of the penis
years of age or hemipenis is composed of
a. dyspareunia a. retention of eggs in the oviduct for a
e. does not occur prolonged period a. mucous glandular tissue
b. dysecdysis

om
c. dyspepsia b. retention of spermatozoa for a prolonged b. fibrovascular tissue
d. dystocia 280. Rattlesnakes, pygmy rattlesnakes, copperheads, period afler mating c. fibrocollagenous connective tissue
and cottonmouth water moccasins giue birth to c. migration of eggs from one oviduct into the d. serous glandular tissue
e. dysautonomia
liuing young, whereas coral snakes lay eggs. opposite one e. bone
Which statement best describes the young of d. slowed sexual maturity caused by subnormal
276. Sexual precocity refers to: these snakes?
environmental temperatures
289. Spermatozoa can remain viable in the
a. delayed sexual maturation a. Alhough the young of these snakes are born e. prolonged penile erection
reprodllctiue tract of some mated female
b. accelerated sexual maturation with fangs, they do not have poison (venom)
reptiles for:

b.c
c. failure to deposit ova or deliver young on uncil they have eaten several meals and have
284. The eggs ofmost reptiles resemble those ot
schedule shed at least twice. a. 2 to 4 weeks
b. Baby snakes, even venomous ones, must a. amphibians b. 2 to 4 monrhs
d. premature ejaculation
learn from their mother how to bite to b. primates c. 2 to 4 years
e. inability to attain or maintain an erection
protect themselves. c. insects d. 2 to 4 hours
c. Baby venomous snakes can deliver serious d. birds e. 2 to 4 minutes
277. In reptiles, fracwre of an intraoviductal egg, with bites even before they emerge from the egg e. fish
leakage a/tile contents into the body cavity, is: or protective fetal membranes.
290. Examples of inherited disorders ill reptiles are:

tto
a. harmless under most circumstances and can d. Baby venomous snakes do not develop fangs
until they have shed their skin at least twice. 285. Which pathogen can be transmitted from a
be ignored a. dicephaly and spina bifida
female ouiparolls reptile to her eggs?
h. an emergency that warrants immediate e. Because these baby snakes are small, they b. albinism and melanism
surgical treatment can be handled without fear of a. Pasteurella c. cleft palate and microphthalmia
c. a consequence of inbreeding or line breeding envenomation. b. Pseudomonas d. molluscum contagiosum and diabetes
d. normal in many snakes, lizards, and tortoises c. Salmonella mellitus
e. usually caused by vitamin B6 deficiency 281. The egg shells of some lizards can be oueriy soft d. Aeromonas e. Canfield's syndrome and amyloidOSiS

o
when they are deposited. The most likely cause of e. Morganella
this softness is:
278. Mastitis in tortoises is: 291. Polyspermia refers to:
a. phosphorous deficiency andlor renal disease 286. Why are adult male tortoises separated from

alk
a. usually related to breeding of a very a. deposition of more than one male's
b. calcium deficiency andlor renal disease females carrying shelled eggs?
immature animal spermatozoa in a female's reproductive tract
c. dietary protein deficiency a. Mismating and confusion over parentage of
b. usually seen only in older tortoises that have b. penetration of more than a single
had many litters of young d. dietary manganese and magnesium the offspring may occur. spermatozoon into an egg
deficiency b. Intraoviductal eggs might become damaged
c. not a clinical entity in this species c. spermatozoa attached to each other by their
e. chronic organophosphate toxicity if mating is repeated. acrosomes
d. caused by chilling during hibernation
e. thought to be related to infection with soil c. Some eggs could be doubly fertilized if d . defective spermatozoa characterized by
fungi 282. Reptiles haue a cloaca through which feces, mating is repeated. multiple tails

279. In most female reptiles, menopause:


a. occurs at about mid-life
w.
urinary wastes, and reproductive products are
passed. The eggs or liuing youngoffemale
reptiles pass out of the female's body through the:
d. Developing embryos within the eggs might
develop into hybrids.
e. The female might attack and injure the male.
e. storage of excess spermatozoa after a mating

292. Whieh species has testicles located in asemtal sac?


a. urodeum
b. occurs only after an animal bas failed to give 287. In reptiles, fertilization occurs:
a. Atlantic white-sided dolphin
b. urethra
birth or lay eggs (Lage1lorhynchus acutus)
c. coprodeum
ww
c. occurs in the fifth or sixth decade of life a. after the eggs have been deposited by the b. common dolphin (Delphinus delphis)
d. vestibuJe female in water c. northern fur seal (Callorhinus ursi1lus)
e. proctodeum b. shortly after the egg shells have been deposited d. Pacific harbor seal (Phoca uitulina)
around the developing embryo and yolk mass
e. Atlantic bottle-nosed dolphin (Tursiops
c. in the oviduct truncatus)
d. within the terminal end of the epididymis
e. within the terminal end of the vasa deferentia

© 1998 Mosby-Year Book, Inc. Photocopying is prohibited by law. Correct mlSluers are Of! pages 234·244.
www.alkottob.com
234 SECTION 9 www.vet4arab.co.cc Tlleriogenology 235

293. Although somewhat variable. gestation in c. mink 5. c High serum progesterone levels are necessary 20. a Fever is not a feature of this disease.
llamas normally lasts: d. llama to maintain pregnancy, and decreased serum 21. b The other answers listed are ancillary
e. bear progesterone levels indicate impending diagnostic methods. Confirmation requires
a. 113 to 125 days

om
parturition. A drop in rectal temperature to isolation of the organism, which is best
h. 140 to 155 days 100S F is not low enough. A rectal temperature recovered from blood.
c. 260 to 280 days 297. WllQt is the gestation length ofguinea pigs? below 99 F accompanies decreased serum
0

22 . d There is no reliable medical treatment for


d. 330 to 360 days a. 21 days progesterone levels (prostaglandin effect). brucellosis. Blood cultures usually are not
e. 390 to 450 days b. 28days 6. a None of the other answers can detect positive until at least 30 days after infection.
pregnancy as early as 18 days. 23. e Separation of the head from the midpiece and
c. 42 days
294. Ferrets have an average litter sizeo[B (range, 2 to 7. b Relaxin is a product ofplacentation. tail is often an artifact resulting from poor sample
d. 45 days
17). Tile deaf, blilld infants are born after an 8. e All the other stages listed have a relatively handling, such as cold shock. The other defects
e. 63 days
average gestation of narrow range of duration. A bitch cycling every 4 listed arise during spermiogenesis in the testes.

b.c
months will have a short anestrus compared 24. e Vaginal hyperplasia occurs during the follicular
a. 21 days 298. In which species is dystocia a major problem? with one that cycles every 9 months, but both are (estrogen) phase and regresses spontaneously.
b. 32 days normal. Uterine prolapse does not regress.
a. rot
c. 42 days 9. c Answers a, b, and e will probably induce 25. b At this age, both testes should be scrotal. This
h. hamster
d. 50 days ovulation. Answer d wiU pharmacologically dog may be fertile. Monorchidism refers to
c. gerbil overcome estrogen-expressed signs of heat.
e. 64 days agenesis of one testis and is very rare or
d. rabbit Answer c will prolong the signs of heat. nonexistent in dogs.
e. guinea pig 10. b Megestrol has been approved for estrus 26. b Cryptorchidism is uncommon in cats. In dogs,
295. Overpopulation is a cOrlStant problem ;nfree-

tto
prevention once proestrus has begun. retained testes have a higher incidence of
ranging and captive feral horses. A contraceptive
that has been used in female feral horses, as well 299. In which species is it absoilltely necessary to 11. a Mibolerone can be used over the long term for neoplasia than do scrotal testes. A diagnosis is
as in dogs, can be administered remotely, does remove the male before parturition? estrus prevention. not possible until scrotal development occurs,
not illferJere with pregnancies already in 12. e None of these drugs has been approved for use and not at birth in any breed. It is not a
a. rat dominant trait.
progress, has no effect on social behavior; and has in cats.
b. mouse 27. c Because of an increased prevalence of
effects tltat are reversible. This contraceptive is: 13. c Sperm mature in the epididymis.
c. gerbil neoplasia in retained testes and possibly also in
14. d Following ovulation, a requisite period of

o
a. porcine zona pellucida d. hamster scrotal testes and because of the hereditary
maturation before fertilization makes this the
b. diethylstilbestrol e. guinea pig nacure of the condition (unilateral cryptorchids
ideal time to inseminate. Mature canine ova are
c. testosterone propionate relatively short lived . are often fertile), bilateral caslration is
recommended. This dog is too old for medical

alk
d. ethinylestradiol homogeneous Silastic 15. d Sperm nwnbers per ejaculate are a function of
300. Which species bears young tllat are precocial therapy. Orchiopexy is considered unethical.
implants testicular size, which is a function of body size.
(fully fllrred, eyes open)?
e. northisterone Thus big dogs produce more sperm than little dogs. 28. a Hermaphroditism is diagnosed by the
a. mouse histologic appearance of the gonads, not by the
16. e Sperm concentration is about the same for all
b. rat gross appearance of the external genitalia.
296. Wl!icll species apparently is not all induced normal males. The total number of sperm per
c. hamster ejaculate and total volume of the ejaculate 29. c Hypothyroidism is commonly associated with
ovulator?
d. gerbil increase with body size, but sperm concentration subfertility or infertility.
a. Bactrian camel varies only slightly. 30. e This bitch is showing signs of estrogen
e. guinea pig
b. ferret w. 17. c Parabasal and uncornified epithelial cells are
synonymous. Their numbers decrease as
estrogen levels increase. Polymorphonuclear
cells disappear, and red blood cell numbers are
stimulation. Once treated, this dog will cycle
normally. Pituitary adenoma is not a common
cause. Nine weeks is far longer than the normal
duration of estrus.
Answers highly variable. 31. a All the other infections listed can cause these
18. d Although not totally immune to the serious problems. Calicivirus infection has not been
1. d Bitches have one cycle at a time and, if they live 3. d Basenjis are one of the few breeds that truly incriminated, although it may cause kitten
ww
side effects of estrogen, young bitches are more
long enough. usually have at least one cycle in cycle once per year. The Tibetan mastiff (rarely mortality.
tolerant. Answers b, c, and e are true and should
each season of the year. The average interestrus seen as a pet) is another. 32. d Subinvolution of placental sites is most
be made known to the owner when assessing the
period for all breeds is 7.4 months. 4. d Although the signs listed have been said to commonly a one-time occurrence in young, first-
risk/benefit ratio before therapy is begun.
2. a Queens have several cycles within a breeding indicate ovulation. only answer d stands up to litter bitches. It cannot be prevented and, by
19. d This drug has a direct effect, causing
season. Breeding seasons are determined in large scientific scrutiny. itself, is not harmful. The only sure immediate
myometrial contraction . The effect of oxytocin is
part by light cycles. cure is hysterectomy. Pyometra frequently occurs
negligible in clinical cases.
without subinvolution.

© 1998 Mosby-Year Book, Inc. Photocopying is prohibited by law.


www.alkottob.com
236 SECTION 9
www.vet4arab.co.cc T/leriogellology 237

33. a Prostatic diseases of all kinds are very 45. b TIle o nly reliable indicator of a functional 62. b On day 3 the embryo is still in the oviduct and 79. e About 85% of heifer calves born twin to a bull
uncommon in cats. in dogs, prostati tis ovarian remna nt is an elevated serum on day 5 it still may be in the oviduct and so calf are freemartins.
frequcndy causes necrozoospermia (dead progesterone level. II is too lale for an elevated cannot be rC(:overed. By day 9 the embryo has 80. c Scrotal circumference is also correlated with

om
sperm). However, it does not usually affect serum estrad iol level. Spayed bitches with lost its zona pellucida and is much more difficult sperm output.
spe rm numbers. hyperadrenocorticism do not cycle. Answer d to identify. 81. b This s ite is most likely to sustain damage when
34. b Needle biopsies do not yield a specimen with would not be present. Parabasal cells would 63. a Estrus las ts only 12 to 18 hours, versus more the erect penis is accidentally bent during a
retained tissue architecture sufficient for predominate. Ultrasonography is totally than 2 days for all the other phases. thrust against an immovable object.
diagnosis. Because cytologic diagnosis is usually unreliable in this circumstance. 64. c Catlie normally give birth to a Single offspring, 82. d All the other diseases listed primarily affect the
not possible, a surgical biopsy is preferred. 46. c An ovarian rem nant is most likely. Most of as opposed to a litter (multiparous). placenta.
35. b Absence of molar teeth is a function offetal these are 011 the right side because the right 65. b Hydramnios occurs as a result ofa defective
ovary is more difficult to exteriorize during 83. b Asymmetry and fluctuation of the uterine
age. The other choices listed are signs of fetal fetus.
routine ovariohysterectomy. Ectopic ovaries are horn could be caused by pyometra. The amniotic
death.

b.c
66. e This is the only indisputable sign of estrus. vesicle is not palpable after 60 days. The uterine
36. d Pain at intromission usually indicates an very rare.
67. a Campylobacteriosis is a venereal disease. artery does not develop fremitus until 3.5 to 4
anatomic problem in the bitch; vaginal strictures 47. a Progesterone prolongs gestation. The other months of gestation.
ure the most common cause of such pain. answers remove the source of progesterone or 68. a The prostate gland of bulls is poorly
the fetuses themselves. developed. 84. a Heifers are first bred to calve at 24 months of
37. d A vi ral etiology has not been proven. age.
Tnlllsmission by licking is common. Many 48. e The duration ofviabiJi ty of frozen cani ne 69. b Diestrus is the only time during which a
semen has not yet been dctermined, but it is well mature corpus luteum is present, with a 85. c The delicate vascular aflachments of the
transmissible venereal tumors are benign and
in excess of 10 years. sufficient number of receptors. amniotic vesicle are vulnerable to damage
regress spontaneously. Most respond well to
70. c As compared with the other s ites listed, the during palpation.
radiation therapy. In some parts of the world, 49. c Forty-eight hours is a reasonable maximum,
86. b The decrease in circulating progesterone

tto
malignancy is more common than in the United using current semen extenders. vagina is relatively pliable and elastic.
Stales. 71. b The response to superovulation is not very concentration permits final maturation of the
50. b All females are XX and most ~torti es~ are ovulatory follicle.
38. a Granulosa-cell tumors occur in females. Only fema le. Those few males arising from chimerism controllable because of variations in drug
Sertoli-cell tumors usually reach such size. The are usually XXV (infertile) , but a few have been dosages, day of the cycle that treatment is 87. d Diestrus lasts 13 or 14 days.
second (soft) mass probably represents a cystic ferti le. Thus fertil e male tortoiseshell cats are initiated, and the animal 's age, lactational status, 88. a The bovine corpus hemorrhagicum is small
prostate gland or uterus masculinus as a result of very rare though not necessarily valuable. breed and nutritional s tatus. and soft.
the esnogens produced by the Sertoli-cell (Umor. 51. d By definition, monestrous females have a 72. d Poor estrus detection is a never-ending 89. a Heifers generally have a 20-day cycle. Therefore

o
39. a Of tile tumors listed. a Scrtoli-celltumor is single heat during a breeding season. manageme nt problem. the chances are I in 20 (5%) that a heifer will be in
most likely to produce e nough estrogen [Q affect 52. a Anestrous means not cycling. 73. b Respiration must be initiated within 5 heat on any given day. With 100 heifers, that
the bone marrow. minutes, o r the calf will die. means 5 would be expected to be in heat.
53. e Bydefinition, polyestrous females have
40. d All the other organisms listed may be isolated 74. d The prevalence of listeriosis in pregnant cows 90. a Slipping the chorioallantoic membrane is the

alk
multiple heats during a breeding season .
on aerobic cultures, but mycoplasms require is too low to warrant development of a only positive sign of pregnancy.
54. c Females rC(:emly in heat are considered to be
special techniques for isolation. commercial vaccine. 91. e Affected bovine fetuses have a ventral
in metestrus.
41 . c Feminizing syndrome usually produces 75. e Pyometra is almost always seen postpartum, abdom inal defect and a ventrodorsally curved
55. b Estrus is the period of heal. spine.
hyperpigme ntation. except when caused by venereal transmission of
56. e Estrogcn is produced by the ovulatory follicle trichomoniasis.
42. c Answers band e do not automatically cause 92. a Hypocalcemia results in a flaccid myometrium.
during estrus.
dystocia; answers a and d are normal. Answer c 76. d For economic reasons, slaughtering the cow is 93. c Bovine ova are fertilized in the oviduct.
57. a Ci rculating levels of luteinizing hormone rise the most a ppropriate action. It would take at
definitely indicates prolonged gestation and 94. a Excessive heat and humidity retard early cell
most probably represents a maternal dystocia, to peak level at the onset of estrus, triggering
w. least 10 more months for this cow already in late
ovulation. division, and the embryo dies before day 16.
although, with lack of owner observation, it lactation to begin "earning her keep" with a new
58. b Unlike surgical embryo collection, nonsurgical 95. b The unaccompanied corpus luteum persists
could be felal. lactation.
collection does not cause adhesion formation for long periods through lack of a luteolytic
43. a There will be no sperm in this dog's ejaculate 77. a In this situation the fetus is not accessible for signaJ because there is no adjacent uterine horn.
and thus can be done repeatedly. fetotomy or through use of a detorsion rod.
(answer a), rather than few (answer b) or dead 96. a Estrus usually begins between midnight and 6
(answer c) sperm . Because most of the ejaculate 59. d Progesterone is produced by the corpus There are no relaxant drugs approved for use in
IUleum, which is maximally developed during AM in dairy cows.
is comprised of prostatic fluid , ejaculate volume food animals. The degree of torsion is too great
ww
diestrus. to correct by rolling. 97. e Semen deposition at this site is unlikely to
is little affected. Ubido is likewise unaffected
60. e ~ Hip-Iocked ~ calves cannot be repelled, damage the endometrium.
because the testes are normal. 78. d There should be no corpus luteum at the time
because they arc firm ly wedged in the birth of artificial insemination, so progesterone levels 98. e The endometrium in this cow is unlikely to be
44. d Erythrocytes may be seen before proestrus
canal. Partial fetolomy would be least traumatic should be low. If the cow is pregnant at the time damaged and is free from infection. and the
and well into diestrus/metestrus. They are
10 the dam. of the next anticipated estrus (22 days later), caruncles are already involuted.
unreliable indicators of the stage of the cycle and
may also be seen in s mears from dogs with 61. c Adhesions are the o llly answer Listed that is there should be a corpus luteum, and 99. a This is the time when the fetus is most
neoplasia or trauma. compatible with regular cycling. progesterone levels should be high. susceptible to peracute, fatal viral infections.

e 1998 Mosby-Year Book, 1nc. Photocopying is prollibited by law.


www.alkottob.com
238 SECTI ON 9
www.vet4arab.co.cc Tl,erioge'lology 239

100. e An impressio n smear of a cotyledon should opposite horn and crushed. Waiting may increase 134. e It is generally assu med that unfertilized ova 151. e Although the other factors listed may
be stained and examined for fungal hyphae. the chance for spontaneous reduction, but if are retained in the oviduct by the ovulating influence the time of onset ofpubcrty, exposure
101. d Mares have multiple estrous cycles during reduction does not occur, chances of me ovary fo r many months before being released of gilts of the proper age to a mature boar is the

om
the spring and so are considered seasonally noncrushed embryo surviving are reduced. into the uterus. most consistent faclOr influencing the onset of
polyestrous. 119. e Endometrial cups are of fe tal origin and 135. a Mares normally have a fertile "foal heat" puberty.
102. a Mares normally cycle during periods of p roduce equine chorionic gonadotropin within 9 days after parturition. 152. b This accurately describes the way A-mode
longer day length, which, in the Northern (pregnant mare gonadotropin). 136. e ~Be llin g" of the glans penis during ultrasonography works.
Hemisphere, is in the spring (March to July). 120. e The mare can be ovariectomized after 120 ejaculation causes the jets of semen to directly 153. e Shorter lactation lengths are associated with
103. d Aflificially increasing day length to 16 hours days of gestation and still maintain pregnancy enter the uterus. longer weaning-to-service intervals and smaller
of light, with 6 hours of darkness, induces with placental progestogens. 137. e Artificial insemination is performed by subsequent liner sizes.
cycling in about 2 months. 12 1. d Tn diestrus the mare has a IOnic uterus and carrying the pipette into the vagina and then 154. a All other answers refer primarily to sliUbinhs.
tonic cervix because of the influence of placing the tip in the uterus, where the semen is

b.c
104. b During late winter and early spring, it is 155. e None of the other regimens works well in
normal for a mare to show prolonged signs of progesterone. deposited. sows.
estrus because of anovulatory follicles. 122. c In estrus the mare has a flaccid uterus and 136. a At least 500 million normal motile sperm 156. d Answers a and b may occur but are less
105. a During the fall it is normal for mares to show flaccid cervix because of the influence of cells are required if an aliquot of semen is to common than answer d. Answers c and e are
prolonged signs of estrus because of estrogen. attain acceptable fertili ty using artificial untrue statemelllS.
anovulatory follicles. 123. a The corpus luteum is not readily palpable in insem inatio n in the mare. 157. b Abortions are primarily the result of
106. b Mares normally go out of estrus 2 days after mares because it is embedded within the ovary. 139. e Stallion sperm can survive for as long as 5 noninfectious causes. Diagnosed cases lend to
ovulation, so the bes t description is that they 124. b The red velvety tissue is the maternal or days in the mare's uterus. be th ose with infectious causes.
ovulate 2 days before the end of estrus. uterine side of the allantochorion. 140. a The stallion's penis is a high-volume, low- 156. b Pigs apparently are seasonal breeders. The

tto
107. d The ovulatory follicle can quickly fill with IZ5. b Stage III , or placental expulsion, should pressure, musculocavernous rype of erectile effects of season are more profound and last
blood after ovulatio n. To determine when the occur within 90 minutes after parturition, or the tissue. longer than the effects of heat stress. Younger
follicle collapses, the ovary must be palpated placenta is considered retained. 141. e Both pressu re and temperature must be sows, outdoor breeding herds, and group-
every 12 hours or the blood that fills the follicle 126. c Mares should be at 330 days of gestation and optimal for the stallion to ejaculate. housed sows are more severely affected.
can be mistaken for the follicle itself. have cetvical relaxation and some ~waxi ng" of 142. d IllIerstitial cells of Leydig in the testis 159. d This is the only answer that accurately
106. d Human chorionic gonadotropin has the teats before parturition can be induced with prod uce testosterone. describes lordosis in swine.
luteinizing action and can induce ovulation in a good success rate.

o
143. d The epididymis is oriented on the testis with 160. b An experienced manager working with a
mares. 127. b Parturition is best induced using oxytocin. the head cranial. the body dorsal and the tail mature boar is 96% accurate in detecting
109. b The fertilized ovum remains in the oviduct Prostagland in Fla tends to cause premature caudal. pregnant sows.
fo r about 5 days before entering the uterus. placen tal separation. 144. b Estradiol is thought to be produced by the 16 1. a This is the only accurate description of the

alk
110. e Approximately 20% of all mares have multiple 126. e Pseudopregnancy is described as a period of Sertoli cells and involved in the feedback events.
ovulations, most of which are double ovulations retained corpus luteum (regardless of whether system of follicle-stimulating hormone 162. e The optim um is not known ; 12 appears to be
within 24 hours of each other. the mare is bred) , which induces tone in the production . the minimum.
Ill. a Prostaglandin is produced by the uterus and cervix. 145. a The stallion's penis is of the 163. d This practice is only effective if the weaning-
endometrium and causes luteolysis. lZ9. a Fetal expulsion (stage II) normally lasts only musculocavernous type. to -service interval is already too long.
lI Z. b Progesterone is produced by the corpus 30 minu tes in mares. 146. c It takes 55 days after the first divisio n of the 164. c Services 3 to 6 days after weaning result in
luteum. 130. a Prostaglandin is luteolytic only when a primary spermatocyte until mature higher farrowing rates and litter sizes. A
113. c The endometrial cups produce equine mature corpus luleu m exists, which is 5 days
w. spermatozoa arc released for ejaculation. ~tro u gh" occurs as a result of setvices 7 to 14
chorionic gonadotropin (formerly pregnant after the end of estrus. 147. e During the nonbreeding season (short day days after weaning.
mare serum gonadotropin). 131. e Superovulation is not readily attainable in length}, the stallion's ejaculate has less volume 165. b This is the only answer that accurately
11 4. e Human chorionic gonadotropin has mares. and fewer spermatozoa than during the describes the sequence of endocrinologic events.
luteinizing activity. 132. c To collect embryos from a mare for embryo breed ing season. 166. c Fetal fluids and tissues are more useful than
115. d The granulosa cells produce estrogen. transfer, it is best to flu sh both uterine horns 148. d Accessory sex glands of stallions include the any of the other selections. The other answers
simultaneously 7 days after ovulation. vesicular glands (seminal vesicles), pros tate might yield results in individual cases, but in
c The equine embryo normally moves for 16
ww
ll6.
133. a To transfer an embryo into a mare, it is only gland, and bulbourethral gland. general they are extremely fru strating
days before fixation.
11 7. d The equine embryo attaches anywhere in the necessary to place the embryo in the uterine 149. b After 7 consecutive days of collection, daily d iagnostic techniques.
body. The location {right or left} of the corpus sperm output stabilizes. 167. d Manifestations of seasonal infertility include
uterus, regardless of the side of ovulation.
luteum makes no difference because the early 150. b A second ejaculate, collected 1 hour after the an increased number of sows with delayed
Il6. a At 14 days postovulation the remaining embryo
embryo is mobile. first, should have the same volume and half as return to estrus after weaning.
is most likely to survive if one twin is moved to the
many spermatozoa. If this is not the case, one of 168. c Ovulation occurs 36 to 40 hours after the
the ejaculates is not representative. onset of estrus in most sows.

~ 1998 Mosby-Year Book, Inc. Phorocopyillg is prohibited by law.


www.alkottob.com
240 SECTION 9
www.vet4arab.co.cc Tllulogenology 241

death or fetal mummification. They require \93. c Boars have a fibroelastic, corkscrew-shaped 210. c Estrus averages about 30 hours in ewes.
\69. b The porcine corpus luteum is unresponsive
to prostaglandins until day 12 of the cycle. It is exposure before breeding so that they can penis. Ovulation occurs in the last part of estrus. The
thought that a lack of or unresponsiveness of develop immunity before early to mid- 194. d In boars a cycle of spermatogenesis takes 35 optimum time for insemination is 12 to 15

om
receptor sites may be involved. Recent studies gestation. days. hours after the onset of estrus.
have shown that the porcine corpus luleum is 182. e Cranial and caudal fetal presentations are 195. e Ova arc fertilized in the oviduct. 211. a Of livestock species, sheep and goats have the
responsive to multiple doses of prostaglandins, normal in pigs. Breech presentations usually do 196. b Laceration of the distal end of the penis, smallest volume of ejaculates. The average ram
beginning at about day 5, but this is still not cause dystocia. Eclampsia is relatively caused by masturbation. homosexual activity. ejaculates I to 1.5 ml of semen. containing J to
impractical for estrus synchronization on uncommon in sows. and biting and fighting associated with pen- 3 billion sperm.
farms. 183. b Exposure to zearalenone is the most likely mating, is the most common physical injury to 212. c Semen can be collected from rams many
170. a This is the only physiologically accurate cause, considering the signs described. the boar's reproductive tract. times each day for several weeks before
description of the events. 184. d Nonproductive days are defined as days on 197. c l11e embryon ic period is the period of depleting epid idymal reserves of sperm
because each ejaculate is of small volume. In

b.c
171. b Estrogens produced by the fetuses which sows are neither gestating nor lactating. greatest susceptibility to teratogens. During the
themselves around day 12 after mating signal a Answers b. c. and d describe higher levels of preattachment and fetal periods. susceptibility contrast. boars and stallions quickly deplete
185.
the sow that pregnancy has occurred. is fairly low. Placental and neonatal periods are their epididymal reserve because they expel
reproductive performance and are all incorrect.
not recognized phases of fetal development. large numbers of sperm in each ejaculate.
172. c At least four embryos must be present in the With answer e, although slower growth may be
uterus on day 12 to produce the amount offetal associated with leaner pigs, higher incidences 19B. b Although the first litter was small. this animal 213. c Bleating and tail wagging indicale estrus in
estrogens necessary for maternal recognition of of diarrhea are not associated. could have more pigs in subsequent liuers. ewes. Answers a, b. and e are observed in cows;
pregnancy. answer d is a sign in mares.
186. b The best biologic measure of sow herd 199. d Despite our desire for lower cuUing rates,
e All other selections have too little volume, too efficiency is pigs per sow. The other measures most herds have a culling rate of 40% to 55%. 214. d Lambs in anterior presentation. with one
173.
many abnormalities, or low sperm may be useful in other areas of herd efficiency. and the rate is higher in many herds. forelimb retained. can usually be delivered by

tto
concentration. Pigs weaned per farrowing crate may be the judicious traction alone.
200. a The second stage of labor lasts 2 to 3 hours.
174. c Mating twice increases the likelihood of a most useful measure of facility use. Pigs are born about 15 minutes apart. 215. b The penis of rams is fibroelastic, with a
mating at the optimal time relative to ovuJation. 187. b Leptospira bratislava has been commonly filiform process at the end of the glans penis.
201. a Ewes cycle every 16 days.
although recent studies suggest single mating identified in serologic surveys in the midwestern 216. a Hypothermia and hypoglycemia from
202. d Ewes normally cycle during periods of
may be adequate for sows in estrus 3 to 6 days United States, although culture results and starvation (mismothering-exposure complex)
shorter day length, which. in the United States
after weaning. definitive diagnoses have not often documented accounts for most neonatal losses in a sheep
and Canada. are in the fal l and winter (August
a Mature boars can be mated 5 to 7 times per its association \'lith reproductive problems. flock.
\75.

o
through March).
week, whereas immature boars should be used Leptospira pomona is most frequently 217. e "Pizzle rot" is the most common condition
documented as a cause of late-term abortions. 203. a Progesterone is produced by the corpus
3 to 5 times per week. affecting the penis and prepuce of rams. The
luteum during diestrus.
d Fetal mummification is primarily caused by 188. d The other answers contain at least one disease is caused by Corynebacterium nma/e,
176. 204. b A mature corpus luteum (CL) must be

alk
viral infections between day 35 and day 70 of organism that is not an important common which grows in the alkaline urine of sheep fed a
presen t if prostaglandin F.... is to induce estrus. high-protein diet.
gestation. herd pathogen.
A mature CL is present omy during breeding
e Approximately 85% to 90% of multiparous sows 189. c Boars ejaculate over a period of 4 to 6 minutes. 21B. a Brucella avis is the major cause of
177- season. The CL is not mature during estrus and
should be in estrus within 7 days of weaning 190. b The main use is production of weaned pigs epididymitis in rams.
immediately after lambing.
following a 21-day lactation. A shoner lactation free of specific diseases. Sow reproductive 219. b Artificial insemination of ewes is difficuh
205. a Placentation is cotyledonary in sheep. diffuse
decreases the percentage of sows in estrus within performance may in fact be worse (answer a). It because of the anatomy of the ovine cervix.
in swine and horses. zonary in dogs and cats.
7 days. is unlikely that all diseases can be eliminated by Deposition of semen cranial to I cm into the
and discoid in primates.
\7S. d Age and testicular weight reflect sexual any procedure (answer c). The cost to produce a cervical canal is impossible. Laparoscopy is
w. 206. d Campylobacteriosis has a wide geographic most common ly used for artificial
maturity in boars. Testicular circumference is wcaned pig may actual ly increase because of
distribution and causes high rates of abortion. insemination.
difficult to measure in boars. The other increased medication costs and decreased sow
The disease often causes severe financial losses.
measurements listed have no relationship to reproductive performance (answer d). The feed 220. c Embryos are best collected on day 6 after
cost to produce a weaned pig probably will not 207. c Progestagen-impregnated vaginal pessaries ovulation. At days 1 and 3 the embryos are too
sexual maturity.
change. Feed utilization improvements as a are widely used for estrus synchronization. immature. By day 9 the embryos have lost their
179. c Seasona1 fluctu ations are the likely cause,
resuh of the technique are gained primarily 208. d Estrus can be induced and synchronized zona pellucida and are difficult to identify. The
considering the percentage of changes
after weaning. during anestrus by administration of embryos are too malUre by day II.
ww
described and the time of year given.
191. b To ensure that crates are kept full in progesterone for 12 to 14 days. followed by 221. dB-mode ultrasound provides accurate
180. d Any of the other organisms listed could administration of pregnant mare serum
potentially cause late-term abortions in December, the number of marings that occur information regardi ng pregnancy status.
about 4 months earlier (August) should be gonodotropin (PMSG) at the time of or 24 hours number of fetuses. fetal defects. and stage of
individual sows. However, leptospirosis is most before progestagen withdrawal.
common and may occur as a herd problem. calculated using the lower historica1 farrowing gestation. Doppler ultrasonography haS been
rate (57%): 53 x 0.57 = 30 sows. 209. d The normal gestation length in ewes is about used for pregnancy diagnosis but cannot
lSI. b Gilts seronegative at the time of breeding are 150 days. with a range among breeds of 145 to
192. b In boars the head of the epididymis lies at the provide detailed information.
at risk of contracting lhe virus during early 155 days.
gestation. with subsequent early embryonic ventral pole of tile testis.

01998 Mosby-Year Book, Inc. PllOtocOpyillg is prollibited by law.


www.alkottob.com
242 SECTION 9
www.vet4arab.co.cc TlleriogetlOlogy 243

251. c Follicle-stimulating hormone stimulates 271. a Reptiles do not have a larval stage.
222. a In ewes the corpus IOleum of pregnancy Tranferring fewer or more than two embryos is development of ovarian foUicles, secretion of 272. c Parthenogenesis results in all-female progeny
initially secretes progesterone: the developing not as successful as transferring two embryos in estrogen by the ovaries. and tubular growth and in reptiles.
placenta assumes this func tion later in gestation. establishing and maintaining pregnancy. spermatogenesis in the testes.

om
After about day 5, pregnancy cannot be 273. b Species that lay eggs that later hatch are
234. c Abortions caused by toxoplasmosis are 252. b Plasma levels of luteinizing hormone are
terminated by ovariectomy or administration of termed oviparous.
characterized by calcified foci of cotyledon generally higher in males than in females
luteolytic doses of prostaglandins. Panurition necrosis. 274. d Hermaphrodites have both testicular and
during the breeding season. ovarian tissllo.
can be induced with gluoocorticoids or estrogens. 235. d Usteriosis is one of the few organisms causing 253. e Prolactin depresses production of luteinizing 275. d Difficulty in giving birth or laying eggs is
223. c Arthrogryposis and hydranencephalyare abortions that also causes the dam to become hormone and follicle-stimulating homlOne in termed dystocia.
caused by Akabane virus disease. sick:. It may also result in macerated fetuses. both sexes.
224. c Breeding soundness evaluation of rams 236. b Rarely reported in the United States, 276. b Sexually precocious animals reach puberty
254. d Altricial young hatch \'lith a low metabolic early.
involves a thorough physical examination, with campylobacteriosis is acquired by ingestion in rate. Body heat increases during development.
empbasis on the reproductive organs 277. b This can lead to peritonitis and death.

b.c
small rum inan ts and not by intromission, as in
(palpation, scrotal Circumference), and semen 255. e The young ofp recocial species hatch with the
cattle. 278. c Mastitis cannot occur in a tortoise because
analysis (sperm motility and morphology). ability to thermoregulate; this capability
237. e Chlamydiosis is widespread throughout the they have no mammary glands.
develops during incubation.
225. c Scroml circumference is correlated with United States. 279. e Menopa use does not occur in reptiles.
sperm production. 256. b Progesterone causes a preovulatory surge of
238. a Older Angora does may habitually abort. luteinizing hormone. 280. c The bite of baby venomous snakes can be
226. c The buck is always the best detector of estrus 239. c Any of the answers may caLise dystocia, but the quite dangerous.
in the doe. Only a small percentage of does 257. c Arginine vasotocin causes oviposition
multiple felUses of goats resull in dystocia from (expulsion of the egg from the oviduct). 281. b Renal disease or calcium deficien cy can
show mounting behavior or have a swollen simultaneous presentation in the birth canal. cause soft egg shells.
vulva. The "buck rag" is only a substitution for 258. c Prolactin induces production of "crop milk"

tto
240. c Mannosidosis is a storage abnormality in passerines. 282. e Eggs or living young are delivered through the
the real thing (the buck). Although a low serum
affecting the central nervous system. The other proctodeum imo and through the cloaca of
progesterone level may indicate estrus, it could 259. d The testes secrete testosterone. which
answers have been associated with retained reptiles.
also be an indication of anestrus. increases the size of the brain nuclei associated
fetal membranes in goats. 283. b Amphigonia retardata refers to retention of
227. b The goat has no known chorionic with song behavior.
241. e Dietary energy levels during early lactation viable sperm in the reproductive tract of
gonadotropin. A1llhe other substances listed 260. c The bursa of Fabricius has no role in red
play no role in pregnancy toxemia. previously mated female animals.
have been used for pregnancy determination. blood cell production.
242. d Ultrasonography is the definitive diagnostic 284. d The eggs of reptiles most closely resemble
228. d Goats have a cOl)'ledonary placenta that is 261. d The molting cycle is associated with

o
procedure for diagnosing pseudopregnancy. those of birds, except that they do not have an
microscopically classified as epitheliochorial at increased testosterone and estrogen levels.
Radiography would only give an indication of air chamber.
the level of the placentome. 262. e Interstitial glandular Leydig ceUs are the
an enlarged uterus, which could be aruibutable 285. c Salmonella is easily transferred from an
229. a Artificial insemination has successfully been to pseudopregnancy, pyometra, or early testicular source of testosterone.
infected female reptile to her ova, similar to

alk
performed in goats for years via transcervical pregnancy. 263. a The left ovary is more well developed than infection in avian species.
insemination using a vaginal speculum to the right ovary.
243. c Gestation in goats normally lasts 147 to 153 286. b Sexually aggressive males are separated from
locate and stabilize the cervix. 264. b Follicle-stimulating hormone influences
days. gravid females because of the risk of damage to
230. d Although all the listed organisms may cause d Catheterization of the buck's bladder is very follicular development and maturation . intraoviductal eggs by the male's erect penis.
244.
abortions in goats, chlamydiosis, 265. a Prolactin depresses production of follicle-
difficult or impossible because of a 287. c Fertilization occurs in the oviduct.
tOxoplasmosis, and Q fever are the most diverticulum in the dorsum of the urethra at the stimulating hormone and luteiniZing hormone
common causes of infectious abortion in the 288. b The erectile portion of the penis or
level of the ischial arch. and initiates brooding behavior.
Un ited States. In theo ry, Brucella melitensis is hemipenis of most reptiles is fibrovascular.
245. b Goats ovulate during the latter part of
w. 266. c Such hens are described as determinate
no longer present in the United States. 289. c So me mated female reptiles can store viable
standing estrus. layers.
Campylobacrer jejllni and Bmcella abortllS only sperm in their reproductive tract for as long as 2
246. c Estrus lasts 12 to 24 hours in most does, but it 267. d Such hens are described as indeterminate to 4 or even more years.
rarely cause abortion.
may last up to 48 hours in some. layers.
231. e To maintain pregnancy, goats totally depend 290. b Albinism and melanism are genetically
247. a Buck: ejaculates are of very low volume but 268. e Plasma cholesterol levels do not increase in linked conditions, but the other conditions
on progesterone produced by the corpus
contain a very high concentration of sperm. the blood of laying birds. listed are not.
luteum throughout gestation.
248. a As a ruminant, the buck: has all fo ur accessory 269. b Testosterone is found in the plasma oflaying 291. b With polyspermia, more than one
ww
232. c Prostaglandin Flo is luteolytic in goats and
sex glands, with the prostate being hens. Levels peak about 8 hours before spermatozoon penetrates the ovum.
can terminate pregnancy at any stage. The
disseminated along th e pelvic urethra. ovulation.
corpus luteum mllst be 5 to 7 days old before it 292. cOnly otariids (sea lions and fur seals) have
will respond to the effects of prostaglandin. 249. d Most polled goats are genotypic females (XX). 270. a Reproductive failure in captive birds usually testicles located in a scrotal sac. All the other
a Goats are seasonally polyestrous, responding associated with failure to develop foUicies that species have testicles located under the skin in
233. b Freemartins are extremely rare in goats, so 250.
twins from embryo transfer are not a problem. to decreasing photoperiod. In the northern ovuJate. the inguinal area.
latitud es, the breeding season is in the late
summer and fall (short-day breeders).

C 1998 Mosby- Year Book, Inc. Photocopying is fJfo11ibited by law.


www.alkottob.com
244 SECTION 9 www.vet4arab.co.cc SECTION

298. e If a guinea pig is bred for the first time after 8

10
293. d Old World camels have a longer gestation
(approximately 390 days). months of age, the pelvic bones cannot separate
294. c Gestation in ferrets lasts 42 days. adequately at parturition and dystocia is likely.

om
295. a Porcine zona pcllucida is an excellent 299. d Female hamsters should be housed alone
contraceptive in feral horses. Diethylstilbestrol after breeding because they may attack and Jdll
causes fetal loss in pregnant mares. Testosterone the male. They should not be disturbed for at
propionate is used in s tallion~ Silastic implants
do not lend themselves to remote delivery.
Northistcrone appears to increase fertility.
least I week following parturition, because any
disturbance could cause them to caImibalize
Iheiryoung. In aUother species listed, the male
Toxicology
helps care for the young.
296. e Most bears apparently exhibit nonna! ovarian GoO. Osweiler
cycles and noninduced ovulation. Implantation 300. e Guinea pigs are born fully furred and with
is delayed in many bear species. open eyes.

b.c
297. e This is the same gestation length as in dogs
and cats.

Recommended Reading
NOTES BeasleyVR et a1: A systems affected approach to small animal toxicology, Urbana, Ill, 1994.
University of Illinois, CoUege of Veterinary Medicine.
Howard JL: Current veterinary therapy: food animal praclice, ed 3, Philadelphia. 1992. WB

tto
Saunders.
Bonagura JO: Kirk's current veterinary therapy XlI: small animal practice, Philad elphia. 1995. WB
Saunders.
Morgan RB: Handbook of small animal pracrice, ed 2. Philadelphia, 1992. WB Saunders.
Osweiler GO: Toxicology. Baltimore. 1996. Williams & Wilkins.

o
Practice answer sheet is on page 273.

alk
Questions
1. A concentration of0.01 % is equivalent to how 3. if the toxic level ofa drug infeed is 100 ppm fora
many parts per million (ppm)? 20-kg pig, what is the estimated roxicityofthe
drug on a milligram per kilogram ofbody weight
a. I ppm basis? Assume the feed is air dried and the pig eats
b. 10 ppm feed at the rate of6% of irs body weight daily.
c. 100 ppm
w. a. 2 mg/kg
d. 1000 ppm
b. 4 mg/kg
e. 10,000 ppm
c. Gmg/kg
d. 8 mg/kg
2. A blood lead concentration reported as 80 ..,.gldl is
the same as:
e. 10 mg/kg
ww
a. 0.08ppm
b. 0.8 ppm
c. 8ppro
d.80ppm
e. 800 ppm

Ii) /998 Mosby-Year Book, Inc. Photocopying is prollibired by!nw. Correct answers are orr pages 252-253. 245
www.alkottob.com
246 SECTION 10 www.vet4arab.co.cc Toxicology 247

Questions 9 through 11 13. Selenium absorption lly crop plants is favored by 18. In cattle. chronicfluoride toxicosis causes:
4. Induction ofemesis is recommended as a
soil tllat is:
detoxifica tion procedure in dogs ingesting any of A 9-mOflth-old male Doberman pinscher puppy is a. diarrhea. pale hair coat, lameness. and hoof
the /offow;ng except: seen because of weigilt loss and periodic vomiting. a. acidic, wet, and poorly drained overgrov.rth

om
a. antifreeze (ethylene glycol) witl! intermitrent periods ofdepressiofl alternating b. acidic, semi-arid, and weU drained b. icterus, hemoglobinuria. and
with seizures or hysteria during the past 6 weeks. All c. alkaline, well aerated, and well drained photosensitization
h. acetaminophen
vaccinations are current, parasites are not a problem, d. alkaline, wet, and poorly aerated c. emaciation, hair loss, and lameness
c. gasoline a nd there is flO history of injury. Ati abdominal e. acidic or alkaline, wet, and poorly aerated d. rumen stasis, nephrosis, and constipation
d. liquid aspirin radiograph reveals two large radiodense objects in the e. lameness, exostosis, and excessive dental wear
e. chocolate stomach. A complete blood count reveals normal
leukocyte numbers, mild anemia, and an excessive 14. [n callie, which ofthefoUowing is least likely ro
number of nucleated red blood cells. be confused w ith chronic selenium toxicosis? 19. Wllicll allribacterial has been associated with
5. TIle antidotal agent N-aceryicysreine is indicated
acute seizures in calves and "ro und heart disease"
for treatment of poisoning with: a. chronic ergotism

b.c
cardiomyopathy in pollitry?
a. cholecalciferol rodenticides 9. Your temative diagnoses should itlelude: b. molybdenosis
c. flu oride poisoning a. arsanilic acid
b. acetaminophen a. lead or arsenic toxicosis
d. laminitis associated with grain overload b. furazolidone
c. brodifacoum b. lead or zinc toxicosis
e. mercury poison ing c. lincomycin
d. chlorpyrifos c. zinc or copper toxicosis d. gentamicin
e. copper d. iron or zinc toxicosis e. chloramphenicol
15. In horses. wllat is the most likely cause ofacute
e. iron or cadmium toxicosis
allorexia, sweating, colic, stiffness. progressive
6. [norganicarsenic toxicosis is manifested clinically

tto
ataxia, paresis, tachycardia, and hypote1lsion? 20. Which organic symhetic herbicide is often
as, 10. W1w t is the best single sample to analyze f or considered dangerous because it induces
a. arsenic poisoning
a. icterus, anemia, and hemoglobinuria confirmation ofrhe suspected toxicosis? accumulation ofnirrites in some weed species?
b. antibiotic toxicosis
b. amaurosis, incoordination, and constipation a. whole blood a. paraquat
c. mone ns in toxicosis
c. cardiomyopathy, hydrothorax, and ascites b. serum h. glyphosate
d. poisoning by red maple leaves
d. photosensitization, dermatitis, and hair loss c. urine c. lindane
e. organophosphate poisoning
e. vomhing, gastroenteritis, diarrhea, and

o
d. stomach contents d. 2,4 -dichlorophenoxy acetic acid
dehydration
e. hair e. pentachlorophenol
16. In rumina nts. urea toxicosis is characterized by:
7. Which combination a/mineral additives is most a. rumina! alkalOSis, systemic acidosis, and

alk
11. Based on the history and clinical signs, wlmt is 21. Which category of insecticidal compollnds
useful in preventing chron ic copper toxicosis in elevated blood ammonia levels
the most likely cause of tile problem? presents a problem of persistent residues in fatty
slleep? b. ruminal acidosis, systemic aIlcaIosis, and tissuesofanimals?
a. cadmium toxicosis elevated blood ammonia levels
a. selenium and molybdenum
b. copper toxicosis c. ruminal alkalosis, systemic alkalos is, and a. carbamates
b. selenium and sulfate
c. iron toxicosis elevated blood ammonia levels b. organochlorines
c. zine and molybdenum
d. lead toxicosis d. ruminal acidosis, systemic alkalosis, and c. organophosphates
d. sulfate and molybdenum
e. zinc toxicosis decreased blood ammonia levels d. pyrethrins
e. arsenic and sulfate w. e. rumina! alkalosis, systemic alkalosis. and e. juvenile hormones
elevated blood urea nitrogen levels
12. Deficiency of what element in tile sow predisposes
8. Chronic diarrhea, achromotrichia , emaciation,
bally pigs to toxicosis lly injectable iron 22. If acute organophosphate insecticide poisoning is
and lameness in cattle suggest toxicosis involving:
preparatiotlS? 17. [11 swine tissue sections. microscopic lesions of suspected. wllat is the best initial sample to
a. arsenic eosinophilic m eningoencephalitis are obtain from a liveanimalfor itlitial diagnostic
a. copper
characteristic of: testing?
b. lead
b. chromium
ww
c. mercury a. antifreeze ingestion a. serum
c. magnesium
d. molybdenum b. aminoglycoside toxicosis b. whole blood
d. selenium
e. selenium c. water deprivation c. urine
e. zinc
d. mercury toxicos is d. stomach conte nts
e. monensin toxicosis e. fat biopsy

0 1998 Mosby-Year Book, Inc. Photocopying is prohibited by la w. Correct answers are on pages 252·253.

www.alkottob.com
250 SECTION 10 www.vet4arab.co.cc Toxicology 251

54. CAuses tremors and myocardial damage in 62. A cat develops incoordination,followed ill 12 66. [n dogs, such neurologic signs as disorientation, d. parakeets
grazing horses; passed in the milk oflnctating hOllrs by vomiting, weakness, tachypnea, oliguria, delirium, agitation, excitement, and e. reptiles
animals hypothermia, and marked metabolicaddosis. incoordination WQuld be expected after ingestion
71lOracic radiographs reveal 110 abnormalities.

m
ofany of the following except:
Mlat is the most likely calise of these signs? 71. Which clinicopatllOlogic value is least likely to be
55. Its seeds cOfltamjflare small grains; causes acute a. Datura seeds abnormai}210 24 hours aftera dog is bitten bya
to chronic liver and pulmonary damage. a. arsenic poisoning b. morning glory seeds rattlesnake?
especially in swine and poultry b. caffeine toxicosis c. marijuana a. serum amylase activity
c. djnitrophenoJ (ONP) poisoning d. Easter lily leaves

o
b. serum creatine phosphokinase activity
For Questions 56 through 60, select the correct d. ethylene glycol poisoning e. ergot alkaloids c. serum -y-glutamylnansferase activity
answer from the five choices below. c. organophosphate poisoning
d. platelet count
a. nitrite 67. Laminitis may develop in horses housed in a stall e. prothrombin time

b.c
b. cyanide 63. lvermectin toxicosis ill collies is characterized by: whose floor is covered with:
c. oxalate a. excitation, profuse salivation, tremors, and a. black cherry shavings 72. The avian toxicant 4-aminopyridine is toxic to
d. tannin vomiting b. black walnut shavings dogs and causes cUnical effects similar to those of
e. pyrrolizidinc alkaloid b. vomiting and hyperesthesia c. oak shavings a. arsenic
c. ataxia, incoordination, weakness, and d. cedar shavings
depression b. ethylene glycol
56. Causes methemoglobinemia e. pentachlorophenol-treated shavings
d. blindness, colic, and seizures c. lead
d. organophosphates

o
57. Inhibits cytochrome oxidase and prevents
e. dyspnea, polypnea, and tachycardia
68. Which toxic principle does not occur in the seeds e. strychnine
miUzation of oxygenated blood of the following respective pkmts?
64. Fluorescence ofblood on exposure to ultraviolet
a. aesculin in Ohio buckeye 73. The toxic principles in chocolate that affect dogs

ott
58. [merJems witlt rumen junction. precipitates light is IIseful in diagnosing:
b. carboxyatractyloside in cocklebw are:
proteins, and causes renal damage if! cattle a. aflatoxicosis of swine c. coniine in poison hemlock a. anagyrine alkaloids
b. ethylene glycol poisoning of cats
d. hydrogen cyanide in wild cherry b. cyclic polypeptides
59. Reduces available serum calcium and produces c. fluoride poisoning of canJe e. nitrate in Sorghum
crystals demonstrable in the vascular walls. d. lead poisoning of waterfowl c. L-CYSleine and tryptophan
kidney. and urine e. selenium toxicosis of horses d. methylxanthine alkaloids
69. Which protein supplement contains a toxic e. solanidine and ergonovine

alk
prinCiple causing congestive heart failure and
60. Produces chronic emo.ciatiOIl. hepatoloxicosis. 65. A group of swine sllOws auuia, paralysis, hoof liver necrosis and is well tolerated by marure
and sometimes hepatic encep/lalopathy in horses (corollary band) and hair lesions, and lesions of cattle but relatively toxic to swine and immature 74. Diarrhea in swine and polioencephalomalacia in
focal symmetric pofiomyeiomalacia. The most (nonruminating) calves? cattle have been observed when livestock consume
likely cause of these signs is toxicosis involving: looter containing high concentrations of:
61. Which of tile following is a major western toxic a. alfalfa meal
piant associated with acwe to SIlbacute a. arsenic b. canola meal
a. arsenic
neurologic sigrlS ill caule, vacuolacion in b. fluoride
b. copper c. cottonseed meal
cerebellar neurons, and congenital skeletal w. c. nitrate
c. lead d. linseed meal
malformations?
d. selenium d. sulfate
e. soybean meal
a. Senecio e. zinc e. zinc
b. Halogeton
c. Delphinium 70. Overheated Teflon-coated frying pans release
IKlPOrs that are especially toxic to; 75. In cats, ingestion of Easter lily leaves causes fatal
d. Cemallrea toxicosis affecting primarily the:
a. cats
ww
e. Astragalus
a. nervous system
b. dogs
b. lungs
c. gerbils
c. liver
d. kidneys
e. gasnointestinai tract

@1998Mosby-YearBook, Inc. PllOtocopying is prohibited by law. Correct answers are on pages 252-253.
www.alkottob.com
252 SECTION 10
www.vet4arab.co.cc Toxicology 253

Answers 41. c Oicumarol inhibits reactivation of vitamin K,


which is necessary for coagulation factor
61. e Swainsonine alkalOids in Astragalus
(locoweed) inhibit a~mannosidase, causing
I. c Convert % to ppm by moving the decimal point 20. d 2,4-Dichlorophenoxy acetic acid is a pla nt synthesis. vacuolated neurons; arthrogryposis is another

m
fo ur places to the right. gro\vth hormone that alters nitrogen 42. b Vomitoxin causes nausea and induces a sequela.
2. b 80 p.g/dl = 800 IJ-g/ L = 0.8 mg/L = 0.8 ppm. metabolism, especially in plants with a tendency learned aversion to the contaminated feed. 62. d Lack of fever differentiates ethyle ne glycol
3. c 20 kg X 0.06 :: 1.2 kg feed/day. 1.2 kg X 100 to accumulate nitrate. 43. d Ergot alkaloids inhibit prolactin in many from dinitrophenol toxicosis.
mg/kg =120 mg total intake. 120 mg + 20 kg = 6 21. b Chlorinated hydrocarbon insecticides are species. 63. c Ivermectin causes depression, with lillie or no
mg/kg. highly lipophilic. 44. e Zearalenone is a physiologic estrogen with tendency toward excitement or se izures.

.co
4. c Ingestion of gasoline. a volatile hydrocarbon, 22. b Cholinesterase activity is most concentrated in estrogenic effects. 64. d Porphyrin metabolism is blocked. The
may lead to inhalation during emesis, with red blood cells. 45. a Aflatoxins are regulated by the Food a nd Drug accumulated porphyrins are de tected by their
subsequcm pneumonia. 23. d Some visual disturbance may accompany Administration because of the ir toxic and fluorescence on exposure to ultraviolet light.
5. b N-acetylcysteine helps replenish glutathione. miosis, bUI true blindness is not a feature. carcinogenic nature. 65. d Selenium affects the skin, hair, and hoof
6. e The principal effects of arsenic are 24 . d The increased stability of this bond makes 46. d Aspergillus ochraceus produces ochratoxin, a through inte rference with sulfur amino acids. It
gastrointestinal, espccially those involving the gut therapy more difficult as time passes. potent nephrotoxin. is the only metal of those listed that causes
mucosa. 25. c Pers iste nce in tissue m ay necessitate therapy 47. c Aspergillus fIavus is a common species poliomyelomalacia.
producing aflatoxin.

ob
7. d Sulfate Is converted to sulfide and forms fo r 3 to 4 weeks. 66. d Easter lily contains a nephrotoxicant and is of
insoluble copper sulfide in the rumen. 26. e Fluoroacel'ate is more toxic to dogs than to 48. a Acremonium coenopliialum is a n endophyte apparent low toxicity in dogs. AJI the othe rs listed
Molybdenum forms a thiomolybda te complex rodents. fungus of fescue, associated with ~ fescue foot. ~ a re neurotoxicants that could cause the signs
that promotes copper excretion. 49. e Fusarium roseum prod uces zearalenone, an described.
27. b Signs are caused by suppression ofinhibitOiY
8. d Molybdenum p romotes copper excretion, lnte m e urons in the spinal reflex arc. estrogen. 67. b An unknown toxic principle is present in black
resulting in copper deficiency. 28. d Bromethalin affects Na ~ K ATPase. leading to 50. b Fusarium moniliforme produces fumonisins. walnut. Only horses manifest this reaction.
9. b Qmy lead and zinc affect the hemogram in a loss of cellular flu id control. which cause malacia of cerebral white m a tter. 68. e Nitrate accumula tes only in vegetative plant

ott
simila r manner. 51. a Japanese yew is a common cause of toxicosis tissues, not in seeds.
29. a Clinical signs do not a ppear until coagulation
10. a Lead toxicosis is best reflected in red blood factors a re depleted. in horses and cattle and is also toxic to dogs. 69. c Cottonseed meal contains a phenolic
ceUs. nO! in serum or urine. 30. c Acute tissue damage is followed by 52. c The toxic prinCiple causing nephrosis has not principle, gossypol, tha t is poorly tolerated by
11 . d The combination of gastroi ntestinal, mineralization when vitamin 0 rodenticides are been defined, but lesions are s imilar to those nonruminan ts, especially swine.
ne urologic, and hematologic changes suggests consumed. seen in oak (acorn) toxicosis. 70. d Teflon fum es sensitize the budgerigar's heart
only lead toxicosis. 31. c Carboxyhemoglobin is red to pink and causes 53. b It causes early oxidant damage and Heinz to epinephrine and endogenous catecholamines.

alk
12. d Several studies have documented that maternal ~ fetal hypoxia. body fo rmation in red blood cells. 71. a Amylase is a pancreatic enzyme. AJI the other
selenium deficiency in the sow predisposes 32. b Carbon dioxide causes loss of consciousness 54. d This plant is hazardous to both adult animals values may be affected by the necrotizing and
piglets to iron lOxicosis. before respiratory failure. and nursing foals or calves. cytotoxic effects of Crotalidae venoms.
13. c Alkaline conditions a nd well ~drained soil favor 33. d Sulfides are irritating and rapidly produce loss 55. e It contains high concentratio ns of 72. e HyperestheSia, tremors, and seizures result
selenium absorption. of consciousness. pyrrolizidine alkalOids, which cause liver when dogs ingest aminopyridine.
14. e Mercury causes central nervous syste m signs, 34. e Oxides of nitrogen often form in silage damage. 73. d Toxic principles in chocolate include the
bl indness. and h igh mortality. The other choices containing high levels of nitrates. 56. a Iron is oxidized to the ferric state by the n itrite stimulants caffeine, theophylline, and
listed are mainly chronic, peripheral effects. ion. theobrom ine.
15. c Monensin affect's m itochondrial function and
ion transport, resulting in metabolic
myodegeneratio n.
16. a Ammonia produces alkaline conditions in the
rumen. A high blood ammon ia level inllibits the
formed from animal wastes.
w.
35. a Ammonia is a major toxicant or pollutant

36. e Amphetamines are commonly used in people.


At high dosages, nervous signs predomina te.
37. a These effects are especially predominant in
cats.
57. b Affected cells cannot accept electrons and
hence cannot bind oxygen.
58. d Tannin has a potent effect on proteins. It is the
toxic principle of acorns or oak buds.
59. c High oxalate levels are present iJl several plants
74. d Sulfates induce osmotic diarrhea that is
usually transient. in cattle, sulfates may be
reduced to sulfides and interfere with thiam in
synthesis or utilization.
'75. d Cats a re very sensitive to the nephrotoxicant
in Easter lily.
ww
tricarboxylic acid cycle, causing me ta bolic prevalent in the midwestern and western United
38. b Aspirin should be used a t the recommended
acidosis. States,
dosages in animals, especially in cats.
17. c This is tlle classic microscopic lesion of wate r 60. e Alkaloids induce hepatomegalocytosis and
39. d Methylxanthine alkaloids (caffeine.
deprivation (salt poisoning) in swine. nuclear dam age and sometimes are
theobromine) are hazardous to dogs.
18. e Fluorides affe ct enamel formation and bone carcinogenic. Chronic liver damage leads to
remodeling. 40. c Large doses of nonsteroidal a ntiinflammatory
encephalopathy.
drugs may induce toxicos is afte r several day-s'
19. b Nitrofura ns, such as furazoli done, interfere
exposure.
with carbohydrate metabolism in L~e central
nervous system and cardiac muscle.

www.alkottob.com
248 SECTION 10 www.vet4arab.co.cc Tox icology 249

23. Cholinesterase inhibitor pesticides typically cause 26. No longer legal to use because of probability of For Questions 36 through 40, select the correct 45. Chronic hepatotoxill alld carcinogell tllat can
all a/the following except: secondary or relay toxicosis; Iligllly toxic to dogs answer from the five choices below. suppress immullefunctioll
a. salivation a. acetaminophen

om
b. miosis 27. Om calise acute tetanic seizures, with tmlrked b. aspirin For Questions 46 through 50, select the correct
hyperesthesia, hyperreflexia, and death from c. ibuprofen answer from the five choices below.
c. dyspnea
respiratory muscle failure
d. blindness d. caffeine a. Acremonium coenophialum
e. bradycardia e. amphetamine b. Fllsarium moniliforme
28. Can cause acute to chronic neurologic signs by c. Aspergillus/lavllS
altering cen tral nervous system/luid balance,
24. When applied to organophosphate insecticide 36. Central nervous system stimulanlused for d. Aspergillus ochraceus
leading to cerebral edema
poisoning, the term aging refers to: appetite suppression and mood elevation e. Fllsarium roseum

b.c
a. loss of insecticidal activity with time 29. Clinical signs may be delayed for 1 or 2days after
b. isomerization afthe organophosphate to a exposure, regardless ofdosage 37. One or two 325-mg tablets can cause 46. Causes chronic rellal failure in swine, characterized
more toxic chemical form methemoglobinemia, facial edema, and hepatic by polyuria, polydipsia, and emaciation
c. hydrolysis afthe choli nesterase- 30. Ca n cause acute renal damage, hypercalcemia, damage
organophosphate bond induced by oxime and tissue mineralization 47. Causes chronic filler damage wilh bile duct
drugs hyperplasia
38. Chronic use or excessive short-term IISeCallSes
d. a chemical change that increases the stability gaseric II lcers, vomiting, depression, anorexia, and
of the organophosphate-cholinesterase bond For Questions 31 through 35, select the correct
answer from the five choices below. toxic hepatitis; more toxic to cats than dogs 48. Causes peripheral gangrene, sloughing ofhooves,

tto
e. altered toxicity of organophosphates from and intolerance to hot weather in cattle. and
spontaneous hydrolysis of ester groups a. ammonia placental lesions and abortion in horses
39. Can cause acute central nervous system
b. carbon dioxide stimlllation, with Ilyperrefiexia, tonic seizures,
25. Newera nricoagulant rodenticides, also known as c. carbon monoxide cach}'cardia, polypnea, and hyperthermia 49. Causes hyperestrogenism and vulvovaginitis in
second-generation anticoagulants, are important d. hydrogen sulfide prepubertal gilts
in veterinary medicine because they: e. nilfogen dioxide
40. Acute toxicosis causes oliguria, renal papillary
a. have been developed to be toxic in rats but

o
necrosis, and llremia;chronic olJemse may cause 50. Causes lellkoellcephalomalacia in horses
not in other classes of m ammals 31. Causes cherry- red blood and may induce gastric ulcers in dogs
b. have effeCls that are readily treated by premature delivery of stillborn piglets
synthetiC vitamin K injection For Questions 5 1 through 55, select the correct
For Questions 41 through 45, select the correct

alk
answer from the five choices below.
c. a re more potent or longer acting than first- 32. Low levels stimulnte respimrion, but high levels answer from the five c hoices below.
generation a nticoagulants, requiring calise loss ofconsciousness, followed by depression a. Japa nese yew
prolonged therapy a. aflatoxin b. red maple
of respiratory centers
d. are more readily detected by chemical b. deoxynivalenol (Vornitoxin) c. redroat p igweed
analysiS than first-generation rodenticides c. dicumarol d. white snakeroot
33. Highly toxic gas released wilen maT/ure pies are
e. do not interact with other drugs or chemicals d. ergot e. Crotalaria
agitated; causes mild respiratory irritation, loss of
olfactory function, and rapid loss ofconsciousness; e. zearalenonc
For Questions 26 through 30, select the correct
answer from the five choices below.
a. anticoagulant rodentkides
w.
dea th is from roxic depression of respiratory centers

34. Hea vier than air;formed in silos IInder some


conditions; causes direct irritation of respiratory
41. Produced in moldy sweet clover; resulting in
vitamin K-responsive coagulopatlly
51. Omamelllal evergreen shmb witll all acute
cardiotoxic prillciple that causes sudden death in
horses and cattle
b. strychnine
c. cholecalciferol traer and lesions in the lllngs 42. Principal effect is dose-relntedfeed refusal 52. Nirrate-accumularing plnnt common in gardens,
d. bromethaJin crop fields, andfeedloes; causes acllte toxic
ww
e. sodium fluoroacetate 35. Stro ng upper respiratory irritant formed by 43. Causes agalactia in periparturient animals by tubular nephrosis with perirenal edema and
decomposition ofhigil-nitrogen swine and inhibition ofproiactin release ascites ill swine and cattle
poultry wastes; most clinical effects are from local
irritation
44. In p igs, causes prolonged anestrus or 53. Causes acute hemolytic anemia in horses
pseudopregnancy, as well as early embryonic death

C 1998 Mosby-Year Booi.:, Inc. Photocopying is prohibited by law. Correct atlSweTS are on pages 252-253.
www.alkottob.com
www.vet4arab.co.cc

m
.co
ob
ott
alk
w.
ww

ISBN 0-8151 -7462-4

28273
www.alkottob.com

You might also like